You are on page 1of 310

TUYỂN CHỌN VẬN DỤNG – VẬN DỤNG CAO 2022 Điện thoại: 0946798489

VẤN ĐỀ 1. HÀM SỐ
• |FanPage: Nguyễn Bảo Vương
• TUYỂN CHỌN CÂU HỎI VD-VDC TỪ CÁC ĐỀ THI THỬ CÁC TRƯỜNG, CÁC SỞ NĂM 2022
Câu 1. (Chuyên Vinh – 2022) Cho hàm số y  f ( x) có đạo hàm là f ( x)   x 2  9 x  x 2  9  với mọi


x   . Có bao nhiêu giá trị nguyên của tham số m để hàm số y  g ( x)  f x3  3x  2m  m2 có không 
quá 6 điểm cực trị ?
A. 2. B. 5. C. 4. D. 7.
Câu 2. (Chuyên Vinh – 2022) Cho hàm số đa thức bậc bốn y  f ( x ) có đồ thị như hình vẽ bên.

 
Có bao nhiêu số nguyên a để phương trình f x 2  4 x  3  a có không ít hơn 10 nghiệm thực
phân biệt?
A. 4. B. 6. C. 2. D. 8.
Câu 3. (Chuyên Vinh -2022) Cho hàm số bậc ba y  f ( x ) . Biết rằng hàm số y  f  1  x 2  có đồ thị
như hình vẽ bên.

 x2 1  2
Số điểm cực trị của hàm số g ( x)  f  2   là
 x  x
A. 5. B. 4. C. 3. D. 7.
Câu 4. (Chuyên Lê Quý Đôn - Điện Biên - 2022) Cho hàm số f  x  có đạo hàm
2 1
f   x    x  1 x  1  x  2  . Giá trị nhỏ nhất của hàm số g  x   f  x   x 3  x  2 có đạo hàm
3
trên đoạn  1; 2 bằng

Facebook Nguyễn Vương https://www.facebook.com/phong.baovuong Trang 1


Blog: Nguyễn Bảo Vương: https://www.nbv.edu.vn/
3 8 4
A. f  2  . B. f 1  . C. f  0   2 . D. f  1  .
4 3 3
Câu 5. (Chuyên Lê Quý Đôn - Điện Biên - 2022) Có bao nhiêu giá trị nguyên của tham số m sao cho
3
m  3 3 m  3log x  log x có 3 nghiệm phân biệt?
phương trình
A. 2. B. 1. C. 3. D. 4.
Câu 6. (Cụm Trường Nghệ An - 2022) Hàm số y  f   có đồ thị là đường cong trong hình vẽ. Số
x
nghiệm thực phân biệt của phương trình f e f  x

 f  x   1 là:

A. 2 . B. 4 . C. 6 . D. 8 .
Câu 7. (Cụm Trường Nghệ An - 2022) Cho hàm số y  f  x  có đồ thị như hình vẽ. Có bao nhiêu giá

 
trị nguyên của tham số m thuộc đoạn  2021; 2021 để hàm số g  x   f x5  4 x  m có ít nhất
5 điểm cực trị.

A. 2022 . B. 2023 . C. 2021 . D. 1012 .


Câu 8.  
(Đại học Hồng Đức 2022) Cho hàm đa thức y   f x2  2 x   có đồ thị cắt trục Ox tại 5 điểm
phân biệt như hình vẽ. Hỏi có bao nhiêu giá trị của tham số m với 2022m  để hàm số
g ( x)  f  x 2  2 | x  1| 2 x  m  có 9 điểm cực trị?

A. 2020. B. 2023. C. 2021. D. 2022.

Trang 2 Fanpage Nguyễn Bảo Vương  https://www.facebook.com/tracnghiemtoanthpt489/


Điện thoại: 0946798489 TUYỂN CHỌN VẬN DỤNG – VẬN DỤNG CAO 2022
Câu 9. (Đại học Hồng Đức – 2022) Cho f ( x)  x  3 x 2  1 . Phương trình f ( f ( x)  1)  1  f ( x)  2
3

có số nghiệm thực là
A. 7. B. 6. C. 4. D. 9.
Câu 10. (THPT Hồ Nghinh – Quảng Nam – 2022) Cho hàm số y  f ( x) có bảng biến thiên như sau

Có bao nhiêu giá trị nguyên của tham số m để phương trình 6 f  x 2  4 x   m có it nhất 3 nghiệm
thực phân biệt thuộc khoảng (0;  ) ?
A. 29. B. 25. C. 24. D. 30.
Câu 11. (THPT Hồ Nghinh – Quảng Nam – 2022) Cho hàm số f ( x ) có đạo hàm
 
f ( x)  ( x  1)2 x 2  4 x . Có bao nhiêu giá trị nguyên dương của tham số m để hàm số

g ( x)  f 2 x 2
 12 x  m  có đúng 5 điểm cực trị?
A. 17. B. 16. C. 18. D. 19.
Câu 12. (THPT Hương Sơn - Hà Tĩnh - 2022) Cho các hàm số y  f  x  ; y  f  f  x   ;
y  f  x 2  2 x  1 có đồ thị lần lượt là  C1  ;  C2  ;  C3  . Đường thẳng x  2 cắt  C1  ;  C2  ;  C3 
lần lượt tại A, B, C . Biết rằng phương trình tiếp tuyến của  C1  tại A và của  C2  tại B lần lượt
là y  2 x  3 và y  8 x  5 . Phương trình tiếp tuyến của  C3  tại C là
A. y  8 x  9 . B. y  12 x  3 . C. y  24 x  27 . D. y  4 x  1 .
Câu 13. (THPT Hương Sơn - Hà Tĩnh - 2022) Cho hàm số y  f  x  có đồ thị như hìnhh vẽ. Hàm số
y  4  f 2  x  có bao nhiêu điểm cực trị?

A. 4 . B. 5 . C. 3 . D. 6 .

Câu 14. (THPT Lê Thánh Tông - HCM-2022) Hàm số y  f  x  có đạo hàm trên  4; 4 , có các điểm
4
cực trị trên  4; 4  là 3;  ;0; 2 và có đồ thị như hình vẽ. Đặt g  x   f  x3  3x   m với m là
3
tham số. Gọi m1 là giá trị của m để max g  x   2022, m2 là giá trị của m để min g  x   2004.
0;1 1;0
Giá trị của m1  m2 bằng

Facebook Nguyễn Vương https://www.facebook.com/phong.baovuongTrang 3


Blog: Nguyễn Bảo Vương: https://www.nbv.edu.vn/

A. 12 . B. 13 . C. 11 . D. 14 .

Câu 15. (THPT Lê Thánh Tông - HCM-2022) Cho hàm số bậc bốn y  f  x  có đồ thị  C1  và
y  f   x  có đồ thị  C2  như hình vẽ dưới.

Số điểm cực đại của đồ thị hàm số g  x   f  e  x f  x   trên khoảng   ;3 là


A. 5 . B. 3 . C. 6 . D. 4 .
Câu 16. (Liên trường Hà Tĩnh – 2022) Cho hàm số f ( x)  x 4  14 x3  36 x 2  (16  m) x với m là tham
số thực. Có bao nhiêu giá trị nguyên của m để hàm số g ( x)  f (| x |) có 7 điểm cực trị?
A. 33. B. 31. C. 32. D. 34.
x x
Câu 17. (Liên trường Hà Tĩnh – 2022) Cho hàm số y  f ( x)  2022  2022  x  sin x . Có bao nhiêu
giá trị nguyên của m để phương trình f ( x  3)  f  x3  4 x  m   0 có ba nghiệm phân biệt?
A. 4. B. 3. C. 2. D. 5.
Câu 18. (THPT Nguyễn Tất Thành-Đh-SP-HN-2022) Cho hàm số f  x  xác định trên  , có đạo hàm
f   x    x 2  4   x  5  , x   và f 1  0 . Có bao nhiêu giá trị nguyên của m để hàm số
g  x   f  x 2  1  m có nhiều điểm cực trị nhất ?

A. 6 . B. 8 . C. 5 . D. 7 .
Câu 19. (THPT Nho Quan A – Ninh Bình – 2022) Cho hàm số f ( x ) có đạo hàm liên tục trên  . Đồ thị
của hàm số y  f (5  2 x ) như hình vẽ bên dưới:

Trang 4 Fanpage Nguyễn Bảo Vương  https://www.facebook.com/tracnghiemtoanthpt489/


Điện thoại: 0946798489 TUYỂN CHỌN VẬN DỤNG – VẬN DỤNG CAO 2022

Có bao nhiêu giá trị thực của tham số m thuộc khoảng (9;9) thỏa mãn 2m và hàm số
1
y  2 f  4 x3  1  m  có 5 điểm cực trị ?
2
A. 26. B. 25. C. 27. D. 24.
Câu 20. (THPT Phù Cừ - Hưng Yên - 2022) Cho hàm số y  f ( x) có đồ thị hàm số y  f ( x ) như hình
vẽ.

Số giá trị nguyên của tham số m để hàm số g ( x)  f  2 x 2  4 | x |  m  3 có 7 điểm cực trị.


A. 1. B. 2. C. 4. D. 3.
Câu 21. (Sở Hà Tĩnh 2022) Cho hàm số f ( x ) liên tục trên  và đồ thị của hàm số y  f (1  x ) như
hình vë bên:

 1 x  2x 1
Số giá trị nguyên của m để phương trình f    m  0 có 4 nghiệm phân biệt là
 x2 x2
A. 3. B. 4. C. 2. D. 5.
Câu 22. (Sở Hà Tình 2022) Cho hàm số bậc ba f ( x ) và hàm số g ( x )  f ( x  1) thoả mãn
 
( x  1) g ( x  3)  ( x  1) g ( x  2) , x   . Số điếm cực trị của hàm số y  f 2 x 2  4 x  5 là
A. 1. B. 3. C. 2. D. 5.
Câu 23. (Sở Thanh Hóa 2022) Cho hàm số f ( x ) có đạo hàm liên tục trên  và f (0)  0 , đồ thị của
f ( x ) như hình vẽ:

Facebook Nguyễn Vương https://www.facebook.com/phong.baovuongTrang 5


Blog: Nguyễn Bảo Vương: https://www.nbv.edu.vn/

Gọi m, n lần lượt là số điểm cực đại, số điểm cực tiểu của hàm số g ( x ) | f (| x |)  3 | x || . Giá trị
của m n bằng
A. 4. B. 8. C. 27. D. 16.
Câu 24. (Sở Thanh Hóa 2022) Cho hàm số f ( x ) có đồ thị của đạo hàm như hình vë:

Giá trị lớn nhất của hàm số h( x)  3 f ( x)  x 3  3 x trên đoạn [ 3; 3] bằng


A. 3 f (1)  2 . B. 3 f (0) . C. 3 f ( 3) . D. 3 f ( 3) .
Câu 25. (Sở Thanh Hóa 2022) Cho hàm số f ( x ) liên tục trên  và có đổ thị như hình vẽ:

Xét T  103 f  a 2  a  1  234 f (af (b)  bf (a )), ( a, b  ) . Biết T có giá trị lónn nhát bằng M
M
đạt tại m cặp ( a; b) , khi đó
bằng
m
1011 1011 337 674
A. . B. . C. . D. .
4 8 2 3
Câu 26. (Sở Thanh Hóa 2022) Cho hàm số bậc bốn f ( x ) có đồ thị của đạo hàm như hình vẽ:

Trang 6 Fanpage Nguyễn Bảo Vương  https://www.facebook.com/tracnghiemtoanthpt489/


Điện thoại: 0946798489 TUYỂN CHỌN VẬN DỤNG – VẬN DỤNG CAO 2022

Số điểm cực tiểu của hàm số g ( x)  4 f  x 2  4   x 4  8 x 2 là


A. 4. B. 7. C. 3. D. 5.
 
Câu 27. (Sở Bắc Giang 2022) Biết rằng f (0)  0 . Hỏi hàm số g ( x)  f x  x3 có bao nhiêu điểm cực
6

đại?

A. 2. B. 4. C. 3. D. 1.
Câu 28. (Sở Bắc Giang 2022) Cho hàm số y  f ( x) liên tục trên  và có đồ thị như hình vẽ dưới đây

Đặt g ( x)  f   
x2  4 x  6  2 x2  4 x  x 2  4 x  6  12 x 2  4 x  6  1 . Tổng giá trị lớn nhất
và giá trị nhỏ nhất của hàm số g ( x ) trên đoạn [1; 4] bằng
A. 12  2 4 . B. 12  12 6 . C. 12  2 4 . D. 12  12 6 .
Câu 29. (Sở Hà Tĩnh 2022) Cho hàm số bậc ba y  f ( x) có f (1)  3 và có đồ thị như hình vẽ bên. Có
bao nhiêu giá trị nguyên của tham số m và m [ 10;10] để phương trình
f ( x)
ln 2
 x[ f ( x )  3mx]  3mx 3  f ( x) có hai nghiệm dương phân biệt?
3mx

Facebook Nguyễn Vương https://www.facebook.com/phong.baovuongTrang 7


Blog: Nguyễn Bảo Vương: https://www.nbv.edu.vn/

A. 18. B. 9. C. 10. D. 15.


3 2
Câu 30. (Sở Ninh Bình 2022) Cho hàm số y  f ( x)  ax  bx  cx  d có bảng biến thiên như sau

Tìm m để phương trình | f ( x  1)  2 | m có 4 nghiệm thỏa mãn x1  x2  x3  1  x4 .


A. 4  m  6 . B. 3  m  6 . C. 2  m  6 . D. 2  m  4 .
Câu 31. (Sở Ninh Bình 2022) Cho f ( x ) là hàm số bậc ba. Hàm số f ( x ) có đồ thị như hình vẽ. Tìm tất
cả các giá trị thực của tham số m để phương trình f  e x  1  x  m  0 có hai nghiệm thực phân
biệt?

A. m  f (2) . B. m  f (0) . C. m  f (0) . D. m  f (2) .


Câu 32. (Sở Bạc Liêu 2022) Cho hàm số y  f  x  liên tục trên  và có đồ thị có 3 điểm cực trị như
hình vẽ.

Trang 8 Fanpage Nguyễn Bảo Vương  https://www.facebook.com/tracnghiemtoanthpt489/


Điện thoại: 0946798489 TUYỂN CHỌN VẬN DỤNG – VẬN DỤNG CAO 2022

Số điểm cực trị của hàm số g  x   f  x 3  3 x  2  là


A. 5 . B. 11 . C. 9 . D. 7 .
Câu 33. (Sở Hà Tĩnh 2022) Cho hàm số y  f  x  là hàm số đa thức bậc bốn và có bảng biến thiên như
1
 3
x4
hình bên. Tìm số điểm cực trị của hàm số g  x   2  f  2 x  1 

A. 7 . B. 5 . C. 4 . D. 6 .
Câu 34. (Sở Hà Tĩnh 2022) Cho hàm số y  f  x  liên tục trên  có đồ thị như hình vẽ..

Tính tổng tất cả các giá trị nguyên của m để phương trình f 1  2sin x   m có đúng hai nghiệm
trên  0;  

A. 3 . B. 2 . C. 0 . D. 6 .
Câu 35. (Sở Lạng Sơn 2022) Biết rằng tập tất cả các giá trị của tham số m để phương trình
m  x  4  x 2  2  5x 2  8x  24 có bốn nghiệm thực phân biệt là khoảng  a; b  . Giá trị a  b
bằng
28 25
A. . B. . C. 4 . D. 9 .
3 3
Câu 36. (Sở Lạng Sơn 2022) Cho hàm số f  x  có bảng biến thiên như sau:

Facebook Nguyễn Vương https://www.facebook.com/phong.baovuongTrang 9


Blog: Nguyễn Bảo Vương: https://www.nbv.edu.vn/

3 2
Hàm số y   f  x    3  f  x   đồng biến trên khoảng nào dưới đây ?
A.   ;1 . B. 1;2  . C.  3;4  . D.  2;3 .
Câu 37. (Sở Phú Thọ 2022) Cho hàm bậc bốn y  f  x  có đạo hàm liên tục trên  , hàm số y  f   x 
có đồ thị như hình vẽ.

Gọi S là tập các giá trị nguyên của tham số m để hàm số y  f  4  2 x  m  6  có đúng 3 điểm
cực tiểu. Tổng các phần tử của S bằng
A. 18 . B. 11 . C. 2 . D. 13 .
Câu 38. (Sở Phú Thọ 2022) Cho hàm số y  f  x  có đồ thị như hình vẽ


Số nghiệm thực phân biệt của phương trình 2 f x  1  2 x  1  3 là 
A. 12 . B. 5 . C. 8 . D. 4 .
x 1
Câu 39. (Sở Thái Nguyên 2022) Cho hàm số y  có đồ thị  C  . Gọi  là tiếp tuyến thay đổi của
2x 1
đồ thị  C  . Khoảng cách từ giao điểm hai đường tiệm cận của đồ thị  C  đến đường thẳng  đạt
giá trị lớn nhất bằng
2
A. . B. 1 . C. 2 . D. 5 .
2
ax  b
Câu 40. (Sở Thái Nguyên 2022) Cho hàm số y  f  x   có đồ thị hàm số f   x  như trong hình
cx  d
vẽ sau:

Trang 10 Fanpage Nguyễn Bảo Vương  https://www.facebook.com/tracnghiemtoanthpt489/


Điện thoại: 0946798489 TUYỂN CHỌN VẬN DỤNG – VẬN DỤNG CAO 2022

Biết rằng đồ thị hàm số f  x  đi qua điểm A  0; 2  . Giá trị f  3 bằng


A. 2 . B. 1 . C. 3 . D. 5 .
Câu 41. (Sở Thái Nguyên 2022) Cho hàm số y  f  x  liên tục trên  có đạo hàm
f   x    x  1  2 x 2  3 x  9  , x   . Hàm số g  x   f  x   x  3x  9 x  6 có bao nhiêu điểm
3 2

cực trị?
A. 2 . B. 1. C. 0 . D. 3 .
Câu 42. (Sở Thái Nguyên 2022) Gọi S là tập hợp tất cả các giá trị nguyên của tham số m sao cho
2 x 3  3 x 2  m  16, x   0;3 . Tổng tất cả các phần tử của S bằng
A.  65 . B.  74 . C. 42 . D. 87 .
Câu 43. (Sở Thái Nguyên 2022) Cho hàm số đa thức bậc bốn y  f  x  có bảng biến thiên như sau

2
 
Số điểm cực trị của hàm số g  x   x3  x  f  x  1  là
A. 11. B. 8. C. 13. D. 10.
1
Câu 44. (Sở Thái Nguyên 2022) Cho hàm số f ( x)  log 3 x  3x  3 . Tổng bình phương các giá trị của
x

 1 
tham số m để phương trình f  2
 
  f x  4 x  7  0 có đúng 3 nghiệm thực phân
 4 | x  m | 3 
biệt bằng
A. 14. B. 13. C. 10. D. 5.
Câu 45. (Sở Vĩnh Phúc 2022) Có bao nhiêu giá trị của tham số m để đồ thị hàm số
x 1
y 2 có đúng hai đường tiệm cận?
x   2m  1 x  m2  3
A. 1 . B. 2 . C. 3 . D. 0 .
Câu 46. (Sở Vĩnh Phúc 2022) Cho hàm số f  x  có đạo hàm f  x  liên tục trên  và đồ thị f   x  như

hình vẽ dưới đây.

Facebook Nguyễn Vương https://www.facebook.com/phong.baovuongTrang 11


Blog: Nguyễn Bảo Vương: https://www.nbv.edu.vn/

1
Bất phương trình f  x   x 3  x 2  3 x  m  0 nghiệm đúng với mọi x   0; 2  khi và chỉ khi
3
22 22
A. m  f  0  . B. m  f  2   . C. m  f  0  . D. m  f  2   .
3 3
2 x  m2
Câu 47. (Sở Vĩnh Phúc 2022) Cho hàm số f  x   , với m là tham số. Gọi m1 , m2  m1  m2  là
x 1
các giá trị của tham số m thỏa mãn 2 max f  x   min f  x   8 . Tổng 2m1  3m2 bằng
0;2 0;2
A. 1. B. 2 . C. 4 . D. 1 .
Câu 48. (Sở Vĩnh Phúc 2022) Một người thợ cần thiết kế một bể cá hình hộp chữ nhật bằng kính, có
chiều cao là 0,8m , thể tích 576dm3 . Biết rằng phần nắp phía trên của bể cá người thợ đó để trống
một ô có diện tích bằng 30% diện tích đáy bể. Biết rằng loại kính mà người thợ sử dụng làm mặt
bên và nắp bể có giá thành 1000000 đồng/m2 và kính để làm mặt đáy có giá thành 1200000
đồng/m2. Giả sử phần tiếp xúc giữa các mặt là không đáng kể. Số tiền mua kính ít nhất để hoàn
thành bể cá gần nhất với số tiền nào dưới đây?
A. 4,1 triệu đồng. B. 3, 2 triệu đồng. C. 2,8 triệu đồng. D. 3,8 triệu đồng.
Câu 49. (Sở Vĩnh Phúc 2022) Cho hàm số y  f  x  có đạo hàm f   x  liên tục trên  và có bảng xét
dấu như hình sau. Hàm số g  x   f  x 2  3x  1 đồng biến trên khoảng nào dưới đây

A.  0;1 . B.  4; 2  . C.  1; 0  . D.  2; 1 .


Câu 50. (Sở Vĩnh Phúc 2022) Cho hàm số y  f  x  liên tục trên  và có bảng xét dấu f   x  như sau

Số điểm cực trị của hàm số g  x   f  x 3  2 x  4  là

A. 2 . B. 3 . C. 5 . D. 10 .

Trang 12 Fanpage Nguyễn Bảo Vương  https://www.facebook.com/tracnghiemtoanthpt489/


Điện thoại: 0946798489 TUYỂN CHỌN VẬN DỤNG – VẬN DỤNG CAO 2022
Câu 51. (THPT Bùi Thị Xuân – Huế - 2022) Cho hàm số y  f ( x) là hàm đa thức bậc bốn. Đồ thị hàm
số f ( x  2) được cho trong hình vẽ bên

Hàm số g ( x)  4 f  x 2   x 6  5 x 4  4 x 2  1 đồng biến trên khoảng nào dưới đây?


A. ( 4; 3) .
B. (2;  ) .
C. ( 2; 2) .
D. (2; 1) .
Câu 52. (THPT Bùi Thị Xuân – Huế - 2022) Cho hàm số y  f ( x) là hàm số bậc ba và có đồ thị
y  f (2  x ) như hình vẽ.

 
Hỏi phương trình f x 2  2 x  1 có tất cả bao nhiêu nghiệm?
A. 8.
B. 7.
C. 9.
D. 6.
Câu 53. (THPT Bùi Thị Xuân – Huế - 2022) Cho hàm số y  f ( x) có đạo hàm
 
f ( x)  ( x  1) 2 x 2  2 x với x   . Có bao nhiêu giá trị nguyên dương của tham số m để hàm
số f  x 2  8 x  m  có 5 điểm cực trị?
A. 15.
B. 16.
C. 17.
D. 18.

Facebook Nguyễn Vương https://www.facebook.com/phong.baovuongTrang 13


Blog: Nguyễn Bảo Vương: https://www.nbv.edu.vn/
Câu 54. (Chuyên Hạ Long 2022) Cho các số thực x, y thoả mãn
max{5;9 x  7 y  20}  x 2  y 2  2 x  8
 .Gọi M , m lần lượt là giá trị lớn nhất và giá trị nhỏ nhất
y 1
của biểu thức P  x  2 y . Tính M  m
A. 1  3 5 .
B. 2 2 .
C. 1  2 2 .
D. 2  3 5 .
mx 2  (m  2) x  5
Câu 55. (Chuyên Hạ Long 2022) Cho hàm số y  . Gọi S là tập hợp các giá trị của
x2  1
m sao cho đồ thị hàm số đã cho có đúng hai điểm cực trị và đường thẳng nối hai điểm cực trị của
25
đồ thị hàm số cắt hai trục tọa độ tạo thành một tam giác có diện tích bằng . Tính tổng các phần
4
tử của S
A. 0.
B. 1
C. 4
D. 2 .
Câu 56. (Chuyên Hoàng Văn Thụ - Hòa Bình – 2022) Cho hàm số y  f ( x ) liên tục trên  và thỏa
mãn f ( 4)  4 . Đồ thị hàm số y  f ( x) như hình vẽ bên dưới. Để giá trị lớn nhất của hàm số

x2
h( x )  f ( x )  x  3m trên đoạn [4;3] không vượt quá 2022 thì tập giác trị của m là
2
A. ( ; 2022] .
B. (674;  ) .
C. ( ; 674] .
D. (2022;  ) .
Câu 57. (Chuyên Lam Sơn 2022) Cho hàm số f ( x)  x 4  2 x3  (m  1) x 2  2 x  m  2022 , với m là
tham số. Có bao nhiêu giá trị nguyên của m thuộc đoạn [ 2021; 2022] để hàm số
y | f ( x  2021)  2022 | có số điểm cực trị nhiều nhất?
A. 2021.
B. 2022.
C. 4040.
D. 2023.
Câu 58. (Chuyên Lam Sơn 2022) Cho hàm số bậc bốn y  f ( x) có đồ thị như hình vẽ dưới đây.

Trang 14 Fanpage Nguyễn Bảo Vương  https://www.facebook.com/tracnghiemtoanthpt489/


Điện thoại: 0946798489 TUYỂN CHỌN VẬN DỤNG – VẬN DỤNG CAO 2022

Có bao nhiêu giá trị nguyên của tham số m  [2021; 2021] để phương trình
2 2
f 2
( x)  x   m 2
 2m  14  f 2

( x)  x 2  4( m  1) 2  36  0 có đúng 6 nghiệm phân biệt.
A. 2022.
B. 4043.
C. 4042.
D. 2021.
Câu 59. (Chuyên Lam Sơn 2022) Cho hàm số bậc ba y  f ( x) có đồ thị như hình vẽ

Tìm số điểm cực trị của hàm số y  f 2 ( g ( x)) với g ( x)  x 2  4 x  2 4 x  x 2


A. 17.
B. 21.
C. 23.
D. 19.
Câu 60. (Chuyên Lam Sơn 2022) Cho hàm đa thức bậc bốn y  f ( x) . Biết đồ thị của hàm số
y  f (3  2 x ) được cho như hình vẽ

Hàm số y  f ( x ) nghịch biến trên khoảng


A. (; 1) .
B. ( 1;1) .
C. (1;5) .
D. (5;  ) .
Câu 61. (Chuyên Lam Sơn 2022) Cho hàm số y  f ( x) liên tục trên  và có bảng biến thiên như hình
vẽ

Facebook Nguyễn Vương https://www.facebook.com/phong.baovuongTrang 15


Blog: Nguyễn Bảo Vương: https://www.nbv.edu.vn/

Đặt g ( x ) | m  f (2022  x ) | . Có bao nhiêu giá trị nguyên của tham số m để hàm số y  g ( x ) có
đúng 5 điểm cực trị?
A. 6.
B. 8.
C. 9,
D. 7.
Câu 62. (Chuyên Lương Văn Tụy – Ninh Bình 2022) Cho hàm số y  f ( x) . Hàm số f ( x ) có bảng
biến thiên như hình vẽ sau Giá trị lớn nhất của hàm số g ( x)  f (2 x)  sin 2 x trền đoạn [ 1;1] là

A. f (1) .
B. f (0) .
C. f (2) .
D. f ( 1) .
Câu 63. (Chuyên Lương Văn Tụy – Ninh Bình – 2022) Cho hàm số y  f ( x ) liên tục trên R và có đồ
thị có 3 điểm cực trị như hình dưới đây. Số điểm cực trị của hàm số g ( x)  f  x 3  3 x  2  là

A. 5.
B. 9.
C. 11.
D. 7.
Câu 64. (Chuyên Nguyễn Trãi – Hải Dương-2022) Cho hàm số y  f ( x ) như hình vẽ. Biết rằng
f (3)  2 f (5)  4 . Hỏi có tất cả bao nhiêu giá trị nguyên của tham số m để phương trình

Trang 16 Fanpage Nguyễn Bảo Vương  https://www.facebook.com/tracnghiemtoanthpt489/


Điện thoại: 0946798489 TUYỂN CHỌN VẬN DỤNG – VẬN DỤNG CAO 2022

1 
f  f ( x)  m   2 x  2m có đúng 3 nghiệm thực phân biệt.
2 
A. 8
B. 6.
C. 3.
D. 7.
Câu 65. (Chuyên Nguyễn Trãi – Hải Dương – 2022) Cho hàm số y  f ( x) có đạo hàm
 
f ( x)  ( x  3) x 2  2 x   . Tìm tất cả các giá trị thực không âm của tham số m để hàm số
  11 
g ( x)  f (| sin x  3 cos x |  m) có nhiều điểm cực trị nhất trên  ; .
 2 12 
 2 
A. m   .
 2 ,  
 
 2 
B. m   .
 2 ,1
 
C. m  ( 2  1, 2)
 2 
D. m  
 2 , 2  .
 
Câu 66. (THPT Đô Lương – Nghệ An – 2022) Hàm số f ( x)  10 x  x và
 
g ( x)  x 3  mx 2  m 2  1 x  2 . Gọi M là giá trị lớn nhất của hàm số y  g ( x  f ( x )) trên đoạn
[0;1] . Khi M đạt giá trị nhỏ nhất thì giá trị của m bằng?
21
A. .
2
B. 6.
C. 21.
D. 5.
Câu 67. (THPT Lương Thế Vinh – Hà Nội – 2022) Cho hàm số y  f ( x) liên tục trên  . Đổ thị hàm
số f ( 3 x ) được cho trong hình bên:

Facebook Nguyễn Vương https://www.facebook.com/phong.baovuongTrang 17


Blog: Nguyễn Bảo Vương: https://www.nbv.edu.vn/

1
Hàm số g ( x)  f ( x)  x 4  x có tối đa bao nhiêu điểm cực đại?
8
A. 2.
B. 4.
C. 5.
D. 3.
Câu 68. (THPT Lương Thế Vinh – Hà Nội – 2022) Cho hàm số f ( x ) có đổ thị như hình vẽ:

 
Gọi S là tập hợp tất cả các giá trị nguyên của tham số m để phương trình f 3  4  x 2  m có

hai nghiệm phân biệt thuộc đoạn [ 3; 3] . Số phần tử của S là


A. 1.
B. 4.
C. 5.
D. 3.
Câu 69. (THPT Lương Thế Vinh – Hà Nội – 2022) Cho hàm số f ( x ) có đạo hàm trên  và f ( x ) có
bảng biến thiên như hình vẽ. Đồ thị y  f ( x) cắt trục hoành tại hai điếm phân biệt có hoành độ
lần lượt là 3; 2 . Có bao nhiêu giá trị nguyên của tham số m thuộc [ 10;10] để hàm số
 
y  f x 2  2 x  m đồng biến trên ( 1;1) .

A. 12.
B. 14.
C. 11.
D. 13.

Trang 18 Fanpage Nguyễn Bảo Vương  https://www.facebook.com/tracnghiemtoanthpt489/


Điện thoại: 0946798489 TUYỂN CHỌN VẬN DỤNG – VẬN DỤNG CAO 2022
Câu 70. (THPT Kim Liên - Hà Nội - 2022) Cho hàm số f  x  có đạo hàm trên  là
f   x    x  3 x  4  . Tính tổng các giá trị nguyên của tham số m   10;5 để hàm số

y  f x 2  3x  m  có nhiều điểm cực trị nhất?
A. 54 . B. 9 . C.  52 . D.  54 .
Câu 71. (THPT Kim Liên - Hà Nội - 2022) Cho hàm số y  f  x  , đồ thị của hàm số y  f   x  là
đường cong trong hình bên. Giá trị nhỏ nhất của hàm số g  x   f  3 x   3 x 2  4 x  1 trên đoạn
 2 2 
 3 ; 3  bằng

1
A. f  0   1 . B. f  6  . C. f  2   . D. f  3   8 .
3
Câu 72. (THPT Kinh Môn - Hải Dương - 2022) Cho hàm số y  f  x  có đạo hàm trên  và đồ thị
hàm số y  f   x  cắt trục hoành tại các điểm có hoành độ 3;  2; a ; b ;3; c ;5 với
4 4
  a  1;1  b  ; 4  c  5 có dạng như hình vẽ bên dưới. Có bao nhiêu giá trị nguyên của
3 3
m để hàm số y  f  2 x  m  2022  có 5 điểm cực trị?

A. 4 . B. 2 . C. 3 . D. Vô số.
Câu 73. (THPT Kinh Môn - Hải Dương - 2022) Tìm m để đường thẳng y  2 x  m cắt đồ thị hàm số
x3
y tại hai điểm A, B sao cho độ dài AB là nhỏ nhất.
x 1
A. 2 . B. 1 . C. 1 . D. 3 .
Câu 74. (THPT Lương Tài 2 - Bắc Ninh - 2022) Cho hàm số y  f  x   2 x  bx 2  cx  d thỏa mãn
3

4b  2c  d  16  0 và 9b  3c  d  54 . Hàm số y  f  x  có tất cả bao nhiêu điểm cực trị?


A. 2 .B 3 . C. 5 . D. 4 .
Facebook Nguyễn Vương https://www.facebook.com/phong.baovuongTrang 19
Blog: Nguyễn Bảo Vương: https://www.nbv.edu.vn/
Câu 75. (THPT Lương Tài 2 - Bắc Ninh - 2022) Cho hàm số y  f  x  là hàm số bậc ba có đồ thị như
hình vẽ. Tìm tất cả các ía trị của tham số m sao cho phương trình f  sin x   f  m  1 có nghiệm.

A.  1  m  3 . B. 2  m  0 . C. 3  m  1 . D. 2  m  2 .
Câu 76. (THPT Võ Nguyên Giáp - Quảng Bình - 2022) Cho hàm số bậc ba y  f  x  có đồ thị như
hình vẽ sau:

Có bao nhiêu giá trị nguyên của tham số m   0; 20 để hàm số g ( x)  f 2  x   2 f  x   m có 9


điểm cực trị?

A. 8 . B. 9 . C. 10 . D. 11 .
Câu 77. (THPT Võ Nguyên Giáp - Quảng Bình - 2022) Cho hàm số y  f  x  liên tục trên  và
f   x    x3  6 x 2  32 . Khi đó hàm số g  x   f  x 2  3 x  nghịch biến trên khoảng
A.  ;   . B. 1;   . C.  2;   . D.  ;1 .
Câu 78. (THPT Yên Lạc - Vĩnh Phúc - 2022) Cho hàm số f  x  , đồ thị của hàm số y  f   x  là
đường cong trong hình bên.

Trang 20 Fanpage Nguyễn Bảo Vương  https://www.facebook.com/tracnghiemtoanthpt489/


Điện thoại: 0946798489 TUYỂN CHỌN VẬN DỤNG – VẬN DỤNG CAO 2022
8x3
Tìm tất cả các giá trị thực của tham số m để bất phương trình f  2 x    4 x  m  0 đúng với
3
 1 1
mọi x    ; 
 2 2
5
A. m  f 1  . B. m  f  0  . C. m  f  0  . D. m  f  3 .
3
x2  1 y  1
Câu 79. (THPT Yên Lạc - Vĩnh Phúc - 2022) Cho x, y là các số thực dương và thỏa mãn  .
y x
y4
Giá trị nhỏ nhất m của biểu thức P  là
x
A. m  3 B. m  2 2 C. m  4 D. m  8
Câu 80. (THPT Yên Lạc - Vĩnh Phúc - 2022) Cho hàm số y  f  x  là hàm đa thức bậc 3 có đồ thị như
hình vẽ. Gọi S là tập hợp tất cả các giá trị nguyên của tham số m   100;100 để hàm số
h  x   f 2  x   4 f  x   3m có đúng 3 điểm cực trị. Tổng tất cả các phần tử của S bằng

A. 5047 . B. 5049 . C. 5043 . D. 5050 .


Câu 81. (THPT Yên Phong 1 - Bắc Ninh - 2022) Cho hàm số f  x    x   4  m 2  x  2020 và
4

g  x    x 3  5 x 2  2020 x  2021 . Có bao nhiêu giá trị nguyên dương của tham số m để
h  x   g  f  x   đồng biến trên  2;   ?

A. 7 . B. 6 . C. 12 . D. 13 .
Câu 82. (THPT Yên Phong 1 - Bắc Ninh - 2022) Cho hàm số y  f  x  liên tục trên  đồng thời thỏa
điều kiện f  0   0 và  f  x   6 x  f  x   9 x 4  3 x 2  4, x   . Tìm giá trị lớn nhất của hàm số
y  f  2 x 2  3 x  1 trên đoạn  0;1 .
5 17 155 167
A. . B. . C. . D. .
2 7 64 69

Theo dõi Fanpage: Nguyễn Bảo Vương  https://www.facebook.com/tracnghiemtoanthpt489/

Hoặc Facebook: Nguyễn Vương  https://www.facebook.com/phong.baovuong

Tham gia ngay: Nhóm Nguyễn Bào Vương (TÀI LIỆU TOÁN)  https://www.facebook.com/groups/703546230477890/

Ấn sub kênh Youtube: Nguyễn Vương


 https://www.youtube.com/channel/UCQ4u2J5gIEI1iRUbT3nwJfA?view_as=subscriber

Tải nhiều tài liệu hơn tại: https://www.nbv.edu.vn/

Facebook Nguyễn Vương https://www.facebook.com/phong.baovuongTrang 21


TUYỂN CHỌN VẬN DỤNG – VẬN DỤNG CAO 2022 Điện thoại: 0946798489

VẤN ĐỀ 2. LOGARIT
• |FanPage: Nguyễn Bảo Vương
• TUYỂN CHỌN CÂU HỎI VD-VDC TỪ CÁC ĐỀ THI THỬ CÁC TRƯỜNG, CÁC SỞ NĂM 2022
Câu 1. (Chuyên Vinh– 2022) Có bao nhiêu số nguyên a sao cho ứng với mỗi a , tồn tại số thực
b  a thỏa mãn 4 a  2b  b và đoạn [ a; b ] chứa không quá 5 số nguyên ?
A. 5.
B. 10.
C. 6.
D. 11.
Câu 2. (Chuyên Vinh – 2022) Có bao nhiêu giá trị nguyên lớn hơn 2 của y sao cho với mỗi y tồn tại
đúng 3 sô nguyên dương x thỏa mãn 3x  y  2 log 2  3x  2  ? .
A. 16.
B. 51.
C. 68.
D. 66.
Câu 3. (Chuyên Vinh – 2022) Số nghiệm nguyên của bất phương trình
 
2log 2 ( x  2)  log 2 2 x 2  1  ( x  1)( x  5) là
A. 5.
B. 6.
C. 7.
D. 4.
Câu 4. (Chuyên Vinh 2022) Gọi m là giá trị nhỏ nhất của hàm số f ( x)  4 x  ( a  2)2 x  2 trên đoạn
[ 1;1] . Tất cả giá trị của a để m  1 là
A. a  1 .
1
B.   a  0 .
2
1
C. a   .
2
D. a  0 .
Câu 5. (Cụm Trường Nghệ An - 2022) Có bao nhiêu cặp số nguyên  x; y  thỏa mãn đẳng thức sau:
2

log 2022  x 4  2 x 2  2023


y  2022 

   2 y  2021 .
A. 3. B. 1. C. 0. D. 2.
Câu 6. (Đại học Hồng Đức – 2022) Cho x là số nguyên dương và y là số thực. Có tất cả bao nhiêu cặp
số ( x; y ) thỏa mãn ln(1  x  2 y )  2 y  3 x  10 ?
A. 10.
B. Vô số.
C. 11.
D. 9.
Câu 7. (Đại học Hồng Đức – 2022) Tổng S của tất cả các nghiệm thuộc khoảng (0; 4 ) của phương
2 2
trình 2022sin x  2022cos x  2 ln(cot x ) là
A. S  18 .
B. S  8 .
C. S  7 .
D. S  16 .

Facebook Nguyễn Vương https://www.facebook.com/phong.baovuong Trang 1


Blog: Nguyễn Bảo Vương: https://www.nbv.edu.vn/
Câu 8. (THPT Hồ Nghinh – Quảng Nam – 2022) Cho các số thực dương x, y thỏa mãn
log x 2  2 y  2 y 2 (9 x  10 y  20)  1 . Gọi M , m lần lượt là giá trị lớn nhất và giá trị nhỏ nhất của
y
S . Tính M  m .
x
5
A. M  m  .
3
B. M  m  5  2
C. M  m  2 7 .
7
D. M  m  .
2
Câu 9. (THPT Hương Sơn - Hà Tĩnh - 2022) Trong mặt phẳng với hệ tọa độ Oxy , điểm M  x ; y  biểu
diễn nghiệm của phương trình log3  9 x  18  x  y  3y . Có bao nhiêu điểm M có tọa độ nguyên
thuộc hình tròn tâm O bán kính R  7 ?
A. 7 . B. 2 . C. 3 . D. 49 .
Câu 10. (THPT Hương Sơn - Hà Tĩnh - 2022) Với giá trị nào của m thì phương trình
1
x
 4m.3x  m  2  0 có hai nghiệm phân biệt x1 ; x2 thỏa mãn x1  x2  1 ?
9 2

3 3
A. m  . B. m   . C. m  7 . D. m  1 .
4 4
Câu 11. (THPT Lê Thánh Tông - HCM-2022) Có tất cả bao nhiêu số b nguyên dương sao cho tồn tại
2 2
đúng hai số thực a thỏa mãn đẳng thức b.2 a  6 a 1  b 2 .2 2 a 12 a 1  3  7 log 2  a 2  6 a  log 2 b  ?
A. 1024 . B. 1023 . C. 2047 . D. 2048 .
Câu 12. (THPT Lê Thánh Tông - HCM-2022) Số nghiệm nguyên của bất phương trình
 3x  36 x  246  5  ln  x  3  0 là
A. 144. B. 145. C. 146. D. 147.
1
Câu 13. (Liên trường Hà Tĩnh – 2022) Cho các số thực a, b thỏa mãn a  , b  1 . Khi biểu thức
2
P  log 2 a b  log b  a  4a  16  đạt giá trị nhỏ nhất thì tổng a  b bằng
4 2

A. 4.
B. 18.
C. 14.
D. 20.
Câu 14. (Liên trường Hà Tĩnh – 2022) Có bao nhiêu giá trị nguyên của m để phương trình
 log 2 ( x  1)  x  2  4 x  2 x3  m  1  0 có ba nghiệm phân biệt
A. 2.
B. 3.
C. 4.
D. 5.
Câu 15. (Liên trường Hà Tĩnh 2022) Tính tổng tất cả các giá trị nguyên dương của m để bất phương
trình 2 x  3  2m  x  2m 3  1 có nhiều nhất 20 nghiệm nguyên
A. 153.
B. 171.
C. 190.
D. 210.
Câu 16. (THPT Nguyễn Tất Thành-Đh-SP-HN-2022) Có bao nhiêu giá trị nguyên của tham số
m   10;10 để bất phương trình

log 22 x  ( m  1) log 2 x  2 m  3  0

Trang 2 Fanpage Nguyễn Bảo Vương  https://www.facebook.com/tracnghiemtoanthpt489/


Điện thoại: 0946798489 TUYỂN CHỌN VẬN DỤNG – VẬN DỤNG CAO 2022
nghiệm đúng với mọi x  1;32
A. 11. B. 12. C. 13. D. 14.
Câu 17. (THPT Nguyễn Tất Thành-Đh-SP-HN-2022) Cho x, y, z  0;2 và thỏa mãn x  2 y  z  6 .
2 2
Tìm giá trị lớn nhất của biểu thức P  32 x x  52 y y  3z  2 x 2  4 y 2
A. m ax P  25 . B. m ax P  27 . C. m ax P  26 . D. m ax P  30 .
Câu 18. (THPT Nho Quan A – Ninh Bình – 2022) Có bao nhiêu số nguyên x sao cho úng với mỗi x có
không quá 255 số nguyên y thỏa mãn log 5  x 2  y   log 2 ( x  y ) ?
A. 1250.
B. 1249.
C. 625.
D. 624.
Câu 19. (THPT Phù Cừ - Hưng Yên - 2022) Tổng các nghiệm nguyên của bất phương trình
 
log 2 x3  log 22 (2 x)  13
 0 là
1  8  ( 2) x  2
A. 16. B. 8. C. 36. D. 136.
2 2
2 x  y 1
Câu 20. (THPT Phù Cừ - Hưng Yên - 2022) Cho các số thực x, y thỏa mãn 2  4 x 1 và
x  y2  2x  2
2 x  y  0 . Giả trị lớn nhất và giá trị nhỏ nhất của biểu thức P  3 x  2 y  1 lần lượt là M và m .
Tính M  m .
A. 6. B. 10. C. 12. D. 8.
1
 x y z

Câu 21. (Sở Hà Tĩnh 2022) Xét các số thục̣ dương x, y, z thoả män ( y  z )  3  81   xy  xz  4 . Giá
 
 
trị nhỏ nhất của biểu thức P  log 2 x  log 2  2 y  z  bằng
2 2

A. 2  log 2 3 .
B. 5  log 2 3 .
C. log 2 11.
D. 4  log 2 3 .
Câu 22. (Sở Thanh Hóa 2022) Cho hàm só f ( x)  2 x  2 x  2022 x 3 . Biết rằng tồn tại só thực m sao cho
bá́ t phương trình f  4 x  mx  37 m   f  ( x  m  37)  2 x   0 nghiệm đúng với mọi x   . Khi đó
m thuộc khoảng nào dưới đây?
A. (30;50) .
B. (10;30) .
C. (50; 70) .
D. (10;10) .
Câu 23. (Sở Thanh Hóa 2022) Gọi S là tập tất cả các số nguyên y sao cho với mỗi y  S có đúng 10 số
nguyên x thoả mãn 2 y  x  log 3  x  y 2  . Tổng các phần tử của S bẳng
A. 7.
B. 4 .
C. 1.
D. 1 .
Câu 24. (Sở Bắc Giang 2022) Có bao nhiêu số nguyên dương x sao cho ứng với mỗi x có đúng 9 số
nguyên y thỏa mãn  2 y 1  x 2  3 y  x   0 ?
A. 64.
B. 67.
C. 128.
Facebook Nguyễn Vương https://www.facebook.com/phong.baovuongTrang 3
Blog: Nguyễn Bảo Vương: https://www.nbv.edu.vn/
D. 53.
Câu 25. (Sở Bắc Giang 2022) Có bao nhiêu số nguyên x sao cho tồn tại số thực y thỏa mãn

2 log 3 ( x  y  1)  log 2 x 2  2 x  2 y 2  1 ? 
A. 4.
B. 2.
C. 3.
D. 1.
Câu 26. (Sở Hà Tĩnh 2022) Có bao nhiêu giá trị nguyên của tham số m  [ 10;10] để phương trình
m 2 m 2 m
23  7 x 2 x
 73  2 x 2 x
 143  7 x 2  14 x  2  7  3m 
có bốn nghiệm phân biệt trong đó có đúng hai nghiệm lớn hơn 1 ?
A. 10.
B. 9.
C. 11.
D. 8.
Câu 27. (Sở Ninh Bình 2022) Cho các số thực a , b thỏa mãn 1  a  b  4 . Tìm giá trị nhỏ nhất của biểu
16
thức P  3log a  b 2  16b  16    log 3b a .
27 a
A. 8.
B. 18.
C. 9.
D. 17.
Câu 28. (Sở Bạc Liêu 2022) Cho phương trình 2log 32 x  log 3 x  1   5x  m  0 ( m là tham số thực). Có
tất cả bao nhiêu giá trị nguyên dương của m để phương trình đã cho có đúng 2 nghiệm phân
biệt?
A. 125 . B. 123 . C. 122 . D. 124 .
Câu 29. (Sở Hà Tĩnh 2022) Có bao nhiêu cặp số  x; y  (trong đó x, y nguyên dương thuộc đoạn
 0;2022 thỏa mãn điều kiện 2 x  log 2  y 2  615  y 2  x  615
A. 1 . B. 3 . C. 4 . D. 2 .
Câu 30. (Sở Lạng Sơn 2022) Với a là tham số thực để bất phương trình 2 x  3x  ax  2 có tập nghiệm là
R , khi đó
A. a  1;3 . B. a   0;1 . C. a   ; 0  . D. a   3;   .
Câu 31. (Sở Lạng Sơn 2022) Biết đồ thị hàm số y  f  x  đối xứng với đồ thị hàm số
 1 
y  a x  a  0, a  1 qua điểm I 1;1 . Giá trị của biểu thức f  2  log a  bằng
 2022 
A. 2022 . B. 2021. C. 2022. D. 2020 .
Câu 32. (Sở Phú Thọ 2022) Xét các số thực dương x , y thỏa mãn
2 2 1 2
2  x 2  y 2  4   log 2022      xy  4  . Khi biểu thức P  x  4 y đạt giá trị nhỏ nhất, giá trị
 x y  2
y
của bằng
x
1 1
A. 4 . B. 2 . C. . D. .
2 4
Câu 33. (Sở Phú Thọ 2022) Có bao nhiêu số nguyên x sao cho ứng với mỗi x có không quá 20 số
2
nguyên y thỏa mãn 4 x 5 y 16  2 x  y  512 và x  y  0 ?
A. 19 . B. 20 . C. 21 . D. 18 .
Câu 34. (Sở Thái Nguyên 2022) Cho bất phương trình
     
log 2 x  x 2  1 .log 2022 x  x 2  1  log m x  x 2  1 . Có bao nhiêu giá trị nguyên thuộc

Trang 4 Fanpage Nguyễn Bảo Vương  https://www.facebook.com/tracnghiemtoanthpt489/


Điện thoại: 0946798489 TUYỂN CHỌN VẬN DỤNG – VẬN DỤNG CAO 2022
khoảng 1; 2022  của tham số m sao cho bất phương trình đã cho nghiệm đúng với mọi x thuộc
5;   ?
A. 2022. B. 2021 . C. 2012 . D. 2010 .
Câu 35. (Sở Thái Nguyên 2022) Cho x, y  0; x  3 y  0 thỏa mãn
 x2  y2  x2  y2
2022  log 2  1  x  3 y  . Tổng của giá trị lớn nhất và giá trị nhỏ nhất của
 x  3y  4
 
biểu thức P  x 2  y 2  14 x  2 y  2022 bằng
A. 4124 B. 4042 . C. 4044 D. 4122

Câu 36. (Sở Vĩnh Phúc 2022) Có bao nhiêu cặp số nguyên dương  x, y  thoả mãn
2
34 x 1
log  4 x 2  4 x  2   3 y  2 x  4 log  2 x  y  1 đồng thời x, y  2021
A. 15 . B. 28 . C. 22 . D. 35 .

Câu 37. (Sở Vĩnh Phúc 2022) Xét các số thực x, y thỏa mãn x 2  y 2  1 và log x 2  y 2  2 x  4 y   1 . Giá trị
lớn nhất của biểu thức P  3 x  y bằng
A. 5  2 10 . B. 5  4 5 . C. 5  5 2 . D. 10  2 5 .
 1 
Câu 38. (Sở Vĩnh Phúc 2022) Cho hàm số f  x   ln  1  2  . Biết rằng
 x 
a
f   2   f   3   ...  f   2019   f   2020   với a , b là các số nguyên dương nguyên tố cùng
b
nhau. Giá trị của 2a  b bằng
A. 2 . B. 4 . C. 2 . D. 4 .

Câu 39. (Sở Vĩnh Phúc 2022) Cho hàm số bậc ba y  f  x  có đồ thị như hình vẽ

Số nghiệm của phương trình log32  f  x   1  log 2 2  f  x   1  2 log 1 f  x   1  6  0 là


2
A. 7 . B. 5 . C. 6 . D. 8 .
Câu 40. (THPT Bùi Thị Xuân – Huế - 2022) Tất cả các giá trị thực của m để bất phương trình
x x  x  12  m log 5 4 x 3 có nghiệm:
A. m  2 3 .
B. m  12 log 3 5 .
C. m  2 3 .
D. 2  m  12 log 3 5 .

Facebook Nguyễn Vương https://www.facebook.com/phong.baovuongTrang 5


Blog: Nguyễn Bảo Vương: https://www.nbv.edu.vn/
Câu 41. (Chuyên Hạ Long 2022) Cho 0  m  1 . Gọi (a; b) là tập hợp các giá trị của m để bất phương
trình log m 1  8m  x   2(1  x) có hữu hạn nghiệm nguyên. Tính b  a
A. 1.
B. 3 2  1 .
C. 2 2  1 .
D. 4 2  1 .
Câu 42. (Chuyên Hoàng Văn Thụ - Hòa Bình – 2022) Có tất cả bao nhiêu giá trị nguyên của y sao cho
tương ứng với mỗi giá trị y luôn tồn tại không quá 15 số nguyên x thỏa mãn điều kiện
log 2021  x  y 2   log 2022  y 2  y  16   log 2 ( x  y ) ?
A. 2021.
B. 4042.
C. 2020.
D. 4041.
Câu 43. (Chuyên Lam Sơn 2022) Có bao nhiêu số nguyên dương m để phương trình
m  e x  1  ln(mx  1)  2e x  e 2 x  1 có 2 nghiệm phân biệt không lớn hơn 5.
A. 26.
B. 27.
C. 29.
D. 28.
Câu 44. (Chuyên Lam Sơn 2022) Cho a , b là các số thực thay đổi thỏa mãn log a 2 b 2  20 (6a  8b  4)  1
c
và c, d là các số thực dương thay đổi thỏa mãn c 2  c  log 2
d
 
 7  2 2d 2  d  3 . Giá trị

nhỏ nhất của biểu thức (a  c  1) 2  (b  d ) 2 là


A. 4 2  1 .
B. 29  1 .
12 5  5
C. .
5
8 5 5
D. .
5
Câu 45. (Chuyên Lương Văn Tụy-Ninh Bình 2022) Cho phương trình 2log 32 x  log3 x  1  5x  m  0
( m là tham số thực). Có tất cả bao nhiêu giá trị nguyên dương của m để phương trình đã cho có
đúng hai nghiệm phân biệt?
A. 125.
B. 123.
C. 122.
D. 124.
1
Câu 46. (Chuyên Lương Văn Tụy-Ninh Bình 2022) Cho các số thực a , b thỏa mãn  b  a  1 . Tìm
3
giá trị nhỏ nhất của biểu thức
4(3b  1)
P  log a  8log 2b a
9 a
A. 7.
B. 8.
C. 6.
D. 9.

Trang 6 Fanpage Nguyễn Bảo Vương  https://www.facebook.com/tracnghiemtoanthpt489/


Điện thoại: 0946798489 TUYỂN CHỌN VẬN DỤNG – VẬN DỤNG CAO 2022
Câu 47. (Chuyên Nguyễn Trãi – Hải Dương – 2022) Gọi S là tập các giá trị của tham số m để bất
phương trình log 0.3  x 2  2( m  3) x  4   log 0.3  3 x 2  2 x  m  thỏa mãn với mọi x thuộc  . Tập
S bằng
A. S  [5; 6) .
B. S  [4; 6] .
C. S  [4;5) .
D. S  [1;5) .
Câu 48. (Chuyên Nguyễn Trãi – Hải Dương – 2022) Cho các số thực a , b, c, d thỏa mãn điều kiện:
 2 2

log 2 a  b  5  1  log 2 (2  2a  b)
 4c 5d 10 c  d  2 Tìm giá trị nhỏ nhất của biểu thức
e e  12  3c  4d
P  ( a  c) 2  (b  d ) 2
2 5
A.
5
B. 2.
C. 2 5  2 .
12
D. .
5
Câu 49. (THPT Lương Thế Vinh – Hà Nội -2022) Gọi S là tập các số nguyên m  [ 2022; 2022] để
phương trình log 22 x  log 2
x  m  m  log 2 x có đúng ba nghiệm phân biệt. Số phần tử của S

A. 1.
B. 2.
C. 2021.
D. 2022.
Câu 50. (THPT Lương Thế Vinh – Hà Nội – 2022) Có bao nhiêu cặp số nguyên (a; b) , trong đó
2b a
 2a   a  2b 
a , b  [1; 2022] thỏa mãn  b    b 1  ?
a2   2 
A. 5.
B. 9.
C. 10.
D. 11.
Câu 51. (THPT Lương Thế Vinh – Hà Nội – 2022) Có bao nhiêu giá trị nguyên của m để bất phương
trình log 22 x  (2m  5) log 2 x  m 2  5m  4  0 có it nhất một nghiệm nguyên và không quá 1791
nghiệm nguyên?
A. 10.
B. 3.
C. 9.
D. 11.
Câu 52. (THPT Trần Phú – Hà Tĩnh – 2022) Có bao nhiêu số tự nhiên x sao cho mỗi giá trị của x tồn

tại số thực y thoả mãn log 3 ( x  y )  log 6 x 2  2 y 2 ?
A. 1.
B. 3.
C. 2.
D. 6.

Facebook Nguyễn Vương https://www.facebook.com/phong.baovuongTrang 7


Blog: Nguyễn Bảo Vương: https://www.nbv.edu.vn/
13 2
Câu 53. (THPT Trần Phú – Hà Tĩnh – 2022) Cho hàm số f ( x )   x 3  x  12 x  e x  2022 . Bất
2
phương trình ẩn m sau đây f log 0,5  log 2 (2m  1)   2021  f [ f (0)] có bao nhiêu nghiệm
nguyên?
A. 14.
B. 10.
C. 11.
D. 7.
Câu 54. (THPT Kim Liên - Hà Nội - 2022) Gọi S là tập nghiệm của phương trình
2x
2 x
 3x  8x  3
 3  m  0 (với m là tham số thực). Có tất cả bao nhiêu giá trị nguyên của
m   2021; 2021 để tập hợp S có hai phần tử ?
A. 2095 . B. 2092 . C. 2093 . D. 2094 .
Câu 55. (THPT Kim Liên - Hà Nội - 2022) Cho bất phương trình
log 5  x 2  4 x  4  m   1  log 5  x 2  2 x  3 với m là tham số. Có bao nhiêu giá trị nguyên của
tham số m để bất phương trình nghiệm đúng với mọi x  1; 3  ?
A. 30 . B. 28 . C. 29 . D. Vô số.
Câu 56. (THPT Kinh Môn - Hải Dương - 2022) Có bao nhiêu giá trị nguyên của tham số
m   2022; 2022 để bất phương trình  3m  112 x   2  m  6 x  3x  0 có nghiệm đúng với
x  0 ?
A. 2021 . B. 4044 . C. 2022 . D. 2020 .
Câu 57. (THPT Kinh Môn - Hải Dương - 2022) Cho hai số thực dương x, y thỏa mãn phương trình
x  y 1
3  ln  9 xy  3 x  3 y. Tìm giá trị nhỏ nhất của biểu thức P  xy
3 xy
1 1
A. 1. B. . C. . D. 9 .
9 3
Câu 58. (THPT Lương Tài 2 - Bắc Ninh - 2022) Có tất cả bao nhiêu số nguyên dương y sao cho tồn tại
số thực x  1; 8 thỏa mãn:

 x  1  2e x  y 2   y  e x  x 2  ?
A. 11 . B. 14 . C. 12 . D. 13 .
Câu 59. (THPT Lương Tài 2 - Bắc Ninh - 2022) Tập nghiệm của bất phương trình
 
4x  65.2 x  64  2  log3 ( x  3)  0 có tất cả bao nhiêu số nguyên?
A. 2. B. 3. C. 4. D. Vô số.
Câu 60. (THPT Võ Nguyên Giáp - Quảng Bình - 2022) Cho ba số thực x, y, z không âm thoả mãn
2 x  4 y  8 z  4 . Gọi M , N lần lượt là giá trị lớn nhất, giá trị nhỏ nhất của biểu thức
x y z
S    . Đặt T  2M  6 N . Khẳng định nào dưới đây đúng?
6 3 2
A. T  1; 2  . B. T   2;3 . C. T   3; 4  . D. T   4;5  .
Câu 61. (THPT Võ Nguyên Giáp - Quảng Bình - 2022) Có bao nhiêu cặp số nguyên  x, y  thỏa mãn
y 2 8
log 2  x 2  2 x  3  7  y2  3 y ?
A. 0 . B. 1 . C. 2 . D. 7 .
Câu 62. (THPT Yên Lạc - Vĩnh Phúc - 2022) Có bao nhiêu giá trị nguyên của tham số m để phương
trình 2 log 3  2 x  1  log 3  mx 2  1 có nghiệm.
A. 1 . B. 3 . C. 7 . D. 9 .

Trang 8 Fanpage Nguyễn Bảo Vương  https://www.facebook.com/tracnghiemtoanthpt489/


Điện thoại: 0946798489 TUYỂN CHỌN VẬN DỤNG – VẬN DỤNG CAO 2022
x 1
 2023 
Câu 63. (THPT Yên Lạc - Vĩnh Phúc - 2022) Cho f ( x)  2023.ln  e  e 2  . Tính giá trị biểu thức
 
H  f  1  f   2   ...  f   2022 
A. 1011. B. 2022. C. e 2022 . D. e1011 .
Câu 64. (THPT Yên Lạc - Vĩnh Phúc - 2022) Có bao nhiêu giá trị nguyên của tham số m để phương
1 x 2 1 x 2
trình 91   m  3 31  2m  1  0 có nghiệm thực
A. 7 . B. 5 . C. 6 . D. 4 .
Câu 65. (THPT Yên Phong 1 - Bắc Ninh - 2022) Cho hàm số y  f  x  . Hàm số y  f   x  có bảng
biến thiên như sau

 
Bất phương trình 2 f  x   ecos x  m đúng x   0;  khi và chỉ khi
 2
   
A. m  2 f  0   e . B. m  2 f    1 . C. m  2 f    1 . D. m  2 f  0   e .
2 2

Theo dõi Fanpage: Nguyễn Bảo Vương  https://www.facebook.com/tracnghiemtoanthpt489/

Hoặc Facebook: Nguyễn Vương  https://www.facebook.com/phong.baovuong

Tham gia ngay: Nhóm Nguyễn Bào Vương (TÀI LIỆU TOÁN)  https://www.facebook.com/groups/703546230477890/

Ấn sub kênh Youtube: Nguyễn Vương


 https://www.youtube.com/channel/UCQ4u2J5gIEI1iRUbT3nwJfA?view_as=subscriber

Tải nhiều tài liệu hơn tại: https://www.nbv.edu.vn/

Facebook Nguyễn Vương https://www.facebook.com/phong.baovuongTrang 9


TUYỂN CHỌN VẬN DỤNG – VẬN DỤNG CAO 2022 Điện thoại: 0946798489

VẤN ĐỀ 3. TÍCH PHÂN


• |FanPage: Nguyễn Bảo Vương
• TUYỂN CHỌN CÂU HỎI VD-VDC TỪ CÁC ĐỀ THI THỬ CÁC TRƯỜNG, CÁC SỞ NĂM 2022
Câu 1. (Chuyên Vinh – 2022) Cho hàm số y  f ( x) có đạo hàm trên đoạn [1; 2] thỏa mãn
2 2 2
f (1)  2, f (2)  1 và   xf ( x)
1
dx  2 . Tich phân 
1
x 2 f ( x)dx bằng
A. 4..
B. 2.
C. 1.
D. 3.
Câu 2. (Chuyên Vinh – 2022) Cho hàm số y  x 4  bx3  cx 2  dx  e(b, c, d , e   ) có các giá trị cực trị
f ( x)
là 1,4 và 9. Diện tích hình phẳng giới hạn bởi đồ thị hàm số g ( x)  với trục hoành bằng
f ( x)
A. 4.
B. 6.
C. 2.
D. 8.
Câu 3. (Chuyên Lê Quý Đôn - Điện Biên - 2022) Cho hàm số
y  f  x   ax3  bx 2  cx  d ,  a, b, c, d  , a  0  có đồ thị  C  . Biết rằng đồ thị  C  tiếp xúc
với đường thẳng y  4 tại điểm có hoành độ âm và đồ thị của hàm số y  f   x  cho bởi hình vẽ
dưới đây. Tính thể tích vật thể tròn xoay được tạo thành khi quay hình phẳng H giới hạn bởi đồ
thị  C  và trục hoành khi quay xung quanh trục Ox .

725 729 1
A. π. B. π. C. 6 π . D. π.
35 35 35
1
Câu 4. (Chuyên Lê Quý Đôn - Điện Biên - 2022) Xét hàm só́ f  x   e x   x f  x  dx . Giá trị của
0

f  ln 2022  bằng bao nhiêu?

A. 2022 . B. 2021 . C. 2023 . D. 2024 .


1
Câu 5. (Chuyên Lê Quý Đôn - Điện Biên - 2022) Cho hàm số f  x  thoả mãn f  2    và
25
2
f   x   4 x3  f  x   vói mọi x   . Giá trị của f 1  f  0  bằng
1 1 1 1
A. . B.  . C.  . D.
90 90 72 72
Câu 6. (Cụm Trường Nghệ An - 2022) Cho hai hàm số f  x  và g  x  liên tục trên  và hàm số
f   x   ax 3  bx 2  cx  d , g   x   qx 2  nx  p với a , q  0 có đồ thị như hình vẽ. Biết diện tích

Facebook Nguyễn Vương https://www.facebook.com/phong.baovuong Trang 1


Blog: Nguyễn Bảo Vương: https://www.nbv.edu.vn/
hình phẳng giới hạn bởi hai đồ thị hàm số y  f   x  và y  g   x  bằng 10 và f  2   g  2  . Biết
a
diện tích hình phẳng giới hạn bởi hai đồ thị hàm số y  f  x  và y  g  x  bằng (với a , b  
b
và a , b nguyên tố cùng nhau). Tính a  b .

A. 18. B. 19. C. 20. D. 13.


Câu 7. (Cụm Trường Nghệ An - 2022) Cho hàm số y  f  x  có đạo hàm liên tục trên  và thoả mãn
x2  4 x 1
f   x   2 f  x    x2  1 e 2
, x  và f 1  e2 . Biết f  3  a.eb  c với a , b , c   .
Tính 2a  3b  4c.
A. 36 . B. 30 . C. 24 . D. 32 .
Câu 8. (Đại học Hồng Đức – 2022) Cho hàm số f ( x)  x  ax  bx  c( a, b, c  ) có hai điểm cực trị
3 2

là 1 và 1. Gọi y  g ( x ) là hàm số bậc hai có đồ thị cắt trục hoành tại hai điểm có hoành độ
trùng với các điểm cực trị của f ( x ) , đồng thời có đỉnh nằm trên đồ thị của f ( x ) với tung độ
bằng 2. Diện tích hình phẳng giới hạn bởi hai đường y  f ( x ) và y  g ( x ) gần với giá trị nào
nhất dưới đây?
A. 10.
B. 12.
C. 13.
D. 11.
Câu 9. (THPT Hồ Nghinh – Quảng Nam – 2022) Cho hàm số f ( x ) nhận giá trị dương và có đạo hàm
2
liên tục trên đoạn [0;1] sao cho f (1)  1 và f ( x)  f (1  x)  e x x
, x  [0;1]. Tính

I 
1  2x 3
 3x 2
 f ( x)dx.
0 f ( x)
1
A. I   .
10
2
B. I  .
5
1
C. I   .
60
1
D. I  .
10
Câu 10. (THPT Hương Sơn - Hà Tĩnh - 2022) Cho hàm số y  f  x  liên tục, nhận giá trị dương trên
x2
 0;   và thỏa mãn f 1  2 ; f  x  2
với mọi x   0;   . Giá trị f  3 bằng
 f  x  
3 3
A. 34 . B. 34 . C. 3 . D. 20 .

Trang 2 Fanpage Nguyễn Bảo Vương  https://www.facebook.com/tracnghiemtoanthpt489/


Điện thoại: 0946798489 TUYỂN CHỌN VẬN DỤNG – VẬN DỤNG CAO 2022
Câu 11. (THPT Hương Sơn - Hà Tĩnh - 2022) Một vật chuyển động trong 4 giờ với vận tốc v  km / h 
phụ thuộc vào thời gian t ( h) có đồ thị của vận tốc như hình bên. Trong khoảng thời gian 2 giờ kể
từ khi bắt đầu chuyển động, đó là một đường parabol có đỉnh I  2;7  và trục đối xứng của
parabol song song với trục tung, khoảng thời gian còn lại, đồ thị là đoạn thẳng IA . Tính quãng
đường s mà vật di chuyển được trong 4 giờ đó (kết quả làm tròn đến hàng phần trăm).

A. s  15,81  km  . B. s  17,33  km  . C. s  23,33  km  . D. s  21, 33  km  .


Câu 12. (THPT Hương Sơn - Hà Tĩnh - 2022) Một biển quảng cáo có dạng hình tròn tâm O , phía trong
được trang trí bởi hình chữ nhật ABCD ; hình vuông MNPQ có cạnh MN  2m và hai đường
parabol đối xứng nhau chung đỉnh O như hình vẽ. Biết chi phí để sơn phần tô đậm là 300.000
đồng/m2 và phần còn lại là 250.000 đồng/m2. Hỏi số tiền để sơn theo cách trên gần nhất với số
tiền nào dưới đây?

A. 3.439.000 đồng. B. 3.628.000 đồng. C. 3.580.000 đồng. D. 3.363.000 đồng.


 2 x  3, khi x  2
Câu 13. (THPT Lê Thánh Tông - HCM-2022) Cho hàm số f  x    3 . Giả sử F  x  là
 4 x  1, khi x  2
nguyên hàm của f  x  trên  và thỏa mãn F  0   3 . Giá trị F  3  5 F  5  bằng
A. 12 . B. 16 . C. 13 . D. 7 .
Câu 14. (THPT Lê Thánh Tông - HCM-2022) Cho hai hàm đa thức f  x   ax 3  bx 2  cx  d và
g  x   mx 2  nx  p . Biết rằng đồ thị hai

hàm số y  f  x  và y  g  x  cắt nhau tại ba điểm có hoành độ lần lượt là 1; 2; 4 đồng thời cắt
trục tung lần lượt tại M , N sao cho MN  6 ( tham khảo hình vẽ).

Facebook Nguyễn Vương https://www.facebook.com/phong.baovuongTrang 3


Blog: Nguyễn Bảo Vương: https://www.nbv.edu.vn/

Hình phẳng giới hạn bởi đồ thị hai hàm số đã cho ( phần gạch sọc) có diện tích bằng
125 253 253 253
A. . B. . C. . D. .
8 24 16 12
Câu 15. (Liên trường Hà Tĩnh – 2022) Cho hàm số f ( x ) thỏa mãn
e3 x  4 f ( x)  f ( x)   2 f ( x) ln 2
 , x  0 và f (0)  1 . Tính I   f ( x)dx
0
 f ( x)  0
11
A. I  .
24
1
B. I   .
12
209
C. I  .
640
201
D. I  .
640
Câu 16. (Liên trường Hà Tĩnh – 2022) Cho hàm số f ( x) xác định trên  \{1;2} thỏa mãn
1 1
f ( x)  2 ; f (3)  f (3)  0 và f (0)  . Giá trị của biểu thức f (4)  f (1)  f (4) bằng
x  x2 3
1 1
A.  ln 2 .
3 3
B. 1  ln S 0 .
1
C.  ln 2 .
3
1 8
D. 1  ln .
3 5
Câu 17. (THPT Nguyễn Tất Thành-Đh-SP-HN-2022) Cho hàm số f  x  có đạo hàm f   x  liên tục
2
trên  và thỏa mãn các điều kiện f  x   0 x   f  0   1 và f   x   4 x 3  f  x   x   .
1
Tính I   x 3 . f  x  dx .
0

ln 2 1 1
A. I  . B. I  ln 2 . C. I  . D. I  .
4 4 6
Câu 18. (THPT Nguyễn Tất Thành-Đh-SP-HN-2022) Cho D là hình phẳng giới hạn bởi đồ thị hàm số
y  ln x, trục hoành và hai đường thẳng x  1; x  m, với m  1. Khi hình phẳng D có diện tích
bằng 1, giá trị của m thuộc khoảng nào dưới đây ?

Trang 4 Fanpage Nguyễn Bảo Vương  https://www.facebook.com/tracnghiemtoanthpt489/


Điện thoại: 0946798489 TUYỂN CHỌN VẬN DỤNG – VẬN DỤNG CAO 2022
y

y  ln x

-1 O 1 m x

7   7 5   5
A.  ; 4  . B.  3;  . C.  ;3 . D.  2;  .
2   2 2   2
2
 2 sin x  1 khi x  0
Câu 19. (THPT Nguyễn Tất Thành-Đh-SP-HN-2022) Cho hàm số f  x    x . Giả
 2 khi x  0
2
sử F  x  là một nguyên hàm của hàm số f  x  trên  và thỏa mãn điều kiện F 1  . Tính
ln 2
F    .
1 1
A. F      2  . B. F       .
ln 2 ln 2
1
C. F     2 . D. F      2  .
ln 2
Câu 20. (THPT Nho Quan A – Ninh Bình – 2022) Cho hàm số y  f ( x)  ax 4  bx 2  c có đồ thị (C ) ,
Biết f ( 1)  0 . Tiếp tuyến d tại điểm có hoành độ x  1 của (C ) cắt (C ) tại 2 điểm có hoành
độ lần lượt là 0 và 2, Gọi S1; S 2 là diện tích hình phẳng (phần gạch chéo trong hình vẽ). Tính S 2 ,
401
biết S1  .
2022

12431
A. .
2022
5614
B.  .
1011
2005
C. .
2022
2807
D. .
1011

Facebook Nguyễn Vương https://www.facebook.com/phong.baovuongTrang 5


Blog: Nguyễn Bảo Vương: https://www.nbv.edu.vn/
Câu 21. (THPT Phù Cừ - Hưng Yên - 2022) Cho hai hàm số f ( x ) và g ( x ) liên tục trên  và hàm số
f ( x)  ax 3  bx 2  cx  d , g ( x)  qx 2  nx  p với a , q  0 có đồ thị như hình vẽ. Biết diện tích
5
hình phẳng giới hạn bởi hai đồ thị hàm số y  f ( x) và y  g ( x ) bằng và f (2)  g (2) . Biết
2
a
diện tích hình phẳng giới hạn bởi hai đồ thị hàm số y  f ( x) và y  g ( x ) bằng (với a , b  
b
và a , b nguyên tố cùng nhau). Tính T  a 2  b 2 .

A. 7. B. 55. C. 5 . D. 16.
Câu 22. (Sở Hà Tĩnh 2022) Cho f ( x ) là hàm đa thức bậc bốn và có đổ thị như hình vẽ. Hình phắng gióri
hạn bởi đổ thị hai hàm số y  f ( x ) ; y  f ( x ) có diện tích bằng

127
A. .
40
107
B. .
5
87
C. .
40
127
D. .
10
Câu 23. (Sở Thanh Hóa 2022) Cho hàm số f ( x ) có đạo hàm liên tục trên  thoả män
2 f ( x )  xf ( x )  3 x  10, x   và f (1)  6 . Biết
4 ln(2  f ( x))
 2
dx  a ln 5  b ln 6  c ln(2  3), (a, b, c  ) . Khi đó a  b  c thuộc khoàng
1 f ( x )  6 f ( x )  9

nào dưới đây?


A. (1; 2) .
B. (2;3) .
C. (0;1) .
D. ( 1; 0) .

Trang 6 Fanpage Nguyễn Bảo Vương  https://www.facebook.com/tracnghiemtoanthpt489/


Điện thoại: 0946798489 TUYỂN CHỌN VẬN DỤNG – VẬN DỤNG CAO 2022
Câu 24. (Sở Thanh Hóa 2022) Cho hàm số f ( x )  0, x  0 và có đạo hàm f ( x ) liên tục trên khoảng
1
(0;  ) thoả mãn f ( x)  (2 x  1) f 2 ( x) , x  0 và f (1)   . Giá trị của biểu thức
2
f (1)  f (2)  f (3)    f (2022) bằng
2022
A. .
2023
2021
B. .
2022
2021
C.  .
2022
2022
D.  .
2023
Câu 25. (Sở Bắc Giang 2022) Cho hàm số y  f ( x) có đạo hàm xác định trên (0;  ) và thỏa mãn
1 a
x  f ( x)  x   ( x  1) f ( x); f (1)  e  1 . Biết rằng  f ( x) dx  ; trong đó a , b là những số nguyên
0 b
a
dương và phân số tối giản. Khi đó giá trị của (2 a  b ) tương ứng bằng:
b
A. 4.
B. 5.
C. 8.
D. 7.
Câu 26. (Sở Bắc Giang 2022) Một bức tường lớn kích thước 8m  8m trước đại sảnh của một toà biệt thự
được sơn loại sơn đặc biệt. Người ta vẽ hai nửa đường tròn đường kính AD, AB cắt nhau tại H ;
đường tròn tâm D , bán kính AD , cắt nửa đường tròn đường kính AB tại K . Biết tam giác
"cong" AHK được sơn màu xanh và các phần còn lại được sơn màu trắng (như hình vẽ) và một
mét vuông sơn trắng, sơn xanh lần lượt có giá là 1 triệu đồng và 1,5 triệu đồng. Tính số tiền phải
trả (làm tròn đến hàng ngàn).

A. 60,567, 000 (đồng). B. 70, 405, 000 (đồng).


C. 67,128, 000 (đồng). D. 86,124, 000 (đồng).
e x  m khi x  0
Câu 27. (Sở Bắc Giang 2022) Cho hàm số f ( x)   2 3 3 (với m là tham số). Biết
 x  x  1 khi x  0
1 b b
hàm số f ( x ) liên tục trên  và  f ( x ) dx  a  e  với a, b, c  * ; tối giản
1 c c
(e  2, 718281828) . Biểu thức a  b  c  m bằng
A. 13.
B. 35.
C. 11 .
D. 36.
Câu 28. (Sở Hà Tĩnh 2022) Cho đường cong (C ) : y  x 3  kx  2 và parabol P : y   x 2  2 tạo thành hai
miền phẳng có diện tích S1 , S 2 như hình vẽ bên.

Facebook Nguyễn Vương https://www.facebook.com/phong.baovuongTrang 7


Blog: Nguyễn Bảo Vương: https://www.nbv.edu.vn/

8
Biết rằng S1  , giá trị của S 2 bằng
3
1
A. .
2
1
B. .
4
3
C. .
4
5
D. .
12
Câu 29. (Sở Hà Tĩnh 2022) Cho hàm số y  f ( x ) có đạo hàm f ( x ) liên tục trên  . Miền hình phẳng
trong hình vẽ được giới hạn bơi đồ thị hàm số y  f ( x ) và trục hoành đồng thời có diện tích
1
S  a . Biết rằng  ( x  1) f ( x)dx  b
0

A. I  a  b  c. B. I  a  b  c .
C. a  b  c . D. I  a  b  c .
Câu 30. (Sở Ninh Bình 2022) Diện tích hình phẳng giới hạn bởi parabol y  x 2  2 x  1 và đường thẳng
y  (m  1) x  5 có giá trị nhỏ nhất bằng
16
A. .
3
48
B. .
3
64
C. .
3
32
D. .
3
Trang 8 Fanpage Nguyễn Bảo Vương  https://www.facebook.com/tracnghiemtoanthpt489/
Điện thoại: 0946798489 TUYỂN CHỌN VẬN DỤNG – VẬN DỤNG CAO 2022
Câu 31. (Sở Ninh Bình 2022) Cho hàm số f ( x)  x5  ax 4  bx3  cx 2  dx  36 . Biết đồ thị hàm số
y  f ( x ), y  f ( x ) và Ox giao nhau tại hai điểm phân biệt có hoành độ lần lượt là 2,3. Diện tích
m
hình phẳng giớihạn bởi đồ thị hàm số y  f ( x) và Ox bằng là một phân số tối giản với
n
m, n  * . Tổng m  n bằng
A. 846.
B. 845.
C. 848.
D. 847.
Câu 32. (Sở Ninh Bình 2022) Gọi X là tập hợp tất cả các giá trị của tham số m để đường thẳng d :
1
y  45m  2 cùng với đồ thị (C ) của hàm số y  x 3  2mx 2  x  1 tạo thành hai miền kín có
3
diện tích lần lượt là S1 , S 2 thỏa mãn S1  S 2 (xem hình vẽ). Số phần tử của tập X là

A. 0.
B. 2.
C. 1.
D. 9.
Câu 33. (Sở Bạc Liêu 2022) Cho hàm số y  f  x  xác định và liên tục trên đoạn  3;3 . Biết diện tích
hình phẳng S1 , S 2 giới hạn bởi đồ thị hàm số y  f  x  và đường thẳng y   x  1 lần lượt là
3
M , m . Tính tích phân  f  x  dx .
3

A. 6  m  M . B. 6  m  M . C. M  m  6 . D. m  M  6 .
x
e  1 khi x  0
Câu 34. (Sở Bạc Liêu 2022) Cho hàm số y  f  x    2 . Tích phân
 x  2 x  2 khi x  0
e2
f  ln x  1 a a
I dx   ce biết a , b, c   và tối giản. Tính a  b  c ?
1 x b b
e
A. 35 . B. 29 . C. 36 . D. 27 .

Facebook Nguyễn Vương https://www.facebook.com/phong.baovuongTrang 9


Blog: Nguyễn Bảo Vương: https://www.nbv.edu.vn/
Câu 35. (Sở Hà Tĩnh 2022) Cho hàm số f  x   ax 4  x 3  2 x  2 và hàm số g  x   bx 3  cx 2  2 có đồ
221
thị như hình vẽ bên. Gọi S1; S2 là diện tích các hình phẳng gạch chéo trong hình vẽ, biết S1  .
640
Khi đó S2 bằng

791 571 271 1361


A. . B. . C. . D. .
640 640 320 640
4
Câu 36. (Sở Hà Tĩnh 2022) Cho hàm số f  x  liên tục trên  và f  4   2 ,  f  x  dx  4 . Tính
0
2

 xf   2 x  dx
0

A. I  17 . B. I  1 . C. I  12 . D. I  4 .
 
Câu 37. (Sở Hà Tĩnh 2022) Cho F  x  là nguyên hàm của f  x   sin 2 x trên  thoả mãn F    0 .
4
 
Giá trị biểu thức S  F     2 F   bằng
2
3  3 3 1 3 3 3
A. S   . B. S   . C. S   . D. S   .
4 4 4 4 4 8 2 8
Câu 38. (Sở Lạng Sơn 2022) Cho hàm số y  f  x  có đạo hàm liên tục trên  thỏa mãn f 1  e và
f   x   f  x   x, x   . Giá trị f  2 bằng
2 1 1
A. . B. 1  . C. 1  . D. 2 .
e e e
Câu 39. (Sở Lạng Sơn 2022) Giả sử hàm số y  f  x  liên tục, nhận giá trị dương trên  0;   và thỏa
mãn f 1  e , f  x   f   x  . 3 x  1 , với mọi x  0 . Mệnh đề nào sau đây đúng?
A. 3  f  5   4 . B. 11  f  5   12 . C. 10  f  5   11 . D. 4  f  5   5 .
Câu 40. (Sở Phú Thọ 2022) Cho hàm số y  f  x liên tục trên R \ 2;0 thỏa mãn
x  x  2  . f   x   2 f  x   x 2  2 x và f 1  6 ln 3 . Biết f  3  a  b ln 5  a, b    . Giá trị
a  b bằng?
10 20
A. 20 . B. 10 . C. . D. .
3 3
3x 2 ln  x  1 khi x  0
Câu 41. (Sở Phú Thọ 2022) Cho hàm số f  x    . Biết
2
2 x x  3  1 khi x  0
e
f  ln x 
 dx  a 3  b ln 2  c với a, b, c  . Giá trị của a  b  6c bằng
1 x
e
A. 35 . B. 14 . C.  27 . D. 18 .
Trang 10 Fanpage Nguyễn Bảo Vương  https://www.facebook.com/tracnghiemtoanthpt489/
Điện thoại: 0946798489 TUYỂN CHỌN VẬN DỤNG – VẬN DỤNG CAO 2022
Câu 42. (Sở Vĩnh Phúc 2022) Cho hàm số f  x  có đạo hàm trên  , thoả mãn f  x   1
và f '  x  x 2  1  2 x f  x   1, x   . Biết rằng f  0   0 , khi đó f  2  có giá trị bằng
A. 0. B. 4. C. 8. D. 6.
Câu 43. (THPT Bùi Thị Xuân – Huế - 2022) Cho hai hàm số f ( x)  ax  bx  cx 2  3 x và
4 3

g ( x)  mx 3  nx 2  x , với a, b, c, m, n   . Biết hàm số y  f ( x )  g ( x ) có ba điểm cực trị là 3,1


và 4. Diện tích hình phẳng giới hạn bởi hai đồ thị y  f ( x ) và y  g ( x ) bằng
935
A. .
36
941
B. .
36
937
C. .
36
939
D. .
36
Câu 44. (THPT Bùi Thị Xuân – Huế - 2022) Một công ty có ý định thiết kế một logo hình vuông có độ
dài nửa đường chéo bằng 4. Biểu tượng 4 chiếc lá (được tô màu) được tạo thành bởi các đường
cong đối xứng với nhau qua tâm của hình vuông và qua các đường chéo.
Một trong số các đường cong ở nửa bên phải của logo là một phần của đồ thị hàm số bậc ba dạng
y  ax 3  bx 2  x với hệ số a  0 . Để kỷ niệm ngày thành lập 2 / 3 , công ty thiết kế để tỉ số diện
2
tích được tô màu so với phần không được tô màu bằng . Tính 2a  2b
3

41
A. .
80
1
B. .
2
4
C. .
5
9
D. .
10
Câu 45. (Chuyên Hoàng Văn Thụ - Hòa Bình – 2022) Cho hàm số y  f ( x) có đạo hàm không âm trên
2 2
 
[0;1] , thỏa mãn f ( x )  0 với mọi x  [0;1] và [ f ( x )]2   f ( x )  x 2  1  1  [ f ( x )]2 . Nếu
f (0)  3 thì giá trị f (1) thuộc khoảng nào sau đây?
 7
A.  3;  .
 2

Facebook Nguyễn Vương https://www.facebook.com/phong.baovuongTrang 11


Blog: Nguyễn Bảo Vương: https://www.nbv.edu.vn/
 5
B.  2;  .
 2
5 
C.  ;3  .
2 
3 
D.  ; 2  .
2 
7
Câu 46. (Chuyên Lam Sơn 2022) Cho hàm số f ( x ) với đồ thị là Parabol đỉnh I có tung độ bằng 
12
và hàm số bậc ba g ( x ) . Đồ thị hai hàm số đó cắt nhau tại ba điểm phân biệt có hoành độ x1 , x2 , x3
thoả mãn 18 x1 x2 x3  55 (hình vẽ).

Diện tích miền tô đậm gần số nào nhất trong các số sau đây?
A. 5,7.
B. 5,9.
C. 6,1.
D. 6,3.
 
Câu 47. (Chuyên Lam Sơn 2022) Cho hàm số y  f ( x ) liên tục trên đoạn  0;  thỏa mãn
 2
 
2 cos x  f (1  4sin x)  sin 2 x  f (3  2 cos 2 x)  sin 4 x  4sin 2 x  4 cos x, x   0; 
 2
5
Khi đó I   f ( x)dx bằng
1
A. 2.
B. 0.
C. 8.
D. 16.
Câu 48. (Chuyên Lam Sơn 2022) Cho hàm số y  f ( x ) có đạo hàm liên tục trên (0;  ) thỏa mãn
2
    
f ( x )  f ( x ) . cot x  2 x . sin x . Biết f    . Tính f   .
2 4 6
2
A. .
36
2
B. .
72
2
C. .
54
2
D. .
80
Câu 49. (THPT Đô Lương – Nghệ An – 2022) Cho hàm số y  f ( x ) liên tục trên  1,1 và thỏa mãn
3 1 1
f ( x)  2  ( x  t ) f ( t ) dt . với  x  [  1;1] Tính tích phân I  1 f ( x)dx
2 1
Trang 12 Fanpage Nguyễn Bảo Vương  https://www.facebook.com/tracnghiemtoanthpt489/
Điện thoại: 0946798489 TUYỂN CHỌN VẬN DỤNG – VẬN DỤNG CAO 2022
A. I  3
B. l  4
C. I  2
D. l  1
Câu 50. (THPT Lương Thế Vinh – Hà Nội – 2022) Một téc nước hình trụ, đang chứa nước được đạat
nằm ngang, có chiều dài 3m và đường kính đáy 1m . Hiện tại mặt nước trong téc cách phía trển
đỉnh của téc 0, 25 m (xem hình vẽ). Tính thể tích của nước trong téc (kết quả làm tròn đến hàng
phần nghìn)?

A. 1, 768 m3 .
B. 1,167m3
C. 1,895m3 .
D. 1,896m3
4
Câu 51. (THPT Lương Thế Vinh – Hà Nội – 2022)  min{2 x  1, x  2, 3x  14}dx bằng
0
31
A. .
2
B. 30.
27
C. .
2
D. 36.
Câu 52. (Chuyên Nguyễn Trãi – Hải Dương – 2022) Cho hàm số y  f ( x) có đồ thị (C ), f ( x ) có đạo
hàm xác định và liên tục trên khoảng (0;  ) thỏa mãn điều kiện
f ( x)  ln x  f 2 ( x), x  (0; ) . Biết f ( x )  0, x  (0;  ) và f (e)  2 . Viết phương trình tiếp
tuyến với đồ thị (C ) tại điểm có hoành độ x  1 .
2
A. y   x  2 .
3
2
B. y   .
3
2
C. y  x  1 .
3
2
D. y  .
3
Câu 53. (THPT Trần Phú – Hà Tĩnh – 2022) Cho hàm số y  f ( x )  0 liên tục trên  và f (1)  e3 .
4
Biết f ( x )  (2 x  3) f ( x ), x   . Hỏi phương trình f ( x)  e2 x 3 x  4
có bao nhiêu nghiệm?
A. 4.
B. 3.
C. 2.
D. 0.

Facebook Nguyễn Vương https://www.facebook.com/phong.baovuongTrang 13


Blog: Nguyễn Bảo Vương: https://www.nbv.edu.vn/
1
Câu 54. (THPT Kim Liên - Hà Nội - 2022) Cho hàm số y  f  x  thỏa mãn f  x   0, x  và có
2
1  1
đạo hàm f   x  liên tục trên khoảng  ;   thỏa mãn f   x   8 xf 2  x   0, x  và
2  2
1
f 1  . Tính f 1  f  2   ...  f 1011 .
3
1 2022 2021 2022 1 2021
A. . . B. . C. . D. . .
2 2023 2043 4045 2 2022
Câu 55. (THPT Kim Liên - Hà Nội - 2022) Gọi S là diện tích hình phẳng giới hạn bởi parabol
y  x 2  2 x  1 và các đường thẳng y  m , x  0 , x  1 . Có bao nhiêu giá trị nguyên của tham số
m  4040; 3 để S  2021 ?

A. 2019 . B. 2020 . C. 2021 . D. 2018 .


3 2
Câu 56. (THPT Kinh Môn - Hải Dương - 2022) Cho f  x   ax  bx  cx  d  a  0  là hàm số nhận
giá trị không âm trên đoạn  2;3 có đồ thị y  f   x  như hình vẽ

Biết diện tích hình giới hạn bởi các đồ thị của các hàm g  x   xf 2  x  ; h  x    x 2 f  x  f   x  và
các đường x  2; x  3 bằng 72. Tính f 1 ?
62
A. f 1  2 B. f 1  1 C. f 1  1 D. f 1  
5
3 2
Câu 57. (THPT Lương Tài 2 - Bắc Ninh - 2022) Cho hàm số f  x   x  bx  cx  d với b,c,d là các
số thực. Biết hàm số g  x   f  x   2 f   x   3 f   x  có hai giá trị cực trị là 6 và 42 . Tính diện
f  x   f   x   f   x 
tích hình phẳng giới hạn bởi các đường y  và y  1 .
g  x   18
A. ln 5 . B. ln 7 . C. 2ln 6 . D. 2 ln 5 .
Câu 58. (THPT Lương Tài 2 - Bắc Ninh - 2022) Cho hàm số y  f  x  liên tục trên  có đồ thị như
hình vẽ. Giả sử diện tích phần kẻ sọc trên hình vẽ có diện tích bằng a . Tính theo a giá trị của tích
2
phân I    2 x  1 f   x  dx ?
3

Trang 14 Fanpage Nguyễn Bảo Vương  https://www.facebook.com/tracnghiemtoanthpt489/


Điện thoại: 0946798489 TUYỂN CHỌN VẬN DỤNG – VẬN DỤNG CAO 2022

A. I  50  2 a . B. I  50  a .
C. I   30  2 a . D. I   30  2 a .
 2 x  a khi x  1
Câu 59. (THPT Lương Tài 2 - Bắc Ninh - 2022) Cho hàm số f ( x)   2 thoả mãn
3x  b khi x  1
2

 f ( x) dx  13 . Tính T  a  b  ab ?
0

A. T  11 . B. T   5 . C. T  1 . D. T  1 .
Câu 60. (THPT Võ Nguyên Giáp - Quảng Bình - 2022) Cho hàm số y  f  x  liên tục và nhận giá trị
2
không âm trên  1;2 và thoả mãn f  x   f 1  x  , x   1;2 . Đặt S1   xf  x  dx , S2 là diện
1

tích hình phẳng được giới hạn bởi đồ thị hàm số y  f  x  , trục Ox và hai đường thẳng
x  1; x  2 . Khẳng định nào dưới đây là đúng?
A. S1  2S2 . B. S1  3S2 . C. 2S1  S2 . D. 3S1  S2 .

Câu 61. (THPT Yên Lạc - Vĩnh Phúc - 2022) Cho hàm số y  f  x  liên tục trên  0;   thỏa mãn
1
2 x. f   x   f  x   3 x 2 x , x   0;   . Biết f 1  , tính f  4  .
2
A. 14 . B. 4 . C. 24 . D. 16 .

Theo dõi Fanpage: Nguyễn Bảo Vương  https://www.facebook.com/tracnghiemtoanthpt489/

Hoặc Facebook: Nguyễn Vương  https://www.facebook.com/phong.baovuong

Tham gia ngay: Nhóm Nguyễn Bào Vương (TÀI LIỆU TOÁN)  https://www.facebook.com/groups/703546230477890/

Ấn sub kênh Youtube: Nguyễn Vương


 https://www.youtube.com/channel/UCQ4u2J5gIEI1iRUbT3nwJfA?view_as=subscriber

Tải nhiều tài liệu hơn tại: https://www.nbv.edu.vn/

Facebook Nguyễn Vương https://www.facebook.com/phong.baovuongTrang 15


TUYỂN CHỌN VẬN DỤNG – VẬN DỤNG CAO 2022 Điện thoại: 0946798489

VẤN ĐỀ 4. SỐ PHỨC
• |FanPage: Nguyễn Bảo Vương
• TUYỂN CHỌN CÂU HỎI VD-VDC TỪ CÁC ĐỀ THI THỬ CÁC TRƯỜNG, CÁC SỞ NĂM 2022
Câu 1. (Chuyên Vinh – 2022) Xét các số phức z và w thỏa mãn | z || w | 1 và | z  w | 2 . Giá trị nhỏ
nhất của biểu thức P | zw  2i ( z  w)  4 | bằng
3 2
A. .
2
1 5 2
B. .
4
C. 5  2 2 .
D. 5 .
Lời giải
Chọn A

Gọi A, B lần lượt là các điểm biểu diễn số phức z , w , khi đó với | z  w | 2 ta luôn có OAB
 
là tam giác vuông tại O với OA  OB  0 , khi đó ta luôn có z  w là số thuần ảo tức
z  w  ki (k   )
Khi đó
 w  ki  2 2
 P | zw  2i( z  w)  4 | ki  2i   w   4  kiw  2i (kiw  w)  4  kiw  2kw  2iw  4
 w  w 

| z  w | ki  w | w(ki  1) || w | k 2  1  2  k  1
 w
Đặt w  a  bi (a , b   ) khi đó ta có:
P  w2  (2i  2) w  4  w2  (2  2i ) w  4i  ( w  1  i ) 2  2i
Đặt u  w  1  i  w  u  1  i | w || u  1  i | 1 , khi đó ta suy ra (đặt trước z0  1  i )
2 2 2
   4

P 2  u 2  2i  u 2  z02  u 2  z0 2 u 2  z02 | u |4  z0  u  z0  z0  u   2 u  z0
2

2
 
| u |4 4 | u |2 4  u  z0  z0  u

Mà  u  z   u  z   u  z  1  u  z
2 2
o o 0 o  zo  u  1 | u |2  zo   | u |2 1
2
92 2  1 9 9 3 2
Suy ra
2
 
| u |4 4 | u |2 4  | u |2 1  2 | u |4 2 | u |2 5  2  | u |2      P 
 2  2 2 2
Câu 2. (Chuyên Vinh – 2022) Gọi S là tập hợp tất cả các số phức z thỏa mãn điều kiện z  z | z  z | . Xét
các số phức z1 , z2  S sao cho z1  z2  1 . Giá trị nhỏ nhất của biểu thức P  z1  3i  z2  3i
bằng
A. 2.
B. 1  3 .
C. 2 3 .
D. 20  8 3 .
Lời giải
Chọn A
Đặt z  a  bi; a , b   .

Facebook Nguyễn Vương https://www.facebook.com/phong.baovuong Trang 1


Blog: Nguyễn Bảo Vương: https://www.nbv.edu.vn/
 a 2  b 2  2a
Ta có: z.z | z  z | a 2  b2  2 | a |  2 2
.
 a  b  2a
Gọi A, B lần lượt là hai điểm biểu diễn của số phức z1 , z2 .
z1  z2  1  AB  1.
Khi đó: P  z1  3i  z2  3i  CA  CB , với C (0; 3)
 Pmin  I1C  R  I 2C  R  2; v?i I1 ( 1;0), I 2 (1;0), R  1.
I1 I 2
Dấu " = " xảy ra vì A, B lần lượt là trung điểm CI1 , CI 2 và AB   1 (thỏa mãn)
2

Câu 3. (Chuyên Vinh – 2022) Biết phương trình z 2  mz  m 2  2  0 ( m là tham số thực) có hai nghiệm
phức z1 , z2 . Gọi A, B, C lần lượt là điểm biểu diễn các số phức z1 , z2 và z0  i . Có bao nhiêu giá
trị của tham số m để diện tích tam giác ABC bằng 1 ?
A. 2.
B. 3.
C. 4.
D. 6.
Lời giải
Chọn C
Ta có:   m 2  4  m 2  2   3m 2  8
2 6 2 6
TH1:   0  3m 2  8  0  m . Khi đó, phương trình có hai nghiệm thực phân
3 3
biệt là z1 , z2 .
2 2
Vì A, B  Ox nên AB  z1  z2   z1  z2    z1  z2   4 z1 z2  3m2  8 .
Mặc khác, ta có C (0;1)  d (C ; AB )  1 .
1 3m2  8 2 3
 SABC  AB  d (C ; AB)  1 m   ( n)
2 2 3
 2 6
m 
3
TH2:   0  3m 2  8  0   . Khi đó, phương trình có hai nghiệm phức liên hợp là
 2 6
m 
 3
m  i |  |
z1,2  .
2
Ta có: AB  z1  z2 | i |  | | 3m2  8  3m2  8 và C (0;1) .
m |m|
Phương trình đường thẳng AB là x   0 nên d (C ; AB )  .
2 2

Trang 2 Fanpage Nguyễn Bảo Vương  https://www.facebook.com/tracnghiemtoanthpt489/


Điện thoại: 0946798489 TUYỂN CHỌN VẬN DỤNG – VẬN DỤNG CAO 2022
 m  2
1 | m | 3m 2  8
Do đó, S ABC  AB  d (C ; AB)  1 
2 4  m   2 3 i(l )
 3
Vậy có 4 giá trị thực của tham số m thỏa mãn đề bài.
Câu 4. (Chuyên Lê Quý Đôn - Điện Biên - 2022) Xét các số phức z  a  bi ( a, b   ) thỏa mãn
| z  3  2i | 5 . Tính P  a  b khi | z  3  3i |  | z  7  i | đạt giá trị lớn nhất.
A. 8 B. 6 C. 4 D. 10
Lời giải
Chọn B
2 2
2 2  a 3  b 2 
 Ta có | z  3  2i | 5   a  3   b  2  5    1.
 5   5 
a 3
 5  sin t  a  5 sin t  3
Đặt   *
 b  2
 cos t  b  5 cos t  2
 5
2 2 2 2
 Đặt T | z  3  3i |  | z  7  i |  a  3   b  3   a  7    b  1
Thay * vào ta có:
2 2 2
T  5sin 2 t   5 cos t  5    5 sin t  4    5 cos t  3 
 30  10 5 cos t  30  8 5 sin t  6 5 cos t

 
 2 60  8 5 sin t  16 5 cos t  2 60  8 5  sin t  2 cos t  

Mà  5  sin t  2 cos t  5  T  2 60  8 5.  5     10 2

Suy ra Tmax  10 2 khi:


 2
 cos t 
30  10 5 cos t  30  8 5 sin t  6 5 cos t 4sin t  2 cos t  0  5 a  2
    .
sin t  2 cos t   5 sin t  2 cos t   5 sin t  1 b  4
 5
Vậy P  a  b  6 .
Câu 5. (Chuyên Lê Quý Đôn - Điện Biên - 2022) Trong tập các số phức, phương trình
z 2  6 z  m  0, m   1 . Gọi m0 là một giá trị m để phương trình 1 có hai nghiệm phân biệt
z1 , z2 thoả mãn z1.z1  z2 .z2 . Hỏi trong khoảng  0; 20  có bao nhiêu giá trị m0   ?
A. 10 B. 12 C. 11 D. 13
Lời giải
Chọn A
Để phương trình 1 có hai nghiệm phân biệt z1 , z2 thoả mãn z1. z1  z2 . z2 thì
   0  62  4m  0  m  9

  z1.z1  z2 .z2  z1.z2  z2 .z1  luoân ñuùng 
 2
   0  6  4m  0  m  9  m  9.
 
  z .z  z .z  z 2  z 2   z1  z2  L 
 1 1 2 2 1 2 
   z1   z2  z1  z2  0  3  0  L 
Mà trong khoảng  0; 20  và m0   nên có 10 giá trị m0 thoả mãn.
Facebook Nguyễn Vương https://www.facebook.com/phong.baovuongTrang 3
Blog: Nguyễn Bảo Vương: https://www.nbv.edu.vn/
Câu 6. (Đại học Hồng Đức – 2022) Cho số phức z thoả mãn iz.z  (1  2i ) z  (1  2i ) z  4i  0 . Giá trị lớn
nhất của P  z  1  2 i  z  4  i gần số nào nhất sau đây?
A. 7,4.
B. 4,6.
C. 4,2.
D. 7,7.
Lời giải.
Giả sử z  x  yi, ( x, y   ) . Ta có

iz.z  (1  2i ) z  (1  2i ) z  4i  0  i ( x  yi )( x  yi )  (1  2i )( x  yi )  (1  2i )( x  yi )  4i  0
 i  x 2  y 2   ( x  2 y )  (2 x  y )i  ( x  2 y )  ( 2 x  y )i  4i  0  x 2  y 2  4 x  2 y  4  0.
Suy ra, tập hợp các số phức z có điểm biểu diễn thuộc đường tròn (C ) có tâm I (2; 1) , bán
kính R  3 .
Lại có
P | z  1  2i |  | z  4  i || ( x  1)  ( y  2)i |  | ( x  4)  ( y  1)i |
 x 2  y 2  2 x  4 y  5  x 2  y 2  8 x  2 y  17
Kết hợp với (2) ta được P  9  2( x  y )  21  4( x  y ) .
 21 9 
Đặt t  x  y thì P  f (t )  9  2t  21  4t với t    ;  .
 4 2
Khảo sát hàm số f (t ) hoăc áp dụng bất đẳng thức Bunhiacopski, ta được
 21   21  3 26
P  9  2t  2   2t   (1  2)  9     7, 65.
 2   2 2
5 7  217 17  217
Dấu bằng xảy ra khi t  , từ đó có thể tính được z  i .
4 8 8
Câu 7. (Đại học Hồng Đức – 2022) Trên tập hợp các số phức, xét phương trình z 2  2mz  6m  5  0(m là
tham số thực). Có bao nhiêu giá trị nguyên của m để phương trình đó có hai nghiệm phân biệt
z1 , z2 thỏa mãn z1  z1  z2  z2 ?
A. 5.
B. 3.
C. 6.
D. 4.
Lời giải.

Ta có   m 2  6m  5 .
Phương trình có hai nghiệm phân biệt nên xảy ra hai trường hợp:
- Nếu   0  m  ( ;1)  (5; ) thì phương trình có hai nghiệm thực phân biệt z1 , z2 và
z1  z1 ; z2  z2 nên
 z1  z2 (ko thoai mãn),
z1  z1  z2  z2  z12  z22  
 z1   z2  z1  z2  0  m  0.
- Nếu   0  m  (1;5) , thì phương trình có hai nghiệm phức là hai số phức liên hợp.
Khi đó z1  z2 ; z1  z2 nên z1 z1  z2 z2  z1 z2  z1 z2 luôn đúng với m  (1;5) .
Vầy có 4 giá trị nguyên của m thoả mãn bài toán.
Câu 8. (THPT Lê Thánh Tông - HCM-2022) Xét hai số phức z1 , z2 thỏa mãn các điều kiện
z1  2, z2  3, z1  z2  5 . Giá trị nhỏ nhất của biểu thức P  3z1  z2  10  5i  2 bằng

Trang 4 Fanpage Nguyễn Bảo Vương  https://www.facebook.com/tracnghiemtoanthpt489/


Điện thoại: 0946798489 TUYỂN CHỌN VẬN DỤNG – VẬN DỤNG CAO 2022

B. 10 3  2 5 . B. 3 5  1 . C. 2  2 5 . D. 8  2 5 .
Lời giải
Chọn C
Gọi z1  a  bi, z2  c  di,  a, b, c, d    .

Ta có:

z1  2  a 2  b2  2  a 2  b2  4 .

z2  3  c 2  d 2  3  c 2  d 2  3 .

2 2 2 2
z1  z2  5   a  c  b  d   5   a  c    b  d   5  ac  bd  1 .

Suy ra:

2 2
3 z1  z2   3a  c    3b  d   9  a 2  b 2    c 2  d 2   6  ac  bd   3 5 .

Khi đó:

P  3 z1  z2  10  5i  2   3 z1  z2    10  5i   2  10  5i  3 z1  z2  2  2  2 5 .

Câu 9. (THPT Lê Thánh Tông - HCM-2022) Có tất cả bao nhiêu số phức w thỏa mãn điều kiện 2 ww  1
w
và là số thuần ảo?
w2
A. 4 . B. 6 . C. 3 . D. 2 .
Lời giải
Chọn B
Gọi số phức w  x  yi, x, y   .
Điều kiện: w2  0  w  0 .
2 1 1
Từ giả thiết 2 ww  1  w   x 2  y 2   * .
2 2
3

Mặt khác:
w

w3

 x  yi  
x 3  3 xy 2

3x2 y  y3
i.
2 2 2
w2 w2 .w 2 x 2
 y2  x 2
 y2  x 2
 y2 
w x3  3 xy 2  x0
Đề là số thuần ảo khi và chỉ khi  x 3  3 xy 2   2 .
2 2 2
w 2
x 2
y  x  3y
2
Với x  0  y   , suy ra tồn tại hai số phức.
2
1 2
Với x 2  3 y 2 thay vào (*) ta được: 4 y 2   y   , với mỗi giá trị của y tồn tại hai giá
2 4
trị của x , do đó có 4 cặp  x; y  .
Vậy có tất cả 6 số phức w thỏa mãn bài toán.
Câu 10. (THPT Nguyễn Tất Thành-Đh-SP-HN-2022) Cho hai số phức z và w thay đổi thỏa mãn các
điều kiện z  1  i  z và w  3  4i  1 . Tìm giá trị nhỏ nhất của biểu thức P  z  w  1  i .
A. min P  3 2  1 . B. min P  3 2 . C. min P  5 2 . D. min P  5 2  1 .
Lời giải

Facebook Nguyễn Vương https://www.facebook.com/phong.baovuongTrang 5


Blog: Nguyễn Bảo Vương: https://www.nbv.edu.vn/
Chọn A
Gọi số phức z  x  yi có điểm biểu diễn là M  x ; y  thì M nằm trên đường thẳng
 : x  y  1  0 hay x   y  1
Ta có P  z  w  1  i   z  4  5i    w  3  4i   z  4  5i  w  3  4i
2 2 2 2
P  x  4    y  5 1   y  5   y  5 1  2 y 2  50  1  5 2  1

Câu 11. (THPT Nguyễn Tất Thành-Đh-SP-HN-2022) Cho số phức z thay đổi thỏa mãn
z  1  3i  2 và số phức w  1  2i  z . Biết rằng tập hợp các điểm biểu diễn số phức w là một
đường tròn  C  trong mặt phẳng  Oxy  . Tìm bán kính R của đường tròn  C  .
A. R  5 . B. R  10 . C. R  6 . D. R  2 5 .
Lời giải
Chọn D
Ta có: w  1  2i  z  1  2i   z   1  3i    1  2i  1  3i 
 w  1  2i   z   1  3i     5  5i 
 w   5  5i   1  2i  z  1  3i   w   5  5i   1  2i  z  1  3i   1  2i z  1  3i
 w   5  5i   2 5
Tập hợp các điểm biểu diễn số phức w là một đường tròn  C  có bán kính R  2 5 .
Câu 12. (THPT Nho Quan A – Ninh Bình – 2022) Gọi S là tập hợp tất cả các số phức z sao cho
iz.z  (1  2i ) z  (1  2i ) z  4i  0 và T là tập hợp tất cả các số phức w có phần thực khác 0 sao
w
cho là số thực. Xét các số phức z1 , z2  S và w  T thỏa mãn z1  z2  2 5 và
w  6i
w  z1 w  z1
 . Khi w  z1  w  z1 đạt giá trị nhỏ nhất thì w  z1  w  z1 bằng
z2  z1 z2  z
A. 3 .
B. 2 3 .
C. 3 3 .
D. 4 3 .
Lời giải
Chon D
Giả sử z  x  yi, ( x, y   ) . Ta có
iz.z  (1  2i) z  (1  2i) z  4i  0
 i ( x  yi )( x  yi )  (1  2i )( x  yi)  (1  2i)( x  yi)  4i  0
 i  x 2  y 2   ( x  2 y )  (2 x  y )i  ( x  2 y )  (2 x  y )i  4i  0
 x2  y2  4 x  2 y  4  0
Suy ra S là tập hợp các số phức có điểm biểu diễn thuộc đường tròn (C ) có tâm I (2; 1) , bán
kính R  3 .
Giả sử w  a  bi, ( a, b  ; a  0) . Ta có
w a  bi (a  bi)[a  (b  6)i] a 2  b2  6b 2ab  6a
  2 2
 2  2 i
w  6i a  (6  b)i a  (b  6) a  (6  b) a  (6  b) 2
2

w 2ab  6a
Do đó là số thực khi và chỉ khi 2  0  b  3.
w  6i a  (6  b)2
Suy ra T là tập hợp các số phức có điểm biểu diễn thuộc đường thẳng  : y  3 .

Trang 6 Fanpage Nguyễn Bảo Vương  https://www.facebook.com/tracnghiemtoanthpt489/


Điện thoại: 0946798489 TUYỂN CHỌN VẬN DỤNG – VẬN DỤNG CAO 2022
w  z1 w  z1
Xét các số phức z1 , z2  S và w  T thỏa mãn z1  z2  5 và  .
z2  z1 z2  z1
Giả sử z1  x1  y1i, z2  x2  y2i  x1 , y1 , x2 , y2    và w  x  3i, ( x  , x  0) .
Gọi M 1 , M 2 , M lần lượt là các điểm biểu diễn của z1 , z2 và w .
Khi đó, M 1 , M 2  (C ) và M   , đồng thời w  z1 . w  z1  MM 1MM 2 .

w  z1 w  z1
Do z1  z2  2 5 nên M 1M 2  2 5 và do  nên ba điểm M 1 , M 2 , M thẳng hàng.
z2  z1 z2  z 1
Suy ra MM 1MM 2  IM 2  R 2 .
Vì vậy w  z1  w  z1  IM 2  R 2 .
Do đó, w  z1  w  z1 đạt giá trị nhỏ nhất khi và chỉ khi IM đạt giá trị nhỏ nhất. Lúc đó, M là
hình chiếu vuông góc của I trên  và M  (2;3) .
Gọi H là trung điểm của M 1M 2 , ta có IH  IM12  M1 H 2  32  ( 5)2  2 .
Vì bốn điểm M1 , M 2 , M , H thẳng hàng nên MIH vuông tại H suy ra
MH  IM 2  IH 2  42  22  2 3 và do đó,
w  z1  w  z1  MM 1  MM 2  MH  HM 1  MH  HM 2  2 MH  4 3.
Câu 13. (THPT Nho Quan A – Ninh Bình – 2022) Trên tập hợp các số phức, xét phương trình
z 2  2mz  m  12  0 ( m là tham số thực). Có bao nhiêu giá trị nguyên của m để phương trình
đó có hai nghiệm phân biệt z1 , z2 thỏa mãn z1  z2  2 z1  z2 ?
A. 1.
B. 2.
C. 3.
D. 4.
Lời giải
Phương trình đã cho có   m 2  m  12 .
 m  4
Trường hợp 1:   0  m2  m  12  0   .
m  3
Khi đó, phương trình đã cho có hai nghiệm thực z1 , z2 phân biệt.
2 2
Do đó, z1  z2  2 z1  z2   z1  z2   2 z1  z2 
2 2

 z12  z22  2 z1 z2  2 z12  z22  2 z1 z2    z1  z2   2 z1 z2  2 z1 z2  2  z1  z2   4 z1 z2 
 

Facebook Nguyễn Vương https://www.facebook.com/phong.baovuongTrang 7


Blog: Nguyễn Bảo Vương: https://www.nbv.edu.vn/
2
  z1  z2   6 z1 z2  2 z1 z2  0  4m 2  6( m  12)  2 | m  12 | 0(*) Nếu m  4 hoặc
 m  6
3  m  12 thì (*)  4m2  8(m  12)  0  m2  2m  24  0   .
m  4
Nếu m  12 thì (*)  4m 2  4( m  12)  0  m 2  m  12  0 (không thỏa mãn).
Trường hợp 2:   0  m 2  m  12  0  4  m  3 .
Khi đó, phương trình đã cho có hai nghiệm phân biệt z1 , z2 là hai số phức liên hợp:
m  i m2  m  12 và  m  i m 2  m  12.
Do đó, z1  z2  2 z1  z2

 2 m2   m2  m  12   2 m2  m  12  m  12  m2  m  12  m  0 (thỏa mãn)


Vậy có 3 giá trị nguyên của tham số m thỏa mãn đề bài.
Câu 14. (THPT Phù Cừ - Hưng Yên - 2022) Trên tập hợp các số phức, xét phương trình
z 2  2mz  3m  10  0 ( m là tham số thực). Có bao nhiêu giá trị nguyên của m để phương trình
đó có hai nghiệm z1 , z2 không phải số thực thỏa mãn z1  z2  8 ?
A. 1 B. 2. C. 3. D. 4.
Lời giải
Ta có: z 2  2mz  3m  10  0(*) thì   m 2  3m  10 .
Điều kiện   0  2  m  5 .
Phương trình (*) khi đó có 2 nghiệm z1,2  m  i m2  3m  10 .
10
Do đó z1  z2  8  2 z1  8  z1  4  3m  10  4    m  2.
3
Kết hợp điều kiện 2  m  5 , suy ra 2  m  2
Vậy các giá trị nguyên của thỏa mãn là: m  {1; 0;1; 2} .
Câu 15. (THPT Phù Cừ - Hưng Yên - 2022) Cho số phức z và số phức w  ( z  i )( z  i )  2 z  3i thỏa
mãn w  i 2022  i 2023  w  1  0 . Giá trị lớn nhất của biểu thức T | z  3  i |2  | z  1  3i |2 bằng
m  n 5 với m, n   . Tính P  m.n .
A. P  124 . B. P  876 . C. P  416 . D. P  104 .
Lời giải
Gọi w  x  yi với x, y   .
Hệ thức w  i 2022  i 2023  w  1  0 | w  1| i  w  i 2 | w  1|| i | . | w  i |
| w  1|| w  i || x  yi  1|| x  yi  i | ( x  1) 2  y 2  x 2  ( y  1) 2  x  y
 số phức w có phần thực bằng phần ảo.
Gọi z  a  bi với a, b   .
 w  ( z  i )( z  i )  2 z  3i | z |2 i ( z  z )  1  2 z  3i  a 2  b 2  i(2bi )  1  2(a  bi )  3i
  a 2  b 2  2a  2b  1  (2b  3)i
Suy ra:  a 2  b 2  2a  2b  1  (2b  3)  (a  1) 2  (b  2)2  1 (1).
Suy ra quỹ tích điểm biểu diễn số phức z là đường tròn (C ) có tâm I ( 1; 2) và bán kính R  1 .
Biểu thức
2 2 2 2 2 2 2 2
T | z  3  i |  | z  1  3i | | z  3  i |  | z  1  3i | | z  3  i |  | z  1  3i |  MA  MB , với
điểm M biểu diễn số phức z và nằm trên đường tròn (C ) có tâm I ( 1; 2) và bán kính R  1 và
điểm A(3; 1), B ( 1; 3)
AB 2
Ta có T  MA2  MB 2  2 MK 2  (với K là trung điểm của đoạn AB )
2
Có K (1; 2) và AB  2 5 suy ra T  MA2  MB 2  2 MK 2  10
Trang 8 Fanpage Nguyễn Bảo Vương  https://www.facebook.com/tracnghiemtoanthpt489/
Điện thoại: 0946798489 TUYỂN CHỌN VẬN DỤNG – VẬN DỤNG CAO 2022
Suy ra Tmax  MK max  K là hình chiếu vuông góc của M trên AB  M , I , K thẳng hàng và
I nằm giữa M , K .
Mặt khác ta có IM  (a  1; b  2), IK  (2; 4)  IK  2 5 .
1  5 2 5 5 2 5
Suy ra IM  IK  M  1  ;2    a  1  ;b  2  .
2 5  5 5  5 5
Vậy Tmax  2(2 5  1) 2  10  52  8 5  m  52; n  8  P  m.n  416 .

Câu 16. (Sở Bắc Giang 2022) Giả sử z1 ; z2 là hai trong các số phức z thỏa mãn ( z  6)(8  i.z ) là số
thực. Biết rằng z1  z2  6 . Giá trị nhỏ nhất của z1  3z2 bằng
A. 5  73 .
B. 5  21 .
C. 20  2 73 .
D. 20  4 21 .
Lời giải

Đặt z  x  yi với x; y   . Gọi A; B lần lượt là các điểm biểu diễn số phức z1 ; z2 .
Ta có: z1  z2  6  AB  6 .
Và ( z  6)(8  iz )  ( x  yi  6)(8  xi  y )  [( x  6)  yi ][(8  y )  xi]
 [( x  6)(8  y )  xy ]  [(8  y ) y  ( x  6) x]i  8 x  6 y  48   x 2  y 2  6 x  8 y  i
Theo giả thiết ( z  6)(8  i.z ) là số thực nên x 2  y 2  6 x  8 y  0
Facebook Nguyễn Vương https://www.facebook.com/phong.baovuongTrang 9
Blog: Nguyễn Bảo Vương: https://www.nbv.edu.vn/
Do đó A; B  (C ) : x 2  y 2  6 x  8 y  0 là đường tròn tâm I (3; 4) , bán kính R  5 .
          
Xét điểm M thỏa mãn MA  3MB  0  MO  OA  3MO  3OB  0  OA  3OB  4OM .
Gọi H là trung điểm AB , khi đó: HI 2  R 2  HB 2  16 ,
2
3 73
IM  HI 2  HM 2  4 2     .
2 2
73
Suy ra: Điểm M thuộc đường tròn  C1  tâm I (3; 4) , bán kính R1  .
   2
Ta có: z1  3 z2 | OA  3OB || 4OM | 4OM
 73 
 z1  3 z2 min  4OM min  4 OI  R1  4  5    20  2 73.
 2 
Vậy z1  3 z2 min  20  2 73 .
Câu 17. (Sở Bạc Liêu 2022) Cho các số phức z , w thỏa mãn z  2 , w  3  2i  1 khi đó z 2  2 zw  4
đạt giá trị lớn nhất bằng
A. 16 . B. 24 . C. 4  4 13 . D. 20 .
Lời giải
Chọn B
2
Ta có T  z 2  2 zw  4  z 2  2 zw  z  z 2  2 zw  z.z  z . z  z  2 w  2 z  z  2 w

Gọi z  x  yi  z  z  2 yi .

Vì z  2 nên 4  2 y  4 .
2 2
Gọi w  x  yi  2w  2w  6  4i  2  w  6  4i  2   x  6    y  4   4 .

Gọi A là điểm biểu diễn của z  z  A thuộc trục Oy với 4  y A  4 .

Gọi B là điểm biểu diễn của 2w  B thuộc đường tròn tâm I  6; 4  ; bán kính R  2 .

Khi đó T  2 AB .
Ta có hình vẽ:

Ta có Tmax  2 ABmax  2  IA  R   24 với A  0; 4  .


Câu 18. (Sở Hà Tĩnh 2022) Cho số phức z  x  yi,  x, y    thỏa mãn z  z  2  3 z  z  4i  6 và
z  1  i  z  3  i . Gọi M , m là giá trị lớn nhất và nhỏ nhất của biểu thức P  2 x  3 y  5 . Khi
đó M  m bằng:
17 33 13 22
A. . B. . C.  . D. .
5 5 5 5
Lời giải
Chọn D

Trang 10 Fanpage Nguyễn Bảo Vương  https://www.facebook.com/tracnghiemtoanthpt489/


Điện thoại: 0946798489 TUYỂN CHỌN VẬN DỤNG – VẬN DỤNG CAO 2022
 ) z  z  2  3 z  z  4i  6  2 x  2  3 2 yi  4i  6  x  1  3 y  2  3
x  3y  2  0 khi x  1, y  2
 x  3 y  10  0 khi x  1, y  2


 x  3 y  4  0 khi x  1, y  2
 x  3 y  8  0 khi x  1, y  2
) z  1  i  z  3  i   x  1   y  1 i   x  3   y  1 i
2 2 2 2
  x  1   y  1   x  3   y  1  8x  4 y  8  0  2 x  y  2  0
+) Tập hợp điểm biểu diễn số phức z thỏa mãn yêu cầu bài toán là ngũ giác ABCDE (như hình
vẽ). Giá trị lớn nhất và nhỏ nhất của P  2 x  3 y  5 đạt được tại hai đỉnh của ngũ giác ABCDE

y
d
d1
1
x
d3
d: 2x + y + 2 = 0
d2 A B d4
C d1: x + 3y + 2 = 0
d2: x + 3y + 8 = 0
E Δ
D d3: x - 3y - 4 = 0
d4: x - 3y - 10 = 0
Δ: 2x + 3y = 0

+) Biểu thức P  2 x  3 y  5 đạt giá trị lớn nhất là M  Pmax  7 khi x  4, y  2 (tại C ).
13 2 14
Biểu thức P  2 x  3 y  5 đạt giá trị nhỏ nhất là m  Pmin   khi x  , y   (tại E ).
5 5 5
22
Suy ra M  m  .
5
Câu 19. (Chuyên Hoàng Văn Thụ - Hòa Bình – 2022) Trên tập hợp số phức, xét phương trình
1
z 2  m  1z   m 2  5m  6   0( m là tham số thực). Có bao nhiêu số nguyên m  [ 10;10] đề
4
phương trình trên có hai nghiệm phức z1 , z2 thỏa mãn z1  z2  z1  z2 ?
A. 11.
B. 10.
C. 8.
D. 9.
Lời giải
Điều kiện m  1  0  m  1.  m 2  4 m  5
m  5
 Trường hợp 1:   0  m2  4m  5  0   phương trình có 2 nghiệm thực z1 , z2
 m  1
1
Theo định lý Viet z1  z2    m 2  5m  6  .
4

Facebook Nguyễn Vương https://www.facebook.com/phong.baovuongTrang 11


Blog: Nguyễn Bảo Vương: https://www.nbv.edu.vn/
2 2
z1  z2  z1  z2  z1  z2  z1  z2  4 z1  z2  0
m  6
  m 2  5m  6   0  m 2  5m  6  0  
 m  1
Do m   và m  [ 10;10] nên số giá trị m thỏa mãn là (10  6)  1  1  6 .
 Trường hợp 2 :   0  m 2  4m  5  0  1  m  5 .
phương trình có 2 nghiệm phức z1 , z2
2 2
z1  z2  z1  z2  z1  z2  z1  z2  m  1  m 2  4m  5
 2 m  6
 m  5m  6  0
 2   m  1

 m  3m  4  0 
 1 m  4
Do m  , 1  m  5 và m  [ 10;10] nên số giá trị m thỏa mãn là m  0, m  1, m  2, m  3 .
Vậy có 10 giá trị của m .
Câu 20. (Chuyên Lam Sơn 2022) Cho M , N , P lần lượt là các điểm biểu diễn số phức z1 , z2 , z3 thỏa mãn
điều kiện 5 z1  9  3i  5 z1 , z2  2  z2  3  i , z3  1  z3  3  4 . Khi M , N , P không thẳng
hàng, giá trị nhỏ nhất của nửa chu vi p của tam giác MNP là
10 5
A. .
9
6 5
B. .
5
9 10
C. .
10
5 11
D. .
13
Lời giải
Trong mặt phẳng Oxy , gọi A( 1; 0), B (0;3), C (3; 0) và M , N , P lần lượt là các điểm biểu diễn số
phức z1 , z2 , z3 . Ta có
Tập hợp điểm M biểu diễn số phức z1 là đường thẳng AB .
Tập hợp điểm N biểu diễn số phức z2 là đường thẳng BC .
z3  1  z3  3  4  PA  PC  AC  Tập hợp điểm P biểu diễn số phức z3 là đoạn AC .

MN  NP  PM
Khi đó p  .
2
Gọi P1 , P2 lần lượt đối xứng với P qua AB, BC . Ta có MP  MP1 , NP  NP2 .
Khi đó MN  NP  PM  PM 1  MN  NP2  P1 P2 .
Ta thấy P  BP  P BA     PBA
ABC  CBP  ABC  PBC  2
ABC .
1 2 1

Trang 12 Fanpage Nguyễn Bảo Vương  https://www.facebook.com/tracnghiemtoanthpt489/


Điện thoại: 0946798489 TUYỂN CHỌN VẬN DỤNG – VẬN DỤNG CAO 2022
AB AC  2 5
AC sin BCA
Theo đinh lí Sin:   sin 
ABC  
 sin 
sin BCA ABC AB 5
Gọi H là trung điểm của P1 P2 , khi đó
  2 5 4 5 4 5 12 5
P1P2  2 P2 H  2 BP2  sin P2 BH  2 BP  sin ABC  2 BP   BP  BO  .
5 5 5 5
6 5
Vậy giá trị nhỏ nhất của p là .
5
Câu 21. (THPT Kinh Môn - Hải Dương - 2022) Cho hai số phức z , w thỏa mãn z  w  10 ,
2 z  w  17 và z  3w  146 . Tính giá trị của biểu thức P  z.w  z.w .
A. P  14 . B. P  14 . C. P  16 . D. P  8 .
Lời giải
Chọn D
Gọi z  a  bi và w  x  yi với a, b, x, y   .
Theo đề ta có:
2 2
z  w  10   a  x    b  y   10  a 2  2ax  x 2  b 2  2by  y 2  10 (1)
2 2
2 z  w  17   2a  x    2b  y   17  4a 2  4ax  x 2  4b 2  4by  y 2  17 (2)
2 2
z  3w  146   a  3 x    b  3 y   146  a 2  6ax  9 x 2  b 2  6by  9 y 2  146 (3)
Lấy 35.(1)  8.(2)  3.(3) vế theo vế ta được:
56.ax  56.by  224  ax  by  4
Ta có P  z.w  z.w =  a  bi  .  x  yi    a  bi  .  x  yi   2.ax  2.by  8
Cách 2:
2
z  w  10  z  w  10   z  w  z  w  10  
 
  z  w  z  w  10  z  P  w  10
2 2
1
Tương tự
2 2
2 z  w  17  4 z  2 P  w  17  2
2 2
z  3w  146  z  3P  9 w  146  3
z2 5

Từ 1 ,  2  ,  3   P  8 .
 2
 w  13
Vậy P  8 .

Theo dõi Fanpage: Nguyễn Bảo Vương  https://www.facebook.com/tracnghiemtoanthpt489/

Hoặc Facebook: Nguyễn Vương  https://www.facebook.com/phong.baovuong

Tham gia ngay: Nhóm Nguyễn Bào Vương (TÀI LIỆU TOÁN)  https://www.facebook.com/groups/703546230477890/

Ấn sub kênh Youtube: Nguyễn Vương


 https://www.youtube.com/channel/UCQ4u2J5gIEI1iRUbT3nwJfA?view_as=subscriber

Tải nhiều tài liệu hơn tại: https://www.nbv.edu.vn/

Facebook Nguyễn Vương https://www.facebook.com/phong.baovuongTrang 13


TUYỂN CHỌN VẬN DỤNG – VẬN DỤNG CAO 2022 Điện thoại: 0946798489

VẤN ĐỀ 5. HHKG-THỂ TÍCH KHỐI ĐA DIỆN


• |FanPage: Nguyễn Bảo Vương
• TUYỂN CHỌN CÂU HỎI VD-VDC TỪ CÁC ĐỀ THI THỬ CÁC TRƯỜNG, CÁC SỞ NĂM 2022
Câu 1. (Chuyên Lê Quý Đôn - Điện Biên - 2022) Cho hình chóp S. ABCD có đáy ABCD là hình
bình hành và M là trung điểm cạnh bên SC . Gọi  P  là mặt phẳng chứa AM và song song với BD , mặt
VS . ABMD
phẳng  P  cắt SB, SD lần lượt tại B  và D . Tính tỷ số
VS . ABCD
1 1 3 2
A. . B. . C. . D. .
6 3 4 3
Câu 2. (Cụm Trường Nghệ An - 2022) Cho hình chóp S . ABCD có đáy ABCD là hình chữ nhật,
AB  a, AD  a 3 . Mặt bên SAB là tam giác đều và nằm trong mặt phẳng vuông góc với mặt
đáy. Cosin của góc giữa đường thẳng SD và mặt phẳng  SBC  bằng:
2 5 13 1 3
A. . B. . C. . D. .
5 4 4 4
Câu 3. (Cụm Trường Nghệ An - 2022) Cho lăng trụ ABC. ABC  có thể tích bằng 2. Gọi M , N lần
2
lượt là hai điểm nằm trên hai cạnh AA và BB  sao cho M là trung điểm AA và BN  BB .
3
Đường thẳng CM cắt đường thẳng AC  tại P và đường thẳng CN cắt đường thẳng BC tại Q .
a
Biết thể tích khối đa diện lồi AMPBNQ bằng với a, b  ; a, b nguyên tố cùng nhau. Tính
b
a  2b .
A. 14 . B. 31 . C. 41 . D. 32 .
Câu 4. (Cụm Trường Nghệ An - 2022) Cho tứ diện ABCD có AC  2 CD  DB  2 a . Gọi H và K
lần lượt là hình chiếu vuông góc
của A và B lên đường thẳng CD sao cho H , C , D , K theo thứ tự cách đều nhau. Biết góc tạo bởi
AH và BK bằng 60 . Thể tích khối tứ diện ABCD bằng
a3 3 a3 3 a3 3 a3 3
A. . B. . C. . D. .
6 8 3 4
Câu 5. (THPT Hồ Nghinh – Quảng Nam – 2022) Cho khối hộp chữ nhật ABCD  A BC D . Khoảng
2
cách giữa 2 đường thẳng AB , CB bằng a , khoảng cách giữa 2 đường thẳng A D, B A bằng
5
2 2
a . Khoảng cách giữa 2 đường thẳng BD , AC bằng a . Tính thể tích khối hộp chữ nhật
5 6
đã cho.
A. a 3 .
a3
B. .
2
C. 2a 3 .
D. 2a3 .
Câu 6. (THPT Hương Sơn - Hà Tĩnh - 2022) Cho tứ diện đều ABCD có tất cả các cạnh bằng 1.

Gọi M  là điểm thuộc cạnh BC  sao cho MC  2MB ; N , P 


lần lượt là trung điểm của

Facebook Nguyễn Vương https://www.facebook.com/phong.baovuong Trang 1


Blog: Nguyễn Bảo Vương: https://www.nbv.edu.vn/

BD và AD . Gọi Q  là giao điểm của AC  và  MNP  . Thể tích khối đa diện

ABMNPQ

7 2
 bằng

13 2 2 11 2
A. . B. . C. . D. .
216 432 36 432
Câu 7. (THPT Lê Thánh Tông - HCM-2022) Hình lập phương ABCD. A1 B1C1 D1 có cạnh bằng 6. Gọi
M , N lần lượt là trung điểm của cạnh B1C1 , CD và O, O1 lần lượt là tâm các hình vuông
ABCD, A1 B1C1 D1 . Thể tích khối tứ diện MNOO1 bằng
A. 9. B. 12. C. 18. D. 27.
Câu 8. (THPT Lê Thánh Tông - HCM-2022) Cho lăng trụ ABC . AB C  có tam giác ABC vuông cân
tại A . Hình chiếu vuông góc của A lên mặt đáy trùng với trung điểm của cạnh BC . Biết cạnh
AA  a 3 và tạo với mặt đáy của hình lăng trụ một góc bằng 60 . Khoảng cách từ đỉnh C  đến
mặt  ABC  bằng
3a a 3 a 2a
A. . B. . C. . D. .
4 2 2 3
Câu 9. (Liên trường Hà Tĩnh – 2022) Cho hình chóp tứ giác S . ABCD có đáy là hình vuông; mặt bên
(SAB) là tam giác vuông cân tại S và nằm trong mặt phẳng vuông góc với đáy. Biết khoảng cách
3 5a
giữa hai đường thẳng AB và SD bằng . Tính thể tích V của khối chóp S . ABCD .
5
3 3
A. V  a .
2
6 3 3
B. V  a .
2
27 3
C. V  a .
2
9
D. V  a 3 .
2
Câu 10. (THPT Nguyễn Tất Thành-Đh-SP-HN) Cho hình lăng trụ đứng ABC . AB C  có đáy ABC là
tam giác vuông tại A , AB  a , AC  2 a và AA  a (tham khảo hình vẽ bên). Tính khoảng cách
d giữa hai đường thẳng AB, AC .

Trang 2 Fanpage Nguyễn Bảo Vương  https://www.facebook.com/tracnghiemtoanthpt489/


Điện thoại: 0946798489 TUYỂN CHỌN VẬN DỤNG – VẬN DỤNG CAO 2022
2a 3a 2a 3a
A. d  . B. d  . C. d  . D. d  .
3 2 3 2
Câu 11. (THPT Nguyễn Tất Thành-Đh-SP-HN-2022) Cho hình lập phương ABCD. ABCD có cạnh
a . Gọi M , N và E lần lượt là trung điểm các cạnh AA, C D và CC (tham khảo hình vẽ bên).
Tính thể tích V của khối tứ diện BMNE .

a3 a3 a3 a3
A. V  . B. V  . C. V  . D. V  .
24 6 8 12
Câu 12. (Sở Hà Tĩnh 2022) Cho hình hộp đứng ABCD  ABCD có cạnh AA  2 , đáy ABCD là hình
thoi với ABC là tam giác đều cạnh bằng 4. Gọi M , N , P lần lượt là trung điểm của
BC , C  D, DD và Q thuộc cạnh BC sao cho QC  3QB . Thể tích của khối tứ diện MNPQ
bằng
A. 3 3 .
3 3
B. .
2
3
C. .
4
3
D. .
2
Câu 13. (Sở Thanh Hóa 2022) Cho khối chóp S. ABCD với đáy ABCD là hình bình hành có thể tích
bằng 84a 3 . Gọi M là trung điểm AB; J thuộc cạnh SC sao cho JC  2 JS ; H thuộc cạnh SD
sao cho HD  6 HS . Mặt phẳng ( MHJ ) chia khối chóp thành hai phần. Thể tích khối đạ diện của
phần chứa đỉnh S bằng
A. 17a 3 .
B. 19a 3 .
C. 24a 3 .
D. 21a 3 .
Câu 14. (Sở Hà Tĩnh 2022) Cho lăng trụ ABCD  ABC D có đáy là hình chữ nhật với
AB  6, AD  3, AC  3 và mặt phẳng  AACC  vuông góc với mặt đáy. Biêt hai mặt phẳng
3
 AACC  và  AABB  tạo với nhau góc  có tan   . Thể tích V của khối lăng trụ
4
ABCD  ABCD là
A. 12.
B. 6.
C. 8.
D. 10.

Facebook Nguyễn Vương https://www.facebook.com/phong.baovuongTrang 3


Blog: Nguyễn Bảo Vương: https://www.nbv.edu.vn/
Câu 15. (Sở Ninh Bình 2022) Cho hình chóp tứ giác đều S. ABCD . Một mặt cầu ( J ) ( J và S cùng
phía với ( ABCD ) ) tiếp xúc với ( ABCD ) tại A , đồng thời tiếp xúc ngoài với mặt cầu nội tiếp
hình chóp. Một mặt phẳng ( P ) đi qua J và BC . Gọi  là góc giữa ( P ) và ( ABCD ) . Tính
tan  biết các đường chéo của thiết diện của hình chóp cắt bởi ( P ) lần lượt cắt và vuông góc với
SA, SD .
1
A. .
4
6
B. .
6
3
C. .
6
1
D. .
2
Câu 16. (Sở Ninh Bình 2022) Cho hình chóp S. ABCD có đáy ABCD là hình bình hành, có thể tích là
V . Gọi M là trung điểm của cạnh SA, N là điểm trên cạnh SB sao cho SN  3NB . Mặt phẳng
( P ) thay đổi đi qua các điểm M , N và cắt các cạnh SC , SD lần lượt tại hai điểm phân biệt P, Q .
Tìm giá trị lớn nhất của thể tích khối chóp S .MNPQ .
V
A. .
3
27
B. V.
80
27
C. V.
40
V
D. .
6
Câu 17. (Sở Bạc Liêu 2022) Cho hình chóp S. ABCD có đáy ABCD là hình vuông, AB  1 , cạnh bên
SA  1 và vuông góc với mặt đáy  ABCD  . Kí hiệu M là điểm di động trên đoạn CD và N là
điểm di động trên đoạn CB và góc MAN   45 . Thể tích nhỏ nhất của khối chóp S . AMN là
2 1 2 1 2 1 2 1
A. . B. . C. . D. .
3 9 6 9

Câu 18. (Sở Phú Thọ 2022) Cho hình lăng trụ đứng ABCD. ABCD có đáy là hình vuông cạnh a , góc
 1 
giữa AC và mặt phẳng  ACD  bằng 30 . Gọi M là điểm sao cho AM  AB . Thể tích khối
3
tứ diện ACDM bằng
a3 a3 a3 3 a3 3
A. . B. . C. . D. .
18 3 12 3
Câu 19. (Sở Thái Nguyên 2022) Cho hình chóp tứ giác S. ABCD có đáy là nửa lục giác đều nội tiếp
3a
đường tròn đường kính AD  2a, SA   ABCD  , SA  . Khoảng cách giữa hai đường thẳng
2
BD và SC bằng
3 2a 2a 5 2a 5 2a
A. . B. . C. . D. .
4 4 12 4
Câu 20. (Sở Thái Nguyên 2022) Cho hình chóp S . ABCD có đáy là hình thang vuông tại A và D ,
AB  AD  a , CD  2 a . Hình chiếu của đỉnh S lên mặt  ABCD  trùng với trung điểm của BD .

Trang 4 Fanpage Nguyễn Bảo Vương  https://www.facebook.com/tracnghiemtoanthpt489/


Điện thoại: 0946798489 TUYỂN CHỌN VẬN DỤNG – VẬN DỤNG CAO 2022
3
2a
Biết thể tích khối chóp S . ABCD bằng . Khoảng cách từ điểm A đến mặt phẳng  SBC 
2
bằng
5a 5a 10a 10a
A. . B. . C. . D. .
2 5 5 2
Câu 21. (Sở Thái Nguyên 2022) Cho hình chóp S . ABC có đáy ABC là tam giác vuông tại B ,
  30 . Đỉnh S cách đều ba điểm A, B , C và cạnh bên SB tạo với mặt phẳng
AB  6 3, CAB
 ABC  một góc 45 . Hai điểm M , Q lần lượt thuộc các đoạn AB và SB sao cho
AM  2 MB, QB  2QS . Mặt phẳng   chứa M , Q và song song với đường thẳng BC chia khối
chóp S . ABC thành hai khối đa diện có thể tích lần lượt là V1 , V2 V1  V2  . Giá trị của V2 là
A. 22 3 . B. 20 3 . C. 24 3 . D. 26 3 .
Câu 22. (Sở Vĩnh Phúc 2022) Cho hình chóp S. ABCD có đáy ABCD là hình chữ nhật
AB  a 3, SA  SB  SC  SD  2a . Giá trị lớn nhất của thể tích khối chóp S. ABCD bằng:
13 3 13 2 3 13 6 3 13 3 3
A. a . B. a . C. a . D. a .
12 12 12 12
Câu 23. (Sở Vĩnh Phúc 2022) Cho hình chóp S. ABCD có đáy ABCD là hình thoi, có
AC  a 3, 
ABC  600 . Biết rằng SA  SC , SB  SD và khoảng cách từ A mặt phẳng  SBC 
a 6
bằng . Tính thể tích khối chóp S. ABC bằng:
2
3 6a3 9 6a 3 3 15a 3 3 6a 3
A. . B. . C. . D. .
8 16 40 16
Câu 24. (Chuyên Lam Sơn 2022) Trên cạnh AD của hình vuông ABCD cạnh 1, người ta lấy điểm M
sao cho AM  x (0  x  1) và trên nửa đường thẳng Ax vuông góc với mặt phẳng chứa hình
vuông, người ta lấy điểm S vói SA  y thỏa mãn y  0 và x 2  y 2  1 . Biết khi M thay đổi trên
m
đoạn AD thì thể tích của khối chóp S. ABCM đạt giá trị lớn nhất bằng với m, n  * và
n
m, n nguyên tố cùng nhau. Tính T  m  n .
A. 11.
B. 17.
C. 27.
D. 35.
Câu 25. (Chuyên Lam Sơn 2022) Cho khối bát diện đều có cạnh a . Gọi M , N , P , Q lần lượt là trọng tâm
của các tam giác SAB , SBC , SCD, SDA; gọi M , N , P, Q lần lượt là trọng tâm của các tam giác
S  AB , S  BC , S CD , S  DA (như hình vẽ dưới).

Facebook Nguyễn Vương https://www.facebook.com/phong.baovuongTrang 5


Blog: Nguyễn Bảo Vương: https://www.nbv.edu.vn/

Thể tích của khối lăng trụ MNPQ  M  N  PQ là


2a 3 2 2a 3 2a 3 2 2a 3
A. . B. . C. . D. .
72 81 24 27
Câu 26. (Chuyên Lương Văn Tụy – Ninh Bình 2022) Cho hình chóp S.ABCD có đáy ABCD là hình
vuông, AB  1 , cạnh bên SA  1 và vuông góc với mặt phẳng đáy ( ABCD ) . Kí hiệu M là điểm
di động trên đoạn CD và N là điểm di động trên đoạn CB sao cho MAN  45 . Thể tích nhỏ
nhất của khối chóp S  AMN là
2 1
A. .
3
2 1
B. .
9
2 1
C. .
6
2 1
D. .
9
Câu 27. (Chuyên Lương Văn Tụy-Ninh Bình 2022) Bạn A định làm một cái hộp quà lưu niệm (không
nắp) bằng cách cắt từ một tấm bìa hình tròn bán kính 4 cm để tạo thành một khối lăng trụ lục giác
đều, biết 6 hình chữ nhật có các kích thước là 1cm và xcm (tham khảo hình vẽ). Thể tích của hộp
quà gần nhất với giá trị nào sau đây?

Trang 6 Fanpage Nguyễn Bảo Vương  https://www.facebook.com/tracnghiemtoanthpt489/


Điện thoại: 0946798489 TUYỂN CHỌN VẬN DỤNG – VẬN DỤNG CAO 2022

A. 24,5 cm3 .
B. 25 cm 3 .
C. 25, 5 cm 3 .
D. 24 cm 3 .
Câu 28. (THPT Trần Phú – Hà Tĩnh – 2022) Cho khối chóp S  ABCD , có đáy là hình chứ nhật cạnh
AB  2a 5 và tất cả các cạnh bên của hình chóp bằng 5a . Thể tích lớn nhất của khối chóp đã
cho bằng
20a 3 5
A. .
3
8a 3
B. .
3
40 5a 3
C. .
3
D. 15 5a 3 .
Câu 29. (THPT Kinh Môn - Hải Dương - 2022) Người ta dùng thuỷ tinh trong suốt để làm một cái chặn
giấy hình tứ diện đều. Để trang trí cho nó, người thiết kế đặt trong khối tứ diện 4 quả cầu nhựa
màu xanh có bán kính bằng nhau là r  2( cm) . Biết rằng 4 quả cầu này đôi một tiếp xúc với
nhau và mỗi mặt của tứ diện tiếp xúc với 3 quả cầu, đồng thời không cắt quả cầu còn lại. Nếu bỏ
qua bề dày của các mặt thì người ta cần dùng bao nhiêu thuỳ tinh để làm chặn giấy trên (làm tròn
đến chữ số thập phân thứ 2).
A. 195, 66  cm3  . B. 62, 09  cm3  . C. 30, 03  cm3  . D. 65,55  cm3  .

Câu 30. (THPT Kinh Môn - Hải Dương - 2022) Cho tam giác ABC đều cạnh a , gọi d là đường thẳng
qua A và vuông góc với mặt phẳng  ABC  . Trên d lấy điểm S và đặt AS  x  x  0  . Gọi H
và K lần lượt là trực tâm của các tam giác ABC và SBC . Biết HK cắt d tại điểm S  . Khi SS 
ngắn nhất thì khối chóp S. ABC có thể tích bằng
a3 6 a3 3 a3 2 a3 6
A. . B. . C. . D. .
6 8 27 24

Facebook Nguyễn Vương https://www.facebook.com/phong.baovuongTrang 7


Blog: Nguyễn Bảo Vương: https://www.nbv.edu.vn/
Câu 31. (THPT Lương Tài 2 - Bắc Ninh - 2022) Cho lăng trụ tam giác đều ABC. ABC  có cạnh đáy
bằng 4a . Góc giữa hai mặt phẳng  ABC  và  ABC  bằng 30 0 . Gọi M là trung điểm của cạnh
AB , tính khoảng cách từ điểm M tới mặt phẳng  ABC  ?
a 3 3a
A. . B. 3a . C. a 3 . D. .
2 2
Câu 32. (THPT Võ Nguyên Giáp - Quảng Bình - 2022) Cho hình lăng trụ đứng ABC. ABC có đáy
ABC là tam giác đều. Gọi  là góc tạo bởi AB với mặt phẳng  ACC A  và  là góc giữa mặt
m m
phẳng  ABC   với mặt phẳng  ACC A  . Biết cot 2   cot 2   (với m, n  N* và phân số
n n
tối giản). Khi đó, giá trị của biểu thức T  m  2n bằng
A. 3 . B. 5 . C. 7 . D. 9 .
Câu 33. (THPT Võ Nguyên Giáp - Quảng Bình - 2022) Cho hình hộp đứng ABCD. ABCD có đáy
ABCD là hình vuông. Gọi S là tâm hình vuông ABCD . Gọi M và N lần lượt là trung điểm
của SA và BC . Biết rằng, nếu MN tạo với mặt phẳng  ABCD  một góc 60 và AB  a thì thể
tích S. ABC bằng
a3 30 a3 30 3 a3 3
A. . B. . C. a 30 . D. .
12 3 2
Câu 34. (THPT Yên Lạc - Vĩnh Phúc - 2022) Cho hình chóp S. ABCD , có đáy ABCD là hình vuông,
SAB là tam giác đều và nằm trong mặt phẳng vuống góc với mặt đáy. Biết khoảng cách giữa hai
2a 21
đường thẳng AB và SD bằng . Thế tích của khối chóp S. ABCD bằng
7
2a 3 3 4a 3 3 8a 3 3 a3 3
A. . B. . C. . D. .
3 3 3 3
Câu 35. (THPT Yên Lạc - Vĩnh Phúc - 2022) Một trang tại cần xây dựng một bể chứa nước hình hộp
chữ nhật bằng gạch không nắp ở phía trên. Biết bể có chiều dài gấp hai lần chiều rộng và thể tích
(phần chứa nước) bằng 8 m 3 . Hỏi chiều cao của bể gần nhất với kết quả nào dưới đây để số lượng
gạch dùng để xây bể là nhỏ nhất
A. 1,8 m . B. 1,3m . C. 1,1m . D. 1, 2 m .

Câu 36. (THPT Yên Phong 1 - Bắc Ninh - 2022) Cho hình chóp S . ABCD có đáy ABCD là hình bình
hành. Gọi M là điểm di động trên cạnh AB và N là trung điểm SD . Mặt phẳng   đi qua
V1 3
M , N và song song BC chia khối chóp thành hai khối có tỉ lệ thể tích  , trong đó V1 là thể
V2 5
AM
tích khối đa diện chứa đỉnh A , V2 là thể tích khối đa diện chứa đỉnh B . Tỉ số bằng
AB
1 1 3 3
A. . B. . C. . D. .
3 2 5 7

Theo dõi Fanpage: Nguyễn Bảo Vương  https://www.facebook.com/tracnghiemtoanthpt489/

Hoặc Facebook: Nguyễn Vương  https://www.facebook.com/phong.baovuong

Tham gia ngay: Nhóm Nguyễn Bào Vương (TÀI LIỆU TOÁN)  https://www.facebook.com/groups/703546230477890/

Ấn sub kênh Youtube: Nguyễn Vương


 https://www.youtube.com/channel/UCQ4u2J5gIEI1iRUbT3nwJfA?view_as=subscriber

Tải nhiều tài liệu hơn tại: https://www.nbv.edu.vn/


Trang 8 Fanpage Nguyễn Bảo Vương  https://www.facebook.com/tracnghiemtoanthpt489/
Điện thoại: 0946798489 TUYỂN CHỌN VẬN DỤNG – VẬN DỤNG CAO 2022

Facebook Nguyễn Vương https://www.facebook.com/phong.baovuongTrang 9


TUYỂN CHỌN VẬN DỤNG – VẬN DỤNG CAO 2022 Điện thoại: 0946798489

VẤN ĐỀ 6. KHỐI TRÒN XOAY


• |FanPage: Nguyễn Bảo Vương
• TUYỂN CHỌN CÂU HỎI VD-VDC TỪ CÁC ĐỀ THI THỬ CÁC TRƯỜNG, CÁC SỞ NĂM 2022
Câu 1. (Cụm Trường Nghệ An - 2022) Cắt hình nón  N  bởi mặt phẳng đi qua đỉnh S và tạo với
trục của  N  một góc bằng 30 , ta được thiết diện là tam giác SAB vuông và có diện tích bằng 4a 2 .Chiều
cao của hình nón bằng
A. a 3 . B. 2 a 3 . C. 2 a 2 . D. a 2 .
Câu 2. (Đại học Hồng Đức – 2022) Cho hình nón đỉnh S có độ dài đường cao là R và đáy là đường
tròn tâm O bán kính R . Gọi ( d ) là tiếp tuyến của đường tròn đáy tại A và ( P ) là mặt phẳng
chứa SA và (d ) . Mặt phẳng (Q ) thay đổi qua S cắt đường tròn O tại hai điểm C , D sao cho
CD  3R . Gọi  là góc tạo bởi ( P ) và (Q ) . Tính giá trị lớn nhất của cos  .
3 10
A. .
10
10
B. .
5
2 6
C. .
5
10
D. .
10
Câu 3. (THPT Hương Sơn - Hà Tĩnh - 2022) Một chiếc kem Ốc quế gồm 2 phần, phần dưới là một
khối nón có chiều cao bằng ba lần đường kính đáy, phần trên là nửa khối cầu có đường kính bằng
đường kính khối nón bên dưới (như hình vẽ). Thể tích phần kem phía trên bằng 50cm3 . Thể tích
của cả chiếc kem bằng

A. 200cm3 . B. 150cm3 . C. 125cm3 . D. 500cm3 .


Câu 4. (THPT Lê Thánh Tông - HCM-2022) Cho hình trụ tròn xoay có hai đáy là hai hình tròn
 I ; 7  và  J ; 7 . Biết rằng tồn tại dây cung EF của đường tròn  I ; 7  sao cho tam giác
JEF là tam giác đều và mặt phẳng  JEF  hợp với mặt đáy của hình trụ một góc bằng 60. Thể
tích V của khối trụ đã cho là
A. V  21 . B. V  7 6 . C. V  14 . D. V  28 .

Câu 5. (Liên trường Hà Tĩnh – 2022) Cho hình nón có chiều cao bằng 2 5 . Một mặt phẳng đi qua
đỉnh hình nón và cắt hình nón theo một thiết diện là tam giác đều có diện tích bằng 9 3 . Thể tích
của khối nón được giới hạn bởi hình nón đã cho bằng
32 5
A. .
3
B. 32 .
C. 32 5 .

Facebook Nguyễn Vương https://www.facebook.com/phong.baovuong Trang 1


Blog: Nguyễn Bảo Vương: https://www.nbv.edu.vn/
18 5
D. .
3
Câu 6. (THPT Nguyễn Tất Thành-Đh-SP-HN-2022) Cho hình chóp S. ABCD có đáy ABCD là hình
chữ nhật, AB  2, AD  1 . Gọi M là trung điểm của DC . Biết SA   ABCD  và SA  2 . Tính
bán kính mặt cầu ngoại tiếp hình chóp S.BCM .

3 3 11 13
A. R  3 . B. R  . C. R  . D. R  .
2 2 2
Câu 7. (THPT Nho Quan A – Ninh Bình – 2022) Cho hình nón đỉnh S có đường cao h  a 3 . Một
mặt phẳng ( ) đi qua đỉnh S , cắt đường tròn đáy tại hai điểm A, B sao cho AB  8a và tạo với
mặt đáy một góc 30 . Tính diện tích xung quanh của hình nón.
10 7 2
A. a .
3
B. 20 7 a 2 .
C. 10 7 a 2
D. 5 7 a 2 .
Câu 8. (THPT Phù Cừ - Hưng Yên - 2022) Một tấm tôn hình tam giác ABC có độ dài cạnh
AB  3; AC  2; BC  19 . Điểm H là chân đường cao kẻ từ đình A của tam giác ABC . Người
ta dùng compa có tâm là A , bán kính AH vạch một cung tròn MN . Lấy phần hình quạt gỏ thành
hình nón không có mặt đáy với đỉnh là A , cung MN thành đường tròn đáy của hình nón (nhuc
hình vẽ). Tính thể tich khối nón trên.
2 114 2 3  57 2 19
A. . B. . C. . D. .
361 19 361 361
Câu 9. (Sở Bạc Liêu 2022) Trên bàn có một cốc nước hình trụ chứa đầy nước, có chiều cao bằng 3 lần
đường kính của đáy; một viên bi và một khối nón đều bằng thủy tinh. Biết viên bi là một khối cầu
có đường kính bằng đường kính của đường tròn đáy cốc nước. Người ta từ từ thả vào cốc nước
viên bi và khối nón sao cho đỉnh khối nốn nằm trên mặt cầu (như hình vẽ) thì thấy nước trong cốc
tràn ra ngoài. Tính tỉ số của lượng nước còn lại trong cốc và lượng nước ban đầu.

Trang 2 Fanpage Nguyễn Bảo Vương  https://www.facebook.com/tracnghiemtoanthpt489/


Điện thoại: 0946798489 TUYỂN CHỌN VẬN DỤNG – VẬN DỤNG CAO 2022

4 5 2 1
A. . B. . C. . D. .
9 9 3 2
Câu 10. (Sở Hà Tĩnh 2022) Cắt hình nón  N  bởi mặt phẳng đi qua đỉnh  S  và tạo bởi với trục của
N một góc bằng 30 ta được thiết diện là tam giác SAB vuông và có diện tích 4a 2 . Chiều cao
của hình nón bằng:
A. a 3 . B. 2a 3 . C. 2a 2 . D. a 2 .
Câu 11. (Sở Lạng Sơn 2022) Một cái cột có hình dạng như hình bên (gồm một khối nón và một khối trụ
ghép lại). Chiều cao đo được ghi trên hình, chu vi đáy là 20cm .

Thể tích của cột bằng


52000 5000 5000 13000
A.
3
 cm 3  . B.
3
 cm 3  . C.

 cm 3  . D.
3
 cm 3  .
Câu 12. (Sở Phú Thọ 2022) Cho hình trụ có bán kính đáy bằng a 3 . Cắt hình trụ bởi một mặt phẳng
song song với trục, cách trục một khoảng bằng a ta được thiết diện là một hình vuông. Thể tích
khối trụ đó bằng:
A. 2 a 3 2 . B. 4 a 3 2 . C. 6 a 3 2 . D. 3 a 3 2 .
Câu 13. (Sở Phú Thọ 2022) Cho hình nón  có chiều cao bằng 2a . Cắt  bởi một mặt phẳng đi qua
4a 2 11
đỉnh và cách tâm của đáy một khoảng bằng a ta được thiết diện có diện tích bằng . Thể
3
tích của khối nón đã cho bằng
10πa 3 4πa 3 5 4πa 3 5
A. . B. 10πa3 . C. . D. .
3 3 9
Câu 14. (Sở Thái Nguyên 2022) Cho một dụng cụ đựng chất lỏng như hình 1 có phần trên là mặt xung
quanh và đáy của hình trụ, phần dưới là mặt xung quanh của hình nón. Biết hình trụ có cùng bán
kính đáy R và cùng chiều cao h  24cm với hình nón. Trong hình 1, lượng chất lỏng có chiều cao
bằng 12cm . Lật ngược dụng cụ theo phương vuông góc với mặt đất như hình 2. Khi đó chiều cao
của chất lỏng trong hình 2 là

Facebook Nguyễn Vương https://www.facebook.com/phong.baovuongTrang 3


Blog: Nguyễn Bảo Vương: https://www.nbv.edu.vn/

A. 3cm . B. 2cm . C. 1cm . D. 4cm .

Câu 15. (Sở Thái Nguyên 2022) Cho tứ diện ABCD có AB  BC  CD  2 3 , AC  BD  2 ,


AD  2 2 . Diện tích mặt cầu ngoại tiếp tứ diện đã cho bằng
40 10
A. 6 . B. 24 . C. . D. .
3 3
Câu 16. (Sở Vĩnh Phúc 2022) Cho một hình nón đỉnh S có đáy là đường tròn tâm O , bán kính R  5
2
và góc ở đỉnh là 2 với sin   . Một mặt phẳng  P  vuông góc với SO tại H và cắt hình
3
nón theo một đường tròn tâm H . Gọi V là thể tích của khối nón đỉnh O và đáy là đường tròn
50 a a
tâm H . Biết V  khi SH  với a, b* và là phân số tối giản. Tính giá trị của biểu
81 b b
thức T  3a 2  2b 3 .
A. 12 . B. 23 . C. 21 . D. 32 .
Câu 17. (Sở Vĩnh Phúc 2022) Cho một hình nón có bán kính đáy bằng a . Mặt phẳng  P  đi qua đỉnh S
của hình nón cắt đường tròn đáy tại A và B sao cho AB  a 3 , khoảng cách từ tâm đường tròn
a 2
đáy đến mặt phẳng  P  bằng . Thể tích khối nón đã cho bằng
4
 a3  a3  a3  a3
A. . B. . C. . D. .
12 6 3 24
Câu 18. (Sở Vĩnh Phúc 2022) Cho hình chóp S . ABC có đáy ABC là tam giác đều cạnh 2a , cạnh bên
SA vuông góc với mặt phẳng đáy, góc giữa hai mặt phẳng  SBC  và  ABC  bằng 45 . Diện tích
mặt cầu ngoại tiếp hình chóp S . ABC bằng
25 a 2 25 a 2 25 a 2 25 a 2
A. . B. . C. . D. .
12 3 9 6
Câu 19. (Chuyên Hạ Long 2022) Cho hình trụ tròn xoay có hai đáy là hai hình tròn (O; R ) và  O; R  .
Tồn tại dây cung AB thuộc đường tròn (O ) sao cho O AB là tam giác đều và mặt phẳng
 O AB  hợp với mặt phẳng chứa đường tròn (O) một góc 60 . Khi đó diện tích xung quanh S xq
hình trụ là
4 R 2
A. S xq  .
7
3 R 2
B. S xq  .
7
Trang 4 Fanpage Nguyễn Bảo Vương  https://www.facebook.com/tracnghiemtoanthpt489/
Điện thoại: 0946798489 TUYỂN CHỌN VẬN DỤNG – VẬN DỤNG CAO 2022
2
3 R 7
C. S xq  .
7
6 R 2 7
D. S xq  .
7
Câu 20. (Chuyên Hạ Long 2022) Cho hình chóp S  ABCD có đáy ABCD là hình vuông cạnh
a, SA  a 7 và vuông góc với đáy. Lấy điểm M trên cạnh SC sao cho CM  a . Gọi (C ) là
hình nón có đỉnh C , các điểm B, M , D thuộc mặt xung quanh, điểm A thuộc mặt đáy của hình
nón. Tính diện tích xung quanh của (C ) .
16 7
A.  a2 .
15
8 30 2
B. a .
15
32 2 2
C. a .
15
16 3 2
D. a .
9
Câu 21. (Chuyên Lam Sơn 2022) Một cái bình thủy tinh có phần không gian bên trong là một hình nón
có đỉnh hướng xuống dưới theo chiều thẳng đứng. Rót nước vào bình cho đến khi phần không
gian trống trong bình có chiều cao 2 cm . Sau đó đậy kín miệng bình bởi một cái nắp phẳng và lật
ngược bình để đỉnh hướng lên trên theo chiều thẳng đứng, khi đó mực nước cao cách đỉnh của nón
8 cm (hình vẽ minh họa bên dướí).

Biết chiều cao của nón là h  a  b cm . Tính T  a  b .


A. 22.
B. 58.
C. 86.
D. 72.
Câu 22. (Chuyên Nguyễn Trãi – Hải Dương – 2022) Nhân dịp năm mới để trang trí một cây thông Noel,
ở sân trung tâm có hình nón ( N ) như hình vẽ sau. Người ta cuộn quanh cây bằng một sợi dây đèn
LED nhấp nháy, bóng đèn hình hoa tuyết từ điểm A đến điểm M sao cho sợi dây luôn tựa trên
mặt nón. Biết rằng bán kính đáy hình nón bằng 8m , độ dài đường sinh bằng 24m và M là điểm
  
sao cho 2MS  MA  0 . Hãy tính chiều dài nhỏ nhất của sợi dây đèn cần có.

Facebook Nguyễn Vương https://www.facebook.com/phong.baovuongTrang 5


Blog: Nguyễn Bảo Vương: https://www.nbv.edu.vn/

A. 8 19(m) .
B. 8 13( m) .
C. 8 7 (m) .
D. 9 12(m) .
Câu 23. (THPT Kim Liên - Hà Nội - 2022) Cắt hình trụ T  có bán kính R bởi một mặt phẳng song
song với trục và cách trục một khoảng bằng a  0  a  R  ta được thiết diện là một hình vuông có
diện tích 16a 2 . Diện tích xung quanh của hình trụ T  bằng
A. 4 a 2 5 . B.  a 2 5 . C. 8 a 2 5 . D. 16 a 2 5 .
a 3
Câu 24. (THPT Võ Nguyên Giáp - Quảng Bình - 2022) Cho tứ diện ABCD có AB  và các cạnh
2
a m
còn lại đều bằng a . Biết rằng bán kính mặt cầu ngoại tiếp tứ diện ABCD bằng với
n
m, n* ; m  15 . Tổng T  m  n bằng
A. 15. B. 17. C. 19. D. 21.
Câu 25. (THPT Yên Lạc - Vĩnh Phúc - 2022) Cho hình trụ T  chiều cao bằng 2a , hai đường tròn đáy
của  T  có tâm lần lượt là O và O1 , bán kính bằng a . Trên đường tròn đáy tâm O lấy điểm A ,
trên đường tròn đáy tâm O1 lấy điểm B sao cho AB  5a . Thể tích khối tứ diện OO1 AB bằng
3a 3 3a 3 3a 3 3a 3
A. . B. . C. . D. .
12 3 4 6
Câu 26. (THPT Yên Lạc - Vĩnh Phúc - 2022) Một khối nón có bán kính đáy bằng 2 cm , chiều cao bằng
3 cm . Một mặt phẳng đi qua đỉnh và tạo với đáy một góc 60 chia khối nón làm 2 phần. Tính
thể tích phần nhỏ hơn (kết quả làm tròn đến hàng phần trăm).
A. 2, 47 cm3 . B. 2,36 cm3 . C. 1, 42 cm 3 . D. 1, 53 cm 3 .

Câu 27. (THPT Yên Phong 1 - Bắc Ninh - 2022) Cho hình nón đỉnh S tâm O có độ dài đường sinh
bằng SA  a , đường kính đáy AB . Thiết diện qua đỉnh tạo với đáy một góc 60 cắt đường tròn

Trang 6 Fanpage Nguyễn Bảo Vương  https://www.facebook.com/tracnghiemtoanthpt489/


Điện thoại: 0946798489 TUYỂN CHỌN VẬN DỤNG – VẬN DỤNG CAO 2022
2a 3
đáy theo dây cung MN  . Biết rằng khoảng cách từ A đến MN bằng a . Thể tích khối nón
3
bằng:
a 3 2 a3 6 a 3 6 a 3 6
A. . B. . C. . D. .
12 18 9 3

Theo dõi Fanpage: Nguyễn Bảo Vương  https://www.facebook.com/tracnghiemtoanthpt489/

Hoặc Facebook: Nguyễn Vương  https://www.facebook.com/phong.baovuong

Tham gia ngay: Nhóm Nguyễn Bào Vương (TÀI LIỆU TOÁN)  https://www.facebook.com/groups/703546230477890/

Ấn sub kênh Youtube: Nguyễn Vương


 https://www.youtube.com/channel/UCQ4u2J5gIEI1iRUbT3nwJfA?view_as=subscriber

Tải nhiều tài liệu hơn tại: https://www.nbv.edu.vn/

Facebook Nguyễn Vương https://www.facebook.com/phong.baovuongTrang 7


TUYỂN CHỌN VẬN DỤNG – VẬN DỤNG CAO 2022 Điện thoại: 0946798489

VẤN ĐỀ 7. OXYZ
• |FanPage: Nguyễn Bảo Vương
• TUYỂN CHỌN CÂU HỎI VD-VDC TỪ CÁC ĐỀ THI THỬ CÁC TRƯỜNG, CÁC SỞ NĂM 2022

Câu 1. (Chuyên Vinh – 2022) Trong không gian Oxyz , cho mặt phẳng ( P ) : 2 x  y  2 z  16  0 và mặt
cầu ( S ) : ( x  2)2  ( y  1) 2  ( z  3) 2  21 . Một khối hộp chữ nhật ( H ) có bốn đỉnh nằm trên mặt
phẳng ( P ) và bốn đỉnh còn lại nằm trên mặt cầu ( S ) . Khi ( H ) có thể tích lớn nhất, thì mặt phẳng
chứa bốn đỉnh của ( H ) nằm trên mặt cầu ( S ) là (Q ) : 2 x  by  cz  d  0 . Giá trị b  c  d bằng
A. 15 .
B. 13 .
C. 14 .
D. 7 .
Câu 2. (THPT Kim Liên – Hà Nội – 2022) Trong không gian Oxyz , cho mặt cầu
( S ) : ( x  1)2  ( y  3) 2  ( z  2)2  25 và hai đường thẳng  d1  ,  d2  lần lượt có phương trình
 x  1  mt x  1 t
 
d1 :  y  1 và d 2 :  y  1 . Có bao nhiêu giá trị của m để  d1  ,  d 2  cắt mặt cầu ( S ) tại 4
z  1 t  z  1  mt
 
điểm phân biệt sao cho bốn điểm đó tạo thành tứ giác có diện tích lớn nhất
A. 0.
B. 3.
C. 2.
D. 1.
Câu 3. (Chuyên Vinh – 2022) Trong không gian Oxyz , cho mặt cầu
( S ) : x 2  y 2  z 2  4 x  12 y  6 z  24  0 . Hai điểm M , N thuộc ( S ) sao cho MN  8 và
OM 2  ON 2  112 . Khoảng cách từ O đến đường thẳng MN bằng
A. 4.
B. 3.
C. 2 3 .
D. 3 .
Câu 4. (Chuyên Lê Quý Đôn - Điện Biên - 2022) Trong không gian với hệ tọa độ Oxyz , cho mặt cầu
2 2 2
 S  :  x  1   y  3   z  4   5 và điểm M 1;4; 2  . Xét điểm N thuộc mặt cầu  S  sao
cho đường thẳng MN tiếp xúc với mặt cầu  S  . Khi đó điểm N luôn nằm trên mặt phẳng có
phương trình là:
A. 2 x  y  z  2  0 . B. x  y  z  1  0 .
C. 2 x  y  2 z  2  0 . D. 2 x  y  2 z  2  0 .
Câu 5. (Chuyên Lê Quý Đôn - Điện Biên - 2022) Trong không gian với hệ trục tọa độ Oxyz , cho điểm
A  a; b; c  với a; b; c là các số thực dương thỏa mãn 5  a 2  b 2  c 2   9  ab  2bc  ca  và
a 1
Q  có giá trị lớn nhất. Gọi M , N , P lần lượt là hình chiếu vuông góc của
b  c  a  b  c 3
2 2

A lên các tia Ox, Oy , Oz . Phương trình mặt phẳng  MNP  là


A. 3 x  12 y  12 z  1  0 .B. x  4 y  4 z  12  0 .
C. 3 x  12 y  12 z  1  0 .D. x  4 y  4 z  0 .

Facebook Nguyễn Vương https://www.facebook.com/phong.baovuong Trang 1


Blog: Nguyễn Bảo Vương: https://www.nbv.edu.vn/
Câu 6. (Cụm Trường Nghệ An - 2022) Trong không gian Oxyz , cho ba điểm A(1; 0; 0) , B (0; 2;3) ,
2
C (1;1;1) . Gọi ( P ) là mặt phẳng chứa A , B sao cho khoảng cách từ C đến ( P ) bằng . Tìm
3
tọa độ giao điểm M của ( P ) và trục Oy .
 23   23 
A. M (0; 1;0) hoặc M  0; ;0  . B. M (0;1; 0) hoặc M  0;  ; 0  .
 37   37 
 23   23 
C. M (0; 1; 0) hoặc M  0;  ; 0  . D. M (0;1; 0) hoặc M  0;  ; 0  .
 37   37 
Câu 7. (Cụm Trường Nghệ An - 2022) Trong không gian Oxyz , cho các điểm A  0;0;3 và
B  2; 3; 5 . Gọi  P là mặt phẳng chứa đường tròn giao tuyến của hai mặt cầu
2 2 2
 S1  :  x  1   y  1   z  3  25 và  S2  : x2  y2  z 2  2x  2 y 14  0 . Gọi M , N là hai
điểm thuộc  P sao cho MN  1 . Biết giá trị nhỏ nhất của AM  BN có dạng a b c
( a , b , c   và c là số nguyên tố). Tính a  b  c
A. 80 . B. 93 . C. 89 . D. 90 .
Câu 8. (Đại học Hồng Đức – 2022) Trong không gian Oxyz , cho hai đường thẳng
x  1 y 1 z  2 x 1 y  3 z 1
 d1  :   ,  d2  :   và điểm A(4;1; 2) . Gọi  là đường thẳng
2 1 2 1 2 3

qua A cắt d1 và cách d 2 một khoảng lớn nhất. Lấy u  (a;1; c ) là một véctơ chỉ phương của  .

Độ dài của u là
A. 3 5 .
B. 86 .
C. 3 .
D. 85 .
Câu 9. (THPT Hồ Nghinh – Quảng Nam – 2022) Trong không gian với hệ trục tọa độ Oxyz , cho ba
mặt phẳng ( P ) : x  y  z  5  0 ; (Q ) : x  y  z  1  0 và ( R ) : x  y  z  2  0 . Úng với mỗi cặp
điểm A, B lần lượt thuộc hai mặt phẳng ( P ), (Q ) thì mặt cầu đường kinh AB luôn cắt mặt phẳng
( R ) theo một đường tròn. Tìm bán kính nhỏ nhất của đường tròn đó.
1
A. .
3
2
B. .
3
C. 1.
1
D. .
2
Câu 10. (THPT Hương Sơn - Hà Tĩnh - 2022) Trong không gian với hệ trục tọa độ Oxyz , cho đường
x  2 y 1 z 2 2 2
thẳng d :   và mặt cầu  S  :  x  2    y  1   z  1  6 . Hai mặt phẳng
2 3 1
   
P , Q chứa d và cùng tiếp xúc với  S  lần lượt tại A, B . Gọi I tà tâm mặt cầu  S  . Giá trị
cos 
AIB bằng
1 1 1 1
A.  . B. . C.  . D. .
9 9 3 3

Trang 2 Fanpage Nguyễn Bảo Vương  https://www.facebook.com/tracnghiemtoanthpt489/


Điện thoại: 0946798489 TUYỂN CHỌN VẬN DỤNG – VẬN DỤNG CAO 2022
Câu 11. (THPT Hương Sơn - Hà Tĩnh - 2022) Trong không gian với hệ toạ độ Oxyz , cho mặt phẳng
x y2 z2
   : x  y  2 z  2  0 và đường thẳng  :    Đường thẳng   là hình chiếu vuông
2 2 1
góc của đường thẳng  trên mặt phẳng    có phương trình:
x8 y 6 z  2 x 8 y 6 z  2
A.   . B.   .
3 5 4 3 5 4
x 1 y 1 z 1 x 1 y 1 z 1
C.   . D.   .
7 5 1 7 5 1
Câu 12. (THPT Lê Thánh Tông - HCM-2022) Trong không gian với hệ trục tọa độ Oxyz cho hai mặt
2
cầu  S1  :  x  5   y 2  z 2  25 ,
2
 S2  :  x  5   y 2  z 2  100 và điểm K  8;0;0  . Đường thẳng  di động nhưng luôn tiếp xúc
với  S1  , đồng thời cắt  S2  tại hai điểm M , N . Tam giác KMN có diện tích lớn nhất bằng
A. 90 3 . B. 50 6 . C. 100 2 . D. 100 3 .

Câu 13. (Liên trường Hà Tĩnh-2022) Trong không gian với hệ trục tọa độ Oxyz cho
A( a;0;0), B (0; b;0), C (0;0; c) với a, b, c  0 sao cho 2OA  OB  OC  5 OB 2  OC 2  36 . Tính
a  b  c khi thể tích khối chóp O. ABC đạt giá trị lớn nhất
A. 1.
B. 5
36  36 2
C.
5
D. 7
Câu 14. (THPT Nguyễn Tất Thành-Đh-SP-HN-2022) Trong không gian tọa độ Oxyz , cho đường thẳng
x 1 y 1 z 1
d:   và hai điểm A 6;0;0 , B  0;0; 6 . Khi M thay đổi trên đường thẳng d ,
2 2 1
hãy tìm giá trị nhỏ nhất của biểu thức P  MA  MB
A. min P  6 3 . B. min P  6 2 . C. min P  9 . D. min P  12 .
Câu 15. (THPT Nguyễn Tất Thành-Đh-SP-HN-2022) Trong không gian tọa độ Oxyz cho mặt phẳng
 P  : x  2 y  2 z  11  0 và điểm I  3;3;1 . Gọi  S  là mặt cầu có tâm là điểm I và cắt mặt
phẳng  P  theo một đường tròn có chu vi bằng 8 . Phương trình của mặt cầu  S  là
2 2 2 2 2 2
A.  x  3    y  3    z  1  52 . B.  x  3    y  3    z  1  64 .

2 2 2 2 2 2
C.  x  3    y  3    z  1  64 . D.  x  3    y  3    z  1  52 .

Câu 16. (THPT Nguyễn Tất Thành-Đh-SP-HN-2022) Trong không gian tọa độ cho đường thẳng
x 1 y  2 z
d:   và hai điểm A 1; 1;1 , B  4; 2; 2  . Gọi  là đường thẳng đi qua A và
2 2 1
vuông góc với d sao cho khoảng cách từ điểm B đến  là nhỏ nhất. Phương trình của đường
thẳng  là
x 1 y 1 z 1 x 1 y 1 z 1
A.   . B.   .
1 1 4 1 1 4

x 1 y 1 z 1 x 1 y 1 z 1
C.   . D.   .
1 1 4 1 1 4

Facebook Nguyễn Vương https://www.facebook.com/phong.baovuongTrang 3


Blog: Nguyễn Bảo Vương: https://www.nbv.edu.vn/
Câu 17. (Nho Quan A – Ninh Bình – 2022) Trong không gian với hệ tọa độ Oxyz , xét ba điểm
1 1 1
A( a; 0; 0), B (0; b; 0), C (0; 0; c ) thỏa mãn    1 . Biết rằng mặt cầu
a b c
2 2 2
( S ) : ( x  2)  ( y  1)  ( z  3)  25 cắt mặt phẳng ( ABC ) theo giao tuyến là đường tròn có bán
kính là 4. Giá trị của biểu thức a  b  c là
A. 1.
B. 2.
C. 3.
D. 5.
x  1 t

Câu 18. (THPT Phù Cừ - Hưng Yên - 2022) Trong không gian Oxyz , cho đường thẳng d :  y  1 và
z  t

mặt phẳng ( P ) : 2 x  z  3  0 . Biết đường thẳng  đi qua điểm O (0; 0; 0) gốc toạ độ, có 1 vectơ

chỉ phương u  (1; a; b) , vuông góc với đường thẳng d và hợp với mặt phẳng ( P ) một góc lớn
nhất. Hỏi điểm nào sau đây thuộc đường thẳng  ?
A. P (0;1; 0) . B. M (2; 0; 2) . C. N ( 1;1;1) . D. Q (1; 2; 2) .
Câu 19. (THPT Phù Cừ - Hưng Yên - 2022) Trong không gian Oxyz , cho mặt cầu ( S ) tâm I (2; 1;3)
bán kính R  4 và mặt cầu  S1  : x 2  y 2  z 2  4 x  6 z  2  0 . Biết mặt phẳng ( P ) là giao của hai
mặt cầu ( S ) và  S1  . Gọi M , N là hai điểm thay đổi thuộc mặt phẳng ( P ) sao cho MN  2 .

Giá trị nhỏ nhất của AM  BN bằng a  b 2 , với a, b   và A(0;5; 0), B (3; 2; 4) . Tính giá
b
trị gần đúng của (làm tròn đến hàng phần trăm).
a
A. 0,05. B. 0,07. C. 0,11. D. 0,13.
Câu 20. (Sở Hà Tĩnh 2022) Trong không gian Oxyz , cho hai mặt cầu
4 7 14
 S1  : x 2  ( y  1)2  ( z  2)2  16;  S2  : ( x  1)2  ( y  1)2  z 2  1 và điểm A  ; ;   . Gọi ( P )
3 3 3
là mặt phẳng tiếp xúc với cả hai mặt cầu  S1  ,  S2  và I là tâm của  S1  . Xét điểm M ( a; b; c ) di
động trên ( P ) sao cho IM tiếp xúc với mặt cầu  S2  , khi AM ngắn nhất thì a  b  c bằng
A. 1.
B. 1 .
7
C. .
3
7
D.  .
3
Câu 21. (Sở Thanh Hóa 2022) Trong không gian Oxyz , cho bốn điếm
A(2;3;5), B ( 1;3; 2), C ( 2;1;3), D (5; 7; 4) . Xét điếm M (a; b; c) di động trên mặt phắng (Oxy ) , khi
T  4 MA2  5MB 2  6 MC 2  MD 4 đạt giá trị nhỏ nhất thì a  b  c bằng
A. 11.
B. 11 .
C. 12.
D. 9.
Câu 22. (Sở Bắc Giang 2022) Trong không gian Oxyz , biết rằng không có đường thẳng nào cắt đồng thời
cả 4 đường thẳng
x  6  t
x3 y 3 z x 1 y 1 z x y  2 z 1 
d1 :   ; d2 :   ; d3 :   ; d 4 :  y  a  3t.
1 1 1 1 2 1 1 1 1 z  b  t

Giá trị 2b  a bằng
Trang 4 Fanpage Nguyễn Bảo Vương  https://www.facebook.com/tracnghiemtoanthpt489/
Điện thoại: 0946798489 TUYỂN CHỌN VẬN DỤNG – VẬN DỤNG CAO 2022
A. 2
B. 3
C. 2.
D. 3 .
Câu 23. (Sở Hà Tĩnh 2022) Trong không gian với hệ trục tọa độ Oxyz , cho mặt cầu
( S ) : x 2  y 2  z 2  2 x  4 y  2 z  0 và điểm M (0;1; 0) . Mặt phẳng ( P ) đi qua M và cắt ( S ) theo
đường tròn (C ) có chu vi nhỏ nhất. Gọi N  x0 ; y0 ; z0  là điểm thuộc đường tròn (C ) sao cho
ON  6 . Tính y0 .
A. 3.
B. 1.
C. 2.
D. 4.
Câu 24. (Sở Bạc Liêu 2022) Trong không gian với hệ trục toạ độ Oxyz , cho mặt cầu
2 2 2
 S  :  x  1   y  1   z  2   9 và điểm M 1;3; 1 , biết rằng các tiếp điểm của các tiếp
tuyến kẻ từ M tới các mặt cầu đã cho luôn thuộc một đường tròn  C  có tâm J  a; b; c  . Giá trị
T  2a  b  c bằng
134 62 84 116
A. T  . B. T  . C. T  . D. T  .
25 25 25 25
Câu 25. (Sở Hà Tĩnh 2022) Trong không gian Oxyz, cho hai điểm A  2; 3; 5  , I  2;0; 1 và mặt phẳng

 P  : 2 x  y  2 z  5  0. Điểm M  a; b; c  thay đổi thuộc mặt phẳng  P sao cho IM  5 và độ


dài đoạn AM lớn nhất. Khi đó giá trị của biểu thức T  a  b  2c bằng
1
A. 1 . B. 11 . C. 6 . D.  .
3

x 5 y 7 z 3
Câu 26. (Sở Hà Tĩnh 2022) Trong không gian Oxyz, cho ba đường thẳng d :   ,
1 2 3
x y 1 z  3 x2 y 3 z
d1 :   và d 2 :   . Gọi  là đường thẳng song song với d đồng thời
2 1 2 1 3 2
cắt cả hai đường thẳng d1 và d 2 . Đường thẳng  đi qua điểm nào sau đây?
A.  4;10;17  . B.  4;1; 7  . C.  3; 12;10  . D. 1; 6;6  .
2 2
Câu 27. (Sở Phú Thọ 2022) Trong không gian Oxyz , cho mặt cầu  S  :  x  2   y 2   z  5  24 cắt
mặt phẳng   : x  y  4  0 theo giao tuyến là đường tròn  C  . Điểm M thuộc  C  sao cho
khoảng cách từ M đến A  4; 12;1 nhỏ nhất. Tung độ của điểm M bằng
A. 6 . B. 4 . C. 0 . D. 2 .
Câu 28. (THPT Bùi Thị Xuân – Huế - 2022) Trong không gian Oxyz , cho mặt cầu x 2  y 2  z 2  9 và
x  1 t

điểm M  x0 ; y0 ; z0   d :  y  1  2t . Ba điểm A, B, C phân biệt cùng thuộc mặt cầu sao
 z  2  3t

cho MA, MB, MC là tiếp tuyến của mặt cầu. Biết rằng mặt phẳng ( ABC ) đi qua điểm D (1;1; 2) .
Tổng T  x02  y02  z02 bằng
A. 21.
B. 30.
C. 20.
D. 26.
Câu 29. (Chuyên Hạ Long 2022) Trong không gian với hệ tọa độ Oxyz , cho điểm N (2;3; 4) . Một mặt
cầu bất kỳ đi qua O và N cắt các trục tọa độ Ox, Oy , Oz lần lượt tại A, B, C  0 . Biết rằng khi
Facebook Nguyễn Vương https://www.facebook.com/phong.baovuongTrang 5
Blog: Nguyễn Bảo Vương: https://www.nbv.edu.vn/
mặt cầu thay đổi nhưng vẫn thỏa đề bài, trọng tâm G của tam giác ABC luôn nằm trên một mặt
phẳng cố định. Mặt phẳng cố định này chắn các trục tọa độ thành một tứ diện, tính thể tích của
khối tứ diện đó.
24389
A. .
3888
24389
B. .
4374
24389
C. .
8748
24389
D. .
2916
Câu 30. (Chuyên Hoàng Văn Thụ - Hòa Bình – 2022) Trong không gian Oxyz cho hai điểm
A(4; 6; 2), B (2; 2; 0) và mặt phẳng ( P ) : x  y  z  0 . Xét đường thẳng d thay đổi thuộc ( P ) và
đi qua B , gọi H là hình chiếu vuông góc của A trên d . Biết rằng khi d thay đổi thì H thuộc
một đường tròn cố định. Diện tích của hình tròn đó bằng
A. 4 .
B.  .
C. 6 .
D. 3 .
Câu 31. (Chuyên Lam Sơn – 2022) Trong không gian với hệ trục tọa độ Oxyz , cho 3 đường thẳng
 x  1  2t1  x  3  t2  x  4  2t3
  
 d1  ,  d2  ,  d3  có phương trình  d1  :  y  1  t1 ,  d 2  :  y  1  2t2 ,  d3  :  y  4  2t3 tiếp xúc
 z  1  2t  z  2  2t 
 1  2  z  1  t3
với 3 đường thẳng đó. Giá trị nhỏ nhất của R gần số nào nhất trong các số sau:
A. 2,1.
B. 2,2.
C. 2,3.
D. 2,4.
Câu 32. (Chuyên Lam Sơn 2022) Trong không gian với hệ trục tọa độ Oxyz cho điểm I (1; 0; 0) , điểm
x  2
7 4 4 
M  ; ;  và đường thẳng d :  y  t .N (a, b, c) là điểm thuộc đường thẳng d sao cho diện
9 9 9 z  1 t

tích tam giác IMN nhỏ nhất. Khi đó a  b  c có giá trị bằng:
A. 2.
B. 2 .
5
C. .
2
5
D. .
2
Câu 33. (Chuyên Nguyễn Trãi – Hải Dương – 2022) Trong không gian với hệ trục Oxyz , cho mặt cầu
( S ) : x 2  y 2  z 2  1 và hai điểm A(3; 0; 0); B ( 1;1; 0) . Gọi M là điểm thuộc mặt cầu ( S ) . Tính
giá trị nhỏ nhất của biểu thức MA  3MB .
A. 2 34
B. 26
C. 5
D. 34
Câu 34. (THPT Đô Lương – Nghệ An – 2022) Trong không gian Oxyz cho đường tròn (C ) là giao
tuyến của mặt phẳng tọa độ ( xOy ) với mặt cầu ( S ) : ( x  6)2  ( y  6)2  ( z  3) 2  41 . Gọi d là

Trang 6 Fanpage Nguyễn Bảo Vương  https://www.facebook.com/tracnghiemtoanthpt489/


Điện thoại: 0946798489 TUYỂN CHỌN VẬN DỤNG – VẬN DỤNG CAO 2022
đường thẳng đi qua các điểm A(0; 0;12), B (0; 4;8) . Với M , N là các điểm thay đổi thứ tự trên
(C ) và d . Giá trị nhỏ nhất của độ dài đoạn MN gần với giá trị nào nhất sau đây?
A. 3,5.
B. 2,35.
C. 1,25.
D. 2,92.
Câu 35. (THPT Lương Thế Vinh – Hà Nội – 2022) Trong không gian Oxyz , cho hai điếm
A(2; 1; 1), B (0;1; 2) và mặt phắng ( P ) : 2 x  y  2 z  2  0 . Điếm M thuộc mặt phắng ( P ) sao
cho AMB Ión nhất, khì đó cos  AMB bằng
5
A. .
13
12
B. .
13
12
C.  .
13
5
D.  .
13
Câu 36. (THPT Kim Liên - Hà Nội - 2022) Trong không gian Oxyz , cho hai điểm A  1; 2;3 và
B  3; 2;5  . Xét hai điểm M và N thay đổi thuộc mặt phẳng  Oxy  sao cho MN  2023 . Tìm giá
trị nhỏ nhất của AM  BN .
A. 2 17 . B. 65 . C. 25 97 . D. 205 97 .
Câu 37. (THPT Kim Liên - Hà Nội - 2022) Trong không gian Oxyz , cho mặt phẳng
 P  : 2 x  2 y  z  5  0 và mặt cầu  S  có tâm I 1;2; 2  . Biết  P  cắt  S  theo giao tuyến là
đường tròn  C  có chu vi 8π . Tìm bán kính của mặt cầu T  chứa đường tròn  C  và T  đi
qua điểm M 1;1;1 .
265 5 5
A. R  5 . B. R  . C. R  . D. R  4 .
4 4

Câu 38. (THPT Kinh Môn - Hải Dương - 2022) Trong không gian Oxyz , cho mặt cầu  S1  có tâm
I  2;1;1 có bán kính bằng 4 và mặt cầu
 S 2  có tâm J  2;1;5  có bán kính bằng 2. Gọi  P  là mặt phẳng thay đổi tiếp xúc với hai mặt
cầu  S1  ,  S 2  . Đặt M , m lần lượt là giá trị lớn nhất, giá trị nhỏ nhất của khoảng cách từ điểm O
đến mặt phẳng  P  . Giá trị M  m bằng
A. 8 . B. 9 . C. 8 3 . D. 15 .
Câu 39. (THPT Lương Tài 2 - Bắc Ninh - 2022) Trong không gian với hệ tọa độ Oxyz , cho điểm
A 2;4;  2 và mặt phẳng  P  : m 2  1 x   m2  1 x  2mz  4  0 . Biết rằng,khi tham số m thay
đổi thì mặt phẳng  P  luôn tiếp xúc với hai mặt cầu cố định cùng đi qua A là  S1  và  S2  . Gọi
M và N lần lượt nằm trên  S1  và  S2  . Tìm giá trị lớn nhất của MN ?
A. 16 2 . B. 8  8 2 . C. 8 2 . D. 8  6 2 .
Câu 40. (THPT Võ Nguyên Giáp - Quảng Bình - 2022) Trong không gian Oxyz , cho ba điểm
A(1;2;3), B  0;1;0  , C (1;0; 2) và mặt phẳng ( P ) : x  y  z  2  0 . Điểm M  a; b; c  nằm trên ( P )
sao cho biểu thức MA2  2 MB 2  3MC 2 đạt giá trị nhỏ nhất. Khi đó giá trị của biểu thức
T  a  b  9c bằng
13 13
A. . B. . C. 13 . D. 13 .
9 9
Facebook Nguyễn Vương https://www.facebook.com/phong.baovuongTrang 7
Blog: Nguyễn Bảo Vương: https://www.nbv.edu.vn/

Theo dõi Fanpage: Nguyễn Bảo Vương  https://www.facebook.com/tracnghiemtoanthpt489/

Hoặc Facebook: Nguyễn Vương  https://www.facebook.com/phong.baovuong

Tham gia ngay: Nhóm Nguyễn Bào Vương (TÀI LIỆU TOÁN)  https://www.facebook.com/groups/703546230477890/

Ấn sub kênh Youtube: Nguyễn Vương


 https://www.youtube.com/channel/UCQ4u2J5gIEI1iRUbT3nwJfA?view_as=subscriber

Tải nhiều tài liệu hơn tại: https://www.nbv.edu.vn/

Trang 8 Fanpage Nguyễn Bảo Vương  https://www.facebook.com/tracnghiemtoanthpt489/


TUYỂN CHỌN VẬN DỤNG – VẬN DỤNG CAO 2022 Điện thoại: 0946798489

VẤN ĐỀ 1. HÀM SỐ
• |FanPage: Nguyễn Bảo Vương
• TUYỂN CHỌN CÂU HỎI VD-VDC TỪ CÁC ĐỀ THI THỬ CÁC TRƯỜNG, CÁC SỞ NĂM 2022
Câu 1. (Chuyên Vinh – 2022) Cho hàm số y  f ( x ) có đạo hàm là f ( x)   x 2  9 x  x 2  9  với mọi


x   . Có bao nhiêu giá trị nguyên của tham số m để hàm số y  g ( x)  f x3  3 x  2m  m2 
có không quá 6 điểm cực trị ?
A. 2.
B. 5.
C. 4.
D. 7.
Lời giải
Chọn B
   
Do g ( x)  f  x 3  3 x  2m  m 2  f x 3  3 x  2m  m 2  g ( x) nên hàm số này là hàm số
chẵn tức để hàm số g ( x ) có không quá 6 điểm cực trị (cụ thể là tối đa 5 cực trị) thì hàm
 
h( x)  f x3  3 x  2m  m 2 có tối đa 2 điểm cực trị dương.
Tức phương trình h( x)   3 x 2
  
 3 f  x 3  3x  2m  m 2  0 có tối đa 2 nghiệm bội lẻ dương.
3 2 3 2
 x  3 x  2m  m  0  x  3 x  m  2m  y3
 3  3 2
 x  3 x  2m  m 2  9  x  3 x  m  2m  9   y1 *
 3 
 x  3 x  2m  m 2  3  x 3  3 x  m 2  2m  3  y2
 3 2  3 2
 x  3 x  2m  m  3  x  3 x  m  2m  3  y4
Như vậy để thỏa mãn đề bài thì bốn đường thẳng lần lượt là y1 , y2 , y3 , y4 phải cắt đồ thị
y  x 3  3 x tại tối đa hai nghiệm dương. Xét hàm số y  x 3  3 x có y  3 x 2  3  0, x   và
y (0)  0 .
Nhận thấy m 2  2m  3  (m  1)2  2  0 luôn đúng nên hệ * có tối thiểu 1 nghiệm, từ đó ta có:
Trường hợp 1: m 2  2m  0  m  [0; 2] thì hệ * có 1 nghiệm tức hàm số luôn có 3 điểm cực trị
m  0
Trường hợp 2: m 2  2m  0   thì hệ * đang có 2 nghiệm dương. Do hàm số có tối đa 5
m  2
điểm cực trị nên chỉ có tối đa 2 nghiệm dương tức ta có điều kiện đủ là:
 m 2  2m  9  0
 2  m  [1;3]
 m  2m  3  0
So với điều kiện ta suy ra m  {1;3} .
Từ hai trường hợp ta suy ra m  {1; 0;1; 2;3} tức có 5 giá trị nguyên m thỏa.
Câu 2. (Chuyên Vinh – 2022) Cho hàm số đa thức bậc bốn y  f ( x) có đồ thị như hình vẽ bên.

Facebook Nguyễn Vương https://www.facebook.com/phong.baovuong Trang 1


Blog: Nguyễn Bảo Vương: https://www.nbv.edu.vn/

 
Có bao nhiêu số nguyên a để phương trình f x 2  4 x  3  a có không ít hơn 10 nghiệm thực
phân biệt?
A. 4.
B. 6.
C. 2.
D. 8.
Lời giải
Chọn A
x2  4 x  x2  4 x  0  x  0; 4
Đạtt t  x 2  4 x  3; ta có t ( x)  2
 (2 x  4); t ( x )  0    .
x  4x 2 x  4  0 x  2
Bảng biến thiên

Nhận thấy: - Với t  3 thì vô nghiệm x .


- Với t  3 thì có 2 nghiệm x .
- Vói t  ( 3;1) thì có 4 nghiệm x .
- Với t  1 thì có 3 nghiệm x .
- Với t  1 thì có 2 nghiệm x .

Trang 2 Fanpage Nguyễn Bảo Vương  https://www.facebook.com/tracnghiemtoanthpt489/


Điện thoại: 0946798489 TUYỂN CHỌN VẬN DỤNG – VẬN DỤNG CAO 2022

Khi đó ta có phương trình f (t )  a (1). Từ đồ thị hàm số f ( x ) ta có


+ Nếu a  2 thì (1) có 2 nghiệm phân biệt t  1 hoặc vô nghiệm  Phương trình đã cho có số
nghiệm không lớn hơn 4.
+ Nếu a  2 thì (1) có 3 nghiệm phân biệt trong đó 1 nghiệm t  ( 3; 0) và có 2 nghiệm
t  1 .  Phương trình đã cho có 8 nghiệm.
 Nếu a  ( 2; 0) thì (1) có 4 nghiệm phân biệt trong đó có 2 nghiệm t  ( 3;1) và 2 nghiệm
t  1  Phương trình đã cho có 12 nghiệm phân biệt.
+ Nếu a  0 thì (1) có 4 nghiệm phân biệt trong đó có 1 nghiệm t  ( 3;1) và 1 nghiệm t  1
và nghiệm t  1; t  3  Phương trình đã cho có 11 nghiệm phân biệt.
+ Nếu a  (0; 2] thì (1) có 4 nghiệm phân biệt trong đó có 2 nghiệm t  ( 3;1) và 1 nghiệm
t  3 và 1 nghiệm t  1  Phương trình đã cho có 10 nghiệm phân biệt.
a  2
 Nếu  thì (1) có 2 nghiệm phân biệt trong đó 1 nghiệm t  3 và 1 nghiệm t  1
a  
 Phương trình đã cho có 2 nghiệm phân biệt.
Vậy với 2  a  2 thì phương trình đã cho có không it hơn 10 nghiệm thực phân biệt, do đó có 4
số nguyên a cần tìm.
Câu 3. (Chuyên Vinh -2022) Cho hàm số bậc ba y  f ( x ) . Biết rằng hàm số y  f  1  x 2  có đồ thị như
hình vẽ bên.

 x2 1  2
Số điểm cực trị của hàm số g ( x)  f  2   là
 x  x
A. 5.
B. 4.
C. 3.
D. 7.
Lời giải
Ta có
Facebook Nguyễn Vương https://www.facebook.com/phong.baovuongTrang 3
Blog: Nguyễn Bảo Vương: https://www.nbv.edu.vn/
2   x2 1  2 2  1   x2 1  
g  ( x)   f  2      f  2   1
x3  x  x
2
x2  x  x  
1  x2 1   x2 1   1 
 g  ( x)  0   f   2   1  0  f   2   x  f  1  2   x
x  x   x   x 
1

Đặt ta được f  1  t 2  .
t

1 1
Xét hàm số h(t )  (t  0)  h(t )   2  0, t  0
t t
1
Vẽ đồ thị hàm h(t )  trên cùng hệ trục toạ độ với hàm số y  f  1  t 2 
t
Từ đồ thị suy ra g '  x   0 có 5 nghiệm đơn.
 x2 1  2
Vậy hàm số g  x   f  2   có 5 điểm cực trị
 x  x
Câu 4. (Chuyên Lê Quý Đôn - Điện Biên - 2022) Cho hàm số f  x có đạo hàm
2 1
f   x    x  1 x  1  x  2  . Giá trị nhỏ nhất của hàm số g  x   f  x   x 3  x  2 có đạo hàm
3
trên đoạn  1;2 bằng
3 8 4
A. f  2   . B. f 1  . C. f  0   2 . D. f  1  .
4 3 3
Lời giải
Chọn B
2
Ta có: g   x   f   x   x 2  1   x  1 x  1  x  2   x 2  1   x  1 x  1  x 2  3 x  3
 x  1
  x  1
g   x   0   x  1 x  1  x  3 x  3  0   x  1
2
 .
 x 2  3x  3  0 x 1

Bảng xét dấu cho y .

Dựa vào bảng biến thiên ta thấy hàm số đạt giá trị nhỏ nhất tại x  1 suy ra giá trị nhỏ nhất của
8
hàm số g  x  là g 1  f 1  .
3
Câu 5. (Chuyên Lê Quý Đôn - Điện Biên - 2022) Có bao nhiêu giá trị nguyên của tham số m sao cho
3
phương trình m  3 3 m  3log x  log x có 3 nghiệm phân biệt?
A. 2. B. 1. C. 3. D. 4.
Lời giải
Chọn C
Điều kiện: x  0 .
Đặt: t  3 m  3log x  t 3  m  3log x  m  t 3  3log x .
Phương trình đã cho trở thành: 3
t 3  3log x  3t  log x
 t 3  3log x  3t  log 3 x

Trang 4 Fanpage Nguyễn Bảo Vương  https://www.facebook.com/tracnghiemtoanthpt489/


Điện thoại: 0946798489 TUYỂN CHỌN VẬN DỤNG – VẬN DỤNG CAO 2022
3 3
 t  3t  log x  3log x 1 .
Xét hàm số f  u   u 3  3u liên tục trên  .
 f   u   3u 2  3  0, u   .
 Hàm số y  f  u  đồng biến trên  .  2 
Khi đó, phương trình 1 trở thành: f  t   f  log x   3
Từ  2  và  3  t  log x  3 m  3log x  log x  m  log 3 x  3log x  4  .
Đặt: v  log x

Ta thấy: ứng với một nghiệm v thuộc  sẽ cho ra một nghiệm x thuộc  0;  .
3
Phương trình  4  trở thành: m  v  3v . Đặt: g  v   v3  3v  g   v   3v 2  3  0  v  1 .
Bảng biến thiên:

Yêu cầu bài toán  m  g  v  có ba nghiệm phân biệt


  2  m  2 . Mà m   , nên m  1;0;1 .
Vậy có 3 giá trị nguyên của tham số m thoả mãn yêu cầu bài toán.
Câu 6. (Cụm Trường Nghệ An - 2022) Hàm số y  f  x  có đồ thị là đường cong trong hình vẽ. Số
nghiệm thực phân biệt của phương trình f e  f  x

 f  x   1 là:

A. 2 . B. 4 . C. 6 . D. 8 .
Lời giải
Chọn C
 x  1
Dựa vào đồ thị hàm số ta có f  x   1  
x  1

Facebook Nguyễn Vương https://www.facebook.com/phong.baovuongTrang 5


Blog: Nguyễn Bảo Vương: https://www.nbv.edu.vn/
Đặt f  x  t , khi đó ta có phương trình  
f e f  x  f  x   1 trở thành

 e t  t  1
f e  t   1   t
t

e  t  1
Xét hàm số g  t   et  t là hàm số đồng biến trên  nên ta có phương trình et  t  1 có nghiệm
duy nhất t  0 .
Xét phương trình et  t  1 , dựa vào sự tương giao của đồ thị hàm số g  t   et  t và đường
thẳng y  1 ta có phương trình có nghiệm duy nhất t  a   2;  1 .

Dựa vào sự tương giao của đồ thị ta có:

Với t  0  f ( x)  0 nên phương trình có 2 nghiệm thì phương trình có 4 nghiệm.


Với t  a   2;  1  f ( x)  a   2;  1 nên phương trình có 2 nghiệm phân biệt.
Vậy phương trình đã cho có 6 nghiệm.
Câu 7. (Cụm Trường Nghệ An - 2022) Cho hàm số y  f  x  có đồ thị như hình vẽ. Có bao nhiêu giá trị


nguyên của tham số m thuộc đoạn  2021;2021 để hàm số g  x   f x5  4 x  m có ít nhất 5
điểm cực trị.

Trang 6 Fanpage Nguyễn Bảo Vương  https://www.facebook.com/tracnghiemtoanthpt489/


Điện thoại: 0946798489 TUYỂN CHỌN VẬN DỤNG – VẬN DỤNG CAO 2022

A. 2022 . B. 2023 . C. 2021 . D. 1012 .


Lời giải
Chọn C
Hàm số y  x 5  4 x  y   5 x 4  4  0 , x   .
Do đó y  x 5  x là hàm số lẻ và đồng biến trên  , x 5  4 x  0  x  0; x 5  4 x  0  x  0 .
 
Vậy hàm số g  x   f x5  4 x  m là hàm số chẵn, có đồ thị nhận trục Oy là trục đối xứng.

   
Hàm số g  x   f x5  4 x  m có ít nhất 5 điểm cực trị  h  x   f x5  4 x  m có ít nhất hai
điểm cực trị dương.
 x5  4 x  m  0  x5  4 x   m
   
Ta có h  x   5x 4  4 f  x5  4 x  m  h  x   0   5  5
 x  4x  m  2  x  4x  2  m
Yêu cầu bài toán  tổng số giao điểm có hoành độ dương khác nhau của đồ thị hàm số
y  x 5  4 x với hai đường thẳng y   m; y  2  m ít nhất là 2
m  0
  m  0.
2  m  0
Do m nguyên và thuộc đoạn  2021;2021 nên có 2021 giá trị m thỏa mãn đề bài.

 
Câu 8. (Đại học Hồng Đức 2022) Cho hàm đa thức y   f x 2  2 x   có đồ thị cắt trục Ox tại 5 điểm
phân biệt như hình vẽ. Hỏi có bao nhiêu giá trị của tham số m với 2022m  để hàm số
g ( x)  f  x 2  2 | x  1| 2 x  m  có 9 điểm cực trị?

A. 2020.
B. 2023.
C. 2021.
D. 2022.
Lời giải.
Ta có:

Facebook Nguyễn Vương https://www.facebook.com/phong.baovuongTrang 7


Blog: Nguyễn Bảo Vương: https://www.nbv.edu.vn/
 f  x 2  2 x     (2 x  2) f   x 2  2 x   a( x  3)( x  2)( x  1)( x)( x  1) (a  0)
 
a a
 f   x 2  2 x   ( x  3)( x  2) x( x  1)   x 2  2 x  3 x 2  2 x  .
2 2
2 a
Đặt t  x  2 x  f (t )  (t  3)t .
2
Ta có g ( x)  f  x  2 | x  1| 2 x  m   f | x  1|2 2 | x  1|  m  1 .
2

Ta thấy g (2  x )  g ( x ), x   nên đồ thị hàm số y  g ( x ) nhận đường thẳng x  1 làm trục đối
xứng. Do đó số điểm cực trị của hàm số g ( x ) bằng 2a  1 với a là số điểm cực trị lớn hơn 1 của
hàm số g ( x ) . Theo bài ra ta có 2a  1  9  a  4 . Vi vậy ta cần tìm m để hàm số g ( x ) có đúng
4 điểm cực trị lớn hơn 1.
Khi x  1 thì g ( x)  f  x 2  4 x  m  2  .
x  2
g ( x)  (2 x  4) f  x  4 x  m  2 , g ( x)  0   x 2  4 x  m  2  0(1).
 2

 x 2  4 x  m  2  3 (2)
Đặt u ( x)  x 2  4 x  m  2 , ta có bảng biến thiên

Yêu cầu bài toán trở thành tìm m để 2 phương trình (1), (2) có đúng 3 nghiệm phân biệt khác 2,
điều này xảy ra khi và chỉ khi m  2  0  m  1  1  m  2,
suy ra 2022  2022m  4044  2022m  {2023; 2024;; 4043},
do đó có 2021 giá trị của m thỏa mãn bài toán.
Câu 9. (Đại học Hồng Đức – 2022) Cho f ( x)  x 3  3 x 2  1 . Phương trình f ( f ( x)  1)  1  f ( x)  2 có
số nghiệm thực là
A. 7.
B.6.
C. 4.
D. 9.
Lời giải.

Đặt t  f ( x)  1  t  x 3  3 x 2  2 (*)

x  0
Suy ra t   3x 2  6 x . Khi đó t   0   . Ta có, bảng biến thiên
x  2

Khi đó f ( f ( x)  1)  1  f ( x)  2 trở thành:


t  1 t  1
f (t )  1  t  1   2
  3 2
 f (t )  1  t  2t  1 t  4t  2t  1  0
Từ bảng biến thiên ta có

Trang 8 Fanpage Nguyễn Bảo Vương  https://www.facebook.com/tracnghiemtoanthpt489/


Điện thoại: 0946798489 TUYỂN CHỌN VẬN DỤNG – VẬN DỤNG CAO 2022
+) Với t  a  ( 1; 0) , phương trình (*) có 3 nghiệm phân biệt.
+) Với t  b  (0;1) , phương trình (*) có 3 nghiệm phần biệt khác 3 nghiệm trên.
+) Với t  c  (4;5) , phương trình (*) có 1 nghiệm khác 6 nghiệm trên.
Vậy phương trình đã cho có 7 nghiệm.
Câu 10. (THPT Hồ Nghinh – Quảng Nam – 2022) Cho hàm số y  f ( x ) có bảng biến thiên như sau

Có bao nhiêu giá trị nguyên của tham số m để phương trình 6 f  x 2  4 x   m có it nhất 3 nghiệm
thực phân biệt thuộc khoảng (0;  ) ?
A. 29.
B. 25.
C. 24.
D. 30.
Lời giải
Chọn D
m
Ta có: 6 f  x 2  4 x   m  f  x 2  4 x  
6
Đặt u  x 2  4 x  u  0  x  2 .

m
Để phương trình f  x 2  4 x   có ít nhất 3 nghiệm phân biệt thuộc (0;  ) :
6
m
 3   2  18  m  12
6
Câu 11. (THPT Hồ Nghinh – Quảng Nam – 2022) Cho hàm số f ( x) có đạo hàm
f ( x)  ( x  1) 2
x 2
  4 x . Có bao nhiêu giá trị nguyên dương của tham số m để hàm số

g ( x)  f 2 x 2
 12 x  m  có đúng 5 điểm cực trị?
A. 17.
B. 16.
C. 18.
D. 19.
Lời giải
Ta có g ( x)  f  2 x  12 x  m   g ( x)  (4 x  12)  f   2 x 2  12 x  m  .
2

x  3
 
Suy ra g ( x)  0  (4 x  12)  f  2 x 2  12 x  m  0   2
 
 f  2 x  12 x  m  0

Facebook Nguyễn Vương https://www.facebook.com/phong.baovuongTrang 9


Blog: Nguyễn Bảo Vương: https://www.nbv.edu.vn/
x  3 x  3
 2  2 x 2  12 x  m  0 1
2 x  12 x  m  0 
 2  2
 2 x  12 x  m  4  2 x  12 x  m  4  0  2 
 2  2
 2 x  12 x  m  1  2 x  12 x  m  1  0  3
Vì phương trình (3) có nghiệm kép nên ta chỉ xét 2 phương trình (1) và (2).
Nhận xét: phương trình (1) và (2) không có nghiệm chung.
Yêu cầu bài toán suy ra phương trình (1) va (2) đều có 2 nghiệm phân biệt khác nhau và khác 3.
(1)  0 36  2m  0 m  18
 2  m  18
2.3  12.3  m  0 m  18 
     m  18
 (2)  0 36  2(m  4)  0 m  22
 2 m  22 m  22
2.3  12.3  m  4  0
Vì m nguyên dương nên m  {1; 2;3;;17} .
Câu 12. (THPT Hương Sơn - Hà Tĩnh - 2022) Cho các hàm số y  f  x ; y  f  f  x ;
y  f  x  2 x  1 có đồ thị lần lượt là  C1  ;  C2  ;  C3  . Đường thẳng x  2 cắt  C1  ;  C2  ;  C3 
2

lần lượt tại A, B, C . Biết rằng phương trình tiếp tuyến của  C1  tại A và của  C2  tại B lần lượt
là y  2 x  3 và y  8 x  5 . Phương trình tiếp tuyến của  C3  tại C là
A. y  8 x  9 . B. y  12 x  3 . C. y  24 x  27 . D. y  4 x  1 .
Lời giải
Chọn C
+ Phương trình tiếp tuyến của đồ thị hàm số y  f  x  tại điểm x  2 :
y  f   2  x  2   f  2   f   2  x  2 f   2   f  2  .
Thực hiện phép đồng nhất thức với phương trình tiếp tuyến y  2 x  3 ta được:
 f   2   2  f   2   2
  .
2 f   2   f  2   3  f  2   7
+ Phương trình tiếp tuyến của đồ thị hàm số y  f  f  x   tại điểm x  2 :
y  f   2  f   f  2    x  2   f  f  2    2 f   7  x  2   f  7   2 f   7  x  4 f   7   f  7  .
Thực hiện phép đồng nhất thức với phương trình tiếp tuyến y  8 x  5 ta được:
2 f   7   8  f   7   4
  .
4 f   7   f  7   5  f  7   21
+ Phương trình tiếp tuyến của đồ thị hàm số y  f  x 2  2 x  1 tại điểm x  2 là
y  6 f   7  x  2   f  7   24  x  2   21  24 x  27 .
Câu 13. (THPT Hương Sơn - Hà Tĩnh - 2022) Cho hàm số y  f  x  có đồ thị như hìnhh vẽ. Hàm số
y  4  f 2  x  có bao nhiêu điểm cực trị?

Trang 10 Fanpage Nguyễn Bảo Vương  https://www.facebook.com/tracnghiemtoanthpt489/


Điện thoại: 0946798489 TUYỂN CHỌN VẬN DỤNG – VẬN DỤNG CAO 2022
A. 4 . B. 5 . C. 3 . D. 6 .

Lời giải
Chọn B

2
 4  f 2  x   2 f  x  . f   x   f  x  . f   x 
y  4  f  x   y   
2 4  f 2  x 2 4  f 2  x 4  f 2  x

 f  x  0
Xét y  0  f  x  . f   x   0   .
 f  x  0

 x  a   2;  1

Dựa vào đồ thị hàm số ta có: f  x   0   x  b  1; 2 
 x  0

 x  1
f  x  0  
x 1

Vậy y  0 có 5 nghiệm phân biệt. Nên hàm số y  4  f 2  x  có 5 điểm cực trị.

Câu 14. (THPT Lê Thánh Tông - HCM-2022) Hàm số y  f  x  có đạo hàm trên  4; 4 , có các điểm
4
cực trị trên  4;4  là 3;  ;0; 2 và có đồ thị như hình vẽ. Đặt g  x   f  x 3  3 x   m với m là
3
tham số. Gọi m1 là giá trị của m để max g  x   2022, m2 là giá trị của m để min g  x   2004.
0;1 1;0
Giá trị của m1  m2 bằng

A. 12 . B. 13 . C. 11 . D. 14 .

Lời giải
Chọn D
 Trước tiên, xét hàm y  x 3  3 x , có BBT như sau:

Facebook Nguyễn Vương https://www.facebook.com/phong.baovuongTrang 11


Blog: Nguyễn Bảo Vương: https://www.nbv.edu.vn/

 x3  3x  3  x  x1   1; 0
 
 x3  3 x   4
 x  x2   1; 0
 Có g   x    3x  3 f   x  3x   0  
2 3
3
 3 x0
 x  3x  0 
 x3  3x  2  x  x3   0;1

 Trên  0;1 , có g  0   f  0   m  3  m; g  x3   f  2   m  3  m; g 1  f  4   m  1  m


Dễ thấy max g  x   3  m  2022 , suy ra m1  m  2022  3  2019.
0;1
 Trên  1;0 , có
g  0   f  0   m  3  m; g  1  f  4   m  1  m; g  x1   f  3  m  4  m;
 4
g  x2   f     m  2  m.
 3
Dễ thấy min g  x   1  m  2004 , suy ra m2  m  2004  1  2005.
1;0
 Vậy m1  m2  2019  2005  14.
Câu 15. (THPT Lê Thánh Tông - HCM-2022) Cho hàm số bậc bốn y  f  x  có đồ thị  C1  và
y  f   x  có đồ thị  C 2  như hình vẽ dưới.

Số điểm cực đại của đồ thị hàm số g  x   f  e  x f  x   trên khoảng   ;3 là


A. 5 . B. 3 . C. 6 . D. 4 .
Lời giải
Chọn D
 f  x   f   x   0  f  x  f  x
 x 
x x x  e f  x   2  f  x  0
g   x    e f  x   e f   x   . f   e f  x    0    x  x
.
 e f  x   0  f  x    2 e
e  x f  x   2  f  x   2e x
 

Trang 12 Fanpage Nguyễn Bảo Vương  https://www.facebook.com/tracnghiemtoanthpt489/


Điện thoại: 0946798489 TUYỂN CHỌN VẬN DỤNG – VẬN DỤNG CAO 2022
+ f  x   f   x  có bốn nghiệm đơn trong đó 3 nghiệm phân biệt nhỏ hơn 3 (có một nghiệm
x  0 ) và một nghiệm lớn hơn 3 .
+ f  x   0 có hai nghiệm đơn phân biệt và một nghiệm bội chẵn x  0 .

+ f  x   2e x có một nghiệm đơn.


+ f  x   2e x có hai nghiệm đơn phân biệt.
Như vậy, trên khoảng   ;3 đạo hàm g   x  đổi dấu qua 8 điểm nên số điểm cực đại và cực tiểu
bằng nhau và bằng 4 .
Câu 16. (Liên trường Hà Tĩnh – 2022) Cho hàm số f ( x)  x 4  14 x3  36 x 2  (16  m) x với m là tham
số thực. Có bao nhiêu giá trị nguyên của m để hàm số g ( x)  f (| x |) có 7 điểm cực trị?
A. 33.
B. 31.
C. 32.
D. 34.
Lời giải

Xét hàm số: f ( x )  x 4  14 x8  36 x 2  (16  m) x .


Tập xác định: D   .
f ( x)  4 x8  42 x 2  72 x  16  m
Hàm số g ( x)  f (| x |) có 7 điểm cực trị  Hàm số f ( x) có 3 điểm cực trị dương.
 Phương trình f ( x)  0 có 3 nghiệm dương phân biệt.
Xét phương trình f ( x)  0  4 x 3  42 x 2  72 x  16  m (1)
x 1
Đặt h( x)  4 x8  42 x 2  72 x  16  h( x)  12 x 2  84 x  72  h( x )  0  
x  6
Ta có bảng biến thiên

Facebook Nguyễn Vương https://www.facebook.com/phong.baovuongTrang 13


Blog: Nguyễn Bảo Vương: https://www.nbv.edu.vn/

Yêu cầu bài toán  (1) có 3 nghiệm dương phân biệt khi và chỉ khi đường thẳng y  m cắt đồ thị
hàm số y  h( x) tại 3 điểm phân biệt có hoành độ dương.
Dựa vào BBT ta có 16  m  50 .
Vì m là số nguyên nên m  17;18...49 nên có 33 số nguyên.
Câu 17. (Liên trường Hà Tĩnh – 2022) Cho hàm số y  f ( x)  2022 x  2022 x  x  sin x . Có bao nhiêu
giá trị nguyên của m để phương trình f ( x  3)  f  x 3  4 x  m   0 có ba nghiệm phân biệt?
A. 4.
B. 3.
C. 2.
D. 5.
Lời giải
Chọn B
Xét hàm số
y  f ( x)  2022 x  2022 x  x  sin x
 f ( x)  2022 x ln 2022  2022 x ln 2022  1  cos x  0x  
Suy ra f ( x) đồng biến trên 
Ta có f (  x )  2022  x  2022 x  x  sin x    2022 x  2022  x  x  sin x    f ( x )
Xét phương trình
f ( x  3)  f  x 3  4 x  m   0  f  x 3  4 x  m    f ( x  3)  f ( x  3).
Vì f ( x) đồng biến nên
f  x3  4 x  m   f ( x  3)  x 3  4 x  m   x  3  x 3  3 x  3   m (1)
YCBT phương trình (1) phải có ba nghiệm phân biệt
Xét hàm số f ( x)  x 3  3 x  3 , ta có bảng biến thiên:

 m  4
Dựa vào BBT suy ra 1   m  5  5  m  1   m  3

 m  2
Vậy có 3 giá trị nguyên của m
Câu 18. (THPT Nguyễn Tất Thành-Đh-SP-HN-2022) Cho hàm số f  x  xác định trên  , có đạo hàm
f   x    x 2  4   x  5  , x   và f 1  0 . Có bao nhiêu giá trị nguyên của m để hàm số
g  x   f  x 2  1  m có nhiều điểm cực trị nhất ?

Trang 14 Fanpage Nguyễn Bảo Vương  https://www.facebook.com/tracnghiemtoanthpt489/


Điện thoại: 0946798489 TUYỂN CHỌN VẬN DỤNG – VẬN DỤNG CAO 2022
A. 6 . B. 8 . C. 5 . D. 7 .
Lời giải
Chọn D
Xét k  x   f  x 2  1  m  k   x   2 xf   x 2  1 .
x  0
x  0 x 1
 2 
x 1  2
k   x   0  2 xf   x 2  1  0   2   x  1 .
 x  1  2 
  x  2
 x 2  1  5  x  2
x4 5 3
Lại có f  x    f   x  dx    x 2  4   x  5  dx 
 x  2 x 2  20 x  C .
4 3
x4 5 199
Vì f 1  0 nên f  x    x3  2 x 2  20 x  .
4 3 12
Bảng biến thiên

Nhận xét:
Số điểm cực trị của hàm số y  f  x  bằng số điểm cực trị của hàm số y  f  x  cộng với số
giao điểm của đồ thị hàm số y  f  x  với trục Ox .
+) Hàm số g  x   f  x 2  1  m có nhiều điểm cực trị nhất khi và chỉ khi g  x   f  x 2  1  m
cắt Ox nhiều điểm nhất
 995
 12  m  0 995 461
  m .
 461  m  0 12 6
 6
Vì m    m  82; 81;; 76 . Vậy có 7 giá trị nguyên của m .
Câu 19. (THPT Nho Quan A – Ninh Bình – 2022) Cho hàm số f ( x ) có đạo hàm liên tục trên  . Đồ thị
của hàm số y  f (5  2 x ) như hình vẽ bên dưới:

Có bao nhiêu giá trị thực của tham số m thuộc khoảng (9;9) thỏa mãn 2m   và hàm số
1
y  2 f  4 x 3  1  m  có 5 điểm cực trị ?
2
A. 26.
B. 25.
Facebook Nguyễn Vương https://www.facebook.com/phong.baovuongTrang 15
Blog: Nguyễn Bảo Vương: https://www.nbv.edu.vn/
C. 27.
D. 24.
Lời giải
5t
Đặt t  5  2 x  x  . Bảng biến thiên của hàm số f (t ) :
2

Từ bảng biến thiên ta thấy hàm số y  f (t ) có 3 điểm cực trị.


Đặt: g ( x)  f  4 x 3  1  g ( x)  12 x 2 f   4 x3  1
 x  0
g ( x)  0   (có 3 nghiệm đơn)
 f  
4 x 3
 1 
 0  *
 hàm số y  f  4 x 3  1 có 3 điểm cực trị.
1 y m 1
Hàm số y  2 f  4 x3  1  m  có 5 điểm cực trị  Hàm số 
 f 4 x3  1    có 5
2 2 2 4
m 1
 
điểm cực trị  Phương trình f 4 x 3  1    0 (1) có 2 nghiệm đơn hoặc nghiệm bội lẻ.
2 4
Đặt t  4 x  1  t   12 x . Suy ra t là hàm số đồng biến trên  . Ứng với mỗi giá trị của t ta có
3 2

một giá trị của x . Số nghiệm của phương trình bằng số nghiệm của phương trình
m 1
f (t )    0.
2 4
m 1
Dựa vào bảng biến thiên, phương trình f (t )    0 có 2 nghiệm đơn hoặc nghiệm bội lẻ khi
2 4
1 m 9
4  2  4  m  4
 2m  8
và chỉ khi   1 17  .
 4  1  m  0  m 1  2m  17
 2 2
4 2
Kết hợp yêu cầu m thuộc khoảng (9;9) và 2m  ta có 26 giá trị thực của m thỏa mãn đề bài.
Câu 20. (THPT Phù Cừ - Hưng Yên - 2022) Cho hàm số y  f ( x ) có đồ thị hàm số y  f ( x ) như hình
vẽ.

Số giá trị nguyên của tham số m để hàm số g ( x)  f  2 x 2  4 | x |  m  3 có 7 điểm cực trị.


A. 1. B. 2. C. 4. D. 3.
Lời giải
Ta có g ( x)   2 x 2  4 | x |  m  3  f   2 x 2  4 | x |  m  3 .

Trang 16 Fanpage Nguyễn Bảo Vương  https://www.facebook.com/tracnghiemtoanthpt489/


Điện thoại: 0946798489 TUYỂN CHỌN VẬN DỤNG – VẬN DỤNG CAO 2022
Suy ra g ( x)  0   2 x  4 | x |  m  3  f   2 x 2  4 | x |  m  3  0
2

  2 x 2  4 | x |  m  3    0
 (2 x 2  4 | x |  m  3)  0 
    2 x 2  4 | x |  m  3  1
2
 f (2 x  4 | x |  m  3)  0  2
  2 x  4 | x |  m  3  2

 
 2 x 2  4 | x |  m  3   0 1

  2 x 2  4 | x | 3   m  1 (2)
 2
 2 x  4 | x | 3   m  2 (3)
 ) Xét phương trình  2 x 2  4 | x |  m  3   0 (1)
Với x  0  (1)  4 x  4  0  x  1 (thoả mãn).
Với x  0  (1)  4 x  4  0  x  1 (thoả mãn).
Khi đó x  1; x  0; x  1 là 3 điểm cực trị của hàm số.
+) Xét phương trình 2 x 2  4 | x | 3  m  1 (2).
Từ đồ thị suy ra phương trình (2) nếu có nghiệm thì nghiệm là bội chẵn nên hàm số g ( x) không
đổi dấu nên không phải là cực trị.
+) Xét phương trình 2 x 2  4 | x | 3   m  2
Yêu cầu bài toán suy ra phương trình (3) có 4 nghiệm phân biệt khác 0, 1 .
Xét hàm số y  2 x 2  4 | x | 3 có bảng biến thiên

Từ bảng biến thiên suy ra 5  m  2  3  5  m  7 .


Vì m nguyên nên m  6 . Vậy có 1 giá trị nguyên của tham số m thoả mãn.
Câu 21. (Sở Hà Tĩnh 2022) Cho hàm số f ( x ) liên tục trên  và đồ thị của hàm số y  f (1  x ) như
hình vë bên:

 1 x  2x 1
Số giá trị nguyên của m để phương trình f    m  0 có 4 nghiệm phân biệt là
 x2 x2
A. 3.
B. 4.
C. 2.
D. 5.
Lời giải

Facebook Nguyễn Vương https://www.facebook.com/phong.baovuongTrang 17


Blog: Nguyễn Bảo Vương: https://www.nbv.edu.vn/
1 x 1 x 2x 1
Đặt  1 t  t  1  . Phương trình trở thành
x2 x2 x2
f (1  t )  t  m  0  f (1  t )  t  m(*) .
Yêu cầu bài toán tương đương với (*) có 4 nghiệm phân biệt.
Điều này tương đương với đồ thị của hai hàm số (C ) : y  f (1  x ); d : y  x  m cắt nhau tại bốn
điểm phân biệt.
Chú ý đường thẳng y  x  m qua hai điểm ( m; 0); (0;  m) và song song hoặc trùng với đường
thẳng y  x .
Vẽ đường thẳng d : y  x  m trên cùng hệ trục toạ độ với đồ thị (C ) như hình vẽ:

Từ đồ thị suy ra d   C  tại 4 điểm phân biệt khi và chỉ khi 2  m  2  m  1;0;1
Câu 22. (Sở Hà Tình 2022) Cho hàm số bậc ba f ( x ) và hàm số g ( x )  f ( x  1) thoả mãn

( x  1) g ( x  3)  ( x  1) g ( x  2) , x   . Số điếm cực trị của hàm số y  f 2 x 2  4 x  5 là 
A. 1.
B. 3.
C. 2.
D. 5.
Lời giải
Có g ( x )  f ( x  1)  g ( x )  f ( x  1)  g ( x  3)  f ( x  4); g ( x  2)  f ( x  3) .
Thay vào giả thiết đã cho có:
( x  1) g ( x  3)  ( x  1) g ( x  2)  ( x  1) f ( x  4)  ( x  1) f ( x  3)(*) .
Thay x  1 vào hai vế của (*) có f (3)  0 ; thay x  1 vào haì vế của (*) có f (4)  0 .
Do đó f ( x ) là đa thức bậc hai có 2 nghiệm x1  3; x2  4 nên f ( x )  a ( x  3)( x  4) .
Khi đó hàm số y  f  2 x 2  4 x  5  có đạo hàm
     
y  (4 x  4) f  2 x 2  4 x  5  4a ( x  1) 2 x 2  4 x  5  3 2 x 2  4 x  5  4  8a ( x  1)3 2 x 2  4 x  1 
đổi dấu 3 lần nên hàm số có 3 điểm cực trị.
Câu 23. (Sở Thanh Hóa 2022) Cho hàm số f ( x ) có đạo hàm liên tục trên  và f (0)  0 , đồ thị của
f ( x ) như hình vẽ:

Trang 18 Fanpage Nguyễn Bảo Vương  https://www.facebook.com/tracnghiemtoanthpt489/


Điện thoại: 0946798489 TUYỂN CHỌN VẬN DỤNG – VẬN DỤNG CAO 2022

Gọi m, n lần lượt là số điểm cực đại, số điểm cực tiểu của hàm số g ( x ) | f (| x |)  3 | x || . Giá trị
của m n bằng
A. 4.
B. 8.
C. 27.
D. 16.
Lời giải
Xét hàm số u ( x)  f (| x |)  3 | x | là một hàm số chẵn nên chỉ cần xét trên [0; ) đế suy ra bảng
biến thiên của u ( x) trên cả . Với
x 0
x  0  u ( x)  f ( x)  3 x  u( x)  f ( x)  3  0  f ( x)  3   x  0; x  1; x  2 .
Bảng biến thiên:

Trong đó u (1)  u (1)  f (1)  3; u (0)  f (0)  0 .


Suy g ( x ) | u ( x ) | có tất cả 5 điểm cực trị trong đó 2 điểm cực đại và 3 điểm cực tiểu.
Câu 24. (Sở Thanh Hóa 2022) Cho hàm số f ( x ) có đồ thị của đạo hàm như hình vë:

Giá trị lớn nhất của hàm số h( x)  3 f ( x)  x3  3 x trên đoạn [  3; 3] bằng


A. 3 f (1)  2 .
B. 3 f (0) .
C. 3 f ( 3) .
D. 3 f ( 3) .
Lời giải

Facebook Nguyễn Vương https://www.facebook.com/phong.baovuongTrang 19


Blog: Nguyễn Bảo Vương: https://www.nbv.edu.vn/
 
Ta có h( x)  3 f ( x)  3 x 2  3  3  f ( x)  x 2  1   0, x  [ 3; 3] . Vì vẽ thêm parabol
y  x 2  1 qua các điềm (0; 1);(  3; 2); ( 3; 2) . Nhận thấy f ( x ) nằm dưới parabol trên đoạn
[ 3; 3] .

Vậy max h( x)  h( 3)  3 f ( 3) .


  3; 3 
 

Câu 25. (Sở Thanh Hóa 2022) Cho hàm số f ( x ) liên tục trên  và có đổ thị như hình vẽ:

Xét T  103 f  a 2  a  1  234 f (af (b)  bf (a )), ( a, b   ) . Biết T có giá trị lónn nhát bằng M
M
đạt tại m cặp ( a; b) , khi đó bằng
m
1011
A. .
4
1011
B. .
8
337
C. .
2
674
D. .
3
Lời giải
Quan sát đồ thị đã cho ta có max  f ( x)  f (3)  6 .

max f  a 2  a  1  6
  Tmax  (103  234)  6  2022.
max f (af (b)  bf (a ))  6
    f (b)  3  bf (1)  3  4n 0
 a 1  
  a  2
Dấu bằng đạt tại  a 2  a  1  3     ,, ( f (1)  f (2)  0) tức
af (b)  bf (a )  3   3  bf (2) 3
   f (b)     4n 0
  2 2
có 8 cặp (a; b) .

Trang 20 Fanpage Nguyễn Bảo Vương  https://www.facebook.com/tracnghiemtoanthpt489/


Điện thoại: 0946798489 TUYỂN CHỌN VẬN DỤNG – VẬN DỤNG CAO 2022
M 2022 1011
Vậy   .
m 8 4
Câu 26. (Sở Thanh Hóa 2022) Cho hàm số bậc bốn f ( x ) có đồ thị của đạo hàm như hình vẽ:

Số điểm cực tiểu của hàm số g ( x)  4 f  x 2  4   x 4  8 x 2 là


A. 4.
B. 7.
C. 3.
D. 5.
Lời giải
 x2  4 
  
Có g ( x)  8 xf  x 2  4  4 x3  16 x  8 x  f  x 2  4  
2 
.

x x x
Xét f ( x)   0  f ( x)    x  2; x  0; x  4 . Suy ra f ( x)  là đa thức bậc ba có 3
2 2 2
x

nghiệm là x  2; x  0; x  4 nên f ( x)   a ( x  2) x( x  4), a  0; lim f ( x)   .
2 x 

Do đó g ( x)  8ax  x  4  2  x  4  x  4  4   8ax  x  2  x  4  x 2  8  đổi dấu tử âm
2 2 2 2 2

sang dương khi qua các điểm x  2 2; x   2; x  2; x  2 2 nên g ( x ) có 4 điểm cực tiểu.
Câu 27.  
(Sở Bắc Giang 2022) Biết rằng f (0)  0 . Hỏi hàm số g ( x)  f x 6  x3 có bao nhiêu điểm cực
đại?

A. 2.
B. 4.
C. 3.
D. 1.
Lời giải
 x2  0
       
h( x)  f x 6  x3  h( x)  6 x 5 f  x 6  3x 2  3x 2 2 x3 f  x 6  1   3
  6
 2 x f  x  1  0
Đặt: u ( x)  2 x3 f   x 6   1  u( x)  6 x 2 f   x 6   12 x8 f   x 6   0, x  
 f  x6  0
    
6 x 2 f  x 6  0

6
(Từ đồ thị ta có x  0   do đó  8 , x   )
 f  x 6
  
0 12 x f  
 x 6
 0

Facebook Nguyễn Vương https://www.facebook.com/phong.baovuongTrang 21


Blog: Nguyễn Bảo Vương: https://www.nbv.edu.vn/
Nên u ( x)  2 x3 f   x 6   1 đồng biến và liên tục trên 
 lim u ( x)  
(do f ( x ) là hàm đa thức  u ( x ) là hàm đa thức) và  x
 xlim u ( x)  


suy ra phương trình u ( x)  2 x 3 f   x 6   1  0 có nghiệm duy nhất.


1
Giả sử 2 x 3 f   x 6   1  0  x 3 f   x 6  
2
  
có nghiệm là x0 (do f  x06  0  x03  0  x0  0 .

Dựa vào bảng biến thiên ta thấy hàm số g  x   h  x  có 1 điểm cực đại.
Câu 28. (Sở Bắc Giang 2022) Cho hàm số y  f ( x ) liên tục trên  và có đồ thị như hình vẽ dưới đây

Đặt g ( x)  f  
x2  4 x  6  2 x2  4x   x 2  4 x  6  12 x 2  4 x  6  1 . Tổng giá trị lớn nhất
và giá trị nhỏ nhất của hàm số g ( x ) trên đoạn [1; 4] bằng
A. 12  2 4 .
B. 12  12 6 .
C. 12  2 4 .
D. 12  12 6 .
Lời giải
Từ đồ thị suy ra f ( x)  x  2 x  3  f ( x)  4 x 3  4 x
4 2

Đặt t  x 2  4 x  6, x  [1; 4]  t  [ 2; 6] .
Ta có: g ( x)  f   
x2  4 x  6  2 x2  4 x  6  x2  4 x  6  1
Suy ra hàm số đã cho trở thành

Trang 22 Fanpage Nguyễn Bảo Vương  https://www.facebook.com/tracnghiemtoanthpt489/


Điện thoại: 0946798489 TUYỂN CHỌN VẬN DỤNG – VẬN DỤNG CAO 2022
3 2
h(t )  f (t )  2t  1  h(t )  f (t )  6t
t  0  ( 2; 6)

1
h(t )  0  f (t )  6t  0  4t  6t  4t  0  t    ( 2; 6)
2 3 2
 2

t  2  ( 2; 6)
Ta có:
h( 2)  f ( 2)  2  ( 2)3  1  2  4 2
h(2)  f (2)  2  (2)3  1  10
h( 6)  f ( 6)  2  ( 6)3  1  22  12 6
Suy ra giá trị lớn nhất và giá trị nhỏ nhất của hàm số h(t ) trên đoạn [ 2; 6] lần lượt là
22  12 6 và 10 .
Vậy tổng giá trị lớn nhất và nhỏ nhất của g ( x ) trên [1; 4] là tổng giá trị lớn nhất và nhỏ nhất của
h(t ) trên [ 2; 6] và bằng 12  12 6 .
Câu 29. (Sở Hà Tĩnh 2022) Cho hàm số bậc ba y  f ( x ) có f (1)  3 và có đồ thị như hình vẽ bên. Có
bao nhiêu giá trị nguyên của tham số m và m [ 10;10] để phương trình
f ( x)
ln  x[ f ( x )  3mx]  3mx3  f ( x) có hai nghiệm dương phân biệt?
3mx 2

A. 18.
B. 9.
C. 10.
D. 15.
Lời giải.
Do yêu cầu bài toán là phương trình có hai nghiệm dương phân biệt nên ta chỉ xét x  0 . Giả sử
 5 131 
f ( x)  ax 3  bx 2  cx  d . Vi đồ thị đi qua các điểm A   ;  , B (0; 4), C (1;5) nên ta có
 4 64 

Facebook Nguyễn Vương https://www.facebook.com/phong.baovuongTrang 23


Blog: Nguyễn Bảo Vương: https://www.nbv.edu.vn/
 125 25 5 131
 64 a  16 b  4 c  d  64

d  4 (1)
a  b  c  d  5


Ta có f (1)  3  3a  2b  c  3 .(2)
Từ (1) và (2) ta có a  1, b  0, c  0, d  4 , suy ra f ( x)  x 3  4 .
f ( x)
Điều kiện 0 m 0.
3mx 2
f ( x)
ln 2
 x[ f ( x)  3mx]  3mx3  f ( x)
3mx
 ln f ( x)  ln  3mx 2   x  f  x  3mx 2    f ( x)  3mx 2  0
Nếu f ( x)  mx 2 thì log f ( x)  log  mx 2  và xf ( x)  x  mx 2  , x  0  (3) vô nghiệm.
Tương tự nếu f ( x)  mx 2 thì phương trình (3) vô nghiệm.
x3  4
Do đó f ( x)  3mx 2  x 3  4  3mx 2   m , vì x  0 .
3x 2
x3  4
Xét hàm số g ( x)  với x  0 .
3x 2
3 x 4  24 x x  0
g ( x)  4
0
9x  x  2.
Vì x  0 nên ta nhận x  2 . Ta có bảng biến thiên

x3  4
Để phương trình  m có hai nghiệm dương phân biệt thì m  1 .
3x 2
Mà m  và m  [ 10;10] nên m  {2;3;;10} . Vậy có 9 giá trị nguyên của tham số m thoả yêu
cầu bài toán.
Câu 30. (Sở Ninh Bình 2022) Cho hàm số y  f ( x)  ax3  bx 2  cx  d có bảng biến thiên như sau

Tìm m để phương trình | f ( x  1)  2 | m có 4 nghiệm thỏa mãn x1  x2  x3  1  x4 .

Trang 24 Fanpage Nguyễn Bảo Vương  https://www.facebook.com/tracnghiemtoanthpt489/


Điện thoại: 0946798489 TUYỂN CHỌN VẬN DỤNG – VẬN DỤNG CAO 2022
A. 4 m  6.
B. 3  m  6.
C. 2 m  6.
D. 2  m  4.
Lời giải.
Đồ thị hàm số y | f ( x  1)  2 | thu được bằng cách biến đổi đồ thị như sau
- Tịnh tiến đồ thị hàm số y  f ( x) sang phải 1 đơn vị, sau đó tịnh tiến lên trên 2 đơn vị ta được
đồ thị hàm số y  f ( x  1)  2 ;
- Với đồ thị hàm số y | f ( x  1)  2 | : Giữ nguyên phần nằm bên trên trục hoành, lấy đối xứng
phần nằm bên dưới trục hoành qua trục hoành rồi xóa phần nằm bên dưới trục hoành đi.
Do đó ta có bảng biến thiên

Câu 31. (Sở Ninh Bình 2022) Cho f ( x ) là hàm số bậc ba. Hàm số f ( x ) có đồ thị như hình vẽ. Tìm tất
cả các giá trị thực của tham số m để phương trình f  e x  1  x  m  0 có hai nghiệm thực phân
biệt?

A. m f (2) .
B. m f (0) .
C. m f (0) .
D. m f (2) .
Lời giải.
Cách 1. Ta có
f  e x  1  x  m  0  f  e x  1  x  m.
Đặt h( x)  f  e x  1  x thì h( x)  e x f   e x  1  1 . Suy ra
1
h( x)  0  e x f   e x  1  1  0  f   e x  1 
.
ex
1
Đặt t  e x  1, t  1 thì (1) trở thành f (t )  . Ta có đồ thị sau
t 1

Facebook Nguyễn Vương https://www.facebook.com/phong.baovuongTrang 25


Blog: Nguyễn Bảo Vương: https://www.nbv.edu.vn/

Từ đồ thị ta có nghiệm của phương trình (2) là t  0 , suy ra e x  1  0 hay x  0 . Ta có bảng của
h( x) như trên. Từ đó, phương trình h( x )  m có hai nghiệm thực phân biệt khi và chỉ khi
m  f (0) .
Cách 2. Từ đồ thị ta có f ( x )  ( x  1) 2 . Suy ra
1
f ( x)  ( x  1)3  C.
3
Thay vào phương trình, ta được
e3 x e3 x
C  xm  0  m   x  C.
3 3
e3 x
Đặt g ( x)   x  C . Ta có
3
g ( x)  0  e3 x  1  0  x  0.
Ta có bảng biến thiên

Từ bảng biến thiên, phương trình có hai nghiệm thực khi và chỉ khi m  f (0) .
Câu 32. (Sở Bạc Liêu 2022) Cho hàm số y  f  x  liên tục trên  và có đồ thị có 3 điểm cực trị như
hình vẽ.

Số điểm cực trị của hàm số g  x   f  x 3  3x  2  là


A. 5 . B. 11 . C. 9 . D. 7 .
Lời giải
Chọn D

Trang 26 Fanpage Nguyễn Bảo Vương  https://www.facebook.com/tracnghiemtoanthpt489/


Điện thoại: 0946798489 TUYỂN CHỌN VẬN DỤNG – VẬN DỤNG CAO 2022
Ta có g   x    3 x  3 f   x  3 x  2  .
2 3

x  1

 x  1
3 x 2  3  0 
g x  0     x 3  3 x  2  a   3; 1
 f   x  3 x  2   0
3
 x 3  3 x  2  a   1;0 

 x 3  3 x  2  a   0;1

x  1
Đặt h  x   x 3  3 x  2  h  x   3 x 2  3  0   .
 x  1
Bảng biến thiên

Với x3  3x  2  a   3; 1 có 1 nghiệm.


Với x3  3 x  2  a   1;0  có 1 nghiệm.
Với x3  3x  2  a   0;1 có 3 nghiệm.
Vậy g  x   f  x3  3 x  2  có 7 điểm cực trị.
Câu 33. (Sở Hà Tĩnh 2022) Cho hàm số y  f  x  là hàm số đa thức bậc bốn và có bảng biến thiên như
1
 3
x4
hình bên. Tìm số điểm cực trị của hàm số g  x   2  f  2 x  1 

A. 7 . B. 5 . C. 4 . D. 6 .
Lời giải
Chọn C
Ta có:
1 1
4  x4 3  4 2
g x  5
.2 .ln 2.  f  2 x  1   2 x .6.  f  2 x  1  . f   2 x  1
x
1
 2  2 ln 2 
x4
 2.2 .  f  2 x  1   5 f  2 x  1  3 f   2 x  1 
 x 
  f  2 x  1  2  0 1
 
g   x   0   2 ln 2
 x 5 f  2 x  1  3 f   2 x  1  0  2 
 .

Facebook Nguyễn Vương https://www.facebook.com/phong.baovuongTrang 27


Blog: Nguyễn Bảo Vương: https://www.nbv.edu.vn/
2
Xét phương trình  f  2 x  1   0 có các nghiệm đều là nghiệm bội chẵn do đó g   x  không đổi
dấu khi qua các nghiệm đó.
2 ln 2
Xét phương trình f  2 x  1  3 f   2 x  1  0 . Đặt 2 x  1  t ; phương trình tương đương
x5
64 ln 2
5
f  t   3 f   t   0  * .
 t  1
64 ln 2 f  t 
Do f  t  và f   t  không đồng thời bằng 0 nên 5
3  0  * .
 t  1 f t 

Dựa vào bảng biến thiên ta có: f  t   a  t  t1  t  t2  t  t3  t  t4  .

64 ln 2 3 3 3 3
Tính f   t  thay vào * ta được phương trình: 5
    0
 t  1 t  t1 t  t2 t  t3 t  t4

Xét hàm số
64 ln 2 3 3 3 3
h t   5
    ;
 t  1 t  t1 t  t2 t  t3 t  t4
320 ln 2 3 3 3 3
h  t    6
 2
 2
 2
 2
 0 ; t   .
 t  1  t  t1   t  t2   t  t3   t  t4 
Ta có bảng biến thiên của h  t  :

Phương trình có 4 nghiệm nên hàm số có 4 điểm cực trị.


Câu 34. (Sở Hà Tĩnh 2022) Cho hàm số y  f  x  liên tục trên  có đồ thị như hình vẽ..

Tính tổng tất cả các giá trị nguyên của m để phương trình f 1  2sin x   m có đúng hai nghiệm
trên  0;  

A. 3 . B. 2 . C. 0 . D. 6 .
Trang 28 Fanpage Nguyễn Bảo Vương  https://www.facebook.com/tracnghiemtoanthpt489/
Điện thoại: 0946798489 TUYỂN CHỌN VẬN DỤNG – VẬN DỤNG CAO 2022
Lời giải
Chọn D
Xét phương trình f 1  2sin x   m .
Có 2  2sin x  2  1  1  2sin x  1; x   0;    1  2sin x   1;1 .
Mỗi giá trị t  1  2sin x   1;1 ứng với hai giá trị x phân biệt thuộc  0;   .
Từ đồ thị suy ra 3  f 1  2sin x   1  3  m  1 thì f 1  2sin x   m có hai nghiệm phân
biệt.
Mà m   nên m  3; 2; 1;0 . Tổng tất cả các giá trị nguyên của m là 6 .
Câu 35. (Sở Lạng Sơn 2022) Biết rằng tập tất cả các giá trị của tham số m để phương trình
m  x  4  x 2  2  5x 2  8x  24 có bốn nghiệm thực phân biệt là khoảng  a; b  . Giá trị a  b
bằng
28 25
A. . B. . C. 4 . D. 9 .
3 3
Lời giải
Chọn B
2
Ta có: m  x  4  x 2  2  5x 2  8x  24  m  x  4  x 2  2   x  4   4  x 2  2  (1).
*) Với x  4  0  x  4 .
Khi đó 1  4  x 2  2   0 vô nghiệm.
*) Với x  4  0  x  4 .
2
 x  4  4  x2  2  x4 4 x2  2

1  m   m    2
 x  4 x2  2 x2  2  x  4
x4
Đặt t 
x2  2
x4
Xét hàm số f  x  
x2  2
x
x2  2   x  4
2
x 2  2  4x
f  x  2
x 2  x  2 x2  2
2

1
f   x  0  2  4x  0  x  .
2
Bảng biến thiên hàm số f  x 

Từ bảng biến thiên ta được điều kiện của t là 1  t  3


Vậy để có 2 nghiệm x ứng với 1 giá trị t thì 1  t  3
4
 2  suy ra m  t  , t  1;3 .
t
4
Xét hàm số g  t   t  trên 1;3  .
t
4 t  2(n)
g   t   1  2 ; g  t   0   .
t t  2(l)
Facebook Nguyễn Vương https://www.facebook.com/phong.baovuongTrang 29
Blog: Nguyễn Bảo Vương: https://www.nbv.edu.vn/
Bảng biến thiên thu gọn
x 1 2 3
y' 0 +
5 13
y 3
4
13 13
Từ BBT để phương trình (1) có 4 nghiệm thực thì 4  m  . Nên a  4; b  .
3 3
25
Vậy a  b  .
3
Câu 36. (Sở Lạng Sơn 2022) Cho hàm số f  x  có bảng biến thiên như sau:

3 2
Hàm số y   f  x    3  f  x   đồng biến trên khoảng nào dưới đây ?
A.   ;1 . B. 1;2  . C.  3;4  . D.  2;3 .
Lời giải
Chọn C
Ta có y  3 f   x   f 2  x   2 f  x   .
 f  x  0

Phương trình y  0   f  x   0 .
f x 2
  

Dựa vào bảng biến thiên, ta thấy: f   x   0  x  1;2;3;4 ;


f  x   0  x  a  1 hoặc x  4 ;
 x  b  a  b  1

x  c  1;2 
f  x  2   .
x  3

 x  d  4
Ta lập được bảng xét dấu của y :

Trang 30 Fanpage Nguyễn Bảo Vương  https://www.facebook.com/tracnghiemtoanthpt489/


Điện thoại: 0946798489 TUYỂN CHỌN VẬN DỤNG – VẬN DỤNG CAO 2022

Từ bảng xét dấu, ta thấy hàm số đồng biến trên các khoảng   ;a  ;  b ;1 ;  c ;2  ;  3;4  và
 d ;   .
Như vậy phương án đúng là C.
Câu 37. (Sở Phú Thọ 2022) Cho hàm bậc bốn y  f  x  có đạo hàm liên tục trên  , hàm số y  f   x 
có đồ thị như hình vẽ.

Gọi S là tập các giá trị nguyên của tham số m để hàm số y  f  4  2 x  m  6  có đúng 3 điểm
cực tiểu. Tổng các phần tử của S bằng
A. 18 . B. 11 . C. 2 . D. 13 .
Lời giải
Chọn B
+ Đồ thị của hàm số y  f  4  2 x  m  6  đối xứng qua đường thẳng x  2 .
+ Xét hàm số y  f  2 x  m  10  .
 9m
x  2  a
 2 x  m  10  1 
 11  m
y   2 f   2 x  m  10   0   2 x  m  10  1   x  b.
 2
 2 x  m  10  4 
 x  14  m  c
 2

+ Hàm số y  f  4  2 x  m  6  có ba điểm cực tiểu khi và chỉ khi:


Câu 38. (Sở Phú Thọ 2022) Cho hàm số y  f  x  có đồ thị như hình vẽ

Facebook Nguyễn Vương https://www.facebook.com/phong.baovuongTrang 31


Blog: Nguyễn Bảo Vương: https://www.nbv.edu.vn/

 
Số nghiệm thực phân biệt của phương trình 2 f x  1  2 x  1  3 là
A. 12 . B. 5 . C. 8 . D. 4 .
Lời giải
Chọn B
 
Xét phương trình 2 f x  1  2 x  1  3 1 .

Đặt t  x  1  2 x  1 , với x  1.
1
Ta có t   1  ; t  0  x 1  1  x  2
x 1
Bảng biến thiên của hàm t  t  x 

Suy ra với x  1 thì t  1 .


 3
3  f t   2
Khi đó, phương trình 1 trở thành 2 f  t   3  f  t    
2  f t    3
 2

Trang 32 Fanpage Nguyễn Bảo Vương  https://www.facebook.com/tracnghiemtoanthpt489/


Điện thoại: 0946798489 TUYỂN CHỌN VẬN DỤNG – VẬN DỤNG CAO 2022

t  a    x0 ; 1

3 t  b   1;0 
*) Trường hợp 1: f  t    
t  c   0;1
2
t  d  1; x 
 0

3 t  e   ;  x0 
*) Trường hợp 2: f  t     
2 t  f   x0 ;  

Dựa vào bảng biến thiên của hàm số t ta có


Với t  a    x0 ; 1  phương trình 1 vô nghiệm.

Với t  b   1; 0   phương trình 1 có 2 nghiệm phân biệt x1 , x2 .

Với t  c   0;1  phương trình 1 có 1 nghiệm x3 .

Với t  d  1; x0   phương trình 1 có 1 nghiệm x4 .

Với t  e   ;  x0   phương trình 1 vô nghiệm.

Với t  f   x0 ;    phương trình 1 có 1 nghiệm x5 .

Các nghiệm x1 , x2 , x3 , x4 , x5 không trùng nhau.


Vậy phương trình đã cho có 5 nghiệm.
x 1
Câu 39. (Sở Thái Nguyên 2022) Cho hàm số y  có đồ thị  C  . Gọi  là tiếp tuyến thay đổi của
2x 1
đồ thị  C  . Khoảng cách từ giao điểm hai đường tiệm cận của đồ thị  C  đến đường thẳng  đạt
giá trị lớn nhất bằng
2
A. . B. 1 . C. 2 . D. 5 .
2
Lời giải
Chọn A
1 1
Tọa độ giao điểm của hai đường tiệm cận là I  ;  .
2 2

Facebook Nguyễn Vương https://www.facebook.com/phong.baovuongTrang 33


Blog: Nguyễn Bảo Vương: https://www.nbv.edu.vn/
 a 1  1
Gọi tọa độ tiếp điểm là M  a ;  , a  . Khi đó phương trình tiếp tuyến  với đồ thị hàm
 2a  1  2
1 a 1 2
số tại điểm M là: y  2 
x  a   x   2a  1 y  2a 2  4a | 1  0 .
 2a  1 2a  1
1 1 2
  2a  1  2a 2  4a  1
2 2 2a  1 2a  1 2
Khi đó: d  I ,       .
1   2a  1
4
1   2a  1
4
2 2a  1 2

2  2a  1  1 a  1
Dấu "  " xảy ra khi và chỉ khi  2a  1  1    .
 2a  1  1  a  0
2
Vậy max d  I ,    .
2
ax  b
Câu 40. (Sở Thái Nguyên 2022) Cho hàm số y  f  x   có đồ thị hàm số f   x  như trong hình
cx  d
vẽ sau:

Biết rằng đồ thị hàm số f  x  đi qua điểm A  0;2  . Giá trị f  3 bằng
A. 2 . B. 1 . C. 3 . D. 5 .
Lời giải
Chọn D
ax  b ad  bc
Ta có: y  f  x    f  x  2
.
cx  d  cx  d 
b
Vì đồ thị thị hàm số f  x  đi qua điểm A  0;2  suy ra f  0   2   2  b  2d 1 .
d
  1
 f  0    2 
ad  bc 1
2
   2
Dựa vào đồ thị hàm số f   x  ta có:   d 2 .
d
x    2 
 c d  2c  3
ad  2dc 1 a  2c 1 1 1
Thay 1 vào  2  : 2
     a   d  2c   .  2c   2c  3c .
d 2 d 2 2 2
3a  b 3.3c  4c 5c
Vậy, f  3     5.
3c  d 3c  2c c
Câu 41. (Sở Thái Nguyên 2022) Cho hàm số y  f  x  liên tục trên  có đạo hàm
f   x    x  1  2 x 2  3 x  9  , x   . Hàm số g  x   f  x   x  3x  9 x  6 có bao nhiêu điểm
3 2

cực trị?
A. 2 . B. 1. C. 0 . D. 3 .

Trang 34 Fanpage Nguyễn Bảo Vương  https://www.facebook.com/tracnghiemtoanthpt489/


Điện thoại: 0946798489 TUYỂN CHỌN VẬN DỤNG – VẬN DỤNG CAO 2022
Lời giải
Chọn D
Vì hàm số y  f  x  liên tục trên  nên hàm số g  x   f  x   x3  3x2  9 x  6 liên tục trên  .
Ta có: g   x   f   x   3x 2  6 x  9   x  1  2 x 2  3x  9   3  x  1 x  3
  x  1 x  3 2 x  6 .
 x  1
g   x   0   x  3 .
 x  3
Ta có bảng biến thiên

Từ bảng biến thiên suy ra hàm số có 3 điểm cực trị.


Câu 42. (Sở Thái Nguyên 2022) Gọi S là tập hợp tất cả các giá trị nguyên của tham số m sao cho
2 x3  3 x 2  m  16, x   0;3. Tổng tất cả các phần tử của S bằng
A. 65 . B.  74 . C. 42 . D. 87 .
Lời giải
Chọn A
Xét f  x   2 x3  3x 2  m , với x  0;3 .
x  0
Ta có: f   x   6x 2  6 x ; f   x   0   .
 x 1
f  0  m ; f 1  m  1 ; f  3  27  m .
Do đó: f  x    m  1; m  27 .
 m  1  16 m  15
Vậy: f  x   16     m  15; 11 .
m  27  16  m  11
m    m  15;  14;  13;  12;  11 .
Ta có:  15   14   13   12   11  65 .
Câu 43. (Sở Thái Nguyên 2022) Cho hàm số đa thức bậc bốn y  f  x  có bảng biến thiên như sau

2
 
Số điểm cực trị của hàm số g  x   x3  x  f  x  1  là
A. 11. B. 8. C. 13. D. 10.
Lời giải
Chọn D
Từ bảng biến thiên ta thấy rằng f ( x )  0 có 4 nghiệm phân biệt, gọi 4 nghiệm đó lần lượt là
x1 , x2 , x3 , x4 với x1  1  x2  0  x3  1  x4 . Khi đó:
2
 
g ( x)  x 3  x  a  x  1  x1  x  1  x2  x  1  x3  x  1  x4   (với a  0 ).

Facebook Nguyễn Vương https://www.facebook.com/phong.baovuongTrang 35


Blog: Nguyễn Bảo Vương: https://www.nbv.edu.vn/
Ta có g ( x)  0  x  0; 1; x1  1; x2  1; x3  1; x4  1 , trong đó x1  1, x2  1, x3  1, x4  1 là các
nghiệm kép. Ta có bảng biến thiên của g  x  như sau:

Vậy g ( x ) có 10 điểm cực trị.


1
Câu 44. (Sở Thái Nguyên 2022) Cho hàm số f ( x)  log 3 x  3x  3 x . Tổng bình phương các giá trị của
 1 
tham số m để phương trình f 
4 | x  m | 3
 2

  f x  4 x  7  0 có đúng 3 nghiệm thực phân
 
biệt bằng
A. 14. B. 13. C. 10. D. 5.
Lời giải
Chọn A
1 x 1 1x
Ta có f   x    3  ln 3  2  3  ln 3  0, x  0
x ln 3 x
 Hàm số y  f  x  đồng biến trên  0;  1 .
1 1
1 1  
Mặt khác f    log 3  3 x  3x    log 3 x  3 x  3x    f  x  , khi đó
x x  
 1 
f
4 | x  m | 3
2
 2

  f x  4 x  7  0   f (4 | x  m | 3)  f x  4 x  7  0 
 

 f  4 | x  m | 3  f x 2  4 x  7  2 .
 4m   x 2  8 x  4
Từ 1 ,  2   4 x  m  3  x 2  4 x  7   2
.
 4m  x  4
Ta có đồ thị sau:

Trang 36 Fanpage Nguyễn Bảo Vương  https://www.facebook.com/tracnghiemtoanthpt489/


Điện thoại: 0946798489 TUYỂN CHỌN VẬN DỤNG – VẬN DỤNG CAO 2022

 4m  4 m  1

Theo yêu cầu bài toán tương đương  4m  8   m  2 . Vậy 12  2 2  32  14 .

 4m  12  m  3
Câu 45. (Sở Vĩnh Phúc 2022) Có bao nhiêu giá trị của tham số m để đồ thị hàm số
x 1
y 2 có đúng hai đường tiệm cận?
x   2m  1 x  m2  3
A. 1 . B. 2 . C. 3 . D. 0 .
Lời giải
Chọn C
x 1
Đồ thị hàm số y  2 có 1 tiệm cận ngang là y  0 .
x   2m  1 x  m2  3
x 1
Đồ thị hàm số y  có đúng hai đường tiệm cận
x   2m  1 x  m2  3
2

x 1
 Đồ thị hàm số y  có đúng 1 tiệm cận đứng
x   2m  1 x  m2  3
2

 Phương trình x 2   2m  1 x  m2  3  0 có nghiệm kép hoặc phương trình x 2   2m  1 x  m2  3  0


có hai nghiệm phân biệt trong đó có một nghiệm bằng 1.
 0   2m  12  4  m 2  3  0  13
  m
 4
  0    2m  1 2  4  m2  3  0   .
   m3
2
 1   2m  1  m  3  0 
   m 2  2m  3  0  m  1
Vậy có ba giá trị của m thỏa mãn yêu cầu đề bài.
Câu 46. (Sở Vĩnh Phúc 2022) Cho hàm số f  x  có đạo hàm f   x  liên tục trên  và đồ thị f   x  như
hình vẽ dưới đây.

Facebook Nguyễn Vương https://www.facebook.com/phong.baovuongTrang 37


Blog: Nguyễn Bảo Vương: https://www.nbv.edu.vn/

1
Bất phương trình f  x   x 3  x 2  3 x  m  0 nghiệm đúng với mọi x   0; 2  khi và chỉ khi
3
22 22
A. m  f  0  . B. m  f  2   . C. m  f  0  . D. m  f  2   .
3 3
Lời giải
Chọn C
1 1
Bất phương trình f  x   x 3  x 2  3 x  m  0  f  x   x 3  x 2  3 x  m 1
3 3
1 3
Xét g  x   f  x   x  x 2  3 x, x   0; 2  có g   x   f   x   x 2  2 x  3 .
3
Ta thấy trên khoảng  0; 2  thì f   x   3 và  x 2  2 x  3  3  g   x   0, x   0;2  .
Do đó g  x  là hàm đồng biến trên khoảng  0; 2  .
Bất phương trình (1) nghiệm đúng với mọi x   0;2  khi và chỉ khi m  g  0   f  0  .
2 x  m2
Câu 47. (Sở Vĩnh Phúc 2022) Cho hàm số f  x   , với m là tham số. Gọi m1 , m2  m1  m2  là
x 1
các giá trị của tham số m thỏa mãn 2 max f  x   min f  x   8 . Tổng 2m1  3m2 bằng
0;2 0;2
A. 1. B. 2 . C. 4 . D. 1 .
Lời giải
Chọn C
2  m2
Ta có: f   x   2
 0, x   0; 2 .
 x  1
4  m2
 Min f  x   f  0    m 2 ; Max f  x   f  2  
0;2 0;2 3
Do đó:
 4  m2  2
2 max f  x   min f  x   8  2    m  8.
0;2 0;2  3 
 m  4
 m 2  16  0  
m  4
Vậy 2m1  3m2  2.  4   3.4  4 .
Câu 48. (Sở Vĩnh Phúc 2022) Một người thợ cần thiết kế một bể cá hình hộp chữ nhật bằng kính, có
chiều cao là 0,8m , thể tích 576dm3 . Biết rằng phần nắp phía trên của bể cá người thợ đó để trống
một ô có diện tích bằng 30% diện tích đáy bể. Biết rằng loại kính mà người thợ sử dụng làm mặt
bên và nắp bể có giá thành 1000000 đồng/m2 và kính để làm mặt đáy có giá thành 1200000

Trang 38 Fanpage Nguyễn Bảo Vương  https://www.facebook.com/tracnghiemtoanthpt489/


Điện thoại: 0946798489 TUYỂN CHỌN VẬN DỤNG – VẬN DỤNG CAO 2022
đồng/m2. Giả sử phần tiếp xúc giữa các mặt là không đáng kể. Số tiền mua kính ít nhất để hoàn
thành bể cá gần nhất với số tiền nào dưới đây?
A. 4,1 triệu đồng. B. 3, 2 triệu đồng. C. 2,8 triệu đồng. D. 3,8 triệu đồng.
Lời giải
Chọn A
Đặt x, y  m  x, y  0  lần lượt là chiều dài và chiều rộng của đáy bể cá.

18
Ta có: V  x. y.h  0,8 xy  0,576  m3   y  .
25 x
 70 
Số tiền cần để mua kính là:  2.0,8  x  y   xy .1  xy.1, 2
 100 
144 171
 1, 6 x    f  x  (triệu đồng).
125 x 125
144 200 x 2  144 3 2
Ta có: f   x   1, 6  2
 2
; f  x  0  x  m .
125 x 125 x 5
3 2 
Lập BBT của hàm f  x  , ta thấy min f  x   f  triệu đồng.
 5   4,1
 
Câu 49. (Sở Vĩnh Phúc 2022) Cho hàm số y  f  x  có đạo hàm f   x  liên tục trên  và có bảng xét
dấu như hình sau. Hàm số g  x   f  x 2  3x  1 đồng biến trên khoảng nào dưới đây

A.  0;1 . B.  4; 2  . C.  1;0  . D.  2; 1 .


Lời giải
Chọn A
g   x    2 x  3 f   x 2  3 x  1 .
 3
 x
 3 2
x   2 
x  1
 
g   x   0   2 x  3 f   x 2  3 x  1  0   x 2  3 x  1  1   x  2 .
 2 
 x  3 x  1  1 x  0
  x  3


Bảng xét dấu g   x    2 x  3 f   x  3 x  1
2

Từ bảng xét dấu suy ra hàm số g  x   f  x 2  3x  1 đồng biến trên khoảng  0;1 .
Câu 50. (Sở Vĩnh Phúc 2022) Cho hàm số y  f  x  liên tục trên  và có bảng xét dấu f   x  như sau

Facebook Nguyễn Vương https://www.facebook.com/phong.baovuongTrang 39


Blog: Nguyễn Bảo Vương: https://www.nbv.edu.vn/

Số điểm cực trị của hàm số g  x   f  x 3  2 x  4  là

A. 2 . B. 3 . C. 5 . D. 10 .

Lời giải
Chọn B
Dựa vào bảng xét dấu f   x  ta có hàm số f  x  có 2 cực trị tại x  1 và x  4 .

 f  x3  2 x  4  khi x  0

Xét g  x   f  x  2 x  4   
3

 f  x  2 x  4  khi x  0
3

 3 x 2  2  . f   x3  2 x  4  khi x  0

g  x   .
 3 x  2  . f   x  2 x  4  khi x  0
2 3

Xét g   x   0

TH1: x  0

 x 3  2 x  4  1  x 3  2 x  3  0  x 1 0
g   x   0   3 x 2  2  . f   x3  2 x  4   0   3  3 
 x  2x  4  4  x  2x  8  0  x  1, 67  0

Với x  0 có 2 cực trị.

TH2: x  0

 2
 2 2  x
 3 x2  2  0  x 3  3
  
g  x   0  3x2  2 . f  x3  2 x  4  0   x3  2 x  4  1   x3  2 x  3  0   x  2 (lo¹i)
   
 x3  2 x  4  4  x3  2 x  8  0  3
   x  1.89 (lo¹i)
 
 x  2,33 (lo¹i)

Với x  0 có 1 cực trị.

Vậy hàm số g  x   f  x 3  2 x  4  có 3 cực trị.

Câu 51. (THPT Bùi Thị Xuân – Huế - 2022) Cho hàm số y  f ( x) là hàm đa thức bậc bốn. Đồ thị hàm
số f ( x  2) được cho trong hình vẽ bên

Trang 40 Fanpage Nguyễn Bảo Vương  https://www.facebook.com/tracnghiemtoanthpt489/


Điện thoại: 0946798489 TUYỂN CHỌN VẬN DỤNG – VẬN DỤNG CAO 2022

Hàm số g ( x)  4 f  x 2   x 6  5 x 4  4 x 2  1 đồng biến trên khoảng nào dưới đây?


A. ( 4; 3) .
B. (2;  ) .
C. (  2; 2) .
D. (2; 1) .
Lời giải
g ( x)  4 f  x   x  5 x  4 x  1  g ( x)  8 xf   x 2   6 x 5  20 x3  8 x
2 6 4 2

g ( x)  0  8 xf   x 2   6 x 5  20 x 3  8 x  0  2 x  4 f   x 2   3x 4  10 x 2  4   0

2x  0 x  0
  
 4 f  ( x 2 )  3x 4  10 x 2  4  0  f   x2   3 x4  5 x2  1
  4 2
Xét:
3 5
f   x 2   x 4  x 2  1 . Đặt x 2  t  2 , ta có:
4 2
3 5 3 5 3 1
f (t  2)  (t  2) 2  (t  2)  1   t 2  4t  4   (t  2)  1  t 2  t  1
4 2 4 2 4 2
Khi đó số nghiệm của phương trình chính là số giao điểm của đồ thị hàm số y  f (t  2) và
3 1
y  t 2  t 1
4 2
Ta có đồ thị ta có:

 t  2  x  2  2  x  4
3 2 1  
Dựa vào đồ thị ta có: f (t  2)  t  t  1  t  0   x  2  0   x  2
4 2 t  2  x  2  2  x  0

Facebook Nguyễn Vương https://www.facebook.com/phong.baovuongTrang 41


Blog: Nguyễn Bảo Vương: https://www.nbv.edu.vn/
Ta có bảng xét dấu g ( x) như sau:

Vậy hàm số g ( x)  4 f  x 2   x 6  5 x 4  4 x 2  1 đồng biến trên khoảng (2;  ) .


Câu 52. (THPT Bùi Thị Xuân – Huế - 2022) Cho hàm số y  f ( x ) là hàm số bậc ba và có đồ thị
y  f (2  x ) như hình vẽ.

 
Hỏi phương trình f x 2  2 x  1 có tất cả bao nhiêu nghiệm?
A. 8.
B. 7.
C. 9.
D. 6.
Lời giải

 2  x  a (0  a  1)  x  2  a  a1 1  a1  2 
 
f (2  x)  1   2  x  2  x  0
 2  x  c (c  3)  x  2  c  c1  c1  1

2  x  1 x 1
f (2  x)  1   
 2  x  d (d  c  3)  x  2  c  d1  d1  c1  1

 f x 2  2 x  1(1)
 
f x  2x  1  
2
 f x 2  2 x  1(2)
  
 x 2  2 x  a1 , (3) 1  a1  2 

 
f x  2 x  1   x 2  2 x  0(4)
2


 x 2  2 x  c1  c1  1 (VN )

Trang 42 Fanpage Nguyễn Bảo Vương  https://www.facebook.com/tracnghiemtoanthpt489/
Điện thoại: 0946798489 TUYỂN CHỌN VẬN DỤNG – VẬN DỤNG CAO 2022
2
 x  2 x  1(5)
 
f x 2  2 x  1  
 x 2  2 x  d1  d1  c1  1 (VN )

Dựa vào đồ thị thì (3) có 2 nghiệm phân biệt, (4) có 2 nghiệm, (5) có 3 nghiệm.

Các nghiệm này khác nhau đôi 1. Vậy phương trình f x 2  2 x  1 có 7 nghiệm. 
Câu 53. (THPT Bùi Thị Xuân – Huế - 2022) Cho hàm số y  f ( x) có đạo hàm
f ( x)  ( x  1) 2
 x  2 x  với x   . Có bao nhiêu giá trị nguyên dương của tham số m để hàm
2

số f  x  8 x  m  có 5 điểm cực trị?


2

A. 15.
B. 16.
C. 17.
D. 18.
Lời giải
Đặt g ( x)  f  x  8 x  m   g ( x)  (2 x  4)  f   x 2  8 x  m  .
2

2
f ( x )  ( x  1)2  x 2  2 x   g ( x )  (2 x  8)  x 2  8 x  m  1  x 2  8 x  m  x 2  8 x  m  2 
x  4
 2
x  8x  m  1  0
g ( x)  0   2
 x  8x  m  0

 x 2  8 x  m  2  0
2
Các phương trình (1), (2), (3) không có nghiệm chung từng đôi một và  x 2  8 x  m  1  0 với
x  
Suy ra g ( x ) có 5 điểm cực trị khi và chỉ khi (2) và (3) có hai nghiệm phân biệt khác 4
16  m  0 m  16
 
16  m  2  0 m  18
   m  16.
16  32  m  0 m  16
16  32  m  2  0 m  18
m nguyên dương và m  16 nên có 15 giá trị m cần tìm.

Facebook Nguyễn Vương https://www.facebook.com/phong.baovuongTrang 43


Blog: Nguyễn Bảo Vương: https://www.nbv.edu.vn/
Câu 54. (Chuyên Hạ Long 2022) Cho các số thực x, y thoả mãn
max{5;9 x  7 y  20}  x 2  y 2  2 x  8
 .Gọi M , m lần lượt là giá trị lớn nhất và giá trị nhỏ nhất
y 1
của biểu thức P  x  2 y . Tính M  m
A. 1  3 5 .
B. 2 2 .
C. 1  2 2 .
D. 2  3 5 .
Lời giải


 2 2
x  y  5

Từ giả thiết ta có ( x  1) 2  y 2  9 .
 2 2
 x  9    y  7   25
  
2 

2 2

Tập hợp điểm ( x, y ) thoả mãn yêu cầu bài là phần được tô trên hình vẽ kể cả biên. Ta thấy  C1 
cắt  C3  tại hai điểm phân biệt trong đó có điểm (2,1) thoả mãn yêu cầu bài toán.
Xét đường thẳng  đi qua ( x, y ) thoả mãn yêu cầu bài toán: x  2 y  c .
x  2 y đạt GTNN khi  đi qua (2,1) nên m  0 .
 C2  : x2  y 2  2 x  8  ( x  1)2  y 2  9 .
 x  2 y  ( x  1)  (2) y  1  1  (2)   9  1  3
2
5 1.
1 : x  2 y  1  3 5  0.1 cắt  C2  tại điểm thoả mãn bài toán.
Khi đó M  3 5  1 .
Vậy M  m  3 5  1 .
mx 2  (m  2) x  5
Câu 55. (Chuyên Hạ Long 2022) Cho hàm số y  . Gọi S là tập hợp các giá trị của
x2  1
m sao cho đồ thị hàm số đã cho có đúng hai điểm cực trị và đường thẳng nối hai điểm cực trị của

Trang 44 Fanpage Nguyễn Bảo Vương  https://www.facebook.com/tracnghiemtoanthpt489/


Điện thoại: 0946798489 TUYỂN CHỌN VẬN DỤNG – VẬN DỤNG CAO 2022
25
đồ thị hàm số cắt hai trục tọa độ tạo thành một tam giác có diện tích bằng . Tính tổng các phần
4
tử của S
A. 0.
B. 1
C. 4
D. 2 .
Lời giải

(m  2) x  5  m (m  2) x 2  2(m  5) x  m  2
Ta có: y  m   y   2
.
x2  1  
x2  1
 x1 x2  1

Với m  2 ta có y  0 có hai nghiệm phân biệt x1 , x2 thỏa  2(m  5) .
 x1  x2 
m2
Mặt khác, đường thẳng qua hai điểm cực trị của đồ thị hàm số là
2( m  2) x  4.5.1 m  2
: y   x 5.
4.1 2
 10 
Gọi A    Ox  A   ; 0  và B    Oy  B (0;5) .
 m2 
25 1 25 5 10 25 m  2
Do đó: S O.4 B    OB.OA     | m  2 | 4   .
4 2 4 2 | m2| 4  m  6
Do đó m1  m2  4 .
Câu 56. (Chuyên Hoàng Văn Thụ - Hòa Bình – 2022) Cho hàm số y  f ( x ) liên tục trên  và thỏa
mãn f ( 4)  4 . Đồ thị hàm số y  f ( x ) như hình vẽ bên dưới. Để giá trị lớn nhất của hàm số

x2
h( x)  f ( x)   x  3m trên đoạn [ 4;3] không vượt quá 2022 thì tập giác trị của m là
2
A. (; 2022] .
B. (674;  ) .
C. (;674] .
D. (2022;  ) .
Lời giải

Facebook Nguyễn Vương https://www.facebook.com/phong.baovuongTrang 45


Blog: Nguyễn Bảo Vương: https://www.nbv.edu.vn/

h( x )  f ( x )  ( x  1)
Trên (4;1), h( x )  0 , trên (1;3), h( x )  0, h(1)  0
Hàm số h( x) đạt cực tiểu trên đoạn [ 4;3] tại x  1
15
a  h(4)  3m; b  h(3)  f (3)   3m
2
1 3
Gọi S1    x  1  f '  x  dx; S2    f  x    x  1  dx
4 1
Nhận thấy
1 3
 x2   x2 
S1  S 2    x  f ( x)    f ( x)   x 
 2  4  2 1
1 12 7 15
  f (1)  4  f ( 4)  f (3)   f (1)  f ( 4)  f (3)   f (3) 
2 2 2 2
Vậy, b  a, max h( x)  a  3m  2022  m  674
x[ 4;3]

Vậy, tập giá trị của m , là ( ; 674] .


Câu 57. (Chuyên Lam Sơn 2022) Cho hàm số f ( x)  x 4  2 x 3  (m  1) x 2  2 x  m  2022 , với m là
tham số. Có bao nhiêu giá trị nguyên của m thuộc đoạn [ 2021; 2022] để hàm số
y | f ( x  2021)  2022 | có số điểm cực trị nhiều nhất?
A. 2021.
B. 2022.
C. 4040.
D. 2023.
Lời giải
Hàm số y | f ( x  2021)  2022 | có số điểm cực trị nhiều nhất là 7 khi và chỉ khi phương trình
f ( x  2021)  2022 có 4 nghiệm phân biệt hay phương trình f ( x )  2022 có 4 nghiệm phân biệt
Ta có f ( x)  2022  x 4  2 x3  (m  1) x 2  2 x  m  0
 x  1
2 
 ( x  1)( x  1)  x  2 x  m   0   x  1
 x 2  2 x  m  0(*)
Suy ra f ( x )  2022 có 4 nghiệm phân biệt khi và chỉ khi   có 2 nghiệm phân biệt khác 1 và 1
*

tức là
1  m  0
2 m  1
1  2  m  0   do m nguyên thuộc [ 2021; 2022] nên có 2021 giá trị thỏa mãn.
12  2  m  0 m  3

Câu 58. (Chuyên Lam Sơn 2022) Cho hàm số bậc bốn y  f ( x ) có đồ thị như hình vẽ dưới đây.

Trang 46 Fanpage Nguyễn Bảo Vương  https://www.facebook.com/tracnghiemtoanthpt489/


Điện thoại: 0946798489 TUYỂN CHỌN VẬN DỤNG – VẬN DỤNG CAO 2022

Có bao nhiêu giá trị nguyên của tham số m  [ 2021; 2021] để phương trình
2 2
f 2
( x)  x   m 2
 2m  14  f 2

( x)  x 2  4( m  1) 2  36  0 có đúng 6 nghiệm phân biệt.
A. 2022.
B. 4043.
C. 4042.
D. 2021.
Lời giải

Đặt t  f 2 ( x)  x 2 , (t  0) ta có phương trình


t  4
t 2   m2  2m  14  t  4(m  1) 2  36  0   2
t  m  2m  10
+ Với t  4 hay f 2 ( x)  x 2  4  f 2 ( x)  4  x 2  f ( x)  4  x 2 (Do f ( x)  0  .
Số nghiệm của phương trình f ( x)  4  x 2 là số giao điểm của đường cong y  f ( x ) và nửa
đường tròn C (O; 2)

Dựa vào đồ thị ta thấy phương trình có 4 nghiệm phân biệt.


t  m 2  2m  10 hay f 2 ( x)  x 2  m 2  2m  10  f 2 ( x)  m 2  2m  10  x 2
 f ( x )  m 2  2m  10  x 2 (Do f ( x)  0).
Số nghiệm của phương trình là số giao điểm của đường cong y  f ( x ) và nửa đường tròn


C O; m 2  2m  10 

Facebook Nguyễn Vương https://www.facebook.com/phong.baovuongTrang 47


Blog: Nguyễn Bảo Vương: https://www.nbv.edu.vn/

2
f 2
( x)  x 2   m 2
 2m  14  f 2

( x)  x 2  4( m  1) 2  36  0 chỉ có 6 nghiệm phân biệt thì

phương trình f ( x)  m 2  2m  10  x 2 chỉ có 2 nghiệm phân biệt.Dựa vào đồ thị ta có điều kiện
m 2  2m  10  9  m 2  2m  1  0  m  1 . Vậy có 4042 giá trị của m   2021; 2021 .
Câu 59. (Chuyên Lam Sơn 2022) Cho hàm số bậc ba y  f ( x ) có đồ thị như hình vẽ

Tìm số điểm cực trị của hàm số y  f 2 ( g ( x)) với g ( x)  x 2  4 x  2 4 x  x 2


A. 17.
B. 21.
C. 23.
D. 19.
Lời giải

Xét hàm số g ( x)  x 2  4 x  2 4 x  x 2 :
TXĐ :[0; 4]
Trang 48 Fanpage Nguyễn Bảo Vương  https://www.facebook.com/tracnghiemtoanthpt489/
Điện thoại: 0946798489 TUYỂN CHỌN VẬN DỤNG – VẬN DỤNG CAO 2022
2
2(2  x) 4x  x 1
g ( x)  2 x  4   2( x  2) , x  (0; 4) ;
2
4x  x 4 x  x2
x  2 x  2
g ( x)  0   2
 
 4 x  x  1  x  2  3
y  f 2 ( g ( x))  y  2 f ( g ( x))  g ( x ) f ( g ( x)) ;

 f ( g ( x))  0 1

y  0   g ( x)  0  2 
 f ( g ( x ))  0 3
  
 g ( x)  a(4)
(1)   g ( x)  b(5)(0  a  b  1)
 g ( x)  1(6)
 g ( x)  c(7)
(3)   (0  a  c  b  d  1)
 g ( x)  d (8)
Mỗi phương trình (4), (5), (7), (8) có 4 nghiệm phân biệt
Phương trình (6) có nghiệm kép x  1
Phương trình (2) có 3 nghiệm phân biệt
Tất cả các nghiệm của các phương trình (2), (4), (5), (7), (8) là phân biệt và y đổi dấu qua các
nghiệm đó.
y không đổi dấu qua x  1 .
Vậy hàm số đã cho có 19 điểm cực trị.
Câu 60. (Chuyên Lam Sơn 2022) Cho hàm đa thức bậc bốn y  f ( x ) . Biết đồ thị của hàm số
y  f (3  2 x ) được cho như hình vẽ

Hàm số y  f ( x ) nghịch biến trên khoảng


A. (; 1) .
B. ( 1;1) .
C. (1;5) .
D. (5;  ) .
Lời giải
Ta có: f (3  2 x )  ax ( x  1)( x  2) ( a  0) .
Với x  0 thì f (3)  0 .
Với x  1 thì f (1)  0 .
Với x  2 thì f (1)  0 .
 x3
Suy ra: f ( x)  0   x  1 .
 x  1

Facebook Nguyễn Vương https://www.facebook.com/phong.baovuongTrang 49


Blog: Nguyễn Bảo Vương: https://www.nbv.edu.vn/
1
Với x   thì f (4)  0 .
2
Bảng biến thiên:

Vậy hàm số y  f ( x ) nghịch biến trên khoảng ( ; 1) và (1;3) .


Câu 61. (Chuyên Lam Sơn 2022) Cho hàm số y  f ( x) liên tục trên  và có bảng biến thiên như hình
vẽ

Đặt g ( x ) | m  f (2022  x ) | . Có bao nhiêu giá trị nguyên của tham số m để hàm số y  g ( x ) có
đúng 5 điểm cực trị?
A. 6.
B. 8.
C. 9,
D. 7.
Lời giải
Đặt h( x )  m  f (2022  x )
Số điểm cực trị của g ( x ) sẽ bằng số điểm cực trị của h( x ) cộng với số nghiệm bội lẻ của phương
trình h( x )  0 (Nghiệm bội lẻ này phải khác điểm cực trị của hàm số).
Số điểm CT của h( x) bằng số điểm CT của f ( x ) . Nên hàm số h( x) có 2 điểm cực trị. Vậy để
hàm số g ( x ) có 5 điểm cực trị thì pt h( x )  0 , phải có 3 nghiệm lẻ phân biệt.
h( x )  0  f ( x  2022)   m.
BBT của hàm số y  f ( x  2022) :

Ycbt 5  m  3  3  m  5 . Do m    m  {2; 1;; 4} .


Vậy có 7 giá trị m thỏa mãn ycbt.
Câu 62. (Chuyên Lương Văn Tụy – Ninh Bình 2022) Cho hàm số y  f ( x ) . Hàm số f ( x ) có bảng
biến thiên như hình vẽ sau Giá trị lớn nhất của hàm số g ( x)  f (2 x)  sin 2 x trền đoạn [ 1;1] là

Trang 50 Fanpage Nguyễn Bảo Vương  https://www.facebook.com/tracnghiemtoanthpt489/


Điện thoại: 0946798489 TUYỂN CHỌN VẬN DỤNG – VẬN DỤNG CAO 2022

A. f (1) .
B. f (0) .
C. f (2) .
D. f ( 1) .
Lời giải

g ( x )  2 f (2 x )  2sin x  cos x  2 f (2 x )  sin 2 x. Đặt t  2 x  t  [2; 2].


sin t
g (t )  0  2 f (t )  sin t  0  f (t )  , t  [2; 2].
2

Vậy giá trị lớn nhất là g  0   f  0 


Câu 63. (Chuyên Lương Văn Tụy – Ninh Bình – 2022) Cho hàm số y  f ( x ) liên tục trên R và có đồ
thị có 3 điểm cực trị như hình dưới đây. Số điểm cực trị của hàm số g ( x )  f  x 3  3 x  2  là

A. 5.
B. 9.
C. 11.

Facebook Nguyễn Vương https://www.facebook.com/phong.baovuongTrang 51


Blog: Nguyễn Bảo Vương: https://www.nbv.edu.vn/
D. 7.
Lời giải

 x  1
 3
 x  3 x  2  m1 1
 2
  3

Ta có g ( x)  3 x  3 f  x  3 x  2 , g ( x)  0   3
x  3 x  2  m2  2 

 x3  3 x  2  m  3
 3

m1  (4; 1); m2  (1;0); m3  (0;1)


Xét hàm số y  x3  3x  2 , có y  3x 2  3

Với m1  (4; 1)  (1) có 1 nghiệm


Với m2  (1;0)  (2) có 1 nghiệm
Với m3  (0;1)  (3) có 3 nghiệm phân biệt
Vậy g ( x)  0 có 7 nghiệm bội lẻ, nên có 7 điểm cực trị.
Câu 64. (Chuyên Nguyễn Trãi – Hải Dương-2022) Cho hàm số y  f ( x ) như hình vẽ. Biết rằng
f (3)  2 f (5)  4 . Hỏi có tất cả bao nhiêu giá trị nguyên của tham số m để phương trình

1 
f  f ( x)  m   2 x  2m có đúng 3 nghiệm thực phân biệt.
 2 
A. 8
B. 6.
C. 3.
D. 7.
Lời giải
1  f ( x )  2u  2 m
Đặt f ( x)  m  u    f (u )  2u  f ( x )  2 x .
2  f (u )  2 x  2m
Xét hàm số g (t )  f (t )  2t  g (t )  f (t )  2  2  2  0 x   .
1 1
Do đó hàm số g (t ) đồng biến trên   u  x  f ( x)  m  x  h( x)  f ( x)  x  m .
2 2

Trang 52 Fanpage Nguyễn Bảo Vương  https://www.facebook.com/tracnghiemtoanthpt489/


Điện thoại: 0946798489 TUYỂN CHỌN VẬN DỤNG – VẬN DỤNG CAO 2022
1 1
Xét hàm số h( x)  f ( x)  x  h( x)  f ( x)  1 .
2 2
 1
 x  3  h(3)  2 f (3)  (3)  5
1 
 h( x)  0  f ( x)  1  f ( x)  2   x  0
2  1
 x  5  h(5)  f (5)  (5)  4
 2
1
Ta có bảng biến thiên của hàm số h( x)  f ( x)  x như sau:
2

Dựa vào bảng biến thiên ta suy ra phương trình có 3 nghiệm khi 4  m  5 .
Do m    m  {3; 2; 1; 0;1; 2;3; 4}
Vậy có 8 giá trị nguyên của m .
Câu 65. (Chuyên Nguyễn Trãi – Hải Dương – 2022) Cho hàm số y  f ( x ) có đạo hàm
 
f ( x)  ( x  3) x 2  2 x   . Tìm tất cả các giá trị thực không âm của tham số m để hàm số
  11 
g ( x )  f (| sin x  3 cos x |  m) có nhiều điểm cực trị nhất trên  ; .
 2 12 
 2 
A. m   .
 2 ,  
 
 2 
B. m   .
 2 ,1
 
C. m  ( 2  1, 2)
 2 
D. m   .
 2 , 2 
 
Lời giải
 x  3

 
Có f ( x)  0  ( x  3) x 2  2  0   x  2
x   2

 
sin x  3 cos x  2sin  x  
 3
  2 
  
g ( x)   2sin  x     m   f (| sin x  3 cos x |  m)

   3  
 

Facebook Nguyễn Vương https://www.facebook.com/phong.baovuongTrang 53


Blog: Nguyễn Bảo Vương: https://www.nbv.edu.vn/
   
2sin  x    2 cos  x  
 3  3    
g ( x)   f   2sin  x    m 
  
2
  3 

 2sin  x  3  
  
   
  cos  x  3   0
   
     
 cos  x    0  2sin  x    m  3
  3  3
g ( x)  0    

 
 f   2sin  x     m   0 
 2sin  x    m  2
   3    3
 
  2sin  x     m   2
   
  3
 
Xét u  2sin  x  
 3

Để thỏa mãn yêu cầu đề bài thì các phương trình (1), (2), (3), (4) có nhiều nghiệm nhất
  11   
x ;  , suy ra u  2sin  x    (0,1) .
 2 12   3
 0  m  3  1  4  m  3
 
Khi đó 0  2  m  1   2  1  m  2 . Vì m  0  m  ( 2  1, 2) .
 
0   2  m  1   2  1  m   2
Do đó m  ( 2  1, 2) .
Câu 66. (THPT Đô Lương – Nghệ An – 2022) Hàm số f ( x)  10 x  x và
3 2
 2

g ( x)  x  mx  m  1 x  2 . Gọi M là giá trị lớn nhất của hàm số y  g ( x  f ( x )) trên đoạn
[0;1] . Khi M đạt giá trị nhỏ nhất thì giá trị của m bằng?
21
A. .
2
B. 6.
C. 21.
D. 5.
Lời giải
x
Ta có: f ( x)  10 ln10  1  0, x
g ( x)  3 x 2  2mx   m 2  1  0, x do   2m 2  3  0.
y  g ( x  f ( x))  g 10 x  2 x  .

y   g 10 x  2 x     10 x ln10  2   g  10 x  2 x   0, x  [0;1].


Hàm số đạt giá trị lớn nhất tại x  1 khi đó
Trang 54 Fanpage Nguyễn Bảo Vương  https://www.facebook.com/tracnghiemtoanthpt489/
Điện thoại: 0946798489 TUYỂN CHỌN VẬN DỤNG – VẬN DỤNG CAO 2022
M  Maxy  y (1)  g (12)  12  m 12   m  1 12  2
3 2 2

[0,1]

 12m  144m  1738  12  ( m  6) 2  1306  1306


2

M đạt giá trị nhỏ nhất khi m  6 .


Câu 67. (THPT Lương Thế Vinh – Hà Nội – 2022) Cho hàm số y  f ( x ) liên tục trên  . Đổ thị hàm
số f ( 3 x ) được cho trong hình bên:

1
Hàm số g ( x)  f ( x)  x 4  x có tối đa bao nhiêu điểm cực đại?
8
A. 2.
B. 4.
C. 5.
D. 3.
Lời giải
1 1 1
Xèt hàm số h( x)  f ( x)  x 4  x có h( x)  f ( x)  x 3  1  h( x)  0  f ( x)  x 3  1 .
8 2 2

 t  2 x  3 2
1   1
Đặt x  3 t  f ( 3 t )  t  1   t  0   x  0 (đường thẳng y  x  1 cắt đồ thị f ( 3 x )
2 t2  x32 2
 
tại 3 điểm có hoành độ 2;0; 2  . Bảng biến thiên:

Suy ra g ( x) | h( x) | có tối đa 3 điểm cực đại.

Facebook Nguyễn Vương https://www.facebook.com/phong.baovuongTrang 55


Blog: Nguyễn Bảo Vương: https://www.nbv.edu.vn/
Câu 68. (THPT Lương Thế Vinh – Hà Nội – 2022) Cho hàm số f ( x ) có đổ thị như hình vẽ:

 
Gọi S là tập hợp tất cả các giá trị nguyên của tham số m để phương trình f 3  4  x 2  m có

hai nghiệm phân biệt thuộc đoạn [ 3; 3] . Số phần tử của S là


A. 1.
B. 4.
C. 5.
D. 3.
Lời giải
x
 
Xét y  f 3  4  x 2  y 
4 x 2  
f  3  4  x2  0

 x  0
x  0  2
  2
 3  4  x  1  x  0
 f (3  4  x )  0  2
 3  4  x  1
Bảng biến thiên:

Vậy ycbt  1  m  3  m  {0,1, 2, 3} .


Câu 69. (THPT Lương Thế Vinh – Hà Nội – 2022) Cho hàm số f ( x ) có đạo hàm trên  và f ( x ) có
bảng biến thiên như hình vẽ. Đồ thị y  f ( x ) cắt trục hoành tại hai điếm phân biệt có hoành độ
lần lượt là 3; 2 . Có bao nhiêu giá trị nguyên của tham số m thuộc [ 10;10] để hàm số
 
y  f x 2  2 x  m đồng biến trên ( 1;1) .

A. 12.
B. 14.
C. 11.
D. 13.
Trang 56 Fanpage Nguyễn Bảo Vương  https://www.facebook.com/tracnghiemtoanthpt489/
Điện thoại: 0946798489 TUYỂN CHỌN VẬN DỤNG – VẬN DỤNG CAO 2022
Lời giải
Từ bảng biến thiên của f ( x ) và f ( x)  0  x  3; x  2 ta có bảng xét dấu của đạo hàm như
sau:

ycbt  (2 x  2) f   x 2  2 x  m   0, x  [1;1]
 f   x 2  2 x  m   0, x  [1;1]
 x 2  2 x  m  2, x  [1;1]  m  g ( x)  x 2  2 x  2, x  [1;1]
 2  2
 x  2 x  m  3, x  [1;1]  m  h( x)  x  2 x  3, x  [1;1]
 m  min[ 1;1] g ( x)  g (1)  3
  m  {10,, 3, 6, ,10}
 m  max[ 1;1] h( x)  h(1)  6
Câu 70. (THPT Kim Liên - Hà Nội - 2022) Cho hàm số f  x  có đạo hàm trên  là
f   x    x  3 x  4  . Tính tổng các giá trị nguyên của tham số m   10;5 để hàm số

y  f x2  3x  m  có nhiều điểm cực trị nhất?
A. 54 . B. 9 . C.  52 . D.  54 .
Lời giải
Chọn D
 2 
 
Theo bài: y  f x 2  3 x  m  f   x 2  3 x  m   .
 
1  2   2 x  3 f  x 2  3 x  m  0
Ta có y 
2
 2 x  3 f    x 2  3 x  m   
  2 x 2  3x  m
 
2  x 2  3x  m 
 3  3
x  2 x  2
2 x  3  0  2  2
   x  3x  m  3  vô lí    x  3x  m  4
2
 f  x  3x  m  0   2  2
 x  3x  m  4  x  3x  m  4
 
 3
x 
 2
2
 3 25
  x     m 1 .
 2 4
 2
 x  3    7  m  2 
 2 4

Để hàm số y  f x 2  3 x  m  có nhiều điểm cực trị nhất thì phương trình 1 và  2 có nhiều
 25  25
 4 m  0 m  4 7
nghiệm nhất    m .
 7  m  0 m   7 4
 4  4
Mặt khác m nguyên và m   10;5  m  10; 9; 8; 7; 6; 5; 4; 3; 2 .
Vậy tổng các giá trị của m bằng  54 .

Facebook Nguyễn Vương https://www.facebook.com/phong.baovuongTrang 57


Blog: Nguyễn Bảo Vương: https://www.nbv.edu.vn/
Câu 71. (THPT Kim Liên - Hà Nội - 2022) Cho hàm số y  f  x  , đồ thị của hàm số y  f   x  là
đường cong trong hình bên. Giá trị nhỏ nhất của hàm số g  x   f  3 x   3 x 2  4 x  1 trên đoạn
 2 2 
 3 ; 3  bằng

1
A. f  0   1 . B. f  6  . C. f  2   . D. f  3   8 .
3
Lời giải
Chọn C
4 2 4
Ta có g   x   3 f   3 x   6 x  4  g   x   0  f   3 x   2 x   f  3x    3 x   1 .
3 3 3
t  2 2 
Đặt 3 x  t  x  vì x   ;  nên t   2; 2 .
3  3 3
2 4
Phương trình 1 trở thành f   t   t .
3 3
Vẽ trên cùng một hệ trục tọa độ ta được:

2 4 t  1
Xét trên  2; 2 , f   t   t  .
3 3 t  2
 1
 x
 2 2   3 x   1 3
Do đó, trên   ;  có g   x   0   
 3 3   3 x  2 x  2
 3
 2 2
Bảng biến thiên hàm số g  x  trên   ; 
 3 3
Trang 58 Fanpage Nguyễn Bảo Vương  https://www.facebook.com/tracnghiemtoanthpt489/
Điện thoại: 0946798489 TUYỂN CHỌN VẬN DỤNG – VẬN DỤNG CAO 2022

2 1
Từ bảng biến thiên, suy ra min g  x   g    f  2   .
 2 2 
 3 ;3 3 3
 

Câu 72. (THPT Kinh Môn - Hải Dương - 2022) Cho hàm số y  f  x  có đạo hàm trên  và đồ thị
hàm số y  f   x  cắt trục hoành tại các điểm có hoành độ 3;  2; a ; b ;3; c ;5 với
4 4
  a  1;1  b  ; 4  c  5 có dạng như hình vẽ bên dưới. Có bao nhiêu giá trị nguyên của
3 3
m để hàm số y  f  2 x  m  2022  có 5 điểm cực trị?

A. 4 . B. 2 . C. 3 . D. Vô số.
Lời giải
Chọn C
Hàm y  f  2 x  m  2022  là hàm chẵn trên  nên số điểm cực trị của hàm đã cho bằng
2.n  1 , trong đó n là số điểm cực trị trên khoảng  0;   của hàm y  f  2 x  m  2022 
Xét hàm y  f  2 x  m  2022  .
 2019  m
x  2
 x1

x  2020  m
 x2
 2 x  m  2022  3  2
 2 x  m  2022  2  2022  a  m
 x   x3
 2 x  m  2022  a  2
  2022  b  m
y  2 f   2 x  m  2022   0   2 x  m  2022  b   x   x4
 2
 2 x  m  2022  3
  2025  m
 2 x  m  2022  c x   x5
 2
 2 x  m  2022  5
  2022  c  m
x  2
 x6

x  2027  m
 x7
 2
Facebook Nguyễn Vương https://www.facebook.com/phong.baovuongTrang 59
Blog: Nguyễn Bảo Vương: https://www.nbv.edu.vn/
Vì x  3 là nghiệm bội chẵn của phương trình f   x   0 nên x  x5 cũng là nghiệm bội chẵn của
phương trình f   2 x  m  2022   0 .
Hàm y  f  2 x  m  2022  có 6 điểm cực trị x1  x2  x3  x4  x6  x7 .
 2022  b  m
 0
Hàm y  f  2 x  m  2022  có 5 điểm cực trị  x4  0  x6   2
 2022  c  m  0
 2
4
Vì 1  b  , 4  c  5, m    2024  m  2026  m  2024; 2025; 2026 .
3
Câu 73. (THPT Kinh Môn - Hải Dương - 2022) Tìm m để đường thẳng y  2 x  m cắt đồ thị hàm số
x3
y tại hai điểm A, B sao cho độ dài AB là nhỏ nhất.
x 1
A. 2 . B. 1 . C. 1 . D. 3 .
Lời giải
Chọn D
x3
Gọi hàm số y  có đồ thị là  C  và đường thẳng y  2 x  m có đồ thị là  d  .
x 1
x3
Xét phương trình hoành độ giao điểm của  C  và  d  :  2 x  m, x  1.
x 1
 x  3  2 x 2  2 x  mx  m  2 x 2   m  1 x  m  3  0, x  1 1
Để  d  cắt  C  tại hai điểm A, B  Phương trình 1 có hai nghiệm phân biệt khác 1
   0
 với g  x   2 x 2   m  1 x  m  3
 g   1  0
 m  12  4.2.  m  3  0
  m2  6m  25  0, m.
2  0
Giả sử hoành độ giao điểm của  C  và  d  là x1 , x2 . Khi đó A  x1 ; 2 x1  m  B  x2 ; 2 x2  m  .
m 1 m3
Theo hệ thức Vi-ét ta có x1  x2   ; x1 x2 
2 2
2 2 2 2
Ta có AB   x2  x1    2 x2  2 x1   5  x1  x2   5  x1  x2   20 x1 x2
2
 m 1
2
m3 5m 2  10m  5  40m  120 5  m  3  80
AB  5.    20.    2 5.
 2  2 4 2
Dấu "  " xảy ra khi và chỉ khi m  3.
Vậy m  3 thì độ dài AB đạt giá trị nhỏ nhất bằng 2 5.
Câu 74. (THPT Lương Tài 2 - Bắc Ninh - 2022) Cho hàm số y  f  x   2 x3  bx 2  cx  d thỏa mãn
4b  2c  d  16  0 và 9b  3c  d  54 . Hàm số y  f  x  có tất cả bao nhiêu điểm cực trị?
A. 2 .B 3 . C. 5 . D. 4 .
Lời giải
Chọn C
Ta có y  f  x   2 x3  bx2  cx  d liên tục trên  .Vì f  2   4b  2c  d  16  0 ,
f  3  9b  3c  d  54  0 , lim f  x    , lim f  x    nên phương trình f  x   0 có
x  x 

ba nghiệm phân biệt x1  3  x2  2  x3

Suy ra y  f  x  có hai điểm cực trị x1  m  x2  n  x3 .

Trang 60 Fanpage Nguyễn Bảo Vương  https://www.facebook.com/tracnghiemtoanthpt489/


Điện thoại: 0946798489 TUYỂN CHỌN VẬN DỤNG – VẬN DỤNG CAO 2022
Ta có bảng biến thiên

Vậy hàm số y  f  x  có 5 điểm cực trị.


Câu 75. (THPT Lương Tài 2 - Bắc Ninh - 2022) Cho hàm số y  f  x  là hàm số bậc ba có đồ thị như
hình vẽ. Tìm tất cả các ía trị của tham số m sao cho phương trình f  sin x   f  m  1 có nghiệm.

A.  1  m  3 . B. 2  m  0 . C. 3  m  1 . D. 2  m  2 .
Lời giải
Chọn C

Ta có: sin x   1;1 nên f  sin x    1;3 .


nên f  m  1   1;3 . Do đó, phương trình phương trình f  sin x   f  m  1 có nghiệm
1  f  m  1  3  2  m  1  2  3  m  1 .
Câu 76. (THPT Võ Nguyên Giáp - Quảng Bình - 2022) Cho hàm số bậc ba y  f  x  có đồ thị như
hình vẽ sau:

Có bao nhiêu giá trị nguyên của tham số m   0; 20 để hàm số g ( x)  f 2  x   2 f  x   m có 9


điểm cực trị?
Facebook Nguyễn Vương https://www.facebook.com/phong.baovuongTrang 61
Blog: Nguyễn Bảo Vương: https://www.nbv.edu.vn/
A. 8 . B. 9 . C. 10 . D. 11 .
Lời giải
Chọn A
Đặt h( x)  f 2  x   2 f  x   m  g  x   h  x  .
h( x)  2. f  x  f   x   2 f   x   2. f   x   f  x   1 .
 x  1

x  1
 f  x  0 
h( x )  0     x  a   2;  1
 f  x   1  x  b   0;1

 x  c  1;   

Ta có f 1  0 , f  1  4 , f  a   f  b   f  c   1 .
 h  a   h  b   h  c   1  m , h 1  m , h  1  8  m .
 2 m 
lim f  x    và lim f  x     lim h( x)  lim f 2  x  1   2    .
 f  x  f  x  
x  x  x  x 

Ta có BBT của y  h  x  :

Dựa vào BBT ta thấy hàm số y  h  x  có 5 cực trị.


Hàm số y  g ( x ) có 9 điểm cực trị khi và chỉ khi đồ thị hàm số y  h  x  cắt trục hoành tại 4
điểm phân biệt hoặc điểm 1;  m  nằm trên trục hoành.
 m  0  8  m  0  m  8 .
Với m là số nguyên thì m  0;1; 2;3; 4;5;6;7 .
Suy ra có 8 giá trị của tham số m thỏa mãn điều kiện đề bài.
Câu 77. (THPT Võ Nguyên Giáp - Quảng Bình - 2022) Cho hàm số y  f  x  liên tục trên  và
f   x    x3  6 x 2  32 . Khi đó hàm số g  x   f  x 2  3 x  nghịch biến trên khoảng
A.  ;   . B. 1;   . C.  2;  . D.  ;1 .
Lời giải
Chọn C
Ta có:
+) f (t )  (t  2)(t  4)2 .
+) g ( x)   f ( x 2  3 x)   ( x 2  3 x). f ( x 2  3 x)  (2 x  3). f ( x 2  3 x) .
Suy ra:

Trang 62 Fanpage Nguyễn Bảo Vương  https://www.facebook.com/tracnghiemtoanthpt489/


Điện thoại: 0946798489 TUYỂN CHỌN VẬN DỤNG – VẬN DỤNG CAO 2022
 3
 x
2 x  3 2
2 2 x  3  0  2 
*) g ( x)  0  (2 x  3). f ( x  3 x)  0   2
  x  3 x  2   x  1
 f ( x  3x )  0  x 2  3x  4 x  2
 
 x  4
 3
 2 x  3  0  x  2
  2
  x  3 x  2
2

  f ( x  3 x)  0
*) g ( x)  0  (2 x  3). f ( x 2  3x)  0   
2x  3  0   x  3
 
  f ( x 2  3x)  0  2
 2
  x  3 x  2
 3
 x  2

1  x  2  3
 1 x 
  3  2.
 x 
 2 x  2
 x  1

   x  2
Câu 78. (THPT Yên Lạc - Vĩnh Phúc - 2022) Cho hàm số f  x  , đồ thị của hàm số y  f   x  là
đường cong trong hình bên.

8x3
Tìm tất cả các giá trị thực của tham số m để bất phương trình f  2 x    4 x  m  0 đúng với
3
 1 1
mọi x    ; 
 2 2
5
A. m  f 1  . B. m  f  0  . C. m  f  0  . D. m  f  3 .
3
Lời giải
Chọn B
8x3
Đặt g  x  f  2x   4x ta có g  x   2 f   2 x   8x2  4 , giải phương trình
3
2
g x  0  f 2x  2  2x .

Facebook Nguyễn Vương https://www.facebook.com/phong.baovuongTrang 63


Blog: Nguyễn Bảo Vương: https://www.nbv.edu.vn/

 1 1
Theo giả thiết chỉ xét x    ;  nên 2 x   1;1 , trên đồ thị hàm số y  f   x  ta vẽ thêm
 2 2
 1
2 2 x  1  x
parabol y  2  x ta có f  2 x   2   2 x   
2
  2.
2 x  0 
 x  0
 1 1
Bảng biến thiên của hàm số g  x  trên x    ;  như sau
 2 2
1 1
-
x ∞ 2 0 2 +∞
g'(x) + + 0 0 +
g(0) +∞
g(x) ∞ g(0.5)
3
1
3 8.  
8.0 1  1 2 1 5
Ta có: g  0   f  2.0    4.0  f  0  và g    f  2.      4.  f 1 
3 2  2 3 2 3
Nên ta có
1 1
-
x ∞ 2 0 2 +∞
g'(x) + + 0 0 +
f(0) +∞
g(x) ∞ 5
f(1)-
3
Từ YCBT cho ta mệnh đề
8 x3  1 1  1 1
f  2x   4 x  m  0, x    ;   g  x   m, x    ;   m  f  0 
3  2 2  2 2
x2  1 y  1
Câu 79. (THPT Yên Lạc - Vĩnh Phúc - 2022) Cho x, y là các số thực dương và thỏa mãn  .
y x
y4
Giá trị nhỏ nhất m của biểu thức P  là
x
A. m  3 B. m  2 2 C. m  4 D. m  8
Trang 64 Fanpage Nguyễn Bảo Vương  https://www.facebook.com/tracnghiemtoanthpt489/
Điện thoại: 0946798489 TUYỂN CHỌN VẬN DỤNG – VẬN DỤNG CAO 2022
Lời giải
Chọn C
x2  1 y  1 3
Ta có:
y

x
 x3  x   y  y (1).

Xét hàm số f  t   t 3  t , t  0 có f   t   3t 2  1  0, t  0
 f  t  đồng biến trên khoảng  0;   .
Do đó 1  x  y  x2  y .
y  4 x2  4 4 4
Khi đó: P    x   2. x.  4
x x x x
4
Dấu “=” xảy ra khi và chỉ khi x   x  2; y  4
x
Vậy giá trị nhỏ nhất của P là 4 .
Câu 80. (THPT Yên Lạc - Vĩnh Phúc - 2022) Cho hàm số y  f  x  là hàm đa thức bậc 3 có đồ thị như
hình vẽ. Gọi S là tập hợp tất cả các giá trị nguyên của tham số m   100;100 để hàm số
h  x   f 2  x   4 f  x   3m có đúng 3 điểm cực trị. Tổng tất cả các phần tử của S bằng

A. 5047 . B. 5049 . C. 5043 . D. 5050 .


Lời giải
Chọn B
Đặt g  x   f 2  x   4 f  x   3m ; g   x   2 f  x  . f   x   4 f   x  .
 f  x  0
g   x   0  2 f   x   f  x   2   0   .
 f  x   2
Dựa vào đồ thị ta thấy:
x  1
- Với f   x   0   (là 2 điểm cực trị của đồ thị f  x  )
x  3
- Với f  x   2  x  a với a   1;0  .
Suy ra hàm số g  x  có 3 điểm cực trị.
 g  x   0; x  
Vậy để hàm số h  x   g  x  có đúng 3 điểm cực trị thì  .
 g  x   0; x  
 g  x   0; x   t 2  4t  3m  0; t   1
Đặt t  f  x  ; t   . Khi đó   2
 g  x   0; x   t  4t  3m  0; t    2
1  0 4
1   m .
  16  12m  0 3

Facebook Nguyễn Vương https://www.facebook.com/phong.baovuongTrang 65


Blog: Nguyễn Bảo Vương: https://www.nbv.edu.vn/
Vậy tập hợp tất cả các giá trị của m thỏa mãn là 2;3; 4;...;100 .
 2 không xảy ra.
Tổng của tất cả các phần tử là 2  3  4  ...  100  5049 .
Câu 81. (THPT Yên Phong 1 - Bắc Ninh - 2022) Cho hàm số f  x    x 4   4  m 2  x  2020 và
g  x    x3  5 x 2  2020 x  2021 . Có bao nhiêu giá trị nguyên dương của tham số m để
h  x   g  f  x   đồng biến trên  2;   ?

A. 7 . B. 6 . C. 12 . D. 13 .
Lời giải
Chọn B
+) h  x   f   x  g   f  x   .
Xét hàm g  t   t 3  5t 2  2020t  2021  g   t   3t 2  10t  2020  0, t   .
Suy ra g   f  x    0, f  x    .
h  x   g  f  x   đồng biến trên  2;    h  x   0 với x   2;  
 f   x  .g   f  x    0, x   2;  
 f   x  g   f  x    0, x   2;   (vì g   f  x    0f  x    )
 f   x   0, x   2;    4 x3  4  m2  0, x   2;  
 m2  4 x3  4, x   2;    m2  min f  x  .
 2;  
3
+) Xét hàm số k  x   4 x  4 , x   2;  
Bảng biến thiên của hàm số k  x   4 x3  4  k   x   12 x 2  0 trên  2;  .

Từ BBT suy ra m 2  36  6  m  6 thì hàm số đồng biến trên  2;   .


Do m nguyên dương nên m  1; 2;3; 4;5;6 suy ra có 6 giá trị nguyên của tham số m .
Câu 82. (THPT Yên Phong 1 - Bắc Ninh - 2022) Cho hàm số y  f  x  liên tục trên  đồng thời thỏa
điều kiện f  0   0 và  f  x   6 x  f  x   9 x 4  3 x 2  4, x   . Tìm giá trị lớn nhất của hàm số
y  f  2 x 2  3 x  1 trên đoạn  0;1 .
5 17 155 167
A. . B. . C. . D. .
2 7 64 69

Lời giải
Chọn C
2 2
 Có  f  x   6 x  f  x   9 x 4  3 x 2  4   f  x   3 x    3 x 2  2 
 f  x   3x  3x 2  2  f  x   3x 2  3x  2
 2
 2
.
 f  x   3 x  3 x  2  f  x   3 x  3 x  2 (l )

Trang 66 Fanpage Nguyễn Bảo Vương  https://www.facebook.com/tracnghiemtoanthpt489/


Điện thoại: 0946798489 TUYỂN CHỌN VẬN DỤNG – VẬN DỤNG CAO 2022
 1 
 Đặt t  2 x 2  3 x  1, với x   0;1 thì t    ;1 .
 8 
 1  1  1 
Xét hàm g  t   f  t  trên   ;1 , có g   t   f   t   0  6t  3  0  t     ;1 .
 8  2  8 
 1  155  1  5
Có g     ; g    ; g 1  2 .
 8  64  2 4
155
Suy ra, max y  max g  t   .
 
0;1  1 
  ;1
64
 8 

Theo dõi Fanpage: Nguyễn Bảo Vương  https://www.facebook.com/tracnghiemtoanthpt489/

Hoặc Facebook: Nguyễn Vương  https://www.facebook.com/phong.baovuong

Tham gia ngay: Nhóm Nguyễn Bào Vương (TÀI LIỆU TOÁN)  https://www.facebook.com/groups/703546230477890/

Ấn sub kênh Youtube: Nguyễn Vương


 https://www.youtube.com/channel/UCQ4u2J5gIEI1iRUbT3nwJfA?view_as=subscriber

Tải nhiều tài liệu hơn tại: https://www.nbv.edu.vn/

Facebook Nguyễn Vương https://www.facebook.com/phong.baovuongTrang 67


TUYỂN CHỌN VẬN DỤNG – VẬN DỤNG CAO 2022 Điện thoại: 0946798489

VẤN ĐỀ 2. LOGARIT
• |FanPage: Nguyễn Bảo Vương
• TUYỂN CHỌN CÂU HỎI VD-VDC TỪ CÁC ĐỀ THI THỬ CÁC TRƯỜNG, CÁC SỞ NĂM 2022
Câu 1. (Chuyên Vinh– 2022) Có bao nhiêu số nguyên a sao cho ứng với mỗi a , tồn tại số thực b  a thỏa
mãn 4 a  2b  b và đoạn [ a; b] chứa không quá 5 số nguyên ?
A. 5.
B. 10.
C. 6.
D. 11.
Lời giải
Chọn D
Do đoạn [ a; b] chứa không quá 5 số nguyên nên ta có điều kiện đủ là: a  b  a  5 . Khi đó ta có:
Phương trình ban đầu tương đương với: 2b  b  4 a  0 .
Xét hàm số y  f a (b)  2b  b  4a có f a (b)  2b ln 2  1  0, b   tức hàm f a (b) đồng biến
trên  .
Kết hợp điều kiện cần ban đầu ta suy ra hàm số f a (b) đồng biến trên [ a; a  5) .
 f ( a  5)  0 2a  5  a  5  4 a
Như vậy điều kiện tồn tại nghiệm là  a  a a
.
 f a ( a)  0 2  a  4
Trường hợp 1: Nếu a  5  0  a  6 thì 2a 5  a  5  2  1  0  4 (loại)
Trường hợp 2: Nếu a  5  0  a  5 thì khi đó
a5 a a a6 2a a 6
2  a  5  4  4  2  2  2  a  6 Đối chiếu với điều kiện ta suy ra 5  a  5 .
Đến đây với mọi a  [ 5;5] thì bất phương trình 2a 5  a  5  4a luôn xảy ra vì
4 a  2 a 5  2 a  5  a  5 (không có dấu bằng xảy ra).
Xét bất phương trình còn lại: 2 a  a  4 a ta thấy cũng luôn đúng với mọi a  [ 5;5]
Vậy a  [ 5;5] thì thỏa mãn yêu cầu đề bài tức có 11 giá trị nguyên a .
Câu 2. (Chuyên Vinh – 2022) Có bao nhiêu giá trị nguyên lớn hơn 2 của y sao cho với mỗi y tồn tại
đúng 3 sô nguyên dương x thỏa mãn 3x  y  2 log 2  3x  2  ? .
A. 16.
B. 51.
C. 68.
D. 66.
Lời giải
Chọn B
3x  y  2 log 2  3x  2  điều kiện: 3x  2  0  x  log3 2 .

 y  3x  2 log 2 3x  2 . 
Xét hàm số f ( x)  3x  2 log 2  3x  2 

x
 f ( x)  3 ln 3  x
2.3x ln 3 x

 3 ln 3  1  x
2 
x
  3 ln 3 
 
 3x  2 ln 2  2 


3  2 ln 2  
 3  2 
ln 2 
   
 3x  2 ln 2 

2  2 
 f ( x)  0  3x  2   x  log 3   2  a
ln 2  ln 2 
Bảng biến thiên:

Facebook Nguyễn Vương https://www.facebook.com/phong.baovuong Trang 1


Blog: Nguyễn Bảo Vương: https://www.nbv.edu.vn/

 f (3)  y  27  2 log 2 25  y
Ycbt     17, 71  y  68, 3
 f (4)  y 81  2 log 2 79  y
Vì y  2 là số nguyên nên 18  y  68  có 51 số.
Câu 3. (Chuyên Vinh – 2022) Số nghiệm nguyên của bất phương trình
 
2log 2 ( x  2)  log 2 2 x 2  1  ( x  1)( x  5) là
A. 5.
B. 6.
C. 7.
D. 4.
Lời giải
Chọn B
Nhận xét x  1 là nghiệm của bất phương trình.
Với x  1 ta có:
2 log 2 ( x  2)  log 2  2 x 2  1  ( x  1)( x  5)

 log 2  x 2  4 x  4   log 2  2 x 2  1  x 2  4 x  5(2)


a  2 x 2  1
Đặt  2
(a  1; b  1).
b  x  4 x  4
(2)  b  log 2 b  a  log 2 a (3) .
Xét hàm số f (t )  t  log 2 t với t  1 .
1
f (t )  1   0, t  1
2t log 2 t ln 2
Hàm số f (t ) đồng biến trên khoảng (1;  ) nên từ (3) ta có:
b  a  x 2  4 x  4  2 x 2  1   x 2  4 x  5  0  1  x  5
Mà x  1  1  x  5 .
Vậy có 6 giá trị nguyên của x thỏa mãn.
Câu 4. (Chuyên Vinh 2022) Gọi m là giá trị nhỏ nhất của hàm số f ( x)  4 x  (a  2)2 x  2 trên đoạn
[1;1] . Tất cả giá trị của a để m  1 là
A. a  1 .
1
B.   a  0 .
2
1
C. a   .
2
D. a  0 .
Lời giải
Chọn D

Trang 2 Fanpage Nguyễn Bảo Vương  https://www.facebook.com/tracnghiemtoanthpt489/


Điện thoại: 0946798489 TUYỂN CHỌN VẬN DỤNG – VẬN DỤNG CAO 2022
1 
Đặt t  2 x , t   ; 2  , f ( x) trở thành g (t )  t 2  (a  2)t  2
2 
1 
Hàm số g (t ) liên tục trên  ; 2  .
2 
2a
g (t )  2t  a  2  g (t )  0  t 
2
1 2a
Trường họp 1:   2  2  a  1
2 2
2
 2  a  8  (a  2)
Suy ra min g (t )  g   
1 
 ;2  2  4
2 

8  ( a  2) 2
Yêu cầu bài toán  1 0  a  4
4
Vậy 0  a  1 (1)
2a 1
Trường họp 2:   a 1
2 2
1 1 5 1 
Suy ra: min g (t )  g    a   ; 2 
2 2 4 2 
1 5 1
Yêu cầu bài toán  a   1  a  
2 4 2
Vậy a  1 .
2a
Trường họp 3:  2  a  2
2
Suy ra: min g (t )  g (2)  2 a  2
1 
 2 ; 2
 
1
Yêu cầu bài toán  2 a  2  1  a  
2
Vậy không tồn tại a .
Kết hợp 3 trường hợp, ta có a  0 .
Câu 5. (Cụm Trường Nghệ An - 2022) Có bao nhiêu cặp số nguyên  x; y  thỏa mãn đẳng thức sau:
2

log 2022  x 4  2 x 2  2023


y  2022 

   2 y  2021 .
A. 3. B. 1. C. 0. D. 2.
Lời giải
Chọn D
2
 
Ta có x 4  2 x 2  2023  x 2  1  2022  2022 với x   .

 log 2022  x 4  2 x 2  2023  1 với x   .


2
 y  2022  2022
Lại có  2 2
với y  
 y  2022  y  1  2021  2 y  2021
2

 log 2022  x 4  2 x 2  2023   y 2  2022 log


y  2022

    2022  x4  2 x2  2023  2 y  2021 x, y   .

Facebook Nguyễn Vương https://www.facebook.com/phong.baovuongTrang 3


Blog: Nguyễn Bảo Vương: https://www.nbv.edu.vn/
 x  1


Do đó log 2022  x  2 x  2023
4 2 y 2  2022 
 2 y  2021 
 x2 1  0
  y  1 .
    x  1
 y 1 
  y  1
Có hai cặp số nguyên  x; y  thỏa mãn đề bài.
Câu 6. (Đại học Hồng Đức – 2022) Cho x là số nguyên dương và y là số thực. Có tất cả bao nhiêu cặp số
( x; y ) thỏa mãn ln(1  x  2 y )  2 y  3 x  10 ?
A. 10.
B. Vô số.
C. 11.
D. 9.
Lời giải.
x 1
Điều kiện: 1  x  2 y  0  y   .
2

Ta luôn chứng minh được e x  x  1, x   .


Xét hàm số y  g ( x)  e x  x  1  g ( x)  e x  1  0  x  0 .
Bảng biến thiên:

Suy ra g ( x)  0x    e x  x  1x   .


Ta có: ln(1  x  2 y )  2 y  3 x  10  1  x  2 y  e 2 y 3 x 10  (2 y  3 x  10)  1  x  5 .
Do x  * , nên x  {1; 2;3; 4;5} .
Lại có: ln(1  x  2 y )  2 y  3 x  10  ln(1  x  2 y )  2 y  3 x  10  0  f ( y )  0 .
 x 1 
Xét hàm số f ( y )  ln(1  x  2 y )  2 y  3 x  10 trên khoảng   ;  
 2 
2 x  x 1 
Suy ra f ( y )   2; f ( y )  0  y      ;  
1 x  2 y 2  2 
Bảng biến thiên của hàm số f ( y )  ln(1  x  2 y )  2 y  3 x  10 .

Dựa vào bảng biến thiên ta nhận thấy:


Với một giá trị x  {1; 2;3; 4} , phương trình ln(1  x  2 y )  2 y  3 x  10  0 theo ẩn y có 2
nghiệm phân biệt. Với x  5 phương trình ln(1  x  2 y )  2 y  3 x  10 theo ẩn y có 1 nghiệm.
Vậy có 9 nghiệm ( x; y ) thỏa mãn bài toán.
Câu 7. (Đại học Hồng Đức – 2022) Tổng S của tất cả các nghiệm thuộc khoảng (0; 4 ) của phương trình
2 2
2022sin x  2022cos x  2 ln(cot x ) là
Trang 4 Fanpage Nguyễn Bảo Vương  https://www.facebook.com/tracnghiemtoanthpt489/
Điện thoại: 0946798489 TUYỂN CHỌN VẬN DỤNG – VẬN DỤNG CAO 2022
A. S  18 .
B. S  8 .
C. S  7 .
D. S  16 .
Lời giải.
Điều kiện cot x  0 . Ta có
2 2 2 2
2022sin x  2022 cos x
 2 ln(cot x)  2022sin x  2022 cos x  ln  cos 2 x   ln  sin 2 x 
2 2
 2022sin x  ln  sin 2 x   2022 cos x  ln  cos 2 x 
Xét hàm số f (t )  2022t  ln t với t  0
1
f (t )  2022t  ln 2022   0, t  0  hàm số f (t ) đồng biến trên khoảng (0;  ) .
t
 k
Khi đó (1)  f  sin x   f  cos 2 x   sin 2 x  cos 2 x  cos 2 x  0  x  
2
,k  .
4 2

Do cot x  0 nên x   k , k   .
4
  5 9 13 
Mà x  (0; 4 ) suy ra x   ; ; :  . Suy ra S  7 .
4 4 4 4 
Câu 8. (THPT Hồ Nghinh – Quảng Nam – 2022) Cho các số thực dương x, y thỏa mãn
log x 2  2 y  2 y 2 (9 x  10 y  20)  1 . Gọi M , m lần lượt là giá trị lớn nhất và giá trị nhỏ nhất của
y
S . Tính M  m .
x
5
A. M  m  .
3
B. M  m  5  2
C. M  m  2 7 .
7
D. M  m  .
2
Lời giải
Chọn A
9 x  10 y  20  0 y
Điều kiện:  2 2
.Có S   y  Sx .
 x  xy  2 y  1 x
Giả thiết log x2  2 y  2 y2 (9 x  10 y  20)  1  x 2  xy  2 y 2  9 x  10 y  20
 x 2  Sx 2  2 S 2 x 2  9 x  10 Sx  20   2 S 2  S  1 x 2  (9  10 S ) x  20  0 (1).
Để phương trình (1) có nghiệm thì
25  8 10 25  8 10
  0  (9  10 S ) 2  80  2 S 2  S  1  0  60 S 2  100 S  1  0  S .
30 30
 9  10S1
25  8 10  x  2 2S 2  S  1  0
Suy ra M  S1 
30
dấu "=" xảy ra khi và chỉ khi   1 1 

 y  S1 x  0
 9  10 S 2
25  8 10  x  2 2S 2  S  1  0
m  S2 
30
dấu "=" xảy ra khi và chỉ khi   2 2 

 y  S 2 x  0.
5
Vậy M  m  .
3
Facebook Nguyễn Vương https://www.facebook.com/phong.baovuongTrang 5
Blog: Nguyễn Bảo Vương: https://www.nbv.edu.vn/
Câu 9. (THPT Hương Sơn - Hà Tĩnh - 2022) Trong mặt phẳng với hệ tọa độ Oxy , điểm M  x ; y  biểu
diễn nghiệm của phương trình log3  9 x  18  x  y  3y . Có bao nhiêu điểm M có tọa độ nguyên
thuộc hình tròn tâm O bán kính R  7 ?
A. 7 . B. 2 . C. 3 . D. 49 .
Lời giải
Chọn B
Điều kiện: x  2
Ta có log3  9 x  18  x  y  3y  log3  x  2   x  2  y  3y
Đặt t  log 3  x  2   x  2  3t ta được 3t  t  3 y  y
Xét hàm số f  u   3u  u thì f  u  là hàm đồng biến, vậy y  t  log 3  x  2 
Do M có tọa độ nguyên và nằm trên hình tròn tâm O bán kính bằng R  7 nên: x 2  y 2  49
2  x  7

M có tọa độ nguyên nên  x  
log x  2  
 3 
TH1: x  1  y  0 ( thỏa mãn)
TH2: x  1  y  1( thỏa mãn)
TH3: x  7  y  2 ( loại)
Vậy có 2 điểm thỏa mãn yêu cầu là  1;0  , 1;1
Câu 10. (THPT Hương Sơn - Hà Tĩnh - 2022) Với giá trị nào của m thì phương trình
1
x
9  4m.3x  m  2  0 có hai nghiệm phân biệt x1 ; x2 thỏa mãn x1  x2  1 ?
2

3 3
A. m  . B. m   . C. m  7 . D. m  1 .
4 4
Lời giải
Chọn C
1
x
Ta có 9 2
 4m.3x  m  2  0  3.32 x  4m.3x  m  2  0 1 .
Đặt 3x  t ,  t  0  .
Phương trình đã cho trở thành: 3t 2  4mt  m  2  0 *
Để 1 có hai nghiệm phân biệt x1 ; x2 thì phương trình * có 2 nghiệm 0  t1  t2

4m2  3m  6  0
  0 
  4m 3  105
 t1  t2  0   0 m .
t .t  0  3 8
1 2 m  2
 3  0
m2 m2
Ta có:  t1 .t2  3x1.3x2  3x1  x2 mà x1  x2  1 nên ta có: 3   m  7 (tmđk).
3 3
Câu 11. (THPT Lê Thánh Tông - HCM-2022) Có tất cả bao nhiêu số b nguyên dương sao cho tồn tại
2 2
đúng hai số thực a thỏa mãn đẳng thức b.2 a  6 a 1  b 2 .2 2 a 12 a 1  3  7 log 2  a 2  6 a  log 2 b  ?
A. 1024 . B. 1023 . C. 2047 . D. 2048 .
Lời giải
Chọn B

Trang 6 Fanpage Nguyễn Bảo Vương  https://www.facebook.com/tracnghiemtoanthpt489/


Điện thoại: 0946798489 TUYỂN CHỌN VẬN DỤNG – VẬN DỤNG CAO 2022
2
 x  a  6a

Đặt  y  log 2 b  0  b  2 y .
x  y  0

Ta được 2 y.2 x 1  22 y.22 x 1  3  7 log 2  x  y   2 x  y  22 x  y   14 log 2  x  y   6  0 .
Xét hàm số f  t   22t  2t  14log 2 t  6 , t  0 .
14 14
f   t   4t ln 4  2t ln 2  f   t   4t ln 2 4  2t ln 2 2  2  0 t  0
t.ln 2 t ln 2
 f  t   0 có nhiều nhất hai nghiệm dương. Ta thấy f 1  f  2   0 .
t  1 x  y  1  log 2 b  1  6a  a 2
Do vậy, ta được f  t   0     2
.
t  2  x  y  2  log 2 b  2  6a  a

Từ đồ thị, ta có: 10  log 2 b  11  210  b  211


Vậy có 1023 số nguyên dương b thỏa mãn.
Câu 12. (THPT Lê Thánh Tông - HCM-2022) Số nghiệm nguyên của bất phương trình
 3x  36 x  246  5  ln  x  3  0 là
A. 144. B. 145. C. 146. D. 147.
Lời giải
Chọn B
 x  3  0  x  3  x  3  x  3
Điều kiện:      3  x  e 5  3 .
5  ln  x  3   0  
ln x  3   5 x  3  e
5 5
x  e  3
5  ln  x  3  0 1
Ta có:  3x  36 x  246  5  ln  x  3  0   x 6 x
3  3  246  0  2 
1  ln  x  3  5  x  3  e5  x  e5  3 (nhận).
729 3 x  3 x  1
 2   3x  x
 246  0  32x
 246.3 x
 729  0   x 5
 .
3 3  3 x  5
So với điều kiện, ta có các giá trị nguyên thoả mãn là x  2; 1; 0;1  5; 6;...;145 .
Vậy bất phương trình đã cho có 145 nghiệm nguyên.

Facebook Nguyễn Vương https://www.facebook.com/phong.baovuongTrang 7


Blog: Nguyễn Bảo Vương: https://www.nbv.edu.vn/
1
Câu 13. (Liên trường Hà Tĩnh – 2022) Cho các số thực a, b thỏa mãn a  , b  1 . Khi biểu thức
2
P  log 2 a b  log b  a  4a  16  đạt giá trị nhỏ nhất thì tổng a  b bằng
4 2

A. 4.
B. 18.
C. 14.
D. 20.
Lời giải
Chọn B
2 1
Do a 4  4a 2  16  4a 2   a 2  4   0 đúng a  ; Dấu bằng xảy ra khi a  2
2
Suy ra
4 4
P  log 2 a b  2 log b (2a) 2  log 2 a b  4 log b 2a  log 2 a b   2 log 2 a b  4
log 2 a b log 2 a b
Dấu bằng xảy ra khi và chỉ khi
a  2
 a  2 a  2 a  2
log b  4   2
  a  b  18
 2a 
 log 2 a b  2 b  (2a)  b  16
 log 2 a b
Vậy, khi P đạt giá trị nhỏ nhất thì a  b  18
Câu 14. (Liên trường Hà Tĩnh – 2022) Có bao nhiêu giá trị nguyên của m để phương trình
log 2 ( x  1)  x  2  4 x  2 x 3  m  1  0 có ba nghiệm phân biệt
A. 2.
B. 3.
C. 4.
D. 5.
Lời giải
Chọn B
log 2 ( x  1)  x  2  0 1
 Phương trình đã cho   x x3
 4  2  m  1  0(2)
1
+ Xét hàm số f ( x)  log 2 ( x  1)  x  2  0 . Ta có f ( x )   1  0, x  1
( x  1) ln 2
Lại có f (2)  0 suy ra phương trình (1) có đúng 1 nghiệm x  2
+ Yêu cầu bài toán PT (2) phải có hai nghiệm phân biệt khác 2. Suy ra phương trình
t 2  8t  1  m phải có hai nghiệm phân biệt khác 4 thỏa mãn 2  t1  t2
+ Xét hàm số f (t )  t 2  8t  1 có bảng biến thiên:

17   m   13  13  m  17
Câu 15. (Liên trường Hà Tĩnh 2022) Tính tổng tất cả các giá trị nguyên dương của m để bất phương
trình 2 x 3  2m  x  2m3  1 có nhiều nhất 20 nghiệm nguyên
A. 153.
B. 171.
C. 190.

Trang 8 Fanpage Nguyễn Bảo Vương  https://www.facebook.com/tracnghiemtoanthpt489/


Điện thoại: 0946798489 TUYỂN CHỌN VẬN DỤNG – VẬN DỤNG CAO 2022
D. 210.
Lời giải
Ta có BPT đã cho
2m
 2 x 3  x  8.2 m  1  8.2 2 x  2 m  8.2 m  x  2 x   2 x  2 m  2 x  2 3   0
2
Ta có
2 x  2m  x  m
2 x  23  x  3
Bảng xét dấu

Suy ra tập nghiệm của BPT là (3; m) . Suy ra tập các nghiệm nguyên là {2; 1;0;1;; m  1}
YCBT suy ra m  1  17  m  18 . Vậy có 18 giá trị nguyên dương của m là
18
m {1, 2,3, ,18}  S  1  2  3  18  (1  18)   171
2
Câu 16. (THPT Nguyễn Tất Thành-Đh-SP-HN-2022) Có bao nhiêu giá trị nguyên của tham số
m   10;10 để bất phương trình

log 22 x  (m  1) log 2 x  2m  3  0

nghiệm đúng với mọi x  1;32


A. 11. B. 12. C. 13. D. 14.
Lời giải
Chọn B
Đặt log 2 x  t ; x  1;32   t   0;5 . Khi đó

log 22 x  ( m  1) log 2 x  2m  3  0, x  1;32  t 2  ( m  1)t  2m  3  0, t   0;5


t2  t  3
 t 2  t  3  (t  2)m, t   0;5  f (t )   m, t   0;5
t2

t 2  4t  5
Khảo sát hàm số ta có f   t    0  t  5;1 .
(t  2) 2

3 23 3 3
Trên  0;5 : f  0   ; f 1  1; f  5    Min f  t    m  là điều kiện cần tìm.
2 7  
0;5 2 2

Kết hợp giá trị nguyên m   10;10 ta được 12 giá trị nguyên.

Câu 17. (THPT Nguyễn Tất Thành-Đh-SP-HN-2022) Cho x, y, z  0;2 và thỏa mãn x  2 y  z  6 .
2 2
Tìm giá trị lớn nhất của biểu thức P  32 x x  52 y y  3z  2 x 2  4 y 2
A. m ax P  25 . B. max P  27 . C. max P  26 . D. max P  30 .
Lời giải
Chọn B
Xét hàm số f  t   3t  2t trên đoạn  0;1 .

Facebook Nguyễn Vương https://www.facebook.com/phong.baovuongTrang 9


Blog: Nguyễn Bảo Vương: https://www.nbv.edu.vn/
2 
Ta có f   t   3t.ln3  2 ; f '  t   0  t  log 3    a   0;1 .
 ln 3 

Suy ra f  t   1 t  0;1 1


Xét hàm số g  t   5t  4t trên đoạn  0;1 .
4 
Ta có g   t   5t.ln5  4 ; g   t   0  t  log 5    b   0;1 .
 ln 5 

Suy ra g  t   1 t   0;1  2 .
Xét hàm số h  t   3t  4t trên đoạn  0;2 .
4 
Ta có h  t   3t.ln 3  4; h  t   0  t  log 3    c   0; 2  .
 ln 3 

Suy ra h  t   1 t   0;2  3
Do x, y, z  0;2  2 x  x 2 ;2 y  y 2   0;1 ; z   0;2 . Từ 1 ;  2 ;  3 suy ra
2 2
32 x x  2  2 x  x 2   52 y y  4  2 y  y 2   3z  4 z  3
2 2
 32 x x  52 y  y  3z  2 x 2  4 y 2  3  4  x  2 y  z   P  27 .
 x  y  2; z  0
Dấu "  " xảy ra  
 x  z  2; y  1
Vậy max P  27 .

Trang 10 Fanpage Nguyễn Bảo Vương  https://www.facebook.com/tracnghiemtoanthpt489/


Điện thoại: 0946798489 TUYỂN CHỌN VẬN DỤNG – VẬN DỤNG CAO 2022
Câu 18. (THPT Nho Quan A – Ninh Bình – 2022) Có bao nhiêu số nguyên x sao cho úng với mỗi x có
không quá 255 số nguyên y thỏa mãn log 5  x 2  y   log 2 ( x  y ) ?
A. 1250.
B. 1249.
C. 625.
D. 624.
Lời giải
Chọn A
Bất phương trình đã cho tương đương log 2 ( x  y )  log 5  x 2  y   0 (1)
Xét hàm số f ( y )  log 2 ( x  y )  log 5  x 2  y  .
Tập xác định D  ( x;  ) .
1 1
Với mọi x   ta có x 2  x nên f ( y )   2  0, x  D  f ( y ) đồng biến
 
( x  y ) ln 2 x  y ln 5
trên khoảng (  x;  ) .
Do y là số nguyên thuộc ( x;  ) nên y   x  k , k    .
Giả sử y   x  k là nghiệm của bất phương trình (1) thì f ( y )  f (  x  k )  0 .
Mà  x  1   x  2    x  k và f ( y ) đồng biến trên khoảng (  x;  ) , suy ra
f (  x  1)  f (  x  2)    f ( x  k )  0 , nên các số nguyên  x  1,  x  2, ,  x  k đều là
nghiệm của (1), hay nói cách khác bất phương trình (1) sẽ có k số nguyên y thỏa mãn yêu cầu
ứng với mỗi x .
Để có không quá 255 số nguyên y thì f (  x  256)  0  log 2 256  log 5  x 2  x  256   0
1  1561477 1  1561477
 x 2  x  390369  0  x .
2 2
Mà x   nên có 1250 số nguyên x thỏa yêu cầu bài toán.
Câu 19. (THPT Phù Cừ - Hưng Yên - 2022) Tổng các nghiệm nguyên của bất phương trình
 
log 2 x3  log 22 (2 x)  13
 0 là
1  8  ( 2) x  2
A. 16. B. 8. C. 36. D. 136.
Lời giải
Chọn D
 x  0
Điều kiện  x2
 x  0.
8  ( 2)  0

Với điều kiện suy ra bất phương trình:


 
log 2 x3  log 22 (2 x)  13
0
1  8  ( 2) x  2
2 2 1
 3log 2 x  1  log 2 x   13  0    log 2 x   log 2 x  12  0  3  log 2 x  4   x  16
8
(thoả mãn).
Vi x    x  {1; 2;3;;16} .
Do đó tổng các nghiệm nguyên của bất phương trình là 1  2  3  16  136 .
2 2
2 x  y 1
Câu 20. (THPT Phù Cừ - Hưng Yên - 2022) Cho các số thực x, y thỏa mãn 2 2
 4 x 1 và
x  y  2x  2
2x  y  0 . Giả trị lớn nhất và giá trị nhỏ nhất của biểu thức P  3 x  2 y  1 lần lượt là M và m .
Tính M  m .
A. 6. B. 10. C. 12. D. 8.
Lời giải

Facebook Nguyễn Vương https://www.facebook.com/phong.baovuongTrang 11


Blog: Nguyễn Bảo Vương: https://www.nbv.edu.vn/
2 2
2 x  y 1 2 2
Ta có 2 2
 4 x 1  2 x  y  2 x 1  x 2  y 2  2 x  2
x  y  2x  2
Đặt t  x 2  y 2  2 x  1, (t  0) bất phương trình trở thành 2t  t  1  2t  t  1  0
Xét hàm số f (t )  2t  t  1 với t  0 .
 1 
Có f (t )  2t ln 2  1  f (t )  0  t  log 2  .
 ln 2 
Mặt khác f (0)  f (1)  0 .
Ta có bảng biến thiên

Do đó (1)  f (t )  0  0  t  1  0  x 2  y 2  2 x  1  1  0  ( x  1)2  y 2  1 .
( x  1) 2  y 2  1
Suy ra hệ bất phương trình  (1).
2 x  y  0
Tập hợp các điểm thoả mãn (1) thuộc miền mầu sẫm giới hạn bởi hình tròn tâm I (1; 0) bán kinh
R  1 và nửa mặt phẳng bờ là đường thẳng d : 2 x  y  0 chứa điểm I (1; 0) .
Ta có P  3 x  2 y  1  3 x  2 y  1  P  0 là đường thẳng  song song với đường thẳng
d1 : 3 x  2 y  0 .

Từ đồ thị suy ra P đặt max và min khi  tiếp xúc với miền nghiệm của hệ (1)
|4P|  P  4  13
Suy ra d ( I ,  )  1  1  .
13  P  4  13
Vậy M  Pmax  4  13; m  Pmin  4  13  M  m  8 .
1
 
Câu 21. (Sở Hà Tĩnh 2022) Xét các số thục̣ dương x, y, z thoả män ( y  z )  3x  81 y  z   xy  xz  4 . Giá
 
 
trị nhỏ nhất của biểu thức P  log 2 x  log 2  2 y  z  bằng
2 2

A. 2  log 2 3 .
B. 5  log 2 3 .
C. log 2 11 .
D. 4  log 2 3 .

Trang 12 Fanpage Nguyễn Bảo Vương  https://www.facebook.com/tracnghiemtoanthpt489/


Điện thoại: 0946798489 TUYỂN CHỌN VẬN DỤNG – VẬN DỤNG CAO 2022
Lời giải
Chia hai vế cho ( y  z ) ta được:
1 4 4
  4 4
( y  z )  3x  81 y  z   x( y  z )  4  3x  3 y  z  x   3x  x  3 y  z  (*).
  yz yz
 
Xèt hàm số g ( a )  3a  a trên (0;  ) có g ( a )  3a ln 3  1  ln 3  1  0, a  0 do đó g ( a ) đồng
 4  4
biến trên (0;  ) . Vì vậy (*)  g ( x)  g   x .
 yz yz
Đến đây quan sát biểu thức P ta sẽ đánh giá 2 y 2  z 2 theo y  z bằng bất đẳng thức Cauchy -
2
2y 2 z 2 ( y  z)2 2
2 2 4 32
Schwarz 2 y  z     ( y  z)2     2 .
1 1 1 3 3  x  3x
1
2 2
 32  32
 
Do đó P  log 2 x  log 2 2 y 2  z 2  log 2 x  log 2  2   log 2
 3x  3
 5  log 2 3 .

Câu 22. (Sở Thanh Hóa 2022) Cho hàm só f ( x)  2 x  2 x  2022 x 3 . Biết rằng tồn tại só thực m sao cho
bá́ t phương trình f  4 x  mx  37 m   f  ( x  m  37)  2 x   0 nghiệm đúng với mọi x   . Khi đó
m thuộc khoảng nào dưới đây?
A. (30;50) .
B. (10;30) .
C. (50; 70) .
D. (10;10) .
Lời giải

f ( x )  2 x ln 2  2  x ln 2  6066 x 2  0, x; f ( x)  2 x  2 x  2022 x 3    2 x  2 x  2022 x3    f ( x)
.
      
f 4 x  mx  37 m  f ( x  m  37).2 x  0  f 4 x  mx  37 m  f ( x  m  37).2 x 
4 x  mx  37 m  ( x  m  37).2 x  4 x  m  2 x  m( x  37)  ( x  37).2 x  0
 2 x  2 x  m   ( x  37)  2 x  m   0   2 x  m  2 x  x  37   0
ycbt  g ( x)   2 x  m  2 x  x  37   0, x . Do 2 x  x  37  0 có nghiệm x  5 do đó điều kiện
cần là 2 x  m  0 có nghiệm x  5  m  25  32 .
Điều kiện đủ thử lại m  32  g ( x)   2 x  32  2 x  x  37   0, x (thoả mãn).
Câu 23. (Sở Thanh Hóa 2022) Gọi S là tập tất cả các số nguyên y sao cho với mỗi y  S có đúng 10 số
nguyên x thoả mãn 2 y  x  log 3  x  y 2  . Tổng các phần tử của S bẳng
A. 7.
B. 4 .
C. 1.
D. 1 .
Lời giải
Điều kiện: x   y . Khi đó bpt  g ( x )  log 3  x  y 2   2 y  x  0 .
2

1
Có g ( x)   
 2 y  x ln 2  0, x  y 2  0 .
 
x  y 2 ln 3
Bảng biến thiên:

Facebook Nguyễn Vương https://www.facebook.com/phong.baovuongTrang 13


Blog: Nguyễn Bảo Vương: https://www.nbv.edu.vn/

Kẻ thêm y  0  g ( x )  0  x  x0 như bảng biến thiên.


Vậy tập nghiệm của bất phương trinh là S x    y 2 ; x0  chửa đủng 10 số nguyên là các số
 y 2  1,,  y 2  10
  y 2  10  x0   y 2  11
 g   y 2  10   0 y  y 2 10 2
  log 310  2 0  y  y  10  log 2  log 3 10 
   2  y  {4, 3}.
 g   y  11  0
y  y 2 11
 y  y  11  log 2  log 3 11
2
log 311  2 0
Câu 24. (Sở Bắc Giang 2022) Có bao nhiêu số nguyên dương x sao cho ứng với mỗi x có đúng 9 số
nguyên y thỏa mãn  2 y 1  x 2  3 y  x   0 ?
A. 64.
B. 67.
C. 128.
D. 53.
Lời giải
y 1 2
2  x  0
THl:  y  log 2 x 2  1  y  log3 x (1).
3  x  0
Điều kiện cần log 2 x 2  1  log3 x  2 log 2 x  1  log Vì x     x  1. Thử lại x  1 loại.
2 y 1  x 2  0
TH2:  y  log 3 x  y  log 2 x 2  1(2)
3  x  0
Để có đúng 9 số nguyên y ta phải có y  1  log 3 x  y  y  1    y  8  log 2 x 2  1  y  9
3 y 1  x  3 y

  y 9 y 10
 2 2  x  2 2 .
 y 210
2  3 y 1  y  6, 06
Hệ trên vô nghiệm   y 9
  y  4,14. .
3 y  2 2 

y  5
Từ đó, y nguyên ta được hệ có nghiệm khi  .
y  6
Do đó ta chỉ có hai trường hợp sau thỏa mãn bài toán
 y  {5; 6;;13} nghĩa là 4  log 3 x  5;6;;13  log 2 x 2  1  14 , ta được x  {129;181} có
53 số nguyên.
 y  {6; ;14} nghĩa là 5  log 3 x  6; 7;;14  log 2 x 2  1  15 , ta được x  {243; 256} có
14 số nguyên.
Câu 25. (Sở Bắc Giang 2022) Có bao nhiêu số nguyên x sao cho tồn tại số thực y thỏa mãn
 
2 log 3 ( x  y  1)  log 2 x 2  2 x  2 y 2  1 ?
A. 4.
B.2.
Trang 14 Fanpage Nguyễn Bảo Vương  https://www.facebook.com/tracnghiemtoanthpt489/
Điện thoại: 0946798489 TUYỂN CHỌN VẬN DỤNG – VẬN DỤNG CAO 2022
C. 3.
D. 1.
Lời giải

Đặt X  x  1 . Khi đó, ta có 2 log 3 ( X  y )  log 2  X 2  2 y 2   log 3 ( X  y )  log 4  X 2  2 y 2 


 X  y  3t
Đặt log3 ( X  y )  log 4 X 2  2 y 2  t   2 
2 t 
X  2y  4
t
y 2 ( X  y ) 2 32t 2 t 4 2 1
 4t  X 2      9  3  4t  2.9t      t  .
1 1 3 3 9 3 2
1
2 2 2
0  X  y  3
Suy ra  2 2
0  X  2 y  2
Ta có: X 2  2 y 2  2  0  X 2  2   2  X  2  X {1; 0;1} do X nguyên.
+ Với X  0 , ta có
t t
 y  3 t t 4
log 4 2

 2 t
 2.9  4     2  t  log 4 2  y  3 9
.
2 y  4 9
  9

 y  3t  1 2
+ Với X  1 , ta có  2 t
 2. 3t  1  4t  1. *   
2 y  4  1
Ta thấy t  0 là nghiệm của (*)  Phương trình đã cho có nghiệm y  0 .
 y  3t  1
+ Với X  1 , ta có  2 t
.
2 y  4  1
Vì y  3t  1  y  1
Mặt khác, ta có:
X 2  2 y 2  2  0  2 y 2  2  2 y 2  2  y 2  1 | y | 1
Do vậy y  1 là không thỏa mãn nên X  1 không thỏa mãn Vậy X  {0;1} hay x  {1; 0} thì
tồn tại số thực y thỏa mãn 2 log 3 ( x  y  1)  log 2  x 2  2 x  2 y 2  1 .
Câu 26. (Sở Hà Tĩnh 2022) Có bao nhiêu giá trị nguyên của tham số m  [ 10;10] để phương trình
m 2 m 2 m
23  7 x 2 x
 73  2 x 2 x
 143  7 x 2  14 x  2  7  3m 
có bốn nghiệm phân biệt trong đó có đúng hai nghiệm lớn hơn 1 ?
A. 10.
B. 9.
C. 11.
D. 8.
Lời giải.
Ta có
m 2 m 2 m
23  7 x  2 x  73  2 x  2 x  143  7 x 2  14 x  2  7  3m 
2 2
7x 2 x
2x 2 x
2
 2 x 3m 2
 2 x 3m
 3 m  3m
 7 x 2  14 x  2  7  3m  7 x  2x  7  x 2  2 x  3m   2
7 2
Đăt x  2 x  3m  a .
2

Khi đó (*) trở thành 7 a  2 a  7 a  2  7 a  2 a  7 a  2  0 .


Xét hàm số f ( a )  7 a  2a  7 a  2 .
Ta có f (a )  7 a ln 7  2 a ln 2  7 .
Ta có f ( a)  7 a (ln 7) 2  2a (ln 2) 2  0, a   .

Facebook Nguyễn Vương https://www.facebook.com/phong.baovuongTrang 15


Blog: Nguyễn Bảo Vương: https://www.nbv.edu.vn/
Suy ra f ( a ) đồng biến trên  , do đó f ( a )  0 có tối đa 1 nghiệm.
Mà f (0)  ln 7  ln 2  7  0 và f (1)  7 ln 7  2 ln 2  7  0 .
Suy ra f ( a )  0 có nghiệm duy nhất a0  (0;1) .
Suy ra f ( a )  0 có tối đa 2 nghiệm.
Bảng biến thiên của y  f (a )

Từ bảng biến thiên ta có f ( a )  0 có đúng 2 nghiệm a  0 và a  1 .


 a  x 2  2 x  3m  0 3m  x 2  2 x
Từ đó  2 m
  m 2
(**)
 a  x  2 x  3  1 3  x  2 x  1
Để (*) có 4 nghiệm thực phân biệt trong đó có đúng hai nghiệm lớn hơn 1 thì (**) có 4 nghiệm
thực phân biệt trong đó có đúng hai nghiệm lớn hơn -1 hay tương đương với đồ thị hàm số y  3m
cắt đồ thị các hàm số y  x 2  2 x và y  x 2  2 x  1 tại 4 điểm phân biệt trong đó có đúng hai
điểm có hoành độ lớn hơn 1 .

Dựa vào đồ thị ta có 3m  3  m  1 .


Suy ra m  {1; 2;;10} .
Vậy có 10 giá trị của m thỏa mãn bài toán.
Câu 27. (Sở Ninh Bình 2022) Cho các số thực a , b thỏa mãn 1  a  b  4 . Tìm giá trị nhỏ nhất của biểu
16
thức P  3log a  b 2  16b  16    log 3b a .
27 a
A. 8.
B. 18.
C. 9.
D. 17.
Lời giải.

Trang 16 Fanpage Nguyễn Bảo Vương  https://www.facebook.com/tracnghiemtoanthpt489/


Điện thoại: 0946798489 TUYỂN CHỌN VẬN DỤNG – VẬN DỤNG CAO 2022
1 1
Ta có log b a   . (1)
b log a b  1
a log a
a
Với b  (1; 4] ta có
(b  1)  b 2  16   0  b3  b 2  16b  16  0  b3  b 2  16b  16  log a  b 2  16b  16   log a b3
 log a  b 2  16b  16   3log a b  2 
Từ (1) và (2), ta có
16 16 1
P  3log a  b 2  16b  16    log 3b a  9 log a b   .
27 a 27  log a b  13
Đặt t  log a b  1 , ta có
16 1
P  3(t  1)  3(t  1)  3(t  1)   9
27 (t  1)3
16 1
 4 4 27  (t  1)3    9  17.
27 (t  1)3
Đẳng thức xảy ra khi và chỉ khi
b  4 b  4 b  4 b  4 3
    a  4 5
 16 1  4 16   5   3 Vậy min P  17 .
3(t  1)  27  (t  1)3 (t  1)  81 t  3 log b a  5 b  4
  
Câu 28. 
(Sở Bạc Liêu 2022) Cho phương trình 2log 32 x  log 3 x  1  5x  m  0 ( m là tham số thực). Có
tất cả bao nhiêu giá trị nguyên dương của m để phương trình đã cho có đúng 2 nghiệm phân
biệt?
A. 125 . B. 123 . C. 122 . D. 124 .
Lời giải
Chọn B
x  0
Điều kiện:  .
 x  log5 m
Với m  1 , điều kiện của phương trình là x  0 và phương trình trở thành
log3 x  1
2   x  3  nhaän 
 2 log3 x  log3 x  1  0 1
 2 x

2log 3 x  log3 x  1 5  1  0   x  log3 x     1 .
5  1  0  2

x  nhaän 
 x  0(l ) 3

Suy ra m  1 thỏa yêu cầu bài toán.
Với m  2 , điều kiện của phương trình là x  log 5 2 và phương trình trở thành
 2log 2
3 x  log 3 x  1 5x  2  0
log 3 x  1 x  3  nhaän 
2  
 2log x  log3 x  1  0 1 1
 x 3  log 3 x     x   nhaän  .
5  2  0  2  3
 x  log 2(n) 
 5
 x  log5 2  nhaän 
Suy ra m  2 không thỏa yêu cầu bài toán.
Với m  3 , điều kiện phương trình là x  log 5 m , khi đó
 2 log 32 x  log 3 x  1  0
 3
2 log 2
x  log 3 x  1 5 x
 m  0   x
5  m  0

Facebook Nguyễn Vương https://www.facebook.com/phong.baovuongTrang 17


Blog: Nguyễn Bảo Vương: https://www.nbv.edu.vn/
 log 3 x  1 x  3  nhaän 
 
1 1
  log 3 x     x   loaïi  .
 2  3
5 x  m  x
 5  m
Do đó phương trình có đúng 2 nghiệm khi và chỉ khi 3  log 5 m  m  53  125 .
Suy ra m  3; 4;5;...;124 nên có 122 giá trị của m .
Vậy có tất cả 123 giá trị của m .
Câu 29. (Sở Hà Tĩnh 2022) Có bao nhiêu cặp số  x; y  (trong đó x, y nguyên dương thuộc đoạn
 0;2022 thỏa mãn điều kiện 2 x  log 2  y 2  615  y 2  x  615
A. 1 . B. 3 . C. 4 . D. 2 .
Lời giải
Chọn A
  
2 x  x  y 2  615  log 2 y 2  615 . 
 
Đặt t  log 2 y 2  615  y 2  615  2t . Khi đó ta có 2x  x  2t  t .
Xét hàm số f  x   2  x, x   , ta có f   x   2 x ln 2  1  0, x   nên hàm số f  x  đồng biến
x

trên  .
 
Do vậy 2 x  x  2t  t  x  t  x  log 2 y 2  615  y 2  2x  615 .
Theo bài ra
0  y  2022  0  y  2022  0  2  615  20222  log 2 615  x  log 2  20222  615 
2 2 x

 9, 3  x  21, 9 .
Khi x chạy từ 10 tới 21 , ta thấy x  12  y  59 . Vậy có 1 cặp số  x; y  thỏa mãn yêu cầu bài
toán.
Câu 30. (Sở Lạng Sơn 2022) Với a là tham số thực để bất phương trình 2 x  3 x  ax  2 có tập nghiệm là
R , khi đó
A. a  1;3 . B. a   0;1 . C. a   ; 0  . D. a   3;   .
Lời giải
Chọn A
2 x  3x  ax  2  2 x  3 x  ax  2  0, x   (1).
Xét f ( x)  2 x  3x  ax  2 liên tục trên R .
Ta có
f (0)  20  30  a.0  2  0
f ( x)  2 x ln 2  3x ln 3  a .
Để f  x   0, x    f  x   f  0  , x  
 x  0 là điểm cực tiểu
Vì x  0 là điểm cực tiểu f   0   0  ln 3  ln 2  a  a  ln 6
Thử lại:
a  ln 6  f  x   2 x  3x  ln 6.x  2
f   x   2 x ln 2  3x ln 3  ln 6
Ta có vế trái h  x   2 x ln 2  3x ln 3 đồng biến; vế phải k  x   ln 6 là hàm hằng.
 x0
Bảng biến thiên

Trang 18 Fanpage Nguyễn Bảo Vương  https://www.facebook.com/tracnghiemtoanthpt489/


Điện thoại: 0946798489 TUYỂN CHỌN VẬN DỤNG – VẬN DỤNG CAO 2022

x ∞ 0 +∞
f'(x) 0 +
+∞ +∞
f(x)
0

Từ bảng biến thiên  f  x   0, x  


Vậy a  ln 6 .
Câu 31. (Sở Lạng Sơn 2022) Biết đồ thị hàm số y  f  x  đối xứng với đồ thị hàm số
 1 
y  a x  a  0, a  1 qua điểm I 1;1 . Giá trị của biểu thức f  2  log a  bằng
 2022 
A. 2022 . B. 2021. C. 2022. D. 2020 .
Lời giải
Chọn D
1
Với a  0, a  1 , ta có 2  log a  2  log a 2022 .
2022
Xét điểm N  2  log a 2022; f  2  log a 2022   thuộc đồ thị hàm số y  f  x 
Gọi M là điểm đối xứng với điểm N qua I 1;1 thì M  log a 2022; 2  f  2  log a 2022  
Theo đề, M  log a 2022; 2  f  2  log a 2022   thuộc đồ thị hàm số y  a x nên
2  f  2  log a 2022   a loga 2022  2  f  2  log a 2022   2022  f  2  log a 2022   2020 .
 1 
Vậy f  2  log a   2020 .
 2022 
9  m
 2  2
  5  m  7  m  5; 6 .
11  m  2
 2
Câu 32. (Sở Phú Thọ 2022) Xét các số thực dương x, y thỏa mãn
2 2 1 2
2  x 2  y 2  4   log 2022      xy  4  . Khi biểu thức P  x  4 y đạt giá trị nhỏ nhất, giá trị
x y 2
y
của bằng
x
1 1
A. 4 . B. 2 . C. . D. .
2 4
Lời giải
Chọn C
2 2 1 2
2  x 2  y 2  4   log 2022      xy  4 
x y 2
 2x  2 y  2
 4 x 2  4 y 2  16  2 log 2022     xy   8 xy  16
 xy 
2 2
  2 x  2 y   2 log 2022  2 x  2 y    xy   2 log 2022  xy  * .
2
Xét hàm số f  t   t  2 log 2022 t với t  0 .
2
Ta có: f   t   2t   0, t  0 .
t.ln 2022
2y
Do đó: *  f  2 x  2 y   f  xy   2 x  2 y  xy  x  y  2   2 y  x  ,  y  2 .
y2

Facebook Nguyễn Vương https://www.facebook.com/phong.baovuongTrang 19


Blog: Nguyễn Bảo Vương: https://www.nbv.edu.vn/
2y  1 
P  x  4y  4y   4  y  2    10  18 .
y2  y  2 
1 y 1
Vậy Pmin  18 khi y  2   y 3 x  6  .
y2 x 2
Câu 33. (Sở Phú Thọ 2022) Có bao nhiêu số nguyên x sao cho ứng với mỗi x có không quá 20 số
2
nguyên y thỏa mãn 4 x 5 y 16  2 x  y  512 và x  y  0 ?
A. 19 . B. 20 . C. 21 . D. 18 .
Lời giải
Chọn B
2 2
Từ giả thiết ta có 4 x 5 y 16  2 x  y  512  4 x 5 y 16  2 x  y  512  0 .
2
Xét hàm số f  y   4 x 5 y 16  2 x  y  512 .
Vì x  y  0  y   x nên ta xét y    x;   .
2
Có f   y   5.4 x  5 y 16
.ln 4  2 x  y.ln 2  0, y    x;   .
Suy ra hàm số f  y  luôn nghịch biến.
2
Có f   x  1  4 x  5 x 11
 21  512  45  21  512  0, x   .
Bảng biến thiên của f  y  :

Với mỗi số nguyên x , để có không quá 20 số nguyên y thỏa mãn f  y   0 và x  y  0 thì ta


2
phải có f   x  21  0  4 x  5 x 89
 2 21  512  x 2  5 x  89  log 4  512  221 
 x 2  5 x  89  log 4  512  221   0

5  381  4 log 4  512  2 21  5  381  4 log 4  512  2 21 


 x
2 2
 12, 487  x  7, 487 .
Vì x  nên x  12; 11;...;7  có 20 số nguyên x thỏa mãn.
Câu 34. (Sở Thái Nguyên 2022) Cho bất phương trình
 2
  2
 
log 2 x  x  1 .log 2022 x  x  1  log m x  x  1 . Có bao nhiêu giá trị nguyên thuộc2

khoảng 1; 2022  của tham số m sao cho bất phương trình đã cho nghiệm đúng với mọi x thuộc
5;   ?
A. 2022. B. 2021 . C. 2012 . D. 2010 .
Lời giải
Chọn C
Nhận thấy: Với x  5 thì x 2  1  x 2  x  x  x 2  1  0 và x  x 2  1  0 .
  
Ta có: log 2 x  x 2  1 .log 2022 x  x 2  1  log m x  x 2  1   
  
 log 2 x  x 2  1 .log 2022 x  x 2  1  log m 2.log 2 x  x 2  1   
Trang 20 Fanpage Nguyễn Bảo Vương  https://www.facebook.com/tracnghiemtoanthpt489/
Điện thoại: 0946798489 TUYỂN CHỌN VẬN DỤNG – VẬN DỤNG CAO 2022

  
 log 2022 x  x  1  log m 2 1 (vì log 2 x  x 2  1  0 , x  5 ).
2


Xét hàm số f  x   log 2022 x  x 2  1 trên 5;   . 
1
Ta có: f   x    f   x   0 , x  5 .
2
x  1.ln 2022
BBT:

Từ BBT ta thấy bất phương trình đã cho nghiệm đúng với mọi x thuộc 5;  
1
 log m 2  f  5 
log 2 m
 
 log 2022 5  24  log 2 m  log5 24
2022  m  2
log 5 24
2022

log5 2022
(do m  1)  m  2  9,9 . 24

Do m nguyên thuộc khoảng 1; 2022  nên m  10;11;...; 2021 .


Vậy có 2012 giá trị của m thỏa mãn yêu cầu bài toán.
Câu 35. (Sở Thái Nguyên 2022) Cho x, y  0; x  3 y  0 thỏa mãn
 x2  y 2  x2  y2
2022  log 2  1  x  3 y  . Tổng của giá trị lớn nhất và giá trị nhỏ nhất của
 x  3y  4
 
biểu thức P  x 2  y 2  14 x  2 y  2022 bằng
A. 4124 B. 4042 . C. 4044 D. 4122

Lời giải
Chọn A
 x2  y 2  x2  y2
Ta có 2022  log 2  1  x  3 y 
 x  3y  4
 
x2  y 2

 2022 log 2 x 2  y 2  log 2 x  3 y  log 2 2  x  3 y   4
 x2  y 2  x2  y2
 2022  log 2  log 2 x  3 y   x  3 y 
 4  4
 
x2  y 2 x2  y2
 2022 log 2   2022log 2 x  3 y  x  3 y , *
4 4
Ta xét hàm số y  f  t   t  2022 log 2 t , t  0.
2022
Ta có y  f   t   1   0 , t  0. Do đó, hàm số luôn đồng biến trên khoảng xác định.
t ln 2
x2  y 2 x2  y2 2 2
Từ * suy ra x  3 y   x  3y    x  2    y  6   40  C 
4 4
2 2 2 2 2 2
P  x  y  14 x  2 y  2022   x  7    y  1  1972   x  7    y  1  P  1972  C   Dễ
 I  2;6   I   7;1
thấy:  C   ;  C  
 R  40  R  P  1972  0

Facebook Nguyễn Vương https://www.facebook.com/phong.baovuongTrang 21


Blog: Nguyễn Bảo Vương: https://www.nbv.edu.vn/
Mà: II   50  R  I    C  
  Pmin  1972 khi và chỉ khi  C  ;  C   tiếp xúc ngoài với nhau.
Rmin
Ta có R  Pmin  1972  II   40  Pmin  1972  50  Pmin  2062  40 5
Rm ax  Pm ax  1972 khi và chỉ khi  C  ;  C   tiếp xúc trong với nhau với R  R .
Pm ax  1972  R  II   Pmax  1972  40  50  Pm ax  2062  40 5
Vậy: S  Pmax  Pmin  4124.
Câu 36. (Sở Vĩnh Phúc 2022) Có bao nhiêu cặp số nguyên dương  x, y  thoả mãn
2
34 x 1
log  4 x 2  4 x  2   3 y  2 x  4 log  2 x  y  1 đồng thời x, y  2021
A. 15 . B. 28 . C. 22 . D. 35 .

Lời giải

Chọn C
2
34 x 1 log  4 x 2  4 x  2   3 y  2 x  4 log  2 x  y  1
2
 34 x 4 x2
log  4 x 2  4 x  2   32 x  y 1 log  2 x  y  11
Xét f  t   3t .log t ( t  2 ).
3t  1 
f   t   3t ln 3.log t   3t  ln 3.log t    0 , t  2 .
t.ln10  t.ln10 
Do đó 1  4 x 2  4 x  2  2 x  y  1  y  4 x 2  2 x  3
1  3 897 1  3 897
y  2021  4 x 2  2 x  3  2021  x
4 4
Vì x     0; 2022  nên x  1; 2;3;...; 21; 22 .

Vì mỗi giá trị của x có đúng một giá trị của y nên có 22 cặp số nguyên dương  x, y  thoả mãn.
Câu 37. (Sở Vĩnh Phúc 2022) Xét các số thực x, y thỏa mãn x 2  y 2  1 và log x 2  y 2  2 x  4 y   1 . Giá trị
lớn nhất của biểu thức P  3 x  y bằng
A. 5  2 10 . B. 5  4 5 . C. 5  5 2 . D. 10  2 5 .
Lời giải
Chọn C
Ta có x 2  y 2  1 .
 
Khi đó log x2  y2  2 x  4 y   1  log x2  y2  2 x  4 y   log x2  y 2 x 2  y 2  2x  4 y  x2  y 2
2 2
 x 2  2 x  1  y 2  4 y  4  5   x  1   y  2   5 .
Khi đó P  3x  y  P  3  x  1   y  2   5  P  5  3  x  1   y  2 
2 2 2
 
Áp dụng BĐT Bunhiacopxki, ta có: 3  x  1   y  2    32  12  x  1   y  2    50
 
2 2 2 2
 
  P  5  3  x  1   y  2    32  12  x  1   y  2    50 .
 
2
Vậy  P  5   50  5 2  P  5  5 2  5  5 2  P  5  5 2
Suy ra max P  5  5 2 .
3 2 2
Dấu "  " xảy ra khi và chỉ khi x  1  , y  2 .
2 2

Trang 22 Fanpage Nguyễn Bảo Vương  https://www.facebook.com/tracnghiemtoanthpt489/


Điện thoại: 0946798489 TUYỂN CHỌN VẬN DỤNG – VẬN DỤNG CAO 2022
 1 
Câu 38. (Sở Vĩnh Phúc 2022) Cho hàm số f  x   ln  1  2  . Biết rằng
 x 
a
f   2   f   3   ...  f   2019   f   2020   với a , b là các số nguyên dương nguyên tố cùng
b
nhau. Giá trị của 2a  b bằng
A. 2 . B. 4 . C. 2 . D. 4 .

Lời giải
Chọn C
2 1 1
Ta có: f   x     .
x  x  1 ( x  1) x  x  1 x  x  1
Khi đó
f '2  f '3  ...  f '2019  f '2020
1 1 1 1 1 1 1 1
     ...    
1.2 2.3 2.3 3.4 2018.2019 2019.2020 2019.2020 2020.2021
1 1 1010.20211
  
2 2020.2021 2020.2021
Nên a  1010.2021  1, b  2020.2021  2 a  b  2 .
Câu 39. (Sở Vĩnh Phúc 2022) Cho hàm số bậc ba y  f  x  có đồ thị như hình vẽ

Số nghiệm của phương trình log32  f  x   1  log 2 2  f  x   1  2 log 1 f  x   1  6  0 là


2
A. 7 . B. 5 . C. 6 . D. 8 .
Lời giải
Chọn C
Xét phương trình log32  f  x   1  log 2 2  f  x   1  2 log 1 f  x   1  6  0 (1).
2

Điều kiện: f  x   1 .
Ta có 1  log 32  f  x   1  4 log 22  f  x   1  log 2  f  x   1  6  0 .

t  1 log 2  f  x   1  1
Đặt t  log 2  f  x   1 , ta có t 3  4t 2  t  6  0  t  3  log 2  f  x   1  3
t  2 
log 2  f  x   1  2

Facebook Nguyễn Vương https://www.facebook.com/phong.baovuongTrang 23


Blog: Nguyễn Bảo Vương: https://www.nbv.edu.vn/
 1
 f  x    2  2

  f ( x)  7  3

 f  x   3  4

1
Dễ thấy số nghiệm phương trình  2  là số giao điểm của đồ thị hàm số y  f  x  và y   . Suy
2
ra phương trình (2) có 3 nghiệm phân biệt.
Tương tự phương trình  3  có nghiệm duy nhất và phương trình  4  có 2 nghiệm phân biệt.
Hiển nhiên các nghiệm này phân biệt. Do dó phương trình 1 có 6 nghiệm phân biệt.
Câu 40. (THPT Bùi Thị Xuân – Huế - 2022) Tất cả các giá trị thực của m để bất phương trình
x x  x  12  m log 5 4 x 3 có nghiệm:
A. m  2 3 .
B. m  12 log 3 5 .
C. m  2 3 .
D. 2  m  12 log 3 5 .
Lời giải
x  0

 x  12  0

ĐKXĐ: 4  x  0 0 x4

5  4  x  0
5  4  x  1

Ta có 0  x  4  0  4  x  2  3  5  4  x  5  0  log 5 4 x
31
x x  x  12
Khi đó x x  x  12  m log5 4 x
3 m  ( x x  x  12) log 3 (5  4  x )
log5 4 x 3
Xét hàm số g ( x)  ( x x  x  12) log 3 (5  4  x )
 1 1  1
 g ( x)   x  x  log3 (5  4  x )  ( x x  x  12)
 2 2 x  12  2 4  x (5  4  x ) ln 3
 
log 3 5  4  x  0

Ta có:  1 , x   0; 4  g '  x   0, x   0; 4
  0
 
 2 4  x 5  4  x ln 3
 g ( x ) đồng biến trên [0; 4] .
Trang 24 Fanpage Nguyễn Bảo Vương  https://www.facebook.com/tracnghiemtoanthpt489/
Điện thoại: 0946798489 TUYỂN CHỌN VẬN DỤNG – VẬN DỤNG CAO 2022
Để phương trình x x  x  12  m log 5 4 x
3 khi và chỉ khi m  min[0:4] g ( x )  g (0)  2 3 .
Câu 41. (Chuyên Hạ Long 2022) Cho 0  m  1 . Gọi (a; b) là tập hợp các giá trị của m để bất phương
trình log m 1  8m  x   2(1  x) có hữu hạn nghiệm nguyên. Tính b  a
A. 1.
B. 3 2  1 .
C. 2 2  1 .
D. 4 2  1 .
Lời giải
Trường hợp 1: m  1
Ta có: log m 1  8m  x   2(1  x)  1  8m  x  m 2 2 x  m 2  m 2 x  8m  x  1  0

16  m 2  4  16  m 2  4   16  m 2  4 
 0  m x    x  log m    x   log m .
m2  m 2   m 2 
   
Rỏ ràng trong trường hợp này không thể có hữu hạn nghiệm nguyên
Trường hợp 2 : 0  m  1
 x 22 x m2  m 2 x  8m  x  1  0
1  8m  m 
 x

Ta có: log m 1  8m  2(1  x)   x
  x 1
1  8m  0 m 
  8
 x 16  m 2  4 
m  16  m 2  4  16  m 2
m 2   x  log  x   log
  m
m2  m
m2
 x  log 1  x  log 8 
 m  m  x  log m 8
8
Để bất phương trình có hữu hạn nghiệm nguyên thì:
16  m2  4 8 16  m2  32 8 16  m 2  32
log m 8  log m  0  log m  0  1
m2 m2 m2
 8 16  m 2  m 2  32  m 4  0, m  (0;1)
Vậy b  a  1
Câu 42. (Chuyên Hoàng Văn Thụ - Hòa Bình – 2022) Có tất cả bao nhiêu giá trị nguyên của y sao cho
tương ứng với mỗi giá trị y luôn tồn tại không quá 15 số nguyên x thỏa mãn điều kiện
   
log 2021 x  y 2  log 2022 y 2  y  16  log 2 ( x  y ) ?
A. 2021.
B. 4042.
C. 2020.
D. 4041.
Lời giải

 x  y 2  0  x2  y  0
Điều kiện   .
 x  y  0 x  y
   
Ta có bất phương trình log 2021 x  y 2  log 2022 y 2  y  16  log 2 ( x  y )  0
Xét f ( x)  log 2021  x  y 2   log 2022  y  y  16   log ( x  y ) với x  y, y   .
2
2

1 1 x(ln 2  ln 2021)  y ln 2  y 2 ln 2021


Ta có: f ( x)    .
 
x  y 2 ln 2021 ( x  y ) ln 2  
x  y 2  ( x  y )  ln 2021  ln 2
Ta có: x  y  x (ln 2  ln 2021)  y (ln 2  ln 2021)
Suy ra x(ln 2  ln 2021)  y ln 2  y 2 ln 2021    y 2  y  ln 2021  0, y   .
Do đó f ( x )  0, x  y , y   .
Facebook Nguyễn Vương https://www.facebook.com/phong.baovuongTrang 25
Blog: Nguyễn Bảo Vương: https://www.nbv.edu.vn/
Ta có bảng biến thiên của f ( x ) là:

Yêu cầu bài toán  f ( y  16)  0


 log 2021  y 2  y  16   log 2022  y 2  y  16   log 2 16
log 2021  y 2  y  16 
 log 2021  y  y  16  
2
4
log 2021 2022
4
 log 2021  y 2  y  16    2, 00
1  log 2022 2021
4
2 1 log0022 2021
 y  y  16  2021  2021, 99  y  2020,99
Do y   nên y  {2021; 2020;; 2020} .
Vậy có tất cả 4041 giá trị nguyên y thỏa yêu cầu bài toán.
Câu 43. (Chuyên Lam Sơn 2022) Có bao nhiêu số nguyên dương m để phương trình
m  e x  1  ln( mx  1)  2e x  e 2 x  1 có 2 nghiệm phân biệt không lớn hơn 5.
A. 26.
B. 27.
C. 29.
D. 28.
Lời giải
Xét phương trình m  e  1  ln( mx  1)  2e  e 2 x  1(*) điều kiện mx  1  0
x x

e x  1  0
(*)   x
e  1  m  ln(mx  1)
e  1  0  x  0 e x  1  m  ln( mx  1) ,
x

 x  ln(my  1)(1)
Đặt y  ln( mx  1)  e x  1  my. Ta có hệ phương trình 
 y  ln(mx  1)(2)
Trừ (1) và (2) theo vế ta được: x  y  ln( my  1)  ln( mx  1) hay x  ln( mx  1)  y  ln( my  1)
với m  0 thì hàm số f ( x )  x  ln(mx  1) đồng biến trên tập xác định nên
x  ln(mx  1)  y  ln( my  1)  x  y
Thay x  y vào (1) ta được x  ln(mx  1) hay e x  mx  1(4)
Rõ ràng x  0 là 1 nghiệm của phương trình (4).
ex 1
Với x  0 ta có (4)  m 
x
ex 1 xe x  e x  1
Xét hàm số g ( x)  , ta có: Tập xác định D   \ {0} và g ( x) 
x x2
g ( x)  0  xe x  e x  1  0
Hàm số h( x)  xe x  e x  1 có h( x)  xe x nên h( x )  0  x  0
Ta có bảng biến thiên của h( x ) như sau:

Trang 26 Fanpage Nguyễn Bảo Vương  https://www.facebook.com/tracnghiemtoanthpt489/


Điện thoại: 0946798489 TUYỂN CHỌN VẬN DỤNG – VẬN DỤNG CAO 2022

Suy ra h ( x )  0, x do đó g ( x )  0, x  0
Bảng biến thiên của g ( x ) :

Để phương trình e x  1  ln(mx  1) m có 2 nghiệm phân biệt không lớn hơn 5 thì phương trình
e5  1
m  g ( x ) có duy nhất 1 nghiệm bé hơn hoặc bằng 5. Ta có g (5)   29,5
5
0  m  g (5)
Dựa vào bảng biến thiên của g ( x ) ta có  do m   * nên có 28 giá trị thỏa mãn.
 m  1
Câu 44. (Chuyên Lam Sơn 2022) Cho a , b là các số thực thay đổi thỏa mãn log a2 b2  20 (6a  8b  4)  1
c
và c, d là các số thực dương thay đổi thỏa mãn c 2  c  log 2
d
 
 7  2 2d 2  d  3 . Giá trị

nhỏ nhất của biểu thức (a  c  1) 2  (b  d ) 2 là


A. 4 2  1 .
B. 29  1 .
12 5  5
C. .
5
8 5 5
D. .
5
Lời giải
Ta có: log a2 b2  20 (6a  8b  4)  1  a  b  20  6a  8b  4  (a  3) 2  (b  4) 2  1(1)
2 2

Lại có:
 2 c
c  c  log 2  7  2  2d  d  3
2
c
c  c  log 2  7  2  2d  d  3  
2 2
d
d 2d 2  d  3  0; d , c  0( gt )

2 2
c  c  log 2 c  (2d )  2d  log 2 2d c  1  2 d  1
   2
d  1; c  0 d  1; c  2
Đặt M ( a; b ) và N (c  1; d ) . Theo (1) ta được M thuộc đường tròn tâm I (3; 4) bán kính R  1 ;
theo (2) ta được N thuộc nửa đường thẳng y  2 x  1 ứng với x  1.
Khi đó MN  ( a  c  1) 2  (b  d ) 2 .

Facebook Nguyễn Vương https://www.facebook.com/phong.baovuongTrang 27


Blog: Nguyễn Bảo Vương: https://www.nbv.edu.vn/

Vậy MN min  N1 I  R  29  1 .
Câu 45. (Chuyên Lương Văn Tụy-Ninh Bình 2022) Cho phương trình 2 log32 x  log3 x  1   5x  m  0
( m là tham số thực). Có tất cả bao nhiêu giá trị nguyên dương của m để phương trình đã cho có
đúng hai nghiệm phân biệt?
A. 125.
B. 123.
C. 122.
D. 124.
Lời giải
x  0 x  0
Điều kiện  x  x
.
5  m  0 m  5
5x  m  0, x  0

 2 log 2
3 x  log3 x  1 5 x  m  0   5 x  m  0
 2
  2 log 3 x  log 3 x  1  0
5 x  m  0, x  0 5 x  m  0, x  0
 x  x
 5  m  0  5  m  0
 
 1   1
  log3 x  2   log3 x  2
 
  log3 x  1   log3 x  1

Trang 28 Fanpage Nguyễn Bảo Vương  https://www.facebook.com/tracnghiemtoanthpt489/


Điện thoại: 0946798489 TUYỂN CHỌN VẬN DỤNG – VẬN DỤNG CAO 2022
 Khi m  1  x  log 2 1  0 vậy phương trình  2 log 32 x  log3 x  1 5x  m  0 có 2 nghiệm
1

x  3 2
x  3

 m  1  x  log 5 m là 1 nghiệm. Để phương trình có đúng 2 nghiệm thì
1
1
 log 5 m  3  5 3
 m  53  2,53  m  125
3
1
Câu 46. (Chuyên Lương Văn Tụy-Ninh Bình 2022) Cho các số thực a , b thỏa mãn  b  a  1 . Tìm
3
giá trị nhỏ nhất của biểu thức
4(3b  1)
P  log a  8log 2b a
9 a
A. 7.
B. 8.
C. 6.
D. 9.
Lời giải
1
Vì  b  a  1 nên (3b  2)2  0
3
4(3b  1) 4(3b  1)
 b2   log a  log a b 2
9 9
2
2  1 
Ta có 8log a  8 
b 
a  log a b  1 
1
Đặt log a b  x . Vì  b  a  1 nên x  log a b  1 . Khi đó
3
2
4(3b  1)  1  8
P  log a  8log 2b a  log a b 2  8    P  2x 
9 a  log a b  1  ( x  1)2
8 8 8
Mà 2 x  2
 ( x  1)  ( x  1)  2
 2  3  3 ( x  1)  ( x  1)  28
( x  1) ( x  1) ( x  1)2
Suy ra P  8
 2
b  3
Dấu "  " xảy ra  
a  3 2
 3
Vậy min P  8 .
Câu 47. (Chuyên Nguyễn Trãi – Hải Dương – 2022) Gọi S là tập các giá trị của tham số m để bất
phương trình log 0.3  x 2  2( m  3) x  4   log 0.3  3 x 2  2 x  m  thỏa mãn với mọi x thuộc  . Tập
S bằng
A. S  [5; 6) .
B. S  [4; 6] .
C. S  [4;5) .
D. S  [1;5) .
Lời giải
Để bất phương trình thỏa mãn với mọi x thuộc  thì

Facebook Nguyễn Vương https://www.facebook.com/phong.baovuongTrang 29


Blog: Nguyễn Bảo Vương: https://www.nbv.edu.vn/
 x 2  2  m  3 x  4  0, x  
 2
3 x  2 x  m  0, x  
 x 2  2 m  3 x  4  3x 2  2 x  m, x  
  
2  m  3  2 1  m  5
 
 1  1 1  m  5
 m   m    4m5
 3  3 4  m  5
(m  4)2  (m  4)  0 2
m  9m  20  0
Vậy, S  [4;5) .
Câu 48. (Chuyên Nguyễn Trãi – Hải Dương – 2022) Cho các số thực a , b, c, d thỏa mãn điều kiện:
 
log 2 a 2  b 2  5  1  log 2 (2  2a  b)
 4c 5d 10 c  d  2 Tìm giá trị nhỏ nhất của biểu thức
e e  12  3c  4d
P  ( a  c) 2  (b  d ) 2
2 5
A.
5
B. 2.
C. 2 5  2 .
12
D. .
5
Lời giải
Điều kiện: 2  2a  b  0  2a  b  2  0 (1).
Ta có: log 2  a 2  b 2  5   1  log 2 (2  2a  b)  log 2  a 2  b 2  5   log 2 2  log 2 (2  2a  b)
 
 log 2 a 2  b 2  5  log 2 (4  4a  2b)  a 2  b 2  5  4  4a  2b
2 2
 (a  2)  (b  1)  4.
a 2  b2  5
Mặt khác a 2  b 2  5  4  4a  2b  2a  b  2   0 . Do đó điều kiện (1) luôn thỏa
2
mãn.
Lại có: e 4c  5d 10  ec  d  2  12  3c  4d  e 4c  5d 10  4c  5d  10  ec  d  2  c  d  2(*)
Do hàm f (t )  et luôn đồng biến trên R. Suy ra (*)  4c  5d  10  c  d  2  3c  4 d  12 .
Đặt A( a; b); B (c; d )  P  AB .
A di động trên đường tròn (C ) có phương trình: ( x  2) 2  ( y  1)2  4 , tâm I ( 2; 1); R  2 .
B di động trên đường thẳng d : 3 x  4 y  12  0 .
| 2.3  1.4  12 | 22 22 12
Có d ( I , d )    2  Pmin  ABmin  d ( I , d )  R  2 .
32  42 5 5 5
Câu 49. (THPT Lương Thế Vinh – Hà Nội -2022) Gọi S là tập các số nguyên m  [ 2022; 2022] để
phương trình log 22 x  log 2
x  m  m  log 2 x có đúng ba nghiệm phân biệt. Số phần tử của S

A. 1.
B. 2.
C. 2021.
D. 2022.
Lời giải
2
Đặt t  m  log 2 x , (t  0)  m  t  log 2 x phương trình trở thành:

Trang 30 Fanpage Nguyễn Bảo Vương  https://www.facebook.com/tracnghiemtoanthpt489/


Điện thoại: 0946798489 TUYỂN CHỌN VẬN DỤNG – VẬN DỤNG CAO 2022
 log 2 x  t
 
log 22 x  2 log 2 x  t 2  log 2 x  t  log 22 x  log 2 x  t 2  t  0  
log 2 x  1  t
(bấm máy phương trình bậc hai ần là log 2 x và tham số t  1000  .
 m  t2  t
Khi đó  2
(*) .
m  t  t  1

ycbt  (*) có 3 nghiệm phân biệt trên [0;  )  đường thẳng y  m cắt đồ thị của hai hàm số
1
y  t 2  t ; y  t 2  t  1 tại ba điểm phân biệt trên [0;  )    m  0.
4
Câu 50. (THPT Lương Thế Vinh – Hà Nội – 2022) Có bao nhiêu cặp số nguyên (a; b) , trong đó
2b a
 2a   a  2b 
a , b  [1; 2022] thỏa mãn  b 
  b 1  ?
a2   2 
A. 5.
B. 9.
C. 10.
D. 11.
Lời giải
x a x a
 2a   a  x   2a   2 x 
Đặt x  2b , ( x  0)          1.
 a  x   2x  ax a x
+ Nếu x  a  VT  1 (thoả mãn).
x x x
 2a   2 x   4ax 
+ Nếu x  a  VT       2 
 1x  1 (không thoả mãn).
 a  x   a  x   (a  x ) 
a a a
 2a   2 x   4ax 
 Nếu x  a  VT       2 
 1a  1 (không thoả mãn).
 a  x   a  x   ( a  x ) 
b
Vậy x  a  a  2  [1; 2022]  b  log 2 2022  10,98  b {1, ,10} . Với mỗi số nguyên b
tìm được ta có tương ứng một số nguyên a  2b thoả mãn tức có 10 cặp.
Câu 51. (THPT Lương Thế Vinh – Hà Nội – 2022) Có bao nhiêu giá trị nguyên của m để bất phương
trình log 22 x  (2m  5) log 2 x  m 2  5m  4  0 có it nhất một nghiệm nguyên và không quá 1791
nghiệm nguyên?
A. 10.
B. 3.
C. 9.
D. 11.
Lời giải

Facebook Nguyễn Vương https://www.facebook.com/phong.baovuongTrang 31


Blog: Nguyễn Bảo Vương: https://www.nbv.edu.vn/
Biến đổi bất phương trình:
2 2
log 2 x  (2m  5) log 2 x  m  5m  4  0   log 2 x  (m  1)  log 2 x  (m  4)   0

 m  1  log 2 x  m  4  2 m1  x  2m  4  S x  2m 1 ; 2m  4 . 
m 4
 Nếu 2  1  m  4  S x  (0;1) không chứa số nguyên nào (loại).
+ Nếu m  3  S x   22 ; 2  thoả mãn.
+ Nếu m  2  S x   21 ; 4  thoả mãn.
 Nếu 2m 1  1  m  1  2m 1 , 2m  4   nên S x chứa các số nguyên là 2m 1  1,, 2 m 4  1
1 1
 ycbt  1   2m  4  1   2m 1  1  1  1791   2m  128  2,8  log 2    m  7 .
7 7
Vậy m  {3,  , 7} .
Câu 52. (THPT Trần Phú – Hà Tĩnh – 2022) Có bao nhiêu số tự nhiên x sao cho mỗi giá trị của x tồn
tại số thực y thoả mãn log 3 ( x  y )  log 6  x 2  2 y 2  ?
A. 1.
B. 3.
C. 2.
D. 6.
Lời giải
t
 x y 3
Đặt log3 ( x  y )  t   2 2 t
x  2 y  6
t
t x 2 ( y ) 2 ( x  y ) 2 2.9t
2 2 3 3
Suy ra 6  x  2y          t 1
1 1 1 3 2 2
1
2 2
 x 2  2 y 2  6  x 2  6  x  {0,1, 2} . Thử lại
 
 x  0  log 3 (  y )  log 6 2 y 2  y  0,1 thoả mãn (nhận).
 
 x  1  log 3 (1  y )  log 6 1  2 y 2  y  1 thoả mãn (nhận).
 
 x  2  log 3 (2  y )  log 6 4  2 y 2  y  1 thoả mãn (nhận).
Vậy x  {0,1, 2} là các số tự nhiên cần tìm.
13 2
Câu 53. (THPT Trần Phú – Hà Tĩnh – 2022) Cho hàm số f ( x)   x 3  x  12 x  e x  2022 . Bất
2
phương trình ẩn m sau đây f  log 0,5  log 2 (2m  1)   2021  f [ f (0)] có bao nhiêu nghiệm
nguyên?
A. 14.
B. 10.
C. 11.
D. 7.
Lời giải
2 x
Ta có f ( x)  3 x  13 x  12  e  0, x do đó:
f log 0,5  log 2 (2m  1)   2021  f [ f (0)]
 log 0,5  log 2 (2m  1)   2021  f (0)  2023
 log 0,5  log 2 (2m  1)   2  0  log 2 (2m  1)  (0,5)2  4
15
 1  2m  1  16  0  m   m  {1,, 7}.
2

Trang 32 Fanpage Nguyễn Bảo Vương  https://www.facebook.com/tracnghiemtoanthpt489/


Điện thoại: 0946798489 TUYỂN CHỌN VẬN DỤNG – VẬN DỤNG CAO 2022
Câu 54. (THPT Kim Liên - Hà Nội - 2022) Gọi S là tập nghiệm của phương trình
x
2
2 x
 3x  8x  3
 3  m  0 (với m là tham số thực). Có tất cả bao nhiêu giá trị nguyên của
m   2021; 2021 để tập hợp S có hai phần tử ?
A. 2095 . B. 2092 . C. 2093 . D. 2094 .
Lời giải
Chọn D
Xét phương trình: 2x  3x  8x  3  0 .
Đặt f  x   2 x  3x  8 x  3 .
f   x   2 x ln 2  3x ln 3  8  f   x   2 x ln 2 2  3x ln 2 3  0 , x   .
Do vậy phương trình f  x   0 có nhiều nhất hai nghiệm.
x 1
Ta thấy f 1  f  2   0  f  x   0   .
x  2
2x

Xét phương trình: 2x  3x  8x  3   3  m  0 1 .
2x
Nếu m  0   3  m  0, x    1 có tập nghiệm S  1; 2 .
x
 32  m  0  x  log2  log3 m
 
 x  1  x  1
Nếu m  0  1     .
 x  2  x  2
 x  log log m
 2 3   x  log2  log3 m
Khi đó S có hai phần từ khi và chỉ khi 1  log 2  log3 m  2  9  m  81.
Vậy m  2021;0   9;80 có 2094 số.
Câu 55. (THPT Kim Liên - Hà Nội - 2022) Cho bất phương trình
log 5  x  4 x  4  m   1  log 5  x  2 x  3  với m là tham số. Có bao nhiêu giá trị nguyên của
2 2

tham số m để bất phương trình nghiệm đúng với mọi x  1; 3  ?


A. 30 . B. 28 . C. 29 . D. Vô số.
Lời giải
Chọn C
2
 x  2 x  3  0
Điều kiện  2 x  1;3  x 2  4 x  4  m  0 , x  1;3
 x  4 x  4  m  0
2
 x 2  4 x  4   m , x  1;3  min  x  2    m  0  m  m  0 .
1;3

Ta có log 5  x 2  4 x  4  m   1  log 5  x 2  2 x  3  ,  x  1; 3 


 log 5  x 2  4 x  4  m   log 5  5 x 2  10 x  15  , x  1;3 
 x 2  4 x  4  m  5 x 2  10 x  15 , x  1;3  m  4 x 2  14 x  11, x  1;3
 m  4.12  14.1  11  29  m   0; 29 .
Vậy có 29 giá trị.
Câu 56. (THPT Kinh Môn - Hải Dương - 2022) Có bao nhiêu giá trị nguyên của tham số
m   2022; 2022 để bất phương trình  3m  112 x   2  m  6 x  3x  0 có nghiệm đúng với
x  0 ?
A. 2021 . B. 4044 . C. 2022 . D. 2020 .
Lời giải
Chọn A
Ta có  3m  112 x   2  m  6 x  3x  0   3m  1 4 x   2  m  2 x  1  0 .
Facebook Nguyễn Vương https://www.facebook.com/phong.baovuongTrang 33
Blog: Nguyễn Bảo Vương: https://www.nbv.edu.vn/
Đặt 2 x  t . Vì x  0  t  1 .
Bất phương trình trở thành:
2
t  2t  1
 3m  1 t 2   2  m  t  1  0  m  3t 2  t   t 2  2t  1  0  m   2 (Vì
3t  t
3t 2  t  0, t  1 ).
t 2  2t  1
Xét hàm số f  t    với t  1 .
3t 2  t
7t 2  6t  1
Ta có: f   t   2
.

3t 2  t 
2
7t  6t  1
Ta thấy: f   t   2
 0, t  1 nên f  t  đồng biến trên khoảng 1;   .
 3t 2
t
t 2  2t  1
Do đó m   , t  1 khi và chỉ khi m  f 1  2 .
3t 2  t
Vì m   2022; 2022 và m nên m  2022; 2021;...; 2 . Vậy có 2021 giá trị thoả mãn.
Câu 57. (THPT Kinh Môn - Hải Dương - 2022) Cho hai số thực dương x, y thỏa mãn phương trình
x  y 1
3  ln  9 xy  3 x  3 y. Tìm giá trị nhỏ nhất của biểu thức P  xy
3 xy
1 1
A. 1. B. . C. . D. 9 .
9 3
Lời giải
Chọn A
x  y 1
Ta có phương trình 3  ln  9 xy  3 x  3 y
3 xy
 ln  x  y  1  3  x  y  1  ln  3xy   9 xy 1
Xét hàm số f  t   ln t  3t , t   0;   .
1
f '  t    3  0  t   0;   .
t
Suy ra f  t  là hàm số tăng.
1  f  x  y  1  f  3xy   x  y  1  3xy .
Theo bất đẳng thức Cauchy ta có
 xy  1
3xy  x  y  1  2 xy  1  3xy  2 xy  1  0   .
 xy  1
 3
So sánh với điều kiện x, y dương ta được xy  1  xy  1
Vậy giá trị nhỏ nhất của biểu thức P là 1.
Câu 58. (THPT Lương Tài 2 - Bắc Ninh - 2022) Có tất cả bao nhiêu số nguyên dương y sao cho tồn tại
số thực x  1; 8 thỏa mãn:

 x  1  2e x  y 2   y  e x  x 2  ?
A. 11 . B. 14 . C. 12 . D. 13 .
Lời giải
Chọn D
Xét f  x    x  1  2e x  y 2   y  e x  x 2  trên 1;8 với y là tham số.
Ta có:

Trang 34 Fanpage Nguyễn Bảo Vương  https://www.facebook.com/tracnghiemtoanthpt489/


Điện thoại: 0946798489 TUYỂN CHỌN VẬN DỤNG – VẬN DỤNG CAO 2022
y
f   x   2 xe x  ye x  y 2  2 yx   e x  y   2 x  y   0  x 
2
Ta thấy f 1   y  e  1  0 do y nguyên dương ;
f  8   7  2e8  y 2   y  e8  64   7 y 2   e8  64  y  14e8
y
TH1: Khi  1  y  2  f   x   0, x  1;8 . Lập bảng biến thiên cho f  x  , từ yêu cầu bài
2
toán  f  8  0  y  13,85  y 1; 2
y
TH2: Khi  8  y  16  f   x   0 , x  1; 8  f  8  f 1  0 suy ra phương trình vô
2
nghiệm trên 1;8
y y
TH3: Khi 1   8  2  y  16  xCT  . Lập bảng biến thiên cho f  x  , từ yêu cầu bài toán
2 2
 f  8  0  y  13,85  y  3; 4; 5;...13
Như vậy có tất cả 13 giá trị y thỏa mãn.
Câu 59. (THPT Lương Tài 2 - Bắc Ninh - 2022) Tập nghiệm của bất phương trình
 
4 x  65.2 x  64  2  log 3 ( x  3)  0 có tất cả bao nhiêu số nguyên?
A. 2. B. 3. C. 4. D. Vô số.
Lời giải
Chọn C
Điều kiện xác định: x  3  0  x   3 .
 
Ta có: 4 x  65.2 x  64  2  log3 ( x  3)  0
 2 x  1  x  0
  x 
x x
 4  65.2  64  0    2  64  x  6
   x  6
 2  log 3 ( x  3)  0  log 3 ( x  3)  2 
 x x  x
x  0
4  65.2  64  0 1  2  64  0  x  6 x  0

       x  0   .
 2  log 3 ( x  3)  0  x6
 log 3 ( x  3)  2  x  6  x  6 
 x  x
 4  65.2 x  64  0   x  0
 2 1 
  x  x  6
 2  log 3 ( x  3)  0  2  64

 log 3 ( x  3)  2 x  6
Kết hợp điều kiện ta có tập nghiệm của bất phương trình là S   3;0  6 . Do đó có tất cả 4 số
nguyên thoả mãn.
Câu 60. (THPT Võ Nguyên Giáp - Quảng Bình - 2022) Cho ba số thực x, y, z không âm thoả mãn
2 x  4 y  8 z  4 . Gọi M , N lần lượt là giá trị lớn nhất, giá trị nhỏ nhất của biểu thức
x y z
S    . Đặt T  2M  6 N . Khẳng định nào dưới đây đúng?
6 3 2
A. T  1; 2  . B. T   2;3 . C. T   3; 4  . D. T   4;5  .
Lời giải
Chọn A
Đặt a  2x , b  4 y , c  8z . Ta có: a  b  c  4 .
Vì x, y, z không âm và a  b  c  4 nên a , b, c thuộc đoạn 1; 2 .
x y z x  2 y  3 z log 2 ( abc )
S    
6 3 2 6 6

Facebook Nguyễn Vương https://www.facebook.com/phong.baovuongTrang 35


Blog: Nguyễn Bảo Vương: https://www.nbv.edu.vn/
64 4
3 log 2 log 2
 a  b  c  64 27 3.
Ta có: abc     S 
 3  27 6 2
 a  1 b  1  0  ab  a  b  1  abc   3  c  c .
Xét hàm số f (c)  c 2  3c, c  1; 2 .
Bảng biến thiên:

log 2 (abc) log 2 2 1


Vậy abc  f (c)  2 . Khi đó S    .
6 6 6
4 8
 1  log 2  1, 41  1; 2  .
Do đó: T  2 M  6 N  log 2
3 3
Câu 61. (THPT Võ Nguyên Giáp - Quảng Bình - 2022) Có bao nhiêu cặp số nguyên  x, y  thỏa mãn
y 2 8
log 2  x 2  2 x  3  7  y2  3y ?
A. 0 . B. 1 . C. 2 . D. 7 .
Lời giải
Chọn B
y 2 8

Ta có: log 2 x 2  2 x  3   7  y 2  3 y   y 2  8 log 2  x 2  2 x  3  7  y 2  3 y
7  y2  3 y
 log 2  x 2  2 x  3  . *
y2  8
2x  2
Xét hàm số f  x   log 2  x 2  2 x  3  f   x   .
 x  2 x  3 ln 2
2

 f  x  nghịch biến trên  ; 1 và đồng biến trên  1;    min f  x   f  1  1 .
x
2 2
7  y  3y 3 y  30 y  24
Xét hàm số g  y    g y  
2
y 8  y 2  8
 y  5  33  0, 74
g  y   0  
 y  5  33
Bảng biến thiên của g  y  trên  :

7
 
Ta có g 5  33  1, 01; g  0   ; g 1  1  Max g  y   y 1  1
8 y

Trang 36 Fanpage Nguyễn Bảo Vương  https://www.facebook.com/tracnghiemtoanthpt489/


Điện thoại: 0946798489 TUYỂN CHỌN VẬN DỤNG – VẬN DỤNG CAO 2022
 x  1
Do đó với x, y   thì *   .
y 1
Vậy chỉ có 1 cặp số nguyên  x, y  thỏa mãn yêu cầu bài toán.
Câu 62. (THPT Yên Lạc - Vĩnh Phúc - 2022) Có bao nhiêu giá trị nguyên của tham số m để phương
trình 2 log3  2 x  1  log 3  mx 2  1 có nghiệm.
A. 1 . B. 3 . C. 7 . D. 9 .

Lời giải
Chọn C
1
Điều kiện xác định x 
2
2
 
Với điều kiện trên, 2 log 3  2 x  1  log3 mx 2  1  log3  2 x  1  log 3 mx 2  1  
2 2
 4 x  4 x  1  mx  1
4
 4   m (*)
x
4 1
Xét hàm số f  x   4  với x  .
x 2
4 1
Ta có f   x   2  0, x 
x 2
4
Bảng biến thiên của hàm số f  x   4 
x

Dựa vào bảng biến thiên, phương trình đã cho có nghiệm  4  m  4 .


Vậy có 7 giá trị nguyên của tham số m .
 x 1

Câu 63. (THPT Yên Lạc - Vĩnh Phúc - 2022) Cho f ( x)  2023.ln  e 2023  e 2  . Tính giá trị biểu thức
 
H  f  1  f   2   ...  f   2022 
A. 1011. B. 2022. C. e 2022 . D. e1011 .
Lời giải
Chọn A
x
x 1
 2023  e 2023
Ta có f ( x)  2023.ln  e  e2   f  x  x 1
.
  e 2023
e 2

1 2 2022
e 2023 e 2023 e 2023
Khi đó H  f  1  f   2   ...  f   2022   1 1
 2 1
 ...  2022 1
2023 2 2023 2
e e e e 2023  e 2
e
ex e1 x ex e ex e ex  e
Để ý x  1 x  x     1
e  e e  e e  e e  ex e ex  e e  ex ex  e
Từ đó

Facebook Nguyễn Vương https://www.facebook.com/phong.baovuongTrang 37


Blog: Nguyễn Bảo Vương: https://www.nbv.edu.vn/
1 2022 2 2020
 2023 2023
  2023 2023

 e e   e e 
H  2022  ...   ...  2020
 1 1 1   2 1 1 
 e 2023  e 2 e 2023  e 2   e 2023  e 2 e 2023  e 2 
  
2022
H  1  1  ...  1   1011 .
2
Câu 64. (THPT Yên Lạc - Vĩnh Phúc - 2022) Có bao nhiêu giá trị nguyên của tham số m để phương
1 x 2 1 x 2
trình 91   m  3 31  2m  1  0 có nghiệm thực
A. 7 . B. 5 . C. 6 . D. 4 .
Lời giải
Chọn A
Điều kiện: 1  x  1 .
1 x 2
Đặt: t  31 , t  3;9 .
t 2  3t  1
Phương trình trở thành: t 2   m  3 t  2m  1  0  m(t  2)  t 2  3t  1  m  (1)
t2
t 2  3t  1
Xét hàm số f (t )  , t  3;9 .
t2
t 2  4t  5
Ta có f '(t )  2
 0, t  3;9  Hàm số luôn đồng biến trên đoạn 3;9
t  2
55
Vậy f (3)  f (t )  f (9) hay 1  f (t )  .
7
55
Phương trình 1 có nghiệm t  3;9  1  m  .
7
Vì m    m  1; 2;3; 4;5; 6;7 .
Câu 65. (THPT Yên Phong 1 - Bắc Ninh - 2022) Cho hàm số y  f  x  . Hàm số y  f   x  có bảng
biến thiên như sau

 
Bất phương trình 2 f  x   ecos x  m đúng x   0;  khi và chỉ khi
 2
   
A. m  2 f  0   e . B. m  2 f    1 . C. m  2 f    1 . D. m  2 f  0   e .
2 2

Lời giải
Chọn B
 Có 2 f  x   ecos x  m  2 f  x   ecos x  m .
 
 Xét hàm g  x   2 f  x   ecos x , có g   x   2 f   x   sin x.ecos x  0, x   0;  .
 2
 
Suy ra g  x   2 f  x   ecos x đồng biến trên  0;  .
 2
Vậy bất phương trình đã cho nghiệm đúng với mọi

       cos  
x   0;  .  m  max g  x   g    2 f    e 2  2 f    1 .
 2   
0; 2  2 2 2
 

Trang 38 Fanpage Nguyễn Bảo Vương  https://www.facebook.com/tracnghiemtoanthpt489/


Điện thoại: 0946798489 TUYỂN CHỌN VẬN DỤNG – VẬN DỤNG CAO 2022

Theo dõi Fanpage: Nguyễn Bảo Vương  https://www.facebook.com/tracnghiemtoanthpt489/

Hoặc Facebook: Nguyễn Vương  https://www.facebook.com/phong.baovuong

Tham gia ngay: Nhóm Nguyễn Bào Vương (TÀI LIỆU TOÁN)  https://www.facebook.com/groups/703546230477890/

Ấn sub kênh Youtube: Nguyễn Vương


 https://www.youtube.com/channel/UCQ4u2J5gIEI1iRUbT3nwJfA?view_as=subscriber

Tải nhiều tài liệu hơn tại: https://www.nbv.edu.vn/

Facebook Nguyễn Vương https://www.facebook.com/phong.baovuongTrang 39


TUYỂN CHỌN VẬN DỤNG – VẬN DỤNG CAO 2022 Điện thoại: 0946798489

VẤN ĐỀ 3. TÍCH PHÂN


• |FanPage: Nguyễn Bảo Vương
• TUYỂN CHỌN CÂU HỎI VD-VDC TỪ CÁC ĐỀ THI THỬ CÁC TRƯỜNG, CÁC SỞ NĂM 2022
Câu 1. (Chuyên Vinh – 2022) Cho hàm số y  f ( x ) có đạo hàm trên đoạn [1; 2] thỏa mãn
2 2 2
f (1)  2, f (2)  1 và   xf ( x)
1
dx  2 . Tich phân 
1
x 2 f ( x)dx bằng
A. 4..
B. 2.
C. 1.
D. 3.
Lời giải
Chọn D
2
2 4 4 2 2
Ta có:  2
dx    2;  f ( x)dx  f ( x) 1  1
1 x x1 1

2

2 2 4 2 2
Vì1   xf ( x )   4 f ( x )  2 dx  0 nên   xf ( x)   dx  0
x  1
 x
2 2
 f ( x)   2  f ( x)   C
x x
2
Mà f (1)  2  C  0  f ( x) 
x
2
2
2
2 2 2
Khi đó  x f ( x) dx   x  2 dx  2 x 1  3 .
1 1 x
Câu 2. (Chuyên Vinh – 2022) Cho hàm số y  x 4  bx3  cx 2  dx  e(b, c, d , e  ) có các giá trị cực trị là
f ( x)
1,4 và 9. Diện tích hình phẳng giới hạn bởi đồ thị hàm số g ( x)  với trục hoành bằng
f ( x)
A. 4.
B. 6.
C. 2.
D. 8.
Lời giải
Chon B
Gọi m, n, p ( m  n  p ) lần lượt là các điểm cực trị của hàm số y  f ( x ) .
Ta có bảng xét dấu của f ( x ) như sau:
Khi đó hàm số đạt cực tiểu m, p và đạt cực đại tại x  n  f ( n)  9 .
f ( x)
Diện tích hình phẳng giới hạn bởi đồ thị hàm số g ( x)  với trục hoành là
f ( x)
p n p n p
S   |g ( x) | dx   g ( x)dx   g ( x)dx   g ( x)dx   g ( x)dx
m m n m n

n
 2[ f ( x)]  2[ f ( x)]np
m

 2[ f (n)  f (m)]  2[ f ( p )  f (n)]  4 f (n)  2( f (m)  f ( p))  4.3  2  (1  2)  6.


Câu 3. (Chuyên Lê Quý Đôn - Điện Biên - 2022) Cho hàm số
y  f  x   ax  bx  cx  d ,  a, b, c, d  , a  0  có đồ thị  C  . Biết rằng đồ thị  C  tiếp xúc
3 2

với đường thẳng y  4 tại điểm có hoành độ âm và đồ thị của hàm số y  f   x  cho bởi hình vẽ
dưới đây. Tính thể tích vật thể tròn xoay được tạo thành khi quay hình phẳng H giới hạn bởi đồ
thị  C  và trục hoành khi quay xung quanh trục Ox .

Facebook Nguyễn Vương https://www.facebook.com/phong.baovuong Trang 1


Blog: Nguyễn Bảo Vương: https://www.nbv.edu.vn/

725 729 1
A. π. B. π. C. 6 π . D. π.
35 35 35
Lời giải
Chọn B
Phương trình f   x   0 có hai nghiệm phân biệt là: 1;1
 f   x   k  x  1 x  1
Mà f   0   3  k.1.  1  3  k  3 . Do đó: f   x   3  x  1 x  1  f   x   3x 2  3 .
 f  x    f   x  dx    3x 2  3 dx  x3  3x  C .
Đồ thị  C  của hàm số y  f  x  tiếp xúc với đường thẳng y  4 tại điểm có hoành độ âm, nghĩa
là đường thẳng  d  : y  4 là tiếp tuyến của đồ thị  C  tại điểm có hoành độ x0 âm.
 x0  1
 f   x0   kd  3 x02  3  0   . Vì x0 âm, nên x0  1 .
 x0  1
3
Ta có: f  x0   4   1  3  1  C  4  C  2 .
x  1
Do đó: f  x   x3  3 x  2 . Xét: f  x   0  x3  3x  2  0   .
 x  2
 y  f  x  ; y  0
Hình phẳng  H  được giới hạn bởi:  .
 x  1; x  2
Thể tích vật thể tròn xoay được tạo thành khi quay hình phẳng H quanh trục Ox là:
1
729
V  π  f 2  x  dx  π.
2
35
1
Câu 4. (Chuyên Lê Quý Đôn - Điện Biên - 2022) Xét hàm só́ f  x   e x   x f  x  dx . Giá trị của
0

f  ln 2022  bằng bao nhiêu?

A. 2022 . B. 2021 . C. 2023 . D. 2024 .


Lời giải
Chọn D
Từ yêu cầu đề bài và đáp án, ta có thể đặt: f  x   e x  C
1 1
Khi đó: f  x   e x   x f  x  dx  e x  C  e x   x  e x  C  dx
0 0
1 1
 e x  C  e x   xe x dx   Cxdx 1 .
0 0

1
x
u  x du  dx
Với : 0 xe d x , đặt:  x
  x
.
dv  e dx v  e
1 1 1 1 1 1
Khi đó: 
0
    e dx   xe 
xe x dx  xe x
0 0
x x
0
 e x   x  1 e x
0 0

Trang 2 Fanpage Nguyễn Bảo Vương  https://www.facebook.com/tracnghiemtoanthpt489/


Điện thoại: 0946798489 TUYỂN CHỌN VẬN DỤNG – VẬN DỤNG CAO 2022
1
1 C 2
1  e x  C  e x   x  1 e x 0  x
2 0
1
 C  C
 C   e x  x  1  x 2   C   1  C  2
 2 0 2
 f  x   e x  2  f  ln 2022   2024 .
1
Câu 5. (Chuyên Lê Quý Đôn - Điện Biên - 2022) Cho hàm số f  x  thoả mãn f  2    và
25
2
f   x   4 x3  f  x   vói mọi x   . Giá trị của f 1  f  0  bằng
1 1 1 1
A. . B.  . C.  .D.
90 90 72 72
Lời giải
Chọn A
2 f  x
Ta có: f   x   4 x3  f  x    2
 4 x3
 f  x  
 f ' x 
   dx  4x 3dx   1  x 4  C
2
  f  x    f  x
  
1 1
  x4  C  f  x    4
f  x x C
1 1 1
Với f  2     f  2    C 9
25 16  C 25
1 1 1 1
 f  x   4  f 1  f  0      .
x 9 10 9 90
Câu 6. (Cụm Trường Nghệ An - 2022) Cho hai hàm số f  x  và g  x  liên tục trên  và hàm số
f   x   ax3  bx 2  cx  d , g   x   qx 2  nx  p với a , q  0 có đồ thị như hình vẽ. Biết diện tích
hình phẳng giới hạn bởi hai đồ thị hàm số y  f   x  và y  g   x  bằng 10 và f  2   g  2  . Biết
a
diện tích hình phẳng giới hạn bởi hai đồ thị hàm số y  f  x  và y  g  x  bằng (với a , b  
b
và a , b nguyên tố cùng nhau). Tính a  b .

A. 18. B. 19. C. 20. D. 13.


Lời giải
Chọn D
Phương trình f   x   g   x  có ba nghiệm bội lẻ phân biệt là: 0;1; 2 .

Facebook Nguyễn Vương https://www.facebook.com/phong.baovuongTrang 3


Blog: Nguyễn Bảo Vương: https://www.nbv.edu.vn/
 f   x   g   x   kx  x  1 x  2 
 y  f  x
2

Với  H1  giới hạn bởi:  y  g   x  , ta có: S H1    f   x   g   x  dx
 x  0; x  2 0

2
 k  x  x  1 x  2  dx  10  k  20
0

Do đó: f   x   g   x   20 x  x  1 x  2 
 f   x   g   x   20 x  x 2  3x  2 
 f   x   g   x   20 x3  60 x 2  40 x 1 .

  f   x   g   x   dx    20 x  60 x 2  40 x  dx
3
Lấy tích phân hai vế của 1 , ta được:
 f  x   g  x   5 x 4  20 x3  20 x 2  C  2 
Thay x  2 vào  2  , ta được: f  2   g  2   C  C  0 .
Do đó: f  x   g  x   5 x 4  20 x3  20 x 2 .
x  0
Xét: f  x   g  x   0  5 x 4  20 x 3  20 x 2  0   .
x  2
 y  f  x  g  x 2
 16
Với  H 2  giới hạn bởi:  y  0 , ta có: S H 2    f  x   g  x  dx  .
 x  0; x  2 0
3

Do đó: a  16, b  3 . Vậy a  b  13 .
Câu 7. (Cụm Trường Nghệ An - 2022) Cho hàm số y  f  x  có đạo hàm liên tục trên  và thoả mãn
x2  4 x 1
f   x   2 f  x    x  1 e
2 2
, x   và f 1  e2 . Biết f  3  a.eb  c với a , b , c   .
Tính 2a  3b  4c.
A. 36 . B. 30 . C. 24 . D. 32 .
Lời giải
Chọn A
x 2  4 x 1
Ta có: f   x   2 f  x    x2  1 e 2
, x   .
x2 1
Nhân 2 vế cho e 2 x
ta được: e 2 x
. f   x   2.e . f  x    x  1 e
2 x 2 2
.
x 2 1 x 2 1

 e2 x . f   x    e2 x  . f  x    x 2  1 e 2
 
  e2 x  . f  x    x 2  1 e 2

Lấy nguyên hàm 2 vế ta được:


x 2 1
 e 2 x
. f  x    x 2
 1 e 2
dx  C
x 2 1
 e 2 x
. f  x    x 2
 1 e 2
dx  C
x 2 1
Đặt H   x  1 e  2
 2
dx

Trang 4 Fanpage Nguyễn Bảo Vương  https://www.facebook.com/tracnghiemtoanthpt489/


Điện thoại: 0946798489 TUYỂN CHỌN VẬN DỤNG – VẬN DỤNG CAO 2022
x2 1 x2 1 x 2 1
H    x2  1 e 2
dx   x e 2 2
dx   e 2
dx *
x 2 1
2
Ta tìm  e dx bằng phương pháp nguyên hàm từng phần.
2 2
 x 1  x 1
u  e 2 du  x.e 2 dx
Đặt  ta có: 
dv  dx  v  x
x 2 1 x2 1 x 2 1
2 2 2 2
Suy ra:  e dx  xe x e dx
x2 1 x2 1 x2 1 x 2 1 x 2 1
Thế vào * ta được: H   x  1 e  2
 2 2
dx   x e 2
dx  xe 2 2
x e 2
dx  xe 2
C
x 2 1
 e 2 x
 . f  x   xe 2
C
Mà f 1  e2

 
Ta cho x  1  e2 . f 1  1.e0  C  e2 .e2  1.e0  C  C  0  
x2 1
 e 2 x
 . f  x  xe 2

 
Để tính f  3 ta chọn x  3  e6 . f  3  3e4  f  3  3e10  a.eb  c

a  3

Suy ra b  10  2a  3b  4c  36 .
c  0

Câu 8. (Đại học Hồng Đức – 2022) Cho hàm số f ( x)  x3  ax 2  bx  c( a, b, c  ) có hai điểm cực trị là
1 và 1. Gọi y  g ( x ) là hàm số bậc hai có đồ thị cắt trục hoành tại hai điểm có hoành độ trùng
với các điểm cực trị của f ( x ) , đồng thời có đỉnh nằm trên đồ thị của f ( x ) với tung độ bằng 2.
Diện tích hình phẳng giới hạn bởi hai đường y  f ( x) và y  g ( x ) gần với giá trị nào nhất dưới
đây?
A. 10.
B. 12.
C. 13.
D. 11.
Lời giải.
Gọi I là toạ độ đỉnh của đồ thị hàm số g ( x ) , dễ thấy I (0; 2) và g ( x )  2( x  1)( x  1) hay
g ( x )  2 x 2  2

Ta có: f ( x)  3x 2  2ax  b .
3  2a  b  0 a  0
Theo bài ra, ta có:    f ( x)  x3  3 x  c .
3  2 a  b  0 b   3
Vi I thuộc đồ thị của f ( x ) , nên c  2  f ( x)  x3  3 x  2 .
 x  3
3 
Xét f ( x)  g ( x)  x  2 x  3x  0   x  02

 x  1
Diện tích hình phẳng cần tìm là
0 1 7
S   x 3  2 x 2  3 x dx   x 3  2 x 2  3 x dx   11,8
3 0 6
Facebook Nguyễn Vương https://www.facebook.com/phong.baovuongTrang 5
Blog: Nguyễn Bảo Vương: https://www.nbv.edu.vn/
Câu 9. (THPT Hồ Nghinh – Quảng Nam – 2022) Cho hàm số f ( x ) nhận giá trị dương và có đạo hàm
2
liên tục trên đoạn [0;1] sao cho f (1)  1 và f ( x)  f (1  x)  e x x
, x  [0;1]. Tính

I 
1  2x 3

 3x 2 f ( x)
dx.
0 f ( x)
1
A. I   .
10
2
B. I  .
5
1
C. I   .
60
1
D. I  .
10
Lời giải
Chọn A
2
Ta có f ( x)  f (1  x)  e x  x  ln f ( x)  ln f (1  x)  x 2  x
2
  x 2  x  ln f ( x)   x 2  x  ln f (1  x)   x 2  x 
1 1 1 2
   x 2  x  ln f ( x) dx    x 2  x  ln f (1  x) dx    x 2  x  dx
0 0 0
1 1 1 2
   x 2  x  ln f ( x) dx   x( x  1) ln f (1  x) dx    x 2  x  dx
0 0 0

 dx  dt

Đặt t  1  x  x  1  t   x  0  t  1
x  1  t  0

1 1 1
 I   x( x  1) ln f (1  x)dx   (1  t )t ln f (t )dt   x( x  1) ln f ( x)dx    x 2  x  ln f ( x)dx
0 1 0 0
1 1 2 1 1 1
 
0

 2  x 2  x ln f ( x ) dx     0
x 2  x dx 
30
  x 2  x ln f ( x ) dx 
0 60

 2 x  3x  f ( x)dx  2 x  3x d ln f ( x)
1
3 2
1
  3 2
I 
0 f ( x) 0

1 1 1 1
  2 x 3  3 x 2  ln f ( x)  6   x 2  x  ln f ( x)dx  6   x 2  x  ln f ( x)dx 
0 0 10 0

Câu 10. (THPT Hương Sơn - Hà Tĩnh - 2022) Cho hàm số y  f  x  liên tục, nhận giá trị dương trên
x2
 0;   và thỏa mãn f 1  2 ; f   x   2
với mọi x   0;   . Giá trị f  3 bằng
 f  x  
3 3
A. 34 . B. 34 . C. 3 . D. 20 .
Lời giải
Chọn A
x2 2
Ta có f   x   2
 f   x  .  f  x    x 2 với mọi x   0;   .
 f  x  
Lấy nguyên hàm hai vế ta được:
2 f 3  x  x3 2
 f   x  .  f  x  dx   x dx  3  3  C .
8 1 8 1 7
Theo đề bài f 1  2 nên ta có:   C  C    .
3 3 3 3 3

Trang 6 Fanpage Nguyễn Bảo Vương  https://www.facebook.com/tracnghiemtoanthpt489/


Điện thoại: 0946798489 TUYỂN CHỌN VẬN DỤNG – VẬN DỤNG CAO 2022
3 3
Khi đó f  x   x  7  f  3  34 . 3

Câu 11. (THPT Hương Sơn - Hà Tĩnh - 2022) Một vật chuyển động trong 4 giờ với vận tốc v  km / h 
phụ thuộc vào thời gian t (h ) có đồ thị của vận tốc như hình bên. Trong khoảng thời gian 2 giờ kể
từ khi bắt đầu chuyển động, đó là một đường parabol có đỉnh I  2;7  và trục đối xứng của
parabol song song với trục tung, khoảng thời gian còn lại, đồ thị là đoạn thẳng IA . Tính quãng
đường s mà vật di chuyển được trong 4 giờ đó (kết quả làm tròn đến hàng phần trăm).

A. s  15,81  km  . B. s  17,33  km  . C. s  23,33  km  . D. s  21, 33  km  .


Lời giải
Chọn D
Trong 2 giờ đầu, đồ thị hàm vận tốc là một parabol có đỉnh I  2;7  như hình vẽ nên hàm vận tốc
v  t   t 2  4t  3
Trong 2 giờ sau đồ thị hàm vận tốc là đoạn thẳng IA như hình vẽ nên hàm vận tốc v  t   11  2t
Vậy quãng đường s vật chuyển động sau 4 giờ là
4 2 4
64
s   v  t  dt    t 2  4t  3 dt   11  2t  dt  .
0 0 2
3
Câu 12. (THPT Hương Sơn - Hà Tĩnh - 2022) Một biển quảng cáo có dạng hình tròn tâm O , phía trong
được trang trí bởi hình chữ nhật ABCD ; hình vuông MNPQ có cạnh MN  2m và hai đường
parabol đối xứng nhau chung đỉnh O như hình vẽ. Biết chi phí để sơn phần tô đậm là 300.000
đồng/m2 và phần còn lại là 250.000 đồng/m2. Hỏi số tiền để sơn theo cách trên gần nhất với số
tiền nào dưới đây?

A. 3.439.000 đồng. B. 3.628.000 đồng. C. 3.580.000 đồng. D. 3.363.000 đồng.


Lời giải
Chọn A

Facebook Nguyễn Vương https://www.facebook.com/phong.baovuongTrang 7


Blog: Nguyễn Bảo Vương: https://www.nbv.edu.vn/

2
 y  x
Toạ độ các điểm A, B , C , D là nghiệm của hệ phương trình  2 2
.
 x  y  4
 2  2 17 1  17 
Giải hệ ta được: A  ;   (m; m 2 ) .
 2 2 
 
m2  2 m2
Từ đó: phương trình đường thẳng AE : y  x  2 ; AF : y  ( x  2) .
m m2
Do đó:
1 1
2  x3  2
+) S1   (1  x ) dx   x    .
0  3 0 3
m m
2 x3 1 1
+) S2   x dx   m3  .
1
3 1 3 3
m m m
 m 2  2   m2  2 m2  2  x 2  1
+) S3    x  2   m 2  dx   ( x  m) dx    mx    m3  m .
0 
m   m 0 m  2 0 2
2
2
 m2   1 x  m2  x 2 
+) S4    4  x 2  ( x  2)  dx   x 4  x 2  2arc sin     2 x 
m
m2   2 2 m2 2  m
1 1 m
 m3  m 4  m 2  m 2  2arc sin   .
2 2 2
1 3 1 1 m
Suy ra: S1  S 2  S3  S 4  m   m 4  m 2  m 2  m  2arc sin   .
3 3 2 2
Vậy số tiền để sơn biển quảng cáo là:
250000.22  50000.4(S1  S2  S3  S4 )  3 439 191 đồng.
 2 x  3, khi x  2
Câu 13. (THPT Lê Thánh Tông - HCM-2022) Cho hàm số f  x    3 . Giả sử F  x  là
 4 x  1, khi x  2
nguyên hàm của f  x  trên  và thỏa mãn F  0   3 . Giá trị F  3   5 F  5  bằng
A. 12 . B. 16 . C. 13 . D. 7 .
Lời giải
Chọn A
3 0

Ta có: F  3  F  0   5F  0   5F  5    f  x  dx  5.  f  x  dx
0 5
2 3 0
   2 x  3 dx    4 x3  1  dx  5.   2 x  3 dx  24
0 2 5

Suy ra F  3   5 F  5   4 F  0   24  F  3   5 F  5   12 .

Trang 8 Fanpage Nguyễn Bảo Vương  https://www.facebook.com/tracnghiemtoanthpt489/


Điện thoại: 0946798489 TUYỂN CHỌN VẬN DỤNG – VẬN DỤNG CAO 2022
Câu 14. (THPT Lê Thánh Tông - HCM-2022) Cho hai hàm đa thức f  x   ax 3  bx 2  cx  d và
g  x   mx 2  nx  p . Biết rằng đồ thị hai

hàm số y  f  x  và y  g  x  cắt nhau tại ba điểm có hoành độ lần lượt là 1; 2; 4 đồng thời cắt
trục tung lần lượt tại M , N sao cho MN  6 ( tham khảo hình vẽ).

Hình phẳng giới hạn bởi đồ thị hai hàm số đã cho ( phần gạch sọc) có diện tích bằng
125 253 253 253
A. . B. . C. . D. .
8 24 16 12
Lời giải
Chọn C
Phương trình hoành độ giao điểm của đồ thị y  f  x  và y  g  x  là:

ax 3  bx 2  cx  d  mx 2  nx  p  ax 3   b  m  x 2   c  n  x  d  p  0

Do đồ thị hai hàm số y  f  x  và y  g  x  cắt nhau tại ba điểm có hoành độ lần lượt là 1;2;4
nên ta được a  x  1 x  2  x  4   ax 3   b  m  x 2   c  n  x  d  p .

3
Mà f  0   g  0   yM  yn  MN  6 . Suy ra a  .
4
3
Do đó: f  x   g  x    x  1 x  2 x  4 .
4
4 4
3 253
Khi đó: S   f  x   g  x  dx    x  1 x  2  x  4  dx  .
1 1
4 16

Câu 15. (Liên trường Hà Tĩnh – 2022) Cho hàm số f ( x) thỏa mãn
3x
e  4 f ( x)  f ( x)   2 f ( x)
, x  0 và f (0)  1 . Tính I  
ln 2
f ( x)dx
 0
 f ( x)  0
11
A. I  .
24
1
B. I   .
12
209
C. I  .
640
201
D. I  .
640
Facebook Nguyễn Vương https://www.facebook.com/phong.baovuongTrang 9
Blog: Nguyễn Bảo Vương: https://www.nbv.edu.vn/
Lời giải
f ( x ) 1 1
Ta có: e3 x  4 f ( x)  f ( x)   2 f ( x)  2e 2 x f ( x)  e 2 x 
2 f ( x) e
 
 z  e 2 x  f ( x)   z .
e
1 1
Do đó e2 x  f ( x) là một nguyên hàm của x , tức e 2 x  f ( x)   x  C .
e e
2
 2 1 
Thay x  0 vào ta được C  2 . Tìm được f ( x)   2 x  3 x  .
e e 
2
ln 2  2
ln 2 1  ln 2  4 4 1  209
I   f ( x)dx    2 x  8 x  dx    4 x  3 x  6 x dx  .
0 0
e e  0
e e e  640
Câu 16. (Liên trường Hà Tĩnh – 2022) Cho hàm số f ( x) xác định trên  \{1;2} thỏa mãn
1 1
f ( x)  2 ; f (3)  f (3)  0 và f (0)  . Giá trị của biểu thức f (4)  f (1)  f (4) bằng
x x2 3
1 1
A.  ln 2 .
3 3
B. 1  ln S 0 .
1
C.  ln 2 .
3
1 8
D. 1  ln .
3 5
Lời giải
dx 1  1 1  1 x2
f ( x)   2    dx  ln C
x  x  2 3   x  2 x 1 3 x 1
1 x  2
 3 ln x  1  C1 khi x  2

1 x2 1  2  x 
 f  x   ln  C   ln    C2 khi  1  x  2
3 x 1 3  x 1 
1 x  2
 ln  C3 khi x  1
3 x 1
1 5 1 8
Khi đó: f (3)  f (4)  ln ; f (4)  f (3)  ln
3 4 3 5
1 1
f (3)  f (4)  f (4)  f (3)  ln 2  f (4)  f (4)   ln 2
3 3
1 1 1 1 1
Mặt khác f (1)  f (0)  ln  f (1)   ln
3 4 3 3 4
1
Do đó f (4)  f (1)  f (4)   ln 2 .
3
Câu 17. (THPT Nguyễn Tất Thành-Đh-SP-HN-2022) Cho hàm số f  x  có đạo hàm f   x  liên tục
2
trên  và thỏa mãn các điều kiện f  x   0 x   f  0   1 và f   x   4 x 3  f  x   x   .
1
Tính I   x 3 . f  x  dx .
0

ln 2 1 1
A. I  . B. I  ln 2 . C. I  . D. I  .
4 4 6
Lời giải
Chọn A
Cách 1.

Trang 10 Fanpage Nguyễn Bảo Vương  https://www.facebook.com/tracnghiemtoanthpt489/


Điện thoại: 0946798489 TUYỂN CHỌN VẬN DỤNG – VẬN DỤNG CAO 2022
2 f  x 1
Ta có: f   x   4 x 3  f  x    2  4 x 3    4 x 3 dx  x 4  C
f  x f  x
1 x3
Do f  0   1 nên C  1 . Từ đó: f  x    x 3
f  x  
x4  1 x4  1
1 d  x  1 1
1 1 1 4
x3 1 1
Vậy I   x3 f  x  dx   4 dx   4  ln  x 4  1  ln 2 .
0 0
x 1 4 0 x 1 4 0 4
Câu 18. (THPT Nguyễn Tất Thành-Đh-SP-HN-2022) Cho D là hình phẳng giới hạn bởi đồ thị hàm số
y  ln x , trục hoành và hai đường thẳng x  1; x  m, với m  1. Khi hình phẳng D có diện tích
bằng 1, giá trị của m thuộc khoảng nào dưới đây ?
y

y  ln x

-1 O 1 m x

7   7 5   5
A.  ; 4  . B.  3;  . C.  ;3 . D.  2;  .
2   2 2   2

Lời giải
Chọn C
m m
 Có S   ln x dx   ln xdx   x.ln x  x  |1m  m.ln m  m  1  1
1 1

5 
 m  ln m  1  0  ln m  1  m  e  2, 72   ;3 .
2 
 2 sin 2 x  1 khi x  0
Câu 19. (THPT Nguyễn Tất Thành-Đh-SP-HN-2022) Cho hàm số f  x    x . Giả
 2 khi x  0
2
sử F  x  là một nguyên hàm của hàm số f  x  trên  và thỏa mãn điều kiện F 1  . Tính
ln 2
F    .
1 1
A. F      2  . B. F       .
ln 2 ln 2
1
C. F     2 . D. F      2  .
ln 2
Lời giải
Chọn A
1 0 1 0 1
 1  2x 1
Ta có  f  x  dx    2sin x  1 dx   2 dx    sin 2 x  2 x  
2 x
 2 
  0  2   ln 2 0 ln 2
1 0 1
Mặt khác  f  x  dx   f  x  dx   f  x  dx  f 1  f   
  0

1 1
Khi đó: f 1  f     2   f     2 
ln 2 ln 2

Facebook Nguyễn Vương https://www.facebook.com/phong.baovuongTrang 11


Blog: Nguyễn Bảo Vương: https://www.nbv.edu.vn/
Câu 20. (THPT Nho Quan A – Ninh Bình – 2022) Cho hàm số y  f ( x)  ax 4  bx 2  c có đồ thị (C ) ,
Biết f ( 1)  0 . Tiếp tuyến d tại điểm có hoành độ x  1 của (C ) cắt (C ) tại 2 điểm có hoành
độ lần lượt là 0 và 2, Gọi S1 ; S2 là diện tích hình phẳng (phần gạch chéo trong hình vẽ). Tính S 2 ,
401
biết S1  .
2022

12431
A. .
2022
5614
B.  .
1011
2005
C. .
2022
2807
D. .
1011
Lời giải
Từ đồ thị (C ) nhận thấy a  0; b  0; c  0 .
Ta có: f ( 1)  0 suy ra: a  b  c  0 (1); gọi A( 1; 0)
Phương trình tiếp tuyến tại A( 1; 0) là ( d ) : y  y(1)( x  1)  ( 4 a  2b )( x  1)
Phương trình hoành độ giao điểm của tiếp tuyến (d ) và đồ thị (C ) :
(4a  2b)( x  1)  ax 4  bx 2  c(*)
4a  2b  c
Mà x  0, x  2 là nghiệm của (*) suy ra 
12a  6b  16a  4b  c
c  a  b c   a  b  c  2a
Từ (1) và (2) ta có:   
4a  2b  a  b  b  3a b  3a
0 0
Ta có: S1  
1
 ax 4

 bx 2  c  (4a  2b)( x  1) dx  
1
 ax 4

 3ax 2  2a  2a( x  1) dx
0 a 401 2005
 S1  a 
1
x 4

 3 x 2  2 x dx  
5 2022
a
2022
.
2 2 28a 5614
0
 0
 
 S 2    ( 4a  2b)( x  1)  ax 4  bx 2  c dx  a   x 4  3 x 2  2 x dx 
5

1011
. 
5614
Vậy S 2  .
1011
Câu 21. (THPT Phù Cừ - Hưng Yên - 2022) Cho hai hàm số f ( x ) và g ( x ) liên tục trên  và hàm số
f ( x)  ax 3  bx 2  cx  d , g ( x )  qx 2  nx  p với a , q  0 có đồ thị như hình vẽ. Biết diện tích
5
hình phẳng giới hạn bởi hai đồ thị hàm số y  f ( x ) và y  g ( x ) bằng và f (2)  g (2) . Biết
2

Trang 12 Fanpage Nguyễn Bảo Vương  https://www.facebook.com/tracnghiemtoanthpt489/


Điện thoại: 0946798489 TUYỂN CHỌN VẬN DỤNG – VẬN DỤNG CAO 2022
a
diện tích hình phẳng giới hạn bởi hai đồ thị hàm số y  f ( x) và y  g ( x ) bằng (với a , b  
b
và a, b nguyên tố cùng nhau). Tính T  a 2  b 2 .

A. 7. B. 55. C. 5 . D. 16.
Lời giải
Từ đồ thị hàm số y  f ( x ) và y  g ( x ) suy ra f ( x )  g ( x )  ax ( x  1)( x  2) .
2 5 2 5 2 5 1 5
Mà  f ( x )  g ( x ) dx    |ax ( x  1)( x  2) | dx  | a |  |x ( x  1)( x  2) | dx   | a | | a | 5
0 2 0 2 0 2 2 2

Dựa vào đồ thị hàm y  f ( x ) suy ra a  0 . Do đó | a | 5  a  5 .


Mặt khác, lại có f ( x)  g ( x)  5 x( x  1)( x  2)  5  x 3  3 x 2  2 x 
5 4

   f ( x )  g ( x ) dx   5  x 3  3 x 2  2 x   f ( x )  g ( x ) 
4
 x  4 x3  4 x 2   C
Với x  2  f (2)  g (2)  C  C  0 .
5 4 x  0
Suy ra f ( x)  g ( x)   
x  4 x 3  4 x 2  f ( x)  g ( x)  0   .
4 x  2
Vậy diện tích hình phẳng giới hạn bởi hai đồ thị hàm số y  f ( x) và y  g ( x ) là
2 5  4 a  4
0 4

 

S    x 4  4 x 3  4 x 2 dx   
3 b  3
. Vậy T  a 2  b 2  7 .

Câu 22. (Sở Hà Tĩnh 2022) Cho f ( x ) là hàm đa thức bậc bốn và có đổ thị như hình vẽ. Hình phắng gióri
hạn bởi đổ thị hai hàm số y  f ( x) ; y  f ( x ) có diện tích bằng

127
A. .
40
107
B. .
5
87
C. .
40

Facebook Nguyễn Vương https://www.facebook.com/phong.baovuongTrang 13


Blog: Nguyễn Bảo Vương: https://www.nbv.edu.vn/
127
D. .
10
Lời giải
2 2 1 1
Ta có f ( x )  k ( x  2) ( x  1) ; f ( 1)  1  4k  1  k   f ( x )  ( x  2) 2 ( x  1) 2 .
4 4
1
Do đó f ( x)   2( x  2)( x  1) 2  2( x  1)( x  2) 2  .
4
Phương trình hoành độ giao điểm:
1 1
( x  2)2 ( x  1)2   2( x  2)( x  1)2  2( x  1)( x  2)2 
4 4
1
 ( x  2)( x  1)[( x  2)( x  1)  2( x  1)  2( x  2)]  0
4
1
 ( x  2)( x  1)  x 2  3 x  4   0
4
 x  2; x  1; x  1; x  4.
4 4 1 107
Vì vậy S   f ( x)  f ( x) dx  
2 2 4
 
( x  2)( x  1) x 2  3 x  4 dx 
5
.

Câu 23. (Sở Thanh Hóa 2022) Cho hàm số f ( x ) có đạo hàm liên tục trên  thoả män
2 f ( x )  xf ( x )  3 x  10, x   và f (1)  6 . Biết
4ln(2  f ( x))
 dx  a ln 5  b ln 6  c ln(2  3), (a, b, c  ) . Khi đó a  b  c thuộc khoàng
1 f 2 ( x )  6 f ( x )  9

nào dưới đây?


A. (1; 2) .
B. (2;3) .
C. (0;1) .
D. (1; 0) .
Lời giải
2 10
Ta có 2 f ( x)  xf ( x)  3 x  10  f ( x)  f ( x)  3   x 2 f ( x)  2 xf ( x )  3 x 2  10 x
x x
  x 2 f ( x)    3 x 2  10 x  x 2 f ( x)    3 x 2  10 x dx  x 3  5 x 2  C ; f (1)  6  C  0  f ( x)  x  5.

ln(2  f ( x))
4 4 ln(2  f ( x)) 4 ln(2  x  5)
Khi đó I   2
dx   2
dx   dx .
f ( x)  6 f ( x)  9
1  1 ( f ( x)  3)  1 ( x  2) 2
Trước tiên từng phần
1
4
 1  1 1
 2 x  5 dx
4 4
I   ln(2  x  5)d    ln(2  x  5)  
1
 x2 x2 1
 1 x2 2 x5
1
1 4 1 1
  ln 5  ln 4    2 x  5 dx   ln 5  ln 4  K .
6  1 x2 2 x5 6
Tích phân K chúng ta đổi biến
1
t  2  x  5  dt  dx; x  5  (t  2) 2  x  2  (t  2) 2  3 .
2 x5
5 1
Đổi cận x  1  t  4; x  4  t  5  K   dt .
4
t  (t  2)2  3
1 a 2(t  2) c 1
Đồng nhất hệ số  b   a  1; b   ; c  2 .
 2
t (t  2)  3 t  2 2
(t  2)  3 (t  2)  3 2
Trang 14 Fanpage Nguyễn Bảo Vương  https://www.facebook.com/tracnghiemtoanthpt489/
Điện thoại: 0946798489 TUYỂN CHỌN VẬN DỤNG – VẬN DỤNG CAO 2022
5
1 1 2(t  2)
5 2   1 2 t 2 3 
K      dt   ln | t |  ln (t  2) 2  3 

ln  ∣
2 3 t  2  3 
4 t 2 2
 2 (t  2)  3 (t  2)  3   2
4

1 1  3 3 2 3  1 1
 ln 5  ln 4  ln 6   ln  ln   ln 5  ln 4  ln 6  ln(2  3)
2 3  3 3 2 3  2 3
 1   1 1 
I    ln 5  ln 4    ln 5  ln 4  ln 6  ln(2  3) 
 6   2 3 
5 1 1 5 1 1 2
 ln 5  ln 6  ln(2  3)  a  b  c     .
6 2 3 6 2 3 3

Câu 24. (Sở Thanh Hóa 2022) Cho hàm số f ( x )  0, x  0 và có đạo hàm f ( x ) liên tục trên khoảng
1
(0;  ) thoả mãn f ( x)  (2 x  1) f 2 ( x) , x  0 và f (1)   . Giá trị của biểu thức
2
f (1)  f (2)  f (3)    f (2022) bằng
2022
A. .
2023
2021
B. .
2022
2021
C.  .
2022
2022
D.  .
2023
Lời giải
f ( x) f ( x) 1
Có 2  2 x  1   2 dx   (2 x  1)dx    x2  x  C .
f ( x) f ( x) f ( x)
1 1 1
Có f (1)      1  1  C  C  0  f ( x)   2 .
2 1 x  x

2
 1 1  20122 2022
1 1  1 1  2022
Vì vậy f ( x)    
 x x 1 
  
k 1
f ( k )     
k 1  k k 1 
    
 1 2023  2023
.

Câu 25. (Sở Bắc Giang 2022) Cho hàm số y  f ( x) có đạo hàm xác định trên (0;  ) và thỏa mãn
1 a
x  f ( x)  x   ( x  1) f ( x); f (1)  e  1 . Biết rằng  f ( x) dx  ; trong đó a , b là những số nguyên
0 b
a
dương và phân số tối giản. Khi đó giá trị của (2a  b) tương ứng bằng:
b
A. 4.
B. 5.
C. 8.
D. 7.
Lời giải
Ta có: x  f ( x)  x   ( x  1) f ( x)  xf ( x)  xf ( x)  f ( x)   x 2
Với x  0 ta có: f (0)  0 (1)
Với x  0
xf ( x)  f ( x) f ( x)  f ( x)  f ( x)
Chia cả hai vế cho x 2 : 2
  1      1
x x  x  x
 f ( x)  f ( x)  f ( x)  x 
Nhân hai vế với e x : e x     e x  e  x    e    e  x
 x  x  x 

Facebook Nguyễn Vương https://www.facebook.com/phong.baovuongTrang 15


Blog: Nguyễn Bảo Vương: https://www.nbv.edu.vn/
f ( x)  x
Lấy nguyên hàm hai vế:  e  e x  C
x
f (1) 1
Do f (1)  e  1 nên:  e  e 1  C  C  1
1
f ( x)  x
Vậy
x
 e  e  x  1  f ( x )  x 1  e x (2)  
 
Từ (1) và (2) ta có f ( x)  x 1  e x thỏa mãn yêu cầu đề bài
2 1 1
1 x 1 x2 1 3
0
 x
Khi đó:  x 1  e dx  
2
x
  xe dx 
0 2

 xe x  e x  0

2
.
0 0
Kết luận (2 a  b)  2.3  2  8 .
Câu 26. (Sở Bắc Giang 2022) Một bức tường lớn kích thước 8m  8m trước đại sảnh của một toà biệt thự
được sơn loại sơn đặc biệt. Người ta vẽ hai nửa đường tròn đường kính AD, AB cắt nhau tại H ;
đường tròn tâm D , bán kính AD , cắt nửa đường tròn đường kính AB tại K . Biết tam giác
"cong" AHK được sơn màu xanh và các phần còn lại được sơn màu trắng (như hình vẽ) và một
mét vuông sơn trắng, sơn xanh lần lượt có giá là 1 triệu đồng và 1,5 triệu đồng. Tính số tiền phải
trả (làm tròn đến hàng ngàn).

A. 60,567, 000 (đồng). B. 70, 405, 000 (đồng).


C. 67,128, 000 (đồng). D. 86,124, 000 (đồng).
Lời giải
Chọn hệ toạ độ Oxy như hình vẽ sau

Dễ thấy cung AB có phương trình y  f ( x)  8  16  ( x  4) 2 ; cung AH có phương trình


y  g ( x)  4  16  x 2 và cung AC có phương trình y  h( x)  64  x 2 . Dễ tìm được tọa độ
 24 
các điểm H (4; 4) và K  6, 4;  .
 5 
Diện tích tam giác AHK là
S  S AHE  S HEX  
4

0  
64  x 2  4  16  x 2 dx  
6.4

4  
64  x 2  8  16  ( x  4) 2 dx  6, 255085231.

Số tiền cần trả là S 1, 5   82  S  1  67,12754262 .


Vậy số tiền cần trả là 67,128, 000 (đồng).

Trang 16 Fanpage Nguyễn Bảo Vương  https://www.facebook.com/tracnghiemtoanthpt489/


Điện thoại: 0946798489 TUYỂN CHỌN VẬN DỤNG – VẬN DỤNG CAO 2022
e x  m khi x0
Câu 27. (Sở Bắc Giang 2022) Cho hàm số f ( x)   2 3 3 (với m là tham số). Biết
 x x  1   khi x0
1 b b
hàm số f ( x) liên tục trên  và  f ( x ) dx  a  e  với a , b, c   * ; tối giản
1 c c
(e  2, 718281828) . Biểu thức a  b  c  m bằng
A. 13.
B. 35.
C. 11 .
D. 36.
Lời giải
Hàm số y  f ( x ) có tập xác định là  .
3
Ta có với x  0 khi đó f ( x)  e x  m hoặc x  0 khi đó f ( x)  x 2  x3  1 nên hàm số
y  f ( x ) đã liên tục trên các khoảng (; 0) và (0;  ) với mọi giá trị của tham số m .
Xét tại x  0 , ta được:
3
lim f ( x)  lim e x  m  1  m; lim f ( x)  lim  x 2 x 3  1   0 và f (0)  1  m.
   
x0 x 0 x 0 x 0  
Hàm số f ( x ) liên tục trên  khi và chỉ khi liên tục tại x  0  lim f ( x)  lim f ( x)  f (0)
x 0 x 0
 1  m  0  m  1.
1 1
Khi đó  f ( x)dx    f ( x)dx   f ( x)dx  I  J trong đó:
1 1 0
0 1 0
3 3 1 1
I   x 2  x 3  1 dx    x 3  1 d  x 3 J    e x  1dx   e x  x   e  2.
1 3 1 0 0

1 1 23
Từ đó ta được  f ( x ) dx  e  2   1.e  .
1 12 12
Từ đó ta tìm được a  1; b  23; c  12; m  1 nên a  b  c  m  1  23  12  ( 1)  35 .
Câu 28. (Sở Hà Tĩnh 2022) Cho đường cong (C ) : y  x 3  kx  2 và parabol P : y   x 2  2 tạo thành hai
miền phẳng có diện tích S1 , S 2 như hình vẽ bên.

8
Biết rằng S1  , giá trị của S 2 bằng
3
1
A. .
2
1
B. .
4
3
C. .
4
Facebook Nguyễn Vương https://www.facebook.com/phong.baovuongTrang 17
Blog: Nguyễn Bảo Vương: https://www.nbv.edu.vn/
5
D. .
12
Lời giải.
Phương trình hoành độ giao điểm của (C ) và d
x  0
x 3  kx  2   x 2  2  x  x 2  x  k   0   2
x  x  k  0
Hai đồ thị cắt nhau tại ba điểm phân biệt nên phương trình x 2  x  k  0 có hai nghiệm phân biệt
k  0

x1 , x2 khác 0 và thỏa mãn x1  0  x2 . Do đó ta có  x2  1  x1
k   x 2  x
 1 1

8
Trên đoạn  x1 ;0 , x  kx  2   x  2  x  x  kx  0 . Theo bài ra, diện tích S1  nên
3 2 3 2

3
0
3 2 8
x1 x  x  kx dx  3
0
0 8  x 4 x 3 kx 2  8
   x  x  kx dx     
3 2
     3 x1  4 x1  6kx1   32
4 3 2
x1 3  4 3 2 x 3
1

 3 x14  4 x13  6   x12  x1  x12  32


 3 x14  2 x13  32  0   x1  2   3x13  4 x12  8 x1  16   0  x1  2  vì x1  0  .
Với x1  2  k  2, x2  1 và x3  x 2  2 x  0, x  [0;1] , ta có
1
1  x 4 x3  5
S 2     x  x  2 x dx      x 2  
3 2
0
 4 3  0 12
Câu 29. (Sở Hà Tĩnh 2022) Cho hàm số y  f ( x) có đạo hàm f ( x ) liên tục trên  . Miền hình phẳng
trong hình vẽ được giới hạn bơi đồ thị hàm số y  f ( x ) và trục hoành đồng thời có diện tích
1
S  a . Biết rằng  ( x  1) f ( x)dx  b
0

A. I  a  b  c. B. I  a  b  c .
C. a  b  c . D. I  a  b  c .
Lời giải
Ta có
1 3
S  a   f ( x)dx   f ( x)dx  a  2 f (1)  f (0)  f (3)  a  2 f (1)  f (0)  a  c.
0 1
Áp dụng công thức tích phân từng phần với u  x  1 và dv  f ( x ) dx , ta được
1 1 1
 ( x  1) f ( x)dx  b  ( x  1) f ( x)  
0 0 0
f ( x)dx  b  2 f (1)  f (0)  I  b  a  c  I  b  I  a  b  c.

Trang 18 Fanpage Nguyễn Bảo Vương  https://www.facebook.com/tracnghiemtoanthpt489/


Điện thoại: 0946798489 TUYỂN CHỌN VẬN DỤNG – VẬN DỤNG CAO 2022
Câu 30. (Sở Ninh Bình 2022) Diện tích hình phẳng giới hạn bởi parabol y  x 2  2 x  1 và đường thẳng
y  (m  1) x  5 có giá trị nhỏ nhất bằng
16
A. .
3
48
B. .
3
64
C. .
3
32
D. .
3
Lời giải.
Phương trình hoành độ giao điểm
x 2  2 x  1  (m  1) x  5  x 2  (1  m) x  4  0.
Với mọi m ta đều có ac  4  0 nên phương trình (1) luôn có hai nghiệm x1 , x2 ,  x1  x2  . Theo
x  x  m 1
định lí Vi-et, ta có  1 2 và
 x1 x2  4
2 2
x2  x1   x2  x1    x2  x1   4 x1 x2  (m  1)2  16.
Khi đó hình phẳng luôn tồn tại và có diện tích là
x2 x2
x 2  (1  m) x  4 dx   x  (1  m) x  4 dx
2
S
x1 x1

x2
 x3 x2  1 x2
   (1  m)  4 x    2 x3  3(1  m) x 2  24 x  ∣
 3 2  x1 6 x1

3 3


2
m  2m  17
 x2  x1  
 m 2  2m  17   (m  1) 2  16  
43 32

6 6 6 6 3
32
Dấu bằng xảy ra khi m  1 . Vậy giá trị nhỏ nhất của S bằng .
3
Câu 31. (Sở Ninh Bình 2022) Cho hàm số f ( x)  x5  ax 4  bx 3  cx 2  dx  36 . Biết đồ thị hàm số
y  f ( x ), y  f ( x ) và Ox giao nhau tại hai điểm phân biệt có hoành độ lần lượt là 2,3. Diện tích
m
hình phẳng giớihạn bởi đồ thị hàm số y  f ( x) và Ox bằng là một phân số tối giản với
n
m, n  * . Tổng m  n bằng
A. 846.
B. 845.
C. 848.
D. 847.
Lời giải.
Từ giả thiết ta có x  2, x  3 là nghiệm của f ( x ) và f ( x ) nên f ( x ) có dạng
f ( x)  ( x  2) 2 ( x  3) 2 ( x  k ).
Mà f (0)  36 nên k  1 . Suy ra diện tích hình phẳng cần tìm là
3 3 832
S   | f ( x ) | dx   ( x  2) 2 ( x  3) 2 ( x  1) dx 
1 1 15

Facebook Nguyễn Vương https://www.facebook.com/phong.baovuongTrang 19


Blog: Nguyễn Bảo Vương: https://www.nbv.edu.vn/
Câu 32. (Sở Ninh Bình 2022) Gọi X là tập hợp tất cả các giá trị của tham số m để đường thẳng d :
1
y  45m  2 cùng với đồ thị (C ) của hàm số y  x 3  2mx 2  x  1 tạo thành hai miền kín có
3
diện tích lần lượt là S1 , S2 thỏa mãn S1  S 2 (xem hình vẽ). Số phần tử của tập X là

A. 0.
B. 2.
C. 1.
D. 9.
Lời giải.
Yêu cầu bài toán tương đương đồ thị (C ) có hai điểm cực trị và tâm đối xứng I của (C ) thuộc
d . Ta có f ( x )  x 2  4mx  1 nên hàm số có hai điểm cực cực trị khi và chỉ khi
 1 1
  (2m)2  1  0   m  m   .
 2 2
Lại có
f ( x )  0  2 x  4 m  x  2m.
16 3  16 3 
Với x  2m thì f ( x )  m  2m  1 nên I  2m; m  2m  1 . Suy ra I  d khi và chỉ khi
3  3 
m  3
16 3
m  2m  1  45m  2  16m  141m  9  0  
3
.
3  m  6  33
 4
6  33  6  33 
Dễ thấy m  không thỏa mãn, do đó X  3; .
4  4 
Câu 33. (Sở Bạc Liêu 2022) Cho hàm số y  f  x  xác định và liên tục trên đoạn  3;3 . Biết diện tích
hình phẳng S1 , S 2 giới hạn bởi đồ thị hàm số y  f  x  và đường thẳng y   x  1 lần lượt là
3
M , m . Tính tích phân  f  x  dx .
3

A. 6  m  M . B. 6  m  M . C. M  m  6 . D. m  M  6 .
Lời giải
Chọn D

Trang 20 Fanpage Nguyễn Bảo Vương  https://www.facebook.com/tracnghiemtoanthpt489/


Điện thoại: 0946798489 TUYỂN CHỌN VẬN DỤNG – VẬN DỤNG CAO 2022
1 1 1 1
Ta có S1     x  1  f  x   dx  M     x  1 dx   f  x  dx  M   f  x  dx  M
3 3 3 3

3 3 3 3
S 2    f  x     x  1  dx  m   f  x  dx    x  1 dx  m   f  x  dx  m  6 .
1 1 1 1
3 1 3
Vậy  f  x  dx   f  x  dx   f  x  dx   M  m  6 .
3 3 1

e x  1 khi x  0
Câu 34. (Sở Bạc Liêu 2022) Cho hàm số y  f  x   2 . Tích phân
 x  2 x  2 khi x  0
e2
f  ln x  1 a a
I dx   ce biết a , b, c   và tối giản. Tính a  b  c ?
1 x b b
e
A. 35 . B. 29 . C. 36 . D. 27 .
Lời giải
Chọn C
1
Đặt t  ln x  1  dt  dx .
x
1
Đồi cận x   t  2 và x  e 2  t  1 .
e
1 1 0 1 0 1
Khi đó I   f  t dt   f  x dx   f  x dx   f  x dx  
2 2 2 0 2
x 2
 2 x  2 dx    e x  1dx
0
0
 x3  1 32 a
   x 2  2 x    e x  x    e   ce .
 3  2 0 3 b
Suy ra a  32, b  3, c  1 .
Vậy a  b  c  36 .
Câu 35. (Sở Hà Tĩnh 2022) Cho hàm số f  x   ax 4  x 3  2 x  2 và hàm số g  x   bx 3  cx 2  2 có đồ
221
thị như hình vẽ bên. Gọi S1; S2 là diện tích các hình phẳng gạch chéo trong hình vẽ, biết S1  .
640
Khi đó S2 bằng

791 571 271 1361


A. . B. . C.. D. .
640 640 320 640
Lời giải
Chọn A
Ta có:
1 1
2 2
a b  1 c 1 221
S1    f  x   g  x   dx    ax 4   b  1 x 3  cx 2  2 x  dx     
0 0
160 64 24 4 640

Facebook Nguyễn Vương https://www.facebook.com/phong.baovuongTrang 21


Blog: Nguyễn Bảo Vương: https://www.nbv.edu.vn/
80
 4a  10b  c  71
3
Dựa vào đồ thị ta thấy, các giao điểm của g  x  và trục hoành chính là điểm cực trị của f  x  nên
f   x  và g  x  có ba nghiệm chung. Mặt khác f   x  và g  x  đều là hàm số bậc ba có hệ số tự
do bằng 2 nên: g  x   f   x   4ax 3  3 x 2  2  bx 3  cx 2  2, x  

b  4 a
 .
c  3
1 1
Từ đó ta được 4a  40a  80  71  a   f  x   x 4  x3  2 x  2 .
4 4
2
1  791
Vậy S2    x 4  x3  2 x  2  dx  .
34  640
2
4
Câu 36. (Sở Hà Tĩnh 2022) Cho hàm số f  x  liên tục trên  và f  4   2 ,  f  x  dx  4 . Tính
0
2

 xf   2 x  dx
0

A. I  17 . B. I  1 . C. I  12 . D. I  4 .
Lời giải
Chọn B
2
Xét I   xf   2 x  dx .
0

du  dx
u  x 
Đặt   1 .
 dv  f   
2 x d x v  2 f  2 x 
2 2 2
1 2 1 1 1
I  x. f  2 x  0   f  2 x  dx  f  4    f  2 x  dx  2   f  2 x  dx
2 20 20 20
Đặt t  2 x  dt  2dx .
Đổi cận x  0  t  0; x  2  t  4 .
4 4
1 1
 I  2  f  t  dt  2   f  x  dx  2  1  1 .
40 40
 
Câu 37. (Sở Hà Tĩnh 2022) Cho F  x  là nguyên hàm của f  x   sin 2 x trên  thoả mãn F    0 .
4
 
Giá trị biểu thức S  F     2 F   bằng
2
3  3 3 1 3 3 3
A. S   . B. S   . C. S   . D. S   .
4 4 4 4 4 8 2 8
Lời giải
Chọn B
1  cos 2 x 1 1
Ta có  sin 2 xdx   dx  x  sin 2 x  C .
2 2 4
1 1
 F  x  x  sin 2 x  C
2 4

Trang 22 Fanpage Nguyễn Bảo Vương  https://www.facebook.com/tracnghiemtoanthpt489/


Điện thoại: 0946798489 TUYỂN CHỌN VẬN DỤNG – VẬN DỤNG CAO 2022
  1 
  1
Mà F    0   sin  C  0  C    .
4 8 4 2 8 4
1 1  1
Vậy F  x   x  sin 2 x   .
2 4 8 4
   1  1 1  1   1
S  F     2 F      sin  2     2  .  sin 2.   
2 2 4 8 4 2 2 4 2 8 4
   1 1 3 3
       .
2 8 4 4 2 4 8
Câu 38. (Sở Lạng Sơn 2022) Cho hàm số y  f  x  có đạo hàm liên tục trên  thỏa mãn f 1  e và
f   x   f  x   x, x   . Giá trị f  2 bằng
2 1 1
A. . B. 1  . C. 1  . D. 2 .
e e e
Lời giải
Chọn D
Ta có:
f   x   f  x   x  f   x  .e x  f  x  .e x  xe x   e x . f  x    xe x .
 xe x  e x  C
  e . f  x   dx   xe dx  e . f  x   xe
x x
Nên x x
 ex  C  f  x   .
ex
e1  e1  C
Do f 1  e  f 1  1
 C  e2 .
e
xe x  e x  e 2
Suy ra f  x  
ex
2.e 2  e2  e2
f  2  2.
e2
Câu 39. (Sở Lạng Sơn 2022) Giả sử hàm số y  f  x  liên tục, nhận giá trị dương trên  0;   và thỏa
mãn f 1  e , f  x   f   x  . 3 x  1 , với mọi x  0 . Mệnh đề nào sau đây đúng?
A. 3  f  5   4 . B. 11  f  5   12 . C. 10  f  5   11 . D. 4  f  5   5 .
Lời giải
Chọn C
Do hàm số y  f  x  liên tục, nhận giá trị dương trên  0;   nên f  x   f   x  . 3 x  1
f  x 1 f  x 1 2
   dx   dx  ln f  x   3 x  1  C  *
f  x 3x  1 f  x 3x  1 3
4 1
Ta có: f 1  e nên *  1   C  C   **
3 3
2 1 7
3 x 1 
Từ * và ** suy ra f  x   e 3 3
 f  5   e 3  10, 31 .
Câu 40. (Sở Phú Thọ 2022) Cho hàm số y  f  x liên tục trên R \ 2;0 thỏa mãn
x  x  2  . f   x   2 f  x   x  2 x và f 1  6 ln 3 . Biết f  3  a  b ln 5  a, b    . Giá trị
2

a  b bằng?
10 20
A. 20 . B. 10 . C. . D. .
3 3
Lời giải
Chọn D
2
Xét x  x  2  . f   x   2 f  x   x 2  2 x chia hai vế cho  x  2  ta được:

Facebook Nguyễn Vương https://www.facebook.com/phong.baovuongTrang 23


Blog: Nguyễn Bảo Vương: https://www.nbv.edu.vn/
xf   x  2 f  x  x  x  x
 2
   f  x    .
x  2  x  2 x2 x2  x2
Lấy nguyên hàm hai vế ta được:
x x x
f  x   dx  f  x   x  2 ln x  2  C .
x2 x2 x2
6 ln 3
Mà f 1  6 ln 3 nên ta có:  1  2 ln 3  C  C  1 .
3
x 3 10 10
Khi đó f  x   x  ln x  2  1  f  3  2  2 ln 5  f  3    ln 5 .
x2 5 3 3
10 10 20
Vậy a  b    .
3 3 3
3x 2 ln  x  1 khi x  0
Câu 41. (Sở Phú Thọ 2022) Cho hàm số f  x   . Biết
2
2 x x  3  1 khi x  0
e
f  ln x 
 dx  a 3  b ln 2  c với a, b, c  . Giá trị của a  b  6c bằng
1 x
e
A. 35 . B. 14 . C.  27 . D. 18 .
Lời giải
Chọn C
x  e  t  1
e
f  ln x  1 
Ta có: I   dx . Đặt: t  ln x  dt  dx . Đổi cận:  1 .
1 x x  x  e  t  1
e
1 1 0 1
I  f  t  dt   f  x  dx   f  x  dx   f  x  dx
1 1 1 0
0 1
I   2x
1

x 2  3  1 dx   3 x 2 ln  x  1 dx
0
0 0 1
 I   2 x x 2  3dx   dx   3 x 2 ln  x  1 dx .
1 1 0
0
Ta có: J   2 x x 2  3dx .
1

 x  0  r  3
Đặt: r  x 2  3  r 2  x 2  3  2rdr  2 xdx . Đổi cận:  .
 x  1  r  2
3 3
2r 3 2 16
 J   2r dr  2 3 .
2
3 2
3
1
Ta có: K   3 x 2 ln  x  1 dx .
0

 1
u  ln  x  1 du  dx
Đặt:    x 1 .
2
dv  3x dx v  x  1   x  1  x  x  1
3 2

1 1
1  x3 x 2  5
 K   x  1 ln  x  1    x  x  1 dx  2 ln 2  ln1     1  2 ln 2  .
3 2
0
0  3 2 0 6

Trang 24 Fanpage Nguyễn Bảo Vương  https://www.facebook.com/tracnghiemtoanthpt489/


Điện thoại: 0946798489 TUYỂN CHỌN VẬN DỤNG – VẬN DỤNG CAO 2022
0
16 5 31
Khi đó: I  J  K   dx  2 3   2 ln 2   1  2 3  2 ln 2  .
1
3 6 6
31
. Vậy a  b  6c  27 .
 a  2; b  2; c  
6
Câu 42. (Sở Vĩnh Phúc 2022) Cho hàm số f  x  có đạo hàm trên  , thoả mãn f  x   1
và f '  x  x 2  1  2 x f  x   1, x   . Biết rằng f  0   0 , khi đó f  2  có giá trị bằng
A. 0. B. 4. C. 8. D. 6.
Lời giải
Chọn B
f ' x 2x
Ta có f '  x  x 2  1  2 x f  x   1  
f  x 1 x2  1
f ' x 2x d  f  x  1
Nên  dx   dx    d  x 2  1
f  x 1 2
x 1 f  x 1 2
x 1
 2 f  x   1  2 x2  1  C .
Mà f  0   0  C  0  f  x   1  x 2  1 nên f  x   x 2  f  2   4 .
Câu 43. (THPT Bùi Thị Xuân – Huế - 2022) Cho hai hàm số f ( x)  ax 4  bx 3  cx 2  3 x và
g ( x)  mx 3  nx 2  x , với a , b, c, m, n   . Biết hàm số y  f ( x )  g ( x ) có ba điểm cực trị là 3,1
và 4. Diện tích hình phẳng giới hạn bởi hai đồ thị y  f ( x ) và y  g ( x ) bằng
935
A. .
36
941
B. .
36
937
C. .
36
939
D. .
36
Lời giải
Ta có: f ( x )  ax  bx  cx  3 x  f ( x)  4ax 3  3bx 2  2cx  3 .
4 3 2

Ta có: g ( x)  mx 3  nx 2  x  g ( x)  3mx 2  2mx  1 .


y  f ( x)  g ( x )  y  f ( x)  g ( x)  4ax3  3(b  m) x 2  2(c  n) x  4.
   1
   a
 x  3 4a (3)3  3(b  m)(3)2  2(c  n)(3)  4  0 12

   2
y  0   x  1  4a 13  3(b  m) 12  2(c  n) 1  4  0  b  m 
4a  43  3(b  m)  42  2(c  n)  4  4  0  9
x  4
   11
  cn 
   6
4 4 1 2 11 937
S   f ( x )  g ( x) dx   x 3  x 2  x  4 dx  .
3 3 3 3 3 36
Câu 44. (THPT Bùi Thị Xuân – Huế - 2022) Một công ty có ý định thiết kế một logo hình vuông có độ
dài nửa đường chéo bằng 4. Biểu tượng 4 chiếc lá (được tô màu) được tạo thành bởi các đường
cong đối xứng với nhau qua tâm của hình vuông và qua các đường chéo.
Một trong số các đường cong ở nửa bên phải của logo là một phần của đồ thị hàm số bậc ba dạng
y  ax 3  bx 2  x với hệ số a  0 . Để kỷ niệm ngày thành lập 2 / 3 , công ty thiết kế để tỉ số diện
2
tích được tô màu so với phần không được tô màu bằng . Tính 2a  2b
3
Facebook Nguyễn Vương https://www.facebook.com/phong.baovuongTrang 25
Blog: Nguyễn Bảo Vương: https://www.nbv.edu.vn/

41
A. .
80
1
B. .
2
4
C. .
5
9
D. .
10
Lời giải
Ta có nửa đường chéo hình vuông có độ dài là 4, cạnh hình vuông sẽ là 4 2 và diện tích hình
64
vuông là 32, khi đó ta có được diện tích phần tô màu là .
5
Gọi f ( x)  ax3  bx 2  x là hàm số bậc ba biểu diễn đường cong trên logo.
Ta có x  4 là nghiệm của phương trình nên 64a  16b  4  0  4a  b  1 (1).
Ta có phương trình phương trình f ( x )  0 sẽ có các nghiệm là 0,4 và a  4 vì 4a  0
4 4
 
Nên S   ax3  bx 2  x dx    ax3  bx 2  x dx
0 0
4
 ax 4 bx3 x 2  64 8 64 32
      64a  b  8   64a  b 
 4 3 2 0 3 5 3 5
  1
 4a  b  1 a  20 4
Từ (1) và (2) ta có hệ phương trình  64 32    2a  2b  .
64a  3 b  5 b  9 5
 
 20
Câu 45. (Chuyên Hoàng Văn Thụ - Hòa Bình – 2022) Cho hàm số y  f ( x ) có đạo hàm không âm trên
2 2
 
[0;1] , thỏa mãn f ( x )  0 với mọi x  [0;1] và [ f ( x)]2   f ( x) x 2  1  1  [ f ( x)]2 . Nếu
f (0)  3 thì giá trị f (1) thuộc khoảng nào sau đây?
 7
A.  3;  .
 2
 5
B.  2;  .
 2
5 
C.  ;3  .
2 
3 
D.  ; 2  .
2 
Lời giải
Trang 26 Fanpage Nguyễn Bảo Vương  https://www.facebook.com/tracnghiemtoanthpt489/
Điện thoại: 0946798489 TUYỂN CHỌN VẬN DỤNG – VẬN DỤNG CAO 2022
2
2 2 [ f ( x)]2   f ( x) 1
Ta có: [ f ( x)]   f ( x)   x  1  1  [ f ( x)] 
2 2 2
2
 2
1  [ f ( x)] x 2
1 
f ( x)  f ( x) 1 1 f ( x )  f ( x ) 1 1 1 f ( x )  f ( x ) 1 1
  2
 dx   2 dx   dx   2 dx
1  [ f ( x)]2 x 1 0
1  [ f ( x)] 2 0 x 1 0
1  [ f ( x)] 2 0 x 1
f ( x)  f ( x) 1 f 2 (1)
 Nếu đặt t  1  [ f ( x )]2  dt  dx  VT   dt  1  f 2 (1)  2
2 2
1  [ f ( x)]
+ Nếu đặt x  tan u  dx  1  tan 2 u  du

1   2 5 
 VP   4
0 2
1  tan u
1  tan 2
u  dx 
4
 1  f 2
(1)  2 
4
 f (1) 
16
   3  2, 6   ;3  .
2 

7
Câu 46. (Chuyên Lam Sơn 2022) Cho hàm số f ( x ) với đồ thị là Parabol đỉnh I có tung độ bằng 
12
và hàm số bậc ba g ( x ) . Đồ thị hai hàm số đó cắt nhau tại ba điểm phân biệt có hoành độ x1 , x2 , x3
thoả mãn 18 x1 x2 x3  55 (hình vẽ).

Diện tích miền tô đậm gần số nào nhất trong các số sau đây?
A. 5,7.
B. 5,9.
C. 6,1.
D. 6,3.
Lời giải
1 7 7
Dễ thấy I  ,   và f ( x )  ( x  1)( x  2) .
 2 12  27
Hàm số g ( x ) đạt cực trị tại x  1, x  2 nên
 x3 x 2 
g ( x)  a ( x  1)( x  2)  g ( x)  a    2 x   b
 3 2 
1 7 7 13
Đồ thị hàm số g ( x ) đi qua I nên g         a  b , (1).
2 12 12 12
 x x2
3
 7
Phương trình hoành độ giao điểm: f ( x)  g ( x)  a    2 x   b  ( x  1)( x  2)
 3 2  27
14
b
Theo định lý viet ta có: 18 x1 x2 x3  55  18  27  55  18b  28   55a , (2)
a 3 3
3
3 2
1 x x 1
Từ (1), (2) ta được a  1, b   g ( x)    2 x  . Từ đó suy ra diện tich miền tô đậm sấp
2 3 2 2
sỉ 5,7.
Facebook Nguyễn Vương https://www.facebook.com/phong.baovuongTrang 27
Blog: Nguyễn Bảo Vương: https://www.nbv.edu.vn/
 
Câu 47. (Chuyên Lam Sơn 2022) Cho hàm số y  f ( x) liên tục trên đoạn 0;  thỏa mãn
 2
 
2 cos x  f (1  4sin x)  sin 2 x  f (3  2 cos 2 x)  sin 4 x  4sin 2 x  4 cos x, x  0; 
 2
5
Khi đó I   f ( x)dx bằng
1
A. 2.
B. 0.
C. 8.
D. 16.
Lời giải
Ta có: 2 cos x  f (1  4sin x )  sin 2 x  f (3  2 cos 2 x )  sin 4 x  4 sin 2 x  4 cos x (*)

Lấy tích phân từ 0 đến hai vế của  *  ta được:
2
  
2
 2 cos x  f (1  4sin x)dx   2 sin2 x  f (3  2 cos 2 x)dx   2 (sin 4 x  4sin 2 x  4 cos x)dx
0 0 0
 
1 2 1 1 5 1 5
  f (1  4sin x)d (1  4sin x)   2 f (3  2 cos 2 x)d (3  2 cos 2 x )  0   f (t )dt   f (t )dt  0
2 0 4 0 2 1 4 1
5 5
  f (t )dt  0   f ( x)dx  0
1 1

5
Vậy I   f ( x)dx  0.
1
Câu 48. (Chuyên Lam Sơn 2022) Cho hàm số y  f ( x) có đạo hàm liên tục trên (0;  ) thỏa mãn
2
    
f ( x )  f ( x ) . cot x  2 x . sin x . Biết f    . Tính f   .
2 4 6
2
A. .
36
2
B. .
72
2
C. .
54
2
D. .
80
Lời giải
f ( x )  f ( x)  cot x  2 x  sin x  sin x  f ( x)  f ( x)  cos x  2 x  sin 2 x
 sin x  f ( x)  f ( x)  cos x  2 x  sin 2 x
sin x  f ( x)  f ( x)  cos x
  2x
sin 2 x
 
sin x  f ( x)  f ( x)  cos x
 2 2
dx  
2
2x  dx
6 sin x 6

 
 f ( x)  2 2 f ( x) 2  2  2
 2  dx  x    
6  sin x  6 sin x  4 36
6

   
f  f 
2 6 2 2   
2
       f  
1 1 4 36  6  72
2

Trang 28 Fanpage Nguyễn Bảo Vương  https://www.facebook.com/tracnghiemtoanthpt489/


Điện thoại: 0946798489 TUYỂN CHỌN VẬN DỤNG – VẬN DỤNG CAO 2022
Câu 49. (THPT Đô Lương – Nghệ An – 2022) Cho hàm số y  f ( x ) liên tục trên  1,1 và thỏa mãn
3 1 1
f ( x)  2  ( x  t ) f (t ) dt . với  x  [  1;1] Tính tích phân I  1 f ( x)dx
2 1
A. I 3
B. l 4
C. I 2
D. l 1
Lời giải
a  1 f (t )dt
1 1 1  1
1 ( x  t ) f (t )dt  1 x f (t )dt  1 t f (t )dt  ax  b. Với  1
b   t f (t )dt
 1

3 3
Do đó f ( x)  2  (ax  b)  f ( x )  ( ax  b)  2 .
2 2
1
1 3 1   3  ax 2  
I   f ( x)dx    (ax  b)  2 dx     bx   2 x   3b  4  a  3b  4(1)
1 2
2  2
1
    1
1 3 
b   t  (at  b)  2  dt  a  a  b(2).
1
2 
a  3b  4 a  2 1
Từ (1) và (2) ta có hệ   .Vậy I   f ( x)dx  2
a  b b  2 1

Câu 50. (THPT Lương Thế Vinh – Hà Nội – 2022) Một téc nước hình trụ, đang chứa nước được đạat
nằm ngang, có chiều dài 3m và đường kính đáy 1m . Hiện tại mặt nước trong téc cách phía trển
đỉnh của téc 0, 25 m (xem hình vẽ). Tính thể tích của nước trong téc (kết quả làm tròn đến hàng
phần nghìn)?

A. 1, 768 m3 .
B. 1,167m3
C. 1,895m3 .
D. 1,896m3
Lời giải
Thế tích phần dầu còn lại sẽ bằng diện tích hình phẳng gạch sọc trong hình nhân với chiều dài của
bồn (chiều cao của trụ).

Facebook Nguyễn Vương https://www.facebook.com/phong.baovuongTrang 29


Blog: Nguyễn Bảo Vương: https://www.nbv.edu.vn/

Đường tròn có tâm O (0; 0), R  0, 5 có phương trình là x 2  y 2  0, 25  y   0, 25  x 2 . Diện


tích hình gạch sọc chinh là diện tích hình phẳng giới hạn bởi các đường
y  0, 25  x 2 ; y   0, 25  x 2 ; x  0, 5; x  0, 25 .

Do đó V  S .h  3
0,25

0,5  
0, 25  x 2   0, 25  x 2 dx  1,896m3 .
4
Câu 51. (THPT Lương Thế Vinh – Hà Nội – 2022)  min{2 x  1, x  2, 3x  14}dx bằng
0
31
A. .
2
B. 30.
27
C. .
2
D. 36.
Lời giải
13
Xét 2 x  1  x  2  x  1; x  2  3 x  14  x  3; 3 x  14  2 x  1  x  .
5
Vẽ đồ thị của ba hàm số f ( x )  2 x  1; g ( x )  x  2; h ( x )  3 x  14 trên đoạn [0; 4] trên cùng một
hệ trục toạ độ và quan sát suy ra:

min{2 x  1, x  2, 3 x  14}  2 x  1, x  [0;1]


min{2 x  1, x  2, 3 x  14}  x  2, x  [1;3]
min{2 x  1, x  2, 3 x  14}  3 x  14, x  [3; 4]

Trang 30 Fanpage Nguyễn Bảo Vương  https://www.facebook.com/tracnghiemtoanthpt489/


Điện thoại: 0946798489 TUYỂN CHỌN VẬN DỤNG – VẬN DỤNG CAO 2022
4 1 3 4 27
Khi đó  min{2 x  1, x  2, 3 x  14}dx   (2 x  1)dx   ( x  2)dx   ( 3 x  14)dx  .
0 0 1 3 2
a  b | a  b |
Cách 2: Dùng công thức min{a, b}  và min{a, b, c}  min{a, min{b, c}} khi đó
2
x  2  3 x  14 | ( x  2)  ( 3 x  14) |
min{x  2, 3 x  14}    x  8 | 2 x  6 | .
2
min{2 x  1, x  2, 3 x  14}  min{2 x  1, min{x  2, 3 x  14}}
Suy ra 2 x  1  x  8 | 2 x  6 |  | 2 x  1  ( x  8 | 2 x  6 |) | .

2
4 2 x  1  x  8 | 2 x  6 |  | 2 x  1  (  x  8 | 2 x  6 |) | 27
Tích phân cần tính bằng  dx  .
0 2 2
Câu 52. (Chuyên Nguyễn Trãi – Hải Dương – 2022) Cho hàm số y  f ( x) có đồ thị (C ), f ( x ) có đạo
hàm xác định và liên tục trên khoảng (0;  ) thỏa mãn điều kiện
f ( x)  ln x  f 2 ( x), x  (0; ) . Biết f ( x )  0, x  (0;  ) và f (e)  2 . Viết phương trình tiếp
tuyến với đồ thị (C ) tại điểm có hoành độ x  1 .
2
A. y   x  2 .
3
2
B. y   .
3
2
C. y  x  1 .
3
2
D. y  .
3
Lời giải
2
2 f ( x)  1 
Ta có f ( x )  ln x  f ( x)  2  ln x     ln x
f ( x)  f ( x) 
1
  ln xdx  x ln x  x  C
f ( x) 
1 1
Với x  e ta có  e ln e  e  C mà f (e)  2  C
f (e ) 2
1
Suy ra f ( x) 
1
x ln x  x 
2
 2
 f (1) 
Khi đó  3
 f (1)  ln1  f 2 (1)  0

Phương trình tiếp tuyến với đồ thị (C ) tại điểm có hoành độ x  1 là:

2
y  f ( x )( x  1)  f (1)  .
3
Câu 53. (THPT Trần Phú – Hà Tĩnh – 2022) Cho hàm số y  f ( x )  0 liên tục trên  và f (1)  e3 .
4
Biết f ( x )  (2 x  3) f ( x ), x   . Hỏi phương trình f ( x)  e2 x 3 x  4
có bao nhiêu nghiệm?
A. 4.
B. 3.
C. 2.
D. 0.
Lời giải

Facebook Nguyễn Vương https://www.facebook.com/phong.baovuongTrang 31


Blog: Nguyễn Bảo Vương: https://www.nbv.edu.vn/
f ( x)
f ( x)  (2 x  3) f ( x)   2x  3
f ( x)
f ( x)
 dx   (2 x  3)dx  ln f ( x)  x 2  3x  C , ( f ( x)  0); f (1)  e3
f ( x)
 3  C  2  C  5  ln f ( x)  x 2  3x  5
2 4
 f ( x)  e x 3 x  5
 f ( x)  e 2 x 3 x  4

2 4
 ex 3 x 5
 e2 x 3 x  4
 2 x 4  3x  4  x 2  3x  5  x  1.
1
Câu 54. (THPT Kim Liên - Hà Nội - 2022) Cho hàm số y  f  x  thỏa mãn f  x   0, x  và có
2
1  1
đạo hàm f   x  liên tục trên khoảng  ;   thỏa mãn f   x   8 xf 2  x   0, x  và
2  2
1
f 1  . Tính f 1  f  2   ...  f 1011 .
3
1 2022 2021 2022 1 2021
A. . . B. . C. . D. . .
2 2023 2043 4045 2 2022
Lời giải
Chọn A
f  x
Ta có f   x   8 xf 2  x   0  2  8 x .
f  x
Lấy nguyên hàm hai vế ta được:
f  x 1 1
 f 2  x dx   8 xdx   f 2  x d  f  x     8 xdx   f  x   4 x  C .
2

1
Mà f 1    3  4  C  C  1 .
3
1 1 1 1 
Khi đó f  x   2    .
4 x  1 2  2 x  1 2 x  1 
1  1 1 1 1  1 1 1  1 1 1 
f 1  1   , f  2      , f  3     , ... f 1011    .
2  3 2 3 5 2 5 7  2  2001 2023 
1 1  1 2022
Vậy f 1  f  2   ...  f 1011  1   . .
2  2023  2 2023
Câu 55. (THPT Kim Liên - Hà Nội - 2022) Gọi S là diện tích hình phẳng giới hạn bởi parabol
y  x 2  2 x  1 và các đường thẳng y  m , x  0 , x  1 . Có bao nhiêu giá trị nguyên của tham số
m  4040; 3 để S  2021 ?

A. 2019 . B. 2020 . C. 2021 . D. 2018 .


Lời giải
Chọn D
2
Ta có: y  x 2  2 x  1   x  1  2  2 x   .
mà m  4040; 3 nên m  x 2  2 x  1 x   .
1 1
 x3  1
 
Suy ra S   x  2 x  1  m dx    x 2  x  mx    m .
2

0  3 0 3
1 6062
Khi đó: S  2021   m  2021  m   .
3 3
Vì m  4040; 3 , m   nên có 2018 số nguyên m .

Trang 32 Fanpage Nguyễn Bảo Vương  https://www.facebook.com/tracnghiemtoanthpt489/


Điện thoại: 0946798489 TUYỂN CHỌN VẬN DỤNG – VẬN DỤNG CAO 2022
Câu 56. (THPT Kinh Môn - Hải Dương - 2022) Cho f  x   ax 3  bx 2  cx  d  a  0  là hàm số nhận
giá trị không âm trên đoạn  2;3 có đồ thị y  f   x  như hình vẽ

Biết diện tích hình giới hạn bởi các đồ thị của các hàm g  x   xf 2  x  ; h  x    x 2 f  x  f   x  và
các đường x  2; x  3 bằng 72. Tính f 1 ?
62
A. f 1  2 B. f 1  1 C. f 1  1 D. f 1  
5
Lời giải
Chọn A
+) Ta có f   x   3ax 2  2bx  c  a  0 
 f  0  0  c  0 c  0
 
+) Từ đồ thị có  f   2   0  12a  4b  0  a  1 . Suy ra f  x   x 3  3 x 2  d .
  b  3
 f 1  3  3a  2b  c  3 
+) Diện tích
3 3 3
2 2
 g  x   h  x  dx   xf  x   x f  x  f   x dx   xf  x   f  x   xf   x  dx
2 2 2
3
1 2
  xf  x  d  xf  x     xf  x   |32  72
2
2
1 1 2 2
 . 9 f 2  3  4. f 2  2    9  27  3.9  d   4  8  3.4  d    72
2 2  
d  4
 5d  32d  208  0  
2
 d   52
 5
Vì f  x  là hàm số nhận giá trị không âm trên đoạn  2;3 nên f  x   x 3  3 x 2  4
Vậy f 1  2
Câu 57. (THPT Lương Tài 2 - Bắc Ninh - 2022) Cho hàm số f  x   x3  bx 2  cx  d với b,c,d là các
số thực. Biết hàm số g  x   f  x   2 f   x   3 f   x  có hai giá trị cực trị là 6 và 42 . Tính diện
f  x   f   x   f   x 
tích hình phẳng giới hạn bởi các đường y  và y  1 .
g  x   18
A. ln 5 . B. ln 7 . C. 2ln 6 . D. 2 ln 5 .
Lời giải
Chọn A
Hàm f  x  là hàm số bậc ba nên g  x  là hàm số bậc 3  g   x  là hàm số bậc 2.
Ta có: f '  x   6
g   x   f   x   2 f   x   18 có hai nghiệm là x1 ,x2  x1  x2  và g  x1   42,g  x2   6
Facebook Nguyễn Vương https://www.facebook.com/phong.baovuongTrang 33
Blog: Nguyễn Bảo Vương: https://www.nbv.edu.vn/
Xét phương trình tìm cận của tích phân để tính diện tích:
f  x   f   x   f   x  f   x   2 f   x   18
1 0
g  x   18 g  x   18
 x  x1
 f   x   2 f   x   18  0  g   x   0  
 x  x2
x2 f  x   f   x   f   x  x2 g x x2 g x
Diện tích hình phẳng S    1 dx   dx   dx
x1 g  x   18  
x1 g x  18 x1 g  x   18
x2
 ln g  x   18  ln12  ln 60  ln 5 .
x1

Câu 58. (THPT Lương Tài 2 - Bắc Ninh - 2022) Cho hàm số y  f  x  liên tục trên  có đồ thị như
hình vẽ. Giả sử diện tích phần kẻ sọc trên hình vẽ có diện tích bằng a . Tính theo a giá trị của tích
2
phân I    2 x  1 f   x  dx ?
3

A. I  50  2 a . B. I  50  a . C. I   30  2 a . D. I   30  2 a .
Lời giải
Chọn A
u  2 x  1  du  2dx
Đặt   .
 dv  f   x  dx v  f  x 
2
2
 I   2 x  1 f  x    2  f  x  dx  5 f  2   5 f  3  2a  5.2  5.8  2a  50  2a .
3
3

 2 x  a khi x 1
Câu 59. (THPT Lương Tài 2 - Bắc Ninh - 2022) Cho hàm số f ( x)   2 thoả mãn
3x  b khi x 1
2

 f ( x) dx  13 . Tính T  a  b  ab ?
0

A. T  11 . B. T  5 . C. T  1 . D. T  1 .
Lời giải
Chọn A
Nhận thấy hàm số phải liên tục tại x  1 . Khi đó:
lim f ( x)  lim f ( x)  f (1)  2  a  3  b  a  b  1 .
x 1 x 1
Ta có:
2 1 2 1 2
1 2
13   f ( x ) dx   f ( x ) dx   f ( x ) dx    3 x 2  b  dx    2 x  a  dx   x 3  bx    x 2  ax 
0 1
0 0 1 0 1

  b  1   a  3  a  b  4  a  b  9 .

Trang 34 Fanpage Nguyễn Bảo Vương  https://www.facebook.com/tracnghiemtoanthpt489/


Điện thoại: 0946798489 TUYỂN CHỌN VẬN DỤNG – VẬN DỤNG CAO 2022
a  b  1 a  5
Như vậy   suy ra T  a  b  ab  5  4  5.4  11 .
a  b  9 b  4
Câu 60. (THPT Võ Nguyên Giáp - Quảng Bình - 2022) Cho hàm số y  f  x  liên tục và nhận giá trị
2
không âm trên  1;2 và thoả mãn f  x   f 1  x  , x   1;2 . Đặt S1   xf  x  dx , S2 là diện
1

tích hình phẳng được giới hạn bởi đồ thị hàm số y  f  x  , trục Ox và hai đường thẳng
x  1; x  2 . Khẳng định nào dưới đây là đúng?
A. S1  2S2 . B. S1  3S2 . C. 2S1  S2 . D. 3S1  S2 .

Lời giải
Chọn C
Có S2 là diện tích hình phẳng được giới hạn bởi đồ thị hàm số y  f  x  , trục Ox và hai đường
2
thẳng x  1; x  2  S 2   f  x  dx .
1

2
Xét S1   xf  x  dx .
1

Đặt x  1  t  dx  dt . Đổi cận x  1  t  2 ; x  2  t  1 .


2 2 2 2
Ta có: S1   1  t  f 1  t  dt   1  t  f  t  dt   f  t  dt   t. f  t  dt  S
1 1 1 1
2  S1 .

Do đó, 2S1  S2 .

y  f  x  0;   thỏa mãn
Câu 61. (THPT Yên Lạc - Vĩnh Phúc - 2022) Cho hàm số liên tục trên
2 x. f   x   f  x   3 x 2 x , x   0;   1 f  4
. Biết f 1  , tính .
2
A. 14 . B. 4 . C. 24 . D. 16 .
Lời giải
Chọn D
2 x. f   x   f  x   3 x 2 x , x   0;  
1 3x 2
 x. f   x   f  x  , x   0;  
2 x 2
 3x 2
 
 x. f   x   x . f  x  
2
, x   0;  
2
 3x
  x . f  x    , x   0;  
2
3x 2 x3 x3
 x. f  x    dx   C  x . f  x    C *
2 2 2
1 1 1 1 x3
Thay x  1 vào * ta được: f 1   C  C  f 1     0  f  x  
2 2 2 2 2 x
Vậy f  4   16 .

Theo dõi Fanpage: Nguyễn Bảo Vương  https://www.facebook.com/tracnghiemtoanthpt489/

Facebook Nguyễn Vương https://www.facebook.com/phong.baovuongTrang 35


Blog: Nguyễn Bảo Vương: https://www.nbv.edu.vn/
Hoặc Facebook: Nguyễn Vương  https://www.facebook.com/phong.baovuong

Tham gia ngay: Nhóm Nguyễn Bào Vương (TÀI LIỆU TOÁN)  https://www.facebook.com/groups/703546230477890/

Ấn sub kênh Youtube: Nguyễn Vương


 https://www.youtube.com/channel/UCQ4u2J5gIEI1iRUbT3nwJfA?view_as=subscriber

Tải nhiều tài liệu hơn tại: https://www.nbv.edu.vn/

Trang 36 Fanpage Nguyễn Bảo Vương  https://www.facebook.com/tracnghiemtoanthpt489/


TUYỂN CHỌN VẬN DỤNG – VẬN DỤNG CAO 2022 Điện thoại: 0946798489

VẤN ĐỀ 4. SỐ PHỨC
• |FanPage: Nguyễn Bảo Vương
• TUYỂN CHỌN CÂU HỎI VD-VDC TỪ CÁC ĐỀ THI THỬ CÁC TRƯỜNG, CÁC SỞ NĂM 2022
Câu 1. (Chuyên Vinh – 2022) Xét các số phức z và w thỏa mãn | z || w | 1 và | z  w | 2 . Giá trị
nhỏ nhất của biểu thức P | zw  2i ( z  w)  4 | bằng
3 2
A. .
2
1 5 2
B. .
4
C. 5  2 2 .
D. 5 .
Câu 2. (Chuyên Vinh – 2022) Gọi S là tập hợp tất cả các số phức z thỏa mãn điều kiện z  z | z  z | .
Xét các số phức z1 , z2  S sao cho z1  z2  1 . Giá trị nhỏ nhất của biểu thức
P  z1  3i  z2  3i bằng
A. 2.
B. 1  3 .
C. 2 3 .
D. 20  8 3 .
Câu 3. (Chuyên Vinh – 2022) Biết phương trình z 2  mz  m 2  2  0 ( m là tham số thực) có hai
nghiệm phức z1 , z2 . Gọi A, B , C lần lượt là điểm biểu diễn các số phức z1 , z2 và z0  i . Có bao
nhiêu giá trị của tham số m để diện tích tam giác ABC bằng 1 ?
A. 2.
B. 3.
C. 4.
D. 6.
Câu 4. (Chuyên Lê Quý Đôn - Điện Biên - 2022) Xét các số phức z  a  bi ( a, b   ) thỏa mãn
| z  3  2i | 5 . Tính P  a  b khi | z  3  3i |  | z  7  i | đạt giá trị lớn nhất.
A. 8 B. 6 C. 4 D. 10
Câu 5. (Chuyên Lê Quý Đôn - Điện Biên - 2022) Trong tập các số phức, phương trình
z 2  6 z  m  0, m   1 . Gọi m0 là một giá trị m để phương trình 1 có hai nghiệm phân biệt
z1 , z2 thoả mãn z1.z1  z2 .z2 . Hỏi trong khoảng  0; 20  có bao nhiêu giá trị m0   ?
A. 10 B. 12 C. 11 D. 13
Câu 6. (Đại học Hồng Đức – 2022) Cho số phức z thoả mãn iz.z  (1  2i ) z  (1  2i ) z  4i  0 . Giá trị
lớn nhất của P  z  1  2 i  z  4  i gần số nào nhất sau đây?
A. 7,4.
B. 4,6.
C. 4,2.
D. 7,7.
Câu 7. (Đại học Hồng Đức – 2022) Trên tập hợp các số phức, xét phương trình z 2  2mz  6m  5  0( m
là tham số thực). Có bao nhiêu giá trị nguyên của m để phương trình đó có hai nghiệm phân biệt
z1 , z2 thỏa mãn z1  z1  z2  z2 ?
A. 5.
B. 3.
C. 6.
D. 4.
Facebook Nguyễn Vương https://www.facebook.com/phong.baovuong Trang 1
Blog: Nguyễn Bảo Vương: https://www.nbv.edu.vn/
Câu 8. (THPT Lê Thánh Tông - HCM-2022) Xét hai số phức z1 , z2 thỏa mãn các điều kiện
z1  2, z2  3, z1  z2  5 . Giá trị nhỏ nhất của biểu thức P  3z1  z2  10  5i  2 bằng

B. 10 3  2 5 . B. 3 5  1 . C. 2  2 5 . D. 8  2 5 .
Câu 9. (THPT Lê Thánh Tông - HCM-2022) Có tất cả bao nhiêu số phức w thỏa mãn điều kiện
w
2 ww  1 và là số thuần ảo?
w2
A. 4 . B. 6 . C. 3 . D. 2 .
Câu 10. (THPT Nguyễn Tất Thành-Đh-SP-HN-2022) Cho hai số phức z và w thay đổi thỏa mãn các
điều kiện z  1  i  z và w  3  4i  1 . Tìm giá trị nhỏ nhất của biểu thức P  z  w  1  i .
A. min P  3 2  1 . B. min P  3 2 . C. min P  5 2 . D. min P  5 2  1 .
Câu 11. (THPT Nguyễn Tất Thành-Đh-SP-HN-2022) Cho số phức z thay đổi thỏa mãn
z  1  3i  2 và số phức w  1  2i  z . Biết rằng tập hợp các điểm biểu diễn số phức w là một
đường tròn  C  trong mặt phẳng  Oxy  . Tìm bán kính R của đường tròn  C  .
A. R  5 . B. R  10 . C. R  6 . D. R  2 5 .
Câu 12. (THPT Nho Quan A – Ninh Bình – 2022) Gọi S là tập hợp tất cả các số phức z sao cho
iz.z  (1  2i ) z  (1  2i ) z  4i  0 và T là tập hợp tất cả các số phức w có phần thực khác 0 sao
w
cho là số thực. Xét các số phức z1 , z2  S và w  T thỏa mãn z1  z2  2 5 và
w  6i
w  z1 w  z1
 . Khi w  z1  w  z1 đạt giá trị nhỏ nhất thì w  z1  w  z1 bằng
z2  z1 z2  z
A. 3 .
B. 2 3 .
C. 3 3 .
D. 4 3 .
Câu 13. (THPT Nho Quan A – Ninh Bình – 2022) Trên tập hợp các số phức, xét phương trình
z 2  2 mz  m  12  0 ( m là tham số thực). Có bao nhiêu giá trị nguyên của m để phương trình
đó có hai nghiệm phân biệt z1 , z2 thỏa mãn z1  z2  2 z1  z2 ?
A. 1.
B. 2.
C. 3.
D. 4.
Câu 14. (THPT Phù Cừ - Hưng Yên - 2022) Trên tập hợp các số phức, xét phương trình
z 2  2mz  3m  10  0 ( m là tham số thực). Có bao nhiêu giá trị nguyên của m để phương trình
đó có hai nghiệm z1 , z2 không phải số thực thỏa mãn z1  z2  8 ?
A. 1 B. 2. C. 3. D. 4.
Câu 15. (THPT Phù Cừ - Hưng Yên - 2022) Cho số phức z và số phức w  ( z  i )( z  i )  2 z  3i thỏa
mãn w  i 2022  i 2023  w  1  0 . Giá trị lớn nhất của biểu thức T | z  3  i |2  | z  1  3i |2 bằng
m  n 5 với m, n   . Tính P  m.n .
A. P  124 . B. P  876 . C. P  416 . D. P  104 .
Câu 16. (Sở Bắc Giang 2022) Giả sử z1 ; z2 là hai trong các số phức z thỏa mãn ( z  6)(8  i.z ) là số
thực. Biết rằng z1  z2  6 . Giá trị nhỏ nhất của z1  3z2 bằng
A. 5  73 .
B. 5  21 .
C. 20  2 73 .

Trang 2 Fanpage Nguyễn Bảo Vương  https://www.facebook.com/tracnghiemtoanthpt489/


Điện thoại: 0946798489 TUYỂN CHỌN VẬN DỤNG – VẬN DỤNG CAO 2022
D. 20  4 21 .
Câu 17. (Sở Bạc Liêu 2022) Cho các số phức z , w thỏa mãn z  2 , w  3  2i  1 khi đó z 2  2 zw  4
đạt giá trị lớn nhất bằng
A. 16 . B. 24 . C. 4  4 13 . D. 20 .
Câu 18. (Sở Hà Tĩnh 2022) Cho số phức z  x  yi,  x, y    thỏa mãn z  z  2  3 z  z  4i  6 và
z  1  i  z  3  i . Gọi M , m là giá trị lớn nhất và nhỏ nhất của biểu thức P  2 x  3 y  5 . Khi
đó M  m bằng:
17 33 13 22
A. . B. . C.  . D. .
5 5 5 5
Câu 19. (Chuyên Hoàng Văn Thụ - Hòa Bình – 2022) Trên tập hợp số phức, xét phương trình
1
z 2  m  1z   m 2  5m  6   0( m là tham số thực). Có bao nhiêu số nguyên m  [ 10;10] đề
4
phương trình trên có hai nghiệm phức z1 , z2 thỏa mãn z1  z2  z1  z2 ?
A. 11.
B. 10.
C. 8.
D. 9.
Câu 20. (Chuyên Lam Sơn 2022) Cho M , N , P lần lượt là các điểm biểu diễn số phức z1 , z2 , z3 thỏa mãn
điều kiện 5 z1  9  3i  5 z1 , z2  2  z2  3  i , z3  1  z3  3  4 . Khi M , N , P không thẳng
hàng, giá trị nhỏ nhất của nửa chu vi p của tam giác MNP là
10 5
A. .
9
6 5
B. .
5
9 10
C. .
10
5 11
D. .
13
Câu 21. (THPT Kinh Môn - Hải Dương - 2022) Cho hai số phức z , w thỏa mãn z  w  10 ,
2 z  w  17 và z  3w  146 . Tính giá trị của biểu thức P  z.w  z.w .
A. P  14 . B. P  14 . C. P  16 . D. P  8 .

Theo dõi Fanpage: Nguyễn Bảo Vương  https://www.facebook.com/tracnghiemtoanthpt489/

Hoặc Facebook: Nguyễn Vương  https://www.facebook.com/phong.baovuong

Tham gia ngay: Nhóm Nguyễn Bào Vương (TÀI LIỆU TOÁN)  https://www.facebook.com/groups/703546230477890/

Ấn sub kênh Youtube: Nguyễn Vương


 https://www.youtube.com/channel/UCQ4u2J5gIEI1iRUbT3nwJfA?view_as=subscriber

Tải nhiều tài liệu hơn tại: https://www.nbv.edu.vn/

Facebook Nguyễn Vương https://www.facebook.com/phong.baovuongTrang 3


TUYỂN CHỌN VẬN DỤNG – VẬN DỤNG CAO 2022 Điện thoại: 0946798489

VẤN ĐỀ 5. HHKG-THỂ TÍCH KHỐI ĐA DIỆN


• |FanPage: Nguyễn Bảo Vương
• TUYỂN CHỌN CÂU HỎI VD-VDC TỪ CÁC ĐỀ THI THỬ CÁC TRƯỜNG, CÁC SỞ NĂM 2022
Câu 1. (Chuyên Lê Quý Đôn - Điện Biên - 2022) Cho hình chóp S.ABCD có đáy ABCD là hình bình
hành và M là trung điểm cạnh bên SC . Gọi  P  là mặt phẳng chứa AM và song song với BD ,
VS . ABMD
mặt phẳng  P  cắt SB, SD lần lượt tại B  và D . Tính tỷ số
VS . ABCD
1 1 3 2
A. . B. . C. . D. .
6 3 4 3
Lời giải
Chọn B
S

B'
I

D'
B C

A D

Trong  ABCD  gọi O là giao điểm của AC và BD .


Trong  SAC  gọi I là giao điểm của SO và AM .
Trong  SBD  từ I vẽ đường thẳng song song với BD cắt SB , SD lần lượt tại B  , D , suy ra
mặt phẳng  P  là mặt phẳng  ABMD  .
+ Ta thấy I là giao điểm của hai đường trung tuyến AM và SO của tam giác SAC  I là
SI SB SD 2
trọng tâm tam giác SAC , suy ra:    (định lý Ta-lét vì B D// BD )
SO SB SD 3
V SA.SB.SM 2 1 1 1
Ta có: SABM   .   VSABM  VSABC .
VSABC SA.SB.SC 3 2 3 3
VSADM SA.SD.SM 2 1 1 1
  .   VSADM  VSADC
VSADC SA.SD.SC 3 2 3 3
1 1
VSABMD VSABM  VSADM VSABC  VSADC  VSABCD 1
  3 3  .
VSABCD VSABCD VSABCD VSABCD 3
Câu 2. (Cụm Trường Nghệ An - 2022) Cho hình chóp S . ABCD có đáy ABCD là hình chữ nhật,
AB  a, AD  a 3 . Mặt bên SAB là tam giác đều và nằm trong mặt phẳng vuông góc với mặt
đáy. Cosin của góc giữa đường thẳng SD và mặt phẳng  SBC  bằng:
Facebook Nguyễn Vương https://www.facebook.com/phong.baovuong Trang 1
Blog: Nguyễn Bảo Vương: https://www.nbv.edu.vn/
2 5 13 1 3
A. . B. . C. . D. .
5 4 4 4
Lời giải
Chọn B
S

A D

H α

B C (SBC) S

d  D ,  SBC  
Gọi  là góc giữa đường thẳng SD và mặt phẳng  SBC  , khi đó: sin   , với
SD
0    90 .
a 13
Trong AHD vuông tại A có: HD  AD 2  AH 2  .
2
AB 3 a 3
Trong  SAB đều có: SH   .
2 2
Trong SHD vuông tại H có: SD  SH 2  HD 2  2a .
Trong  SAB  , dựng HK  SB .
 BC  AB
Ta có:  ,  BC   SBC   BC  HK , mà HK  SB , nên HK   SBC  .
 BC  SH
AB
Ta có: d  D,  SBC    d  A,  SBC    .d  H ,  SBC    2 HK .
HB
1 1 1 a 3
Trong AHB vuông tại H có: 2
 2
 2
 HK  .
HK SH BH 4
2 HK 3 13
Do đó: sin    . Vì 0    90 , nên cos   1  sin 2   .
SD 4 4
 13
Vậy cos  SD ,  SBC    .
4
Câu 3. (Cụm Trường Nghệ An - 2022) Cho lăng trụ ABC. ABC  có thể tích bằng 2. Gọi M , N lần lượt là
2
hai điểm nằm trên hai cạnh AA và BB  sao cho M là trung điểm AA và BN  BB . Đường
3
thẳng CM cắt đường thẳng AC  tại P và đường thẳng CN cắt đường thẳng BC tại Q . Biết
a
thể tích khối đa diện lồi AMPB NQ bằng với a, b  ; a, b nguyên tố cùng nhau. Tính
b
a  2b .
A. 14 . B. 31 . C. 41 . D. 32 .
Lời giải
Chọn C

Trang 2 Fanpage Nguyễn Bảo Vương  https://www.facebook.com/tracnghiemtoanthpt489/


Điện thoại: 0946798489 TUYỂN CHỌN VẬN DỤNG – VẬN DỤNG CAO 2022

Ta có:
PA AM 1 C A 1 QB BN 2 C B 1
  hay  ;   hay  .
PC CC 2 C P 2 QC  C C 3 C Q 3
VABC .MNC 1  AM BN C C  1  1 2  13 13
         1   VABC.MNC  VABC. ABC .
VABC . ABC 3  AA BB C C  3  2 3  18 18
1
VC. ABC  VABC. ABC .
3
VC .C PQ S  C P C Q
 C PQ  .  2.3  6  VC .C PQ  6VC .C AB  2VABC . ABC .
VC .C AB SC AB C A C B
 13  23 23
Suy ra: VAMPBNQ  VC .C PQ  VABC.MNC   2   VABC . ABC  .2  .
 18  18 9
Khi đó: a  23, b  9  a  2b  41 .
Câu 4. (Cụm Trường Nghệ An - 2022) Cho tứ diện ABCD có AC  2CD  DB  2 a . Gọi H và K lần
lượt là hình chiếu vuông góc
của A và B lên đường thẳng CD sao cho H , C , D , K theo thứ tự cách đều nhau. Biết góc tạo bởi
AH và BK bằng 60 . Thể tích khối tứ diện ABCD bằng
a3 3 a3 3 a3 3 a3 3
A. . B. . C. . D. .
6 8 3 4
Lời giải
Chọn D

 Có HC  CD  DK  a; AC  2a; BD  2a .
Facebook Nguyễn Vương https://www.facebook.com/phong.baovuongTrang 3
Blog: Nguyễn Bảo Vương: https://www.nbv.edu.vn/
Tam giác AHC vuông tại H nên AH  AC 2  HC 2  4a 2  a 2  a 3
Tam giác BKD vuông tại K nên BK  BD 2  HK 2  4a 2  a 2  a 3.
Tứ diện ABKH có:
cặp cạnh đối AH  BK  a 3 ,  AH ; BK   60 ,và d  AH ; BK   HK  3a .

1 1 3 3 3 3
Suy ra VABKH  AH .BK .d ( AH , BK ).sin( AH , BK )  .a 3.a 3.3a.  a.
6 6 2 4
1 1 3a3 3 a3 3
 Dễ thấy VABCD  VABKH  .  .
3 3 4 4
Câu 5. (THPT Hồ Nghinh – Quảng Nam – 2022) Cho khối hộp chữ nhật ABCD  A BC D . Khoảng cách
2 2
giữa 2 đường thẳng AB , CB bằng a , khoảng cách giữa 2 đường thẳng A D, B A bằng a.
5 5
2
Khoảng cách giữa 2 đường thẳng BD , AC bằng a . Tính thể tích khối hộp chữ nhật đã cho.
6
A. a 3 .
a3
B. .
2
C. 2a 3 .
D. 2a3 .
Lời giải
Chọn C

Giải sử các kích thức của hình hộp chữ nhật là AB  x, AD  y , AA  z với x, y , z  0 .
2a 5
+) Khoảng cảch giữa hai đường thẳng AB và BC bằng .
5
 AB / /CD

Ta có CD   A BCD   AB / /  ABCD   d  AB; BC   d  AB;  A BCD  

 AB   A BCD 
2a 5
 d  A,  ABCD    AH  , với H là hình chiếu của A trên A D .
5
1 1 1 1 1 5
Từ 2
 2
 2
 2  2  2 (1)
AH AA AD y z 4a
2a 5
+) Khoảng cách giữa hai đường thẳng A D  và AB bằng .
5
Tương tự, ta chứng minh được A D / /  ABC  D   d  A D; AB   d  A D,  ABC  D  
2a 5
 A K  với K là hình chiếu của A trên AB .
5
Trang 4 Fanpage Nguyễn Bảo Vương  https://www.facebook.com/tracnghiemtoanthpt489/
Điện thoại: 0946798489 TUYỂN CHỌN VẬN DỤNG – VẬN DỤNG CAO 2022
1 1 1 1 1 5
Từ 2
 2
 2
 2  2  2 (2)
A K A A A B x z 4a
a 3
+) Khoảng cách giữa hai đường thẳng AC và BD bằng .
3
Gọi {O}  AC  BD  O là trung điểm của BD .
Gọi I là trung điểm của DD thì OI là đường trung bình của
BDD  OI / / BD  BD / /( ACI ) .
 d  BD; AC   d  BD; ( ACI )   d  D;( ACI )   d ( D;( ACI )).
Ta thấy DI , DA, DC đôi một vuông góc với nhau nên
1 1 1 1 1 1 4 1 1 4 3
2
 2
 2
 2
 2
 2
 2
 2  2  2  2 (3)
d ( D, ( ACI )) DA DC DI DA DC DD x y z a
1 1 5 1 1
 y 2  z 2  4a 2  x2  a2
  x  a
1 1 5 1 1 
Từ (1),(2),(3) ta có hệ  2  2  2   2  2  y  a
x z 4a y a 
1 1 4 3 1 1  z  2a
 x2  y 2  z 2  a 2  2  2
 z 4a
3
Vậy thể tích khối hộp là V  xyz  a  a  2a  2a .
Câu 6. (THPT Hương Sơn - Hà Tĩnh - 2022) Cho tứ diện đều ABCD có tất cả các cạnh bằng 1. Gọi M
là điểm thuộc cạnh BC sao cho MC  2MB ; N , P lần lượt là trung điểm của BD và AD . Gọi Q
là giao điểm của AC và  MNP  . Thể tích khối đa diện ABMNPQ bằng
7 2 13 2 2 11 2
A. . B. . C. . D. .
216 432 36 432
Lời giải
Chọn B

Gọi I  MN  CD suy ra Q  IP  AC . Gọi K là trung điểm của MC . Dễ thấy MN , DK lần


1 1 1 1 IN 3
lượt là đường trung bình BDK , CMI nên MN  DK  . IM  IM   .
2 2 2 4 IM 4
IP IN 3
Ta có MQ / / PN    .
IQ IM 4
V ID IN IP 1 3 3 9
Ta có: IDNP  . .  . . 
VICMQ IC IM IQ 2 4 4 32
23 23 23 SCMQ 23 CM CQ
 VDNPCMQ  VICMQ  .2.VDCMQ  .2. .VABCD  .2. . .VABCD
32 32 32 SCAB 32 CB CA
23 2 2 23 13 13 2 13 2
 .2. . .VABCD  .VABCD  VABMNPQ  .VABCD  .  .
32 3 3 36 36 36 12 432

Facebook Nguyễn Vương https://www.facebook.com/phong.baovuongTrang 5


Blog: Nguyễn Bảo Vương: https://www.nbv.edu.vn/
Câu 7. (THPT Lê Thánh Tông - HCM-2022) Hình lập phương ABCD. A1B1C1D1 có cạnh bằng 6. Gọi
M , N lần lượt là trung điểm của cạnh B1C1 , CD và O, O1 lần lượt là tâm các hình vuông
ABCD, A1 B1C1 D1 . Thể tích khối tứ diện MNOO1 bằng
A. 9. B. 12. C. 18. D. 27.
Lời giải
Chọn A
A1 D1

O1

B1 M C1

A D

O N

B C

Ta có: OO1   A1 B1C1 D1   OO1  O1M ; OO1   ABCD   OO1  ON .


Mặt khác: O1M  B1C1
Mà B1C1 // BC // ON , nên ON  O1M .
O M  OO1
Ta có  1 ,  O1M   O1ON  .
O1M  ON
BC 1 AB
Ta có: OO1  AA1  6 , ON   3 , nên SO1ON  OO1.ON  9 . Ta có: O1M  1 1  3
2 2 2
1
Vậy thể tích khối chóp MNOO1 là V  O1M .SO1ON  9 .
3
Câu 8. (THPT Lê Thánh Tông - HCM-2022) Cho lăng trụ ABC . AB C  có tam giác ABC vuông cân tại
A . Hình chiếu vuông góc của A lên mặt đáy trùng với trung điểm của cạnh BC . Biết cạnh
AA  a 3 và tạo với mặt đáy của hình lăng trụ một góc bằng 60 . Khoảng cách từ đỉnh C  đến
mặt  ABC  bằng
3a a 3 a 2a
A. . B. . C. . D. .
4 2 2 3
Lời giải
Chọn B

Trang 6 Fanpage Nguyễn Bảo Vương  https://www.facebook.com/tracnghiemtoanthpt489/


Điện thoại: 0946798489 TUYỂN CHỌN VẬN DỤNG – VẬN DỤNG CAO 2022
B'
C'

A'

H
B C

A
Gọi H là hình chiếu vuông góc của điểm A lên mặt phẳng  ABC  .
Ta có: AH là hình chiếu vuông góc của AA trên mặt phẳng  ABC   Góc giữa AA ' và mặt
phẳng  ABC  là 
AAH  
AAH  60 .
Gọi I là tâm của hình bình hành ACC A .
d  C ,  ABC   C I
  1  d  C ,  ABC    d  A,  ABC   .
d  A,  ABC   AI
AH  BC 
Ta có:   AH   ABC   d  A,  ABC    AH .
AH  A ' H 
a 3
Xét tam giác A ' HA : AH  AA.cos 60  .
2
Câu 9. (Liên trường Hà Tĩnh – 2022) Cho hình chóp tứ giác S . ABCD có đáy là hình vuông; mặt bên
(SAB) là tam giác vuông cân tại S và nằm trong mặt phẳng vuông góc với đáy. Biết khoảng cách
3 5a
giữa hai đường thẳng AB và SD bằng . Tính thể tích V của khối chóp S . ABCD .
5
3 3
A. V  a .
2
6 3 3
B. V  a .
2
27 3
C. V  a .
2
9
D. V  a 3 .
2
Lời giải
Chọn D

Facebook Nguyễn Vương https://www.facebook.com/phong.baovuongTrang 7


Blog: Nguyễn Bảo Vương: https://www.nbv.edu.vn/

Gọi I ; J lần lượt là trung điểm của AB; CD; K là hình chiếu của I lên SJ
x
Đặt cạnh đáy bằng AB  x khi đó SI  , IJ  x .
2
Vì AB / / CD nên AB‖( SCD)  d ( AB, SD)  d ( I , ( SCD))  IK . Suy ra
IS .IJ
d ( I ;( SCD))  IK 
IS 2  IJ 2
x
x
3a 5 2  x  3a
 
5 x2
x2 
4
1 x 2 9a 3
Từ đó suy ra V  x  .
32 2
Câu 10. (THPT Nguyễn Tất Thành-Đh-SP-HN) Cho hình lăng trụ đứng ABC . AB C  có đáy ABC là
tam giác vuông tại A , AB  a , AC  2 a và AA  a (tham khảo hình vẽ bên). Tính khoảng cách
d giữa hai đường thẳng AB, AC .

2a 3a 2a 3a
A. d  . B. d  . C. d  . D. d  .
3 2 3 2
Lời giải
Chọn A

Trang 8 Fanpage Nguyễn Bảo Vương  https://www.facebook.com/tracnghiemtoanthpt489/


Điện thoại: 0946798489 TUYỂN CHỌN VẬN DỤNG – VẬN DỤNG CAO 2022

Không mất tính tổng quát, ta chọn hệ trục tọa độ  Oxyz  , như hình vẽ.
Chọn a  1 , khi đó A  0;0;0  , B  0;1;0  , A  0;0;1 , C  2;0;1 .
  
Ta có: AB   0;1;  1 ; AC   2;0;1 , AA   0;0;1 .
    
Ta có:  AB, AC   1;  2;  2    AB, AC  . AA  2 .

Khoảng cách d giữa hai đường thẳng AB  , AC là:


  
 AB, AC  . AA 2
  2
d     .
 AB, AC  2 2
1   2    2 
2 3
 
Câu 11. (THPT Nguyễn Tất Thành-Đh-SP-HN) Cho hình lập phương ABCD. ABCD có cạnh a . Gọi
M , N và E lần lượt là trung điểm các cạnh AA, C D và CC (tham khảo hình vẽ bên). Tính thể
tích V của khối tứ diện BMNE .

a3 a3 a3 a3
A. V  . B. V  . C. V  . D. V  .
24 6 8 12
Lời giải
Chọn A

Facebook Nguyễn Vương https://www.facebook.com/phong.baovuongTrang 9


Blog: Nguyễn Bảo Vương: https://www.nbv.edu.vn/

Dựng hệ trục tọa độ Bxyz như hình vẽ và để làm việc dễ hơn ta chọn a  1 thì ta có:
 1 1   1
B  0;0;1 , M  1;0;  , N  ;1;0  , E  0;1; 
 2 2   2
  1  1  
  1
 BM  1;0;   , BN   ;1; 1 , BE   0;1;  
 2 2   2
1    1
 VBMNE   BM , BN  .BE 
6 24
3
a
Vậy VBMNE  .
24
Câu 12. (Sở Hà Tĩnh 2022) Cho hình hộp đứng ABCD  A BCD có cạnh AA  2 , đáy ABCD là hình
thoi với ABC là tam giác đều cạnh bằng 4. Gọi M , N , P lần lượt là trung điểm của
BC , C  D, DD và Q thuộc cạnh BC sao cho QC  3QB . Thể tích của khối tứ diện MNPQ
bằng
A. 3 3 .
3 3
B. .
2
3
C. .
4
3
D. .
2
Lời giải
Gọi O  AC  BD; O  AC   B D . Chọn hệ trục toạ độ Oxyz sao cho các tia Ox, Oy , Oz lần
  
lượt trùng với các tia OB, OC , OO

Trang 10 Fanpage Nguyễn Bảo Vương  https://www.facebook.com/tracnghiemtoanthpt489/


Điện thoại: 0946798489 TUYỂN CHỌN VẬN DỤNG – VẬN DỤNG CAO 2022

khi đó toạ độ các điểm là


O (0;0; 0), B(2 3;0;0), C (0; 2;0), D (2 3;0;0), O(0;0; 2), B(2 3;0; 2), C (0; 2; 2), D(2 3;0; 2)

3 3 1  1    1 3
M ( 3;1; 2), N ( 3;1; 2), P (2 3;0;1), Q  ; ;0   VMNPQ  | [ MN , MP ]  MQ |  3 3 
 2 2  6 6 2

Câu 13. (Sở Thanh Hóa 2022) Cho khối chóp S. ABCD với đáy ABCD là hình bình hành có thể tích
bằng 84a3 . Gọi M là trung điểm AB; J thuộc cạnh SC sao cho JC  2 JS ; H thuộc cạnh SD
sao cho HD  6 HS . Mặt phẳng ( MHJ ) chia khối chóp thành hai phần. Thể tích khối đạ diện của
phần chứa đỉnh S bằng
A. 17a 3 .
B. 19a 3 .
C. 24a 3 .
D. 21a 3 .
Lời giải
Gọi P  HJ  CD; N  MP  BC ; Q  MP  AD; E  QH  SA khi đó khối đa diện chứa đỉnh S
là SBMEHJN .

Định lý Menelaus là một định lý cơ bản trong hình học tam giác, được phát biểu như sau: Cho tam
giác ABC . Các điểm D , E , F lần lượt nằm trên các đường thẳng BC , CA, AB . Khi đó D, E , F
DB EC FA
thẳng hàng thì   1.
DC EA FB

Facebook Nguyễn Vương https://www.facebook.com/phong.baovuongTrang 11


Blog: Nguyễn Bảo Vương: https://www.nbv.edu.vn/

 NB BM
 1
PC HD JS PC 1 PC 1  NC PC
Áp dụng định lí trên có:   1 6 1   .
PD HS JC PD 2 PD 3  QA AM 1
 
 QD PD 3
ES QA HD ES 1 ES 1
Tương tự   1  6 1  . Ta có
EA QD HS EA 3 EA 2
VSBMEHJN  VS .BMN  VS .MEHJN  VS . BMN  VS .MEH  VS .MHJ  VS .MNJ  .
S BMN 1
VS . BMN  VS . ABCD  VS . ABCD
S ABCD 8
SE SH 1 1 S 1 1 1 1
VS .MEH   VS .MAD    MAD VS . ABCD    VS . ABCD  VS . ABCD
SA SD 3 7 S ABCD 3 7 4 84
SH SJ 1 1 S 1 1 1 1
VS .MHJ   VS .MDC    MDC VS . ABCD    VS . ABCD  VS . ABCD
SD SC 7 3 S ABCD 7 3 2 42
SJ 1 S 1 1 1
VS .MNJ  VS .MNC   MNC VS . ABCD   VS . ABCD  VS . ABCD
SC 3 S ABCD 3 8 24
1 1 1 1  17
 VSBMEHJN       VS . ABCD   84a 3  17a 3 .
 8 84 42 24  84
Câu 14. (Sở Hà Tĩnh 2022) Cho lăng trụ ABCD  A BCD có đáy là hình chữ nhật với
AB  6, AD  3, AC  3 và mặt phẳng  AACC  vuông góc với mặt đáy. Biêt hai mặt phẳng
3
 AACC  và  AABB  tạo với nhau góc  có tan   . Thể tích V của khối lăng trụ
4
ABCD  A BC D là
A. 12.
B. 6.
C. 8.
D. 10.
Lời giải.
Dễ thấy
nên tam giác ACC cân tại A , do đó A F  CC , với F là trung điểm của CC . Gọi E là điểm
 3 
thỏa mãn C  E  C  D .
2

Trang 12 Fanpage Nguyễn Bảo Vương  https://www.facebook.com/tracnghiemtoanthpt489/


Điện thoại: 0946798489 TUYỂN CHỌN VẬN DỤNG – VẬN DỤNG CAO 2022

3 6 6
Khi đó C E  và DE  , suy ra
2 2
27
A E 2  AC 2  A D 2  D E 2  AC 2   CE 2
2
suy ra EA  A F và CC    EAF  , do đó
   A F , EF     AAC  A  ,  CDDC       AAC C  ,  AA B B    
EFA
3 2
Ta có EA  D E 2  AD2  , suy ra A F  A E cot   2 2 và CC   2 AC 2  A F 2  2 ,
2
do đó chiều cao của khối lăng trụ là
AF  CC  4 2
h  d  C ,  A BC  D    d  C , AC    
AC  3
Vậy V  AB  AD  h  8 .
Câu 15. (Sở Ninh Bình 2022) Cho hình chóp tứ giác đều S. ABCD . Một mặt cầu ( J ) ( J và S cùng
phía với ( ABCD ) ) tiếp xúc với ( ABCD ) tại A , đồng thời tiếp xúc ngoài với mặt cầu nội tiếp
hình chóp. Một mặt phẳng ( P ) đi qua J và BC . Gọi  là góc giữa ( P ) và ( ABCD ) . Tính
tan  biết các đường chéo của thiết diện của hình chóp cắt bởi ( P ) lần lượt cắt và vuông góc với
SA, SD .
1
A. .
4
6
B. .
6
3
C. .
6
1
D. .
2
Lời giải.
Gọi R , r lần lượt là bán kính mặt cầu tâm J và bán kính mặt cầu tâm I nội tiếp hình chóp tứ
giác đều. Đặt AB  a , SO  h , với O là tâm hình vuông ABCD . Khi đó do hai mặt cầu ( I ) và
( J ) tiếp xúc ngoài nên OA  2 Rr hay a 2  8 Rr . Gọi giao điểm của JC với SA và SO lần lượt
là E và H . Theo giả thiết thì CE  SA , suy ra hai tam giác HCO và ASO đồng dạng, suy ra

Facebook Nguyễn Vương https://www.facebook.com/phong.baovuongTrang 13


Blog: Nguyễn Bảo Vương: https://www.nbv.edu.vn/

OH OC OA2 4 Rr
  OH   .
OA OS h h
JA R 4 Rr R
Lại từ tính chất đường trung bình, ta có OH   nên  hay h  8r . Gọi N là trung
2 2 h 2
điểm AB . Sử dụng tính chất đường phân giác, ta có
2r OI SI SO h 2h
     .
a ON SN ON  SN a a 2
a  4 h 2
 a 2
 h2 
2 4
h
Thay r  , ta được
8
a 3
7a  4h 2  a 2  12a 2  h 2   .
h 6
Gọi M là trung điểm BC , dễ thấy BC  (OHM ) nên   (( P), ( ABC ))  OMH  , suy ra
OH 4 Rr 8 Rr a 2 3
tan      
OM h  a ah ah 6
2
Câu 16. (Sở Ninh Bình 2022) Cho hình chóp S. ABCD có đáy ABCD là hình bình hành, có thể tích là
V . Gọi M là trung điểm của cạnh SA, N là điểm trên cạnh SB sao cho SN  3NB . Mặt phẳng
( P ) thay đổi đi qua các điểm M , N và cắt các cạnh SC , SD lần lượt tại hai điểm phân biệt P, Q .
Tìm giá trị lớn nhất của thể tích khối chóp S .MNPQ .
V
A. .
3
27
B. V.
80
27
C. V .
40
V
D. .
6
Lời giải.
SC SD
Đặt  x,  y với x, y  1 . Vì hình chóp S. ABCD có đáy ABCD là hình bình hành nên
SP SQ
SA SC SB SD
   .
SM SP SN SQ

Trang 14 Fanpage Nguyễn Bảo Vương  https://www.facebook.com/tracnghiemtoanthpt489/


Điện thoại: 0946798489 TUYỂN CHỌN VẬN DỤNG – VẬN DỤNG CAO 2022

Suy ra
SC 4 SD 2
2    y x
SP 3 SQ 3
Mặt khác ta có
VS .MNPQ VS .MNP VS .MQP 1  SM SN SP SM SQ SP 
         
VS . ABCD 2VS . ABC 2VS . ADC 2  SA SB SC SA SD SC 
11 3 1 1 1 1 1 3 1 1 3 3 
             
2  2 4 x 2 y x  4 x  4 y  4 x  4 3x  2 
9( x  2)

16  3 x 2  2 x 
9( x  2)
Xét hàm số f ( x)  với x  1. Ta có
 
16 3 x 2  2 x
2
9 3 x  12 x  4
f ( x)    0, x  1
16  3 x 2  2 x 2
27
nên hàm số luôn nghịch biến trên nửa khoảng [1;  ) . Suy ra f ( x )  f (1)  , x  1 . Vậy thể
80
27
tích khối chóp S .MNPQ đạt giá trị lớn nhất bằng V , đạt được khi x  1 , tức là khi P  C .
80
Câu 17. (Sở Bạc Liêu 2022) Cho hình chóp S. ABCD có đáy ABCD là hình vuông, AB  1 , cạnh bên
SA  1 và vuông góc với mặt đáy  ABCD  . Kí hiệu M là điểm di động trên đoạn CD và N là
  45 . Thể tích nhỏ nhất của khối chóp S. AMN là
điểm di động trên đoạn CB và góc MAN
2 1 2 1 2 1 2 1
A. . B. . C. . D. .
3 9 6 9

Lời giải
Chọn A

Facebook Nguyễn Vương https://www.facebook.com/phong.baovuongTrang 15


Blog: Nguyễn Bảo Vương: https://www.nbv.edu.vn/
S

A B
45° x
M

D y N C

1
Ta có VS . AMN  SA.S AMN  VS . AMN nhỏ nhất  S AMN nhỏ nhất.
3
2 2
 BM  x  AM  1  x 2 2
Đặt  có  2 2
và MN 2  MC 2  NC 2  1  x   1  y  .
 DN  y  AN  1  y

Áp dụng định lí cosin

2
MN 2  AN 2  AM 2  2 AM . AN .cos 45  1  y 2  1  x 2  2 1  y 2  . 1  x 2  .
2

2 2 2 2
Nên 1  x   1  y   1  x 2  1  y 2  2 1  x 2  . 1  y 2   2  x  y   2 1  x  . 1  y 

 2  x  y  1  x  . 1  y   2  x
2 2 2
 2 xy  y 2   1  x 2  y 2  x 2 y 2

2 2
 x 2  2 xy  y 2  1  2 xy  x 2 y 2   x  y   1  xy   x  y  1  xy vì xy  1 .

2 2

Theo bất đẳng thức Cosi 2 xy  x  y  xy 


 x  y  1  x  y 
 x  y
4 4

2  x  y  2  2 2
  x  y  4 x  y  4  0   .
 x  y  2  2 2

Vậy min  x  y   2  2 2 .

1 1 2 2
Ta có S AMN 
2
AM . AN .sin 45 
2
1  x 1  y .
2 2

2

4
. 2  x  y

1
 SAMN 
2
 
2 2  2  SAMN  2  1 .

1 1 2 1
min SAMN  2  1  min VS . ABCD   SA.SAMN  .1.
3 3
 
2 1 
3
.

Câu 18. (Sở Phú Thọ 2022) Cho hình lăng trụ đứng ABCD. ABCD có đáy là hình vuông cạnh a , góc
 1 
giữa AC và mặt phẳng  ACD  bằng 30 . Gọi M là điểm sao cho AM  AB . Thể tích khối
3
tứ diện ACDM bằng
Trang 16 Fanpage Nguyễn Bảo Vương  https://www.facebook.com/tracnghiemtoanthpt489/
Điện thoại: 0946798489 TUYỂN CHỌN VẬN DỤNG – VẬN DỤNG CAO 2022
a3
a3
a3 3 a3 3
A. . B. . C. . D. .
18 3 12 3
Lời giải
Chọn A

Kẻ AE  AD

CD  AD 
CD  DD 

Ta có   CD   ADDA  , mà AE   ADDA   AE  CD .
AD, DD   ADDA  
AD  DD  D 

AE  CD 
AE  AD 

Suy ra   AE   ACD  .
CD, AD   ACD  
CD  AD  D 

Hình chiếu vuông góc của AC lên mặt phẳng  ACD  là EC .


AC ,  ACD    (
AC , EC )  
ACE  30 .

a 2
Xét tam giác ACE vuông ở E  AE  AC.sin 30 
2

Ta có chiều cao của hình chóp ACDM hạ từ đỉnh M


1 1 AE a 2
là h  d  M ,  ACD    d  B,  ACD    d  A,  ACD     .
3 3 3 6

Xét tam giác AAD vuông ở A có

1 1 1 1 1 1 1 1
AE  AD  2
 2
 2
 2
 2
 2
 2
 2  AA  a .
AE AA AD AA AE AD a 2 a
 
 2 

Facebook Nguyễn Vương https://www.facebook.com/phong.baovuongTrang 17


Blog: Nguyễn Bảo Vương: https://www.nbv.edu.vn/
1 1 a2 2
Ta có diện tích tam giác ACD bằng S ACD  . AD.DC  a 2.a  .
2 2 2

1 1 a 2 a2 2 a3
Thể tích khối tứ diện ACDM bằng VACDM   h  S ACD     .
3 3 6 2 18
Câu 19. (Sở Thái Nguyên 2022) Cho hình chóp tứ giác S.ABCD có đáy là nửa lục giác đều nội tiếp
3a
đường tròn đường kính AD  2a, SA   ABCD  , SA  . Khoảng cách giữa hai đường thẳng
2
BD và SC bằng
3 2a 2a 5 2a 5 2a
A. . B. . C. . D. .
4 4 12 4
Lời giải
Chọn B

Trong mặt phẳng  ABCD  , Kẻ Cx / / BD  BD / /  SCx 


 d  BD, SC   d  BD,  SCx    d  O,  SCF   ; với O  BD  AC và F  AB  Cx .
Vì ABCD là nửa lục giác đều nội tiếp đường tròn đường kính AD  2a nên
OA OD AD 1
AB  BC  CD  a và    2  d  O,  SCF    d  A,  SCF   .
OC OB BC 3
CF  SA  Do SA   ABCD  
Ta có:   CF   SAB  1 .
CF  AB  Do AB  BD, BD / /CF 
Trong mặt phẳng  SAF  , kẻ AH  SF  2  .
 SCF    SAB 

Từ 1 và  2  , ta có:  SCF    SAB   SF  AH   SCF   d  A,  SCF    AH .

 AH   SAB  , AH  SF
AB AO 2 3 3a
Xét tam giác AFC có OB / /CF nên    AF  AB  .
AF AC 3 2 2
Xét tam giác SAF vuông tại A nên
1 1 1 8 3 2a 1 2a
2
 2 2
 2  AH   d  BD, SC   AH  .
AH SA AF 9a 4 3 4
Câu 20. (Sở Thái Nguyên 2022) Cho hình chóp S . ABCD có đáy là hình thang vuông tại A và D ,
AB  AD  a , CD  2 a . Hình chiếu của đỉnh S lên mặt  ABCD  trùng với trung điểm của BD .

Trang 18 Fanpage Nguyễn Bảo Vương  https://www.facebook.com/tracnghiemtoanthpt489/


Điện thoại: 0946798489 TUYỂN CHỌN VẬN DỤNG – VẬN DỤNG CAO 2022
3
2a
Biết thể tích khối chóp S . ABCD bằng . Khoảng cách từ điểm A đến mặt phẳng  SBC 
2
bằng
5a 5a 10a 10a
A. . B. . C. . D. .
2 5 5 2
Lời giải
Chọn C
S

D
E C

K
a
H

A
a B

Gọi H hình chiếu của đỉnh S lên mặt  ABCD  . Khi đó: H là trung điểm của BD .
1 3a 2
Ta có: S ABCD  a  a  2a   .
2 2
3V a3 2 3 2
Ta có: SH  S . ABCD  3. : a a 2.
S ABCD 2 2
1 1
Ta có: AB  CD  d  A,  SBC    d  D,  SBC    d  H ,  SBC   .
2 2
Gọi E là trung điểm của CD . Ta có: ABED là hình vuông cạnh bằng a .
1
Xét tam giác BCD có BE  CD  BCD vuông cân tại B  BC  BD .
2
Theo bài ra, ta có: SH  BC . Do đó: BC   SBD  .
Trong  SBD  kẻ HK  SB . Khi đó: HK  d  H ,  SBC   .
1 1 1 1 1 5 2 2a 2 a 10
Ta có: 2
 2
 2
 2
 2
 2
 HK   HK  .
5 5
HK SH HB a 2  a 2
 
2a

 2 
a 10
Vậy d  A,  SBC   .
5
Câu 21. (Sở Thái Nguyên 2022) Cho hình chóp S . ABC có đáy ABC là tam giác vuông tại B ,
  30 . Đỉnh S cách đều ba điểm A, B , C và cạnh bên SB tạo với mặt phẳng
AB  6 3, CAB
 ABC  một góc 45 . Hai điểm M , Q lần lượt thuộc các đoạn AB và SB sao cho
AM  2 MB, QB  2QS . Mặt phẳng   chứa M , Q và song song với đường thẳng BC chia khối
chóp S . ABC thành hai khối đa diện có thể tích lần lượt là V1 , V2 V1  V2  . Giá trị của V2 là
A. 22 3 . B. 20 3 . C. 24 3 . D. 26 3 .
Lời giải
Facebook Nguyễn Vương https://www.facebook.com/phong.baovuongTrang 19
Blog: Nguyễn Bảo Vương: https://www.nbv.edu.vn/
Chọn B

 M      ABC 

Có  BC   ABC       ABC   d1 qua M và d1 //BC .
 BC //

Gọi N  d1  AC .
Q      SBC 

Có  BC   SBC       SBC   d 2 qua Q và d2 //BC .
 BC //

Gọi K  d2  SC .
Thiết diện của   với hình chóp là hình thang MNKQ .
Gọi P  MQ  KN  P  SA   SAB    SAC  .
1 2 1
Có QK  BC; MN  BC  QK  MN .
3 3 2
Lại có: QK //MN .
Suy ra: QK là đường trung bình của tam giác PMN .
PS MA QB PS 1
Ta có: . . 1  .
PA MB QS PA 4
VPSQK PS PQ PK 1 1 1 1 15
 . .  . .   VSQKAMN  VPAMN
VPAMN PA PM PN 4 2 2 16 16
1 1 1
VSABC  SI .SABC  .6. .6.6 3  36 3 .
3 3 2
1 1 4 4 1 64 3 15 32 3
V PAMN  d  P;  ABC   .SAMN  . .6. . 6.6 3   VSQKAMN  .  20 3 .
3 3 3 9 2 3 16 3
VMNBCKQ  VABCD  VSQKAMN  36 3  20 3  16 3 .
Vậy V1  16 3, V2  20 3 .

Trang 20 Fanpage Nguyễn Bảo Vương  https://www.facebook.com/tracnghiemtoanthpt489/


Điện thoại: 0946798489 TUYỂN CHỌN VẬN DỤNG – VẬN DỤNG CAO 2022
Câu 22. (Sở Vĩnh Phúc 2022) Cho hình chóp S. ABCD có đáy ABCD là hình chữ nhật
AB  a 3, SA  SB  SC  SD  2a . Giá trị lớn nhất của thể tích khối chóp S. ABCD bằng:
13 3 13 2 3 13 6 3 13 3 3
A. a . B. a . C. a . D. a .
12 12 12 12
Lời giải
Chọn D
S

A D

B
C

Gọi AD  x  x  0  .
1 1 2
Ta có AC  x 2  3a 2  AH  AC  x  3a 2
2 2
Khi đó
x 2  3a 2 13 2 x 2
SH  4a 2   a  .
4 4 4
1 1 13 2 x 2 2a x 13a 2 x 2
Thể tích khối chóp V  B.h  .a 3x. a   . .  .
3 3 4 4 3 2 4 4
2 2 2
x 13a x
 
x 13a 2 x 2 4  13a
2
Đặt f  x   .   4 4
2 4 4 2 8
2a 13a 2 13 3a 3
Thể tích lớn nhất của khối chóp V  .  .
3 8 12
Câu 23. (Sở Vĩnh Phúc 2022) Cho hình chóp S. ABCD có đáy ABCD là hình thoi, có
AC  a 3, 
ABC  600 . Biết rằng SA  SC , SB  SD và khoảng cách từ A mặt phẳng  SBC 
a 6
bằng . Tính thể tích khối chóp S. ABC bằng:
2
3 6a3 9 6a 3 3 15a 3 3 6a 3
A. . B. . C. . D. .
8 16 40 16
Lời giải
Chọn D

Facebook Nguyễn Vương https://www.facebook.com/phong.baovuongTrang 21


Blog: Nguyễn Bảo Vương: https://www.nbv.edu.vn/
S

A D

B C
K I

Gọi O là giao điểm của 2 đường chéo AC , BD . Khi đó SO   ABCD  .


Dựng AK , OI lần lượt vuông góc với BC .
Dựng OH  SI tại H .
Vì AC  a 3, ABC  600 nên tam giác ABC đều.
 BC  OI
Ta có   BC   SOI   BC  OH .
 BC  SO
Lại có OH  SI  OH   SBC   d  O,  SBC    OH .
1 a 6 a 3. 3 3a 3a
Mặt khác d  A;  SBC    d  O;  SBC    OH  , AK    OI  .
2 4 2 2 4
1 1 1 16 16 3 2a
Nên 2
 2
 2  2  2  SO  .
SO OH OI 6a 9 a 4
2
1 1 a 3
Thể tích khối chóp VS . ABC  SABC .SO  .
 .

3 3 2 a 3 6a 3
 .
3 3 4 4 16
Câu 24. (Chuyên Lam Sơn 2022) Trên cạnh AD của hình vuông ABCD cạnh 1, người ta lấy điểm M
sao cho AM  x (0  x  1) và trên nửa đường thẳng Ax vuông góc với mặt phẳng chứa hình
vuông, người ta lấy điểm S vói SA  y thỏa mãn y  0 và x 2  y 2  1 . Biết khi M thay đổi trên
m
đoạn AD thì thể tích của khối chóp S .ABCM đạt giá trị lớn nhất bằng với m, n  * và
n
m, n nguyên tố cùng nhau. Tính T  m  n .
A. 11.
B. 17.
C. 27.
D. 35.
Lời giải

Trang 22 Fanpage Nguyễn Bảo Vương  https://www.facebook.com/tracnghiemtoanthpt489/


Điện thoại: 0946798489 TUYỂN CHỌN VẬN DỤNG – VẬN DỤNG CAO 2022

1 1 x 1 1
Ta có VS . ABCM  SA  S ABCM   y   ( x  1) 1  x 2 .
3 3 2 6
2
 2
 4 3
Xét f ( x)  ( x  1) 1  x   x  2 x  2 x  1 trên [0;1] .
 x  1
Có f ( x)  4 x 3  6 x 2  2; f ( x)  0   .
 x  0.5
 1  27
Lập bảng xét dấu của f ( x ) trên [0;1] ta được max[0;1] f ( x)  f    .
 2  16
1 27 3
Vậy thể tích lớn nhất của khối S  ABCM là Vmax   .
6 16 8
Câu 25. (Chuyên Lam Sơn 2022) Cho khối bát diện đều có cạnh a . Gọi M , N , P, Q lần lượt là trọng tâm
của các tam giác SAB , SBC , SCD, SDA; gọi M , N , P, Q lần lượt là trọng tâm của các tam giác
S  AB, S  BC , S CD, S  DA (như hình vẽ dưới).

Thể tích của khối lăng trụ MNPQ  M  N  PQ là


2a 3 2 2a 3 2a 3 2 2a 3
A. . B. . C. . D. .
72 81 24 27
Facebook Nguyễn Vương https://www.facebook.com/phong.baovuongTrang 23
Blog: Nguyễn Bảo Vương: https://www.nbv.edu.vn/
Lời giải
Gọi O  AC  BD; I , J lần lượt là trung điểm của AB , BC .

2 1 a 2
Do M , N lần lượt là trọng tâm của các tam giác SAB, SBC nên ta có MN  JJ  AC 
3 3 3
Do SABCDS là bát diện đều nên hoàn toàn tương tự ta có tất cả các cạnh còn lại của của khối
a 2
lăng trụ MNPQ  M  N  PQ cũng bằng .
3
Mặt khác AC  BD , mà MN / / AC / / PQ, MQ / / BD / / NP nên MNPQ là hình vuông. Tương tự
ta có tất cả các mặt còn lại của lăng trụ MNPQ  M  N  PQ cũng là hình vuông. Suy ra lăng trụ
a 2
MNPQ.M  N  PQ là hình lập phương có cạnh bằng .
3
3
 a 2  2a 3 2
Vậy VMNPQMNP Q     .
 3  27
Câu 26. (Chuyên Lương Văn Tụy – Ninh Bình 2022) Cho hình chóp S. ABCD có đáy ABCD là hình
vuông, AB  1 , cạnh bên SA  1 và vuông góc với mặt phẳng đáy ( ABCD ) . Kí hiệu M là điểm
di động trên đoạn CD và N là điểm di động trên đoạn CB sao cho MAN  45 . Thể tích nhỏ
nhất của khối chóp S  AMN là
2 1
A. .
3
2 1
B. .
9
2 1
C. .
6
2 1
D. .
9
Lời giải

Trang 24 Fanpage Nguyễn Bảo Vương  https://www.facebook.com/tracnghiemtoanthpt489/


Điện thoại: 0946798489 TUYỂN CHỌN VẬN DỤNG – VẬN DỤNG CAO 2022

Đặt DM  x; BN  y (0  x, y  1)
1 1
Ta có S AMN  S ABCD  S ABN  S ADM  S CMN  1  [ x  y  (1  x )(1  y )]  (1  xy )
2 2
Xét tam giác vuông CMN : MN 2  (1  x)2  (1  y )2 (1) .
Áp dụng định lí cos cho tam giác AMN :
MN 2  AM 2  AN 2  2  AM  AN  cos 45  1  x 2  1  y 2  2  x 2  1  y 2  1 (2)
Từ (1) và (2) suy ra
(1  x) 2  (1  y ) 2  1  x 2  1  y 2  2  x 2  1  y 2  1

 
 2 x  2 y  2 x2  1 y 2  1 
2 2 2 2
 x  y  x y  1  4 xy (3)
Ta có x 2  y 2  2 xy (4)
 xy  3  2 2( loai )
Từ (3) và (4) suy ra x 2 y 2  1  4 xy  2 xy  ( xy ) 2  6 xy  1  0  
 xy  3  2 2
1
 S AMN  (1  xy )  2  1
2
1 2 1
 VS . AMN   SA  SAMN 
3 3
 x  y
Dấu "=" xảy ra   x  y  32 2
 xy  3  2 2
2 1
Vậy thể tích nhỏ nhất của khối chóp S  AMN bằng
3
Câu 27. (Chuyên Lương Văn Tụy-Ninh Bình 2022) Bạn A định làm một cái hộp quà lưu niệm (không
nắp) bằng cách cắt từ một tấm bìa hình tròn bán kính 4 cm để tạo thành một khối lăng trụ lục giác
đều, biết 6 hình chữ nhật có các kích thước là 1cm và xcm (tham khảo hình vẽ). Thể tích của hộp
quà gần nhất với giá trị nào sau đây?

Facebook Nguyễn Vương https://www.facebook.com/phong.baovuongTrang 25


Blog: Nguyễn Bảo Vương: https://www.nbv.edu.vn/

A. 24, 5 cm3 .
B. 25 cm 3 .
C. 25,5 cm3 .
D. 24 cm 3 .
Lời giải

Xét hình chữ nhật ABCD nội tiếp (O ) , do đó, AC là đường kính của (O) . Ta có AC  8 cm .
Tính được DC  1  x 3  1  x 3  2
Áp dụng định lý Py-ta-go vào tam giác ADC ta có
3 7 3
x 2  (2  x 3) 2  82  4 x 2  4 x 3  60  0  x 
2
x2 3 3 2 27 7  99 3
V  h  S d  1.6   x 3  25, 0094cm3
4 2 4

Trang 26 Fanpage Nguyễn Bảo Vương  https://www.facebook.com/tracnghiemtoanthpt489/


Điện thoại: 0946798489 TUYỂN CHỌN VẬN DỤNG – VẬN DỤNG CAO 2022
Câu 28. (THPT Trần Phú – Hà Tĩnh – 2022) Cho khối chóp S  ABCD , có đáy là hình chứ nhật cạnh
AB  2a 5 và tất cả các cạnh bên của hình chóp bằng 5a . Thể tích lớn nhất của khối chóp đã
cho bằng
20a 3 5
A. .
3
8a 3
B. .
3
40 5a 3
C. .
3
D. 15 5a 3 .
Lời giải
Đặt AD  x, ( x  0) khi đó bán kinh đường tròn ngoại tiếp đáy là
AB 2  AD 2 20a 2  x 2
Rd   . Chiều cao khối chóp là
2 2
20a 2  x 2 1
h  cb2  Rd2  25a 2   80a 2  x 2 .
4 2
Thể tích khối chóp là
 x 2  80a 2  x 2 
5a  
VABCD
1 1
 S ABCD  h   2 5a  x 
1 2 2
80a  x 

5a x 2 80a 2  x 2    2   40 5a
3

3 3 2 3 3 3
.
Câu 29. (THPT Kinh Môn - Hải Dương - 2022) Người ta dùng thuỷ tinh trong suốt để làm một cái chặn
giấy hình tứ diện đều. Để trang trí cho nó, người thiết kế đặt trong khối tứ diện 4 quả cầu nhựa
màu xanh có bán kính bằng nhau là r  2( cm) . Biết rằng 4 quả cầu này đôi một tiếp xúc với
nhau và mỗi mặt của tứ diện tiếp xúc với 3 quả cầu, đồng thời không cắt quả cầu còn lại. Nếu bỏ
qua bề dày của các mặt thì người ta cần dùng bao nhiêu thuỳ tinh để làm chặn giấy trên (làm tròn
đến chữ số thập phân thứ 2).
A. 195, 66  cm3  . B. 62, 09  cm3  . C. 30, 03  cm3  . D. 65,55  cm3  .

Lời giải
Chọn B

Facebook Nguyễn Vương https://www.facebook.com/phong.baovuongTrang 27


Blog: Nguyễn Bảo Vương: https://www.nbv.edu.vn/

I1

I2 O I4
B D
G'
I3
G

 Goi I1 , I 2 , I 3 , I 4 lần lượt là tâm của 4 hình cầu đã cho. Khi đó I1 I 2 I 3 I 4 là một tứ diện đều có
2r 6 4 3
cạnh bằng 2r  2 2 (cm) và chiều cao h   (cm) . Do đó hai tứ diện ABCD và
3 3
I1 I 2 I 3 I 4 đồng dạng với nhau theo tỉ số k .

 Gọi O là trọng tâm của tứ diện ABCD thì O cũng là trọng tâm của tứ diện I1 I 2 I 3 I 4 . Do đó O
là tâm đồng dạng. Giả sử phép đồng dạng tâm O lần lượt biến các đỉnh A, B, C , D thành
I1 , I 2 , I 3 , I 4 . Khi đó, hai mặt phẳng  BCD  và  I 2 I 3 I 4  song song nhau.

 Gọi G, G lần lượt là trọng tâm các tam giác BCD và I 2 I 3 I 4 . Khi đó:

1 3 3 OG
OG   h và OG  OG   r   2 . Suy ra k   1 6
4 3 3 OG 

 Khi đó, thể tích của tứ diện ABCD là:

2
1  2r  3
3 
8 19  9 6   cm 
3
VABCD  k .VI1I 2 I3 I 4 
 1 6 . .
3
 4
.h 
3
3

 Thể tích của mỗi khối cầu là:

4 8 2
Vc   .r 3 
3 3
 cm 3 

 Vậy lượng thuỷ tinh cần dùng là:

Trang 28 Fanpage Nguyễn Bảo Vương  https://www.facebook.com/tracnghiemtoanthpt489/


Điện thoại: 0946798489 TUYỂN CHỌN VẬN DỤNG – VẬN DỤNG CAO 2022

V  VABCD  4.Vc 

8 19  9 6   4. 8 2
 62, 06  cm3 
3 3
Câu 30. (THPT Kinh Môn - Hải Dương - 2022) Cho tam giác ABC đều cạnh a , gọi d là đường thẳng
qua A và vuông góc với mặt phẳng  ABC  . Trên d lấy điểm S và đặt AS  x  x  0  . Gọi H
và K lần lượt là trực tâm của các tam giác ABC và SBC . Biết HK cắt d tại điểm S  . Khi SS 
ngắn nhất thì khối chóp S. ABC có thể tích bằng
a3 6 a3 3 a3 2 a3 6
A. . B. . C. . D. .
6 8 27 24
Lời giải
Chọn D
S

A K C

H M

S'
+) Chứng minh HK   SBC 
SA  BC 
   SAH   BC  HK  BC
AH  BC 
BH  AC 
  BH  SC . Lại có BK  SC nên HK  SC .
BH  SA 
Vậy HK   SBC   HK  SM (với M là trung điểm của BC ).
a 3 a 3
+) Tam giác ABC đều cạnh a nên AM  , AH  .
2 3
AH SA AH . AM a 2
 AS H  AMS    AS    .
AS  AM SA 2x
a2
+) SS   SA  AS   x  a 2
2x
a
min SS   a 2  x 
2
1 1 a a 2 3 a3 6
Khi đó VS . ABC  SA.S ABC  . .  .
3 3 2 4 24
Câu 31. (THPT Lương Tài 2 - Bắc Ninh - 2022) Cho lăng trụ tam giác đều ABC. ABC  có cạnh đáy
bằng 4a . Góc giữa hai mặt phẳng  ABC  và  ABC  bằng 300 . Gọi M là trung điểm của cạnh
AB , tính khoảng cách từ điểm M tới mặt phẳng  ABC  ?
a 3 3a
A. . B. 3a . C. a 3 . D. .
2 2
Lời giải
Chọn A
Facebook Nguyễn Vương https://www.facebook.com/phong.baovuongTrang 29
Blog: Nguyễn Bảo Vương: https://www.nbv.edu.vn/

Gọi I là trung điểm của BC .


 ABC    ABC   BC

Ta có:  AI   ABC  , AI  BC   ABC  ,  ABC    
 AI , AI  
 AIA  300 .
 
 
 A I   ABC  , AI  BC
AA 4a 3
Xét tam giác AIA vuông tại A suy ra tan  AIA   AA  tan 300.  2a .
AI 2
d  M ,  ABC   MB 1
Ta có: AM   ABC   B     .
d  A,  ABC   AB 2
Kẻ AH  AI ,
 ABC    AAI   Do BC   AAI  

Ta có:  ABC    AAI   AI  AH   ABC   d  A,  ABC    AH

 AH   AAI  , AH  AI
1 1 1 1
Xét tam giác AIA vuông tại A suy ra 2
 2
 2  2  AH  3a .
AH AA AI 3a
1 a 3
Vậy d  M ,  ABC    AH  .
2 2
Câu 32. (THPT Võ Nguyên Giáp - Quảng Bình - 2022) Cho hình lăng trụ đứng ABC. ABC  có đáy
ABC là tam giác đều. Gọi  là góc tạo bởi AB với mặt phẳng  ACC A  và  là góc giữa mặt
m m
phẳng  ABC  với mặt phẳng  ACC A  . Biết cot 2   cot 2   (với m, n  N* và phân số
n n
tối giản). Khi đó, giá trị của biểu thức T  m  2n bằng
A. 3 . B. 5 . C. 7 . D. 9 .
Lời giải
Chọn C

Trang 30 Fanpage Nguyễn Bảo Vương  https://www.facebook.com/tracnghiemtoanthpt489/


Điện thoại: 0946798489 TUYỂN CHỌN VẬN DỤNG – VẬN DỤNG CAO 2022

Gọi H là trung điểm của AC dễ thấy BH   ACC A  và 


 BA ,  ACCA   BAH   và
.
  ABC   ,  ACC A    BIH
Gọi AC  a , AA '  x .
2 2
2 2 HA   HI 
Ta có cot   cot      
 BH   BH 
2
 2  2
 x 2   a    
 2   x  4 x2  a2 4 x 2 a2 1
      2
 2  2  .
 a 3   a 3  3a 3a 3a 3
 2   2 
 
m  1
 . Vậy T  m  2n  1  2.3  7.
n  3
Câu 33. (THPT Võ Nguyên Giáp - Quảng Bình - 2022) Cho hình hộp đứng ABCD. ABCD có đáy
ABCD là hình vuông. Gọi S là tâm hình vuông ABCD . Gọi M và N lần lượt là trung điểm
của SA và BC . Biết rằng, nếu MN tạo với mặt phẳng  ABCD  một góc 60 và AB  a thì thể
tích S. ABC bằng
a3 30 a3 30 a3 3
A. . B. . C. a3 30 . D. .
12 3 2
Lời giải
Chọn A

Facebook Nguyễn Vương https://www.facebook.com/phong.baovuongTrang 31


Blog: Nguyễn Bảo Vương: https://www.nbv.edu.vn/
B C
N

I
A H
D

B'
C'

S
A' D'
Gọi I là tâm hình vuông ABCD ; H là trung điểm của AI .
 là
Ta có: SI   ABCD   MH   ABCD   hình chiếu của MN lên  ABCD  là HN  HNM
  60 .
góc giữa MN và mặt phẳng  ABCD   HNM
a 3 3a 2
Xét HNC có: NC  ; CH  AC  ;
2 4 4
2 2
2
2 2 2  a   3a 2  a 3a 2 2 5a
HN  NC  HC  2 NC .HC cos 45        2  
2  4  2 4 2 8
a 5
 NH  .
2 2
 MH a 5 a 15
Xét HMN vuông ở H có: tan HNM  MH  NH .tan60   3 .
HN 2 2 2 2
a 15
Do đó, SI  2 MH  .
2
1 1 a 15 a 2 a 3 30
Thể tích khối chóp S. ABC là: VS . ABC  SI .S ABC     .
3 3 2 2 12
Câu 34. (THPT Yên Lạc - Vĩnh Phúc - 2022) Cho hình chóp S. ABCD , có đáy ABCD là hình vuông,
SAB là tam giác đều và nằm trong mặt phẳng vuống góc với mặt đáy. Biết khoảng cách giữa hai
2a 21
đường thẳng AB và SD bằng . Thế tích của khối chóp S.ABCD bằng
7
2a 3 3 4a 3 3 8a 3 3 a3 3
A. . B. . C. . D. .
3 3 3 3
Lời giải
Chọn B

Trang 32 Fanpage Nguyễn Bảo Vương  https://www.facebook.com/tracnghiemtoanthpt489/


Điện thoại: 0946798489 TUYỂN CHỌN VẬN DỤNG – VẬN DỤNG CAO 2022
Đặt AB  2 x , gọi H là trung điểm của AB từ giả thiết bài toán ta có được SH   ABCD  ,
SH  x 3 .
Ta có AB //CD do đó AB //  SCD  vì vậy d  AB, SD   d  AB,  SCD    d  H ,  SCD   .
 HI  CD
Gọi I là trung điểm của CD ta có  do đó CD   SHI  , kẻ HK  SI ta được
 SH  CD
HK   SCD  vì vậy d  H ,  SCD    HK .
1 1 1 1 1 7 2 21x 2 21a
Ta lại có 2
 2
 2
 2 2  2
suy ra HK   .
HK HS HI 3x 4x 12 x 7 7
Từ đây ta suy ra x  a .
1 1 4a 3 3
Vậy VS . ABCD  SH .S ABCD  a 3.4a 2  .
3 3 3
Câu 35. (THPT Yên Lạc - Vĩnh Phúc - 2022) Một trang tại cần xây dựng một bể chứa nước hình hộp
chữ nhật bằng gạch không nắp ở phía trên. Biết bể có chiều dài gấp hai lần chiều rộng và thể tích
(phần chứa nước) bằng 8 m 3 . Hỏi chiều cao của bể gần nhất với kết quả nào dưới đây để số lượng
gạch dùng để xây bể là nhỏ nhất
A. 1,8 m . B. 1,3m . C. 1,1m . D. 1, 2 m .

Lời giải
Chọn D
Chiều rộng bể và chiều dài bể lần lượt là x, 2 x ( x  0) , chiều cao bể là h , đơn vị m .

4
Khi đó thể tích bể là x.2 x.h  8  h  .
x2

Diện tích cần xây dựng cho bể không nắp là

4 24
S  2 x.x  2.2 x.h  2.x.h  2 x 2  6 xh  2 x 2  6 x. 2
 2x2  .
x x

Để số lượng gạch dùng để xây bể là nhỏ nhất thì diện tích cần xây dựng là nhỏ nhất
24 12 12 12 12
Áp dụng bất đẳng thức AM – GM: 2 x 2   2 x 2    3 3 2 x 2 . .  2 3 288
x x x x x

12
Dấu đẳng thức xảy ra khi 2 x 2   x3  6  x  3 6 .
x

4 4
Lúc này h  2
 2
 1, 21 .
x 3
6

Câu 36. (THPT Yên Phong 1 - Bắc Ninh - 2022) Cho hình chóp S . ABCD có đáy ABCD là hình bình
hành. Gọi M là điểm di động trên cạnh AB và N là trung điểm SD . Mặt phẳng   đi qua
V1 3
M , N và song song BC chia khối chóp thành hai khối có tỉ lệ thể tích  , trong đó V1 là thể
V2 5
AM
tích khối đa diện chứa đỉnh A , V2 là thể tích khối đa diện chứa đỉnh B . Tỉ số bằng
AB
1 1 3 3
A. . B. . C. . D. .
3 2 5 7
Lời giải
Chọn C
Facebook Nguyễn Vương https://www.facebook.com/phong.baovuongTrang 33
Blog: Nguyễn Bảo Vương: https://www.nbv.edu.vn/

AM
+) Đặt  x . Kẻ MP // BC ; NQ //AD .
AB
   cắt hình chóp theo thiết diện là hình thang MPNQ .
+) Đặt S là diện tích ABCD ; h là chiều cao của S . ABCD .
Diện tích AMPD : S AMPD  x.S
1
d  Q;  AMPD    h .
2
1 1 1 1 hS 1
+) Thể tích Q. AMPD : VQ. AMPD  d  Q;  AMPD   .S AMPD  . h.xS  x.  xV .
3 3 2 2 3 2
( V là thể tích S . ABCD ).
3
1 V
3VS . ACD
+) Lại có VS . ACD  V  d  A;  SCD     2 .
2 S SCD S SCD
3
V
1
Vì Q là trung điểm SA nên d  Q;  SCD    d  A;  SCD    4
2 S SCD
1
Diện tích DPN : SDPN  DN .DP.sin NDP   1 . 1 DS .xDC.sin NDP   x .S
SCD .
2 2 2 2
3
V
1 1 4 x 1 3 x xV
Thể tích Q.PDN : VQ .PDN  d  Q;  SCD   .S DPN  . . .S SCD  . V .  .
3 3 S SCD 2 3 4 2 8
xV xV 5xV
 V1  VQ. ANPD  VQ. PND    .
2 8 8
5x  5x 
V2  V  V1  V  V   1   V .
8  8 
5x
V
V1 3 8 3 5x 3  5x 
Theo giả thiết:      1  
V2 5  5x  5 8 5 8 
 1   V
 8 
8 5x 3 3
 .  x .
5 8 5 5
AM 3
Vậy  .
AB 5

Theo dõi Fanpage: Nguyễn Bảo Vương  https://www.facebook.com/tracnghiemtoanthpt489/

Hoặc Facebook: Nguyễn Vương  https://www.facebook.com/phong.baovuong

Trang 34 Fanpage Nguyễn Bảo Vương  https://www.facebook.com/tracnghiemtoanthpt489/


Điện thoại: 0946798489 TUYỂN CHỌN VẬN DỤNG – VẬN DỤNG CAO 2022

Tham gia ngay: Nhóm Nguyễn Bào Vương (TÀI LIỆU TOÁN)  https://www.facebook.com/groups/703546230477890/

Ấn sub kênh Youtube: Nguyễn Vương


 https://www.youtube.com/channel/UCQ4u2J5gIEI1iRUbT3nwJfA?view_as=subscriber

Tải nhiều tài liệu hơn tại: https://www.nbv.edu.vn/

Facebook Nguyễn Vương https://www.facebook.com/phong.baovuongTrang 35


TUYỂN CHỌN VẬN DỤNG – VẬN DỤNG CAO 2022 Điện thoại: 0946798489

VẤN ĐỀ 6. KHỐI TRÒN XOAY


• |FanPage: Nguyễn Bảo Vương
• TUYỂN CHỌN CÂU HỎI VD-VDC TỪ CÁC ĐỀ THI THỬ CÁC TRƯỜNG, CÁC SỞ NĂM 2022
Câu 1. (Cụm Trường Nghệ An - 2022) Cắt hình nón  N  bởi mặt phẳng đi qua đỉnh S và tạo với trục của
 N  một góc bằng 30 , ta được thiết diện là tam giác SAB vuông và có diện tích bằng
4a 2 .Chiều cao của hình nón bằng
A. a 3 . B. 2 a 3 . C. 2 a 2 . D. a 2 .
Lời giải
Chọn A

Gọi M là trung điểm của AB và O là tâm của đường tròn đáy của hình nón, tam giác OAB cân
đỉnh O nên OM  AB và SO  AB suy ra AB   SOM  .
Dựng OK  SM tại M .
OK  AB
Theo trên ta có:   OK   SAB  .
OK  SM
  30 .
Suy ra góc tạo bởi giữa trục SO và mặt phẳng  SAB  là OSM
1 2
Tam giác vuông cân SAB có diện tích bằng 4a 2 suy ra SA  4a 2  SA  2a 2
2
 AB  4 a  SM  2 a .

 SO 3
Xét tam giác vuông SOM có cos OSM  SO  .2a  3a .
SM 2
Vậy chiều cao của hình chóp bằng a 3 .
Câu 2. (Đại học Hồng Đức – 2022) Cho hình nón đỉnh S có độ dài đường cao là R và đáy là đường tròn
tâm O bán kính R . Gọi (d ) là tiếp tuyến của đường tròn đáy tại A và ( P ) là mặt phẳng chứa
SA và ( d ) . Mặt phẳng (Q ) thay đổi qua S cắt đường tròn O tại hai điểm C , D sao cho
CD  3R . Gọi  là góc tạo bởi ( P ) và (Q ) . Tính giá trị lớn nhất của cos  .
3 10
A. .
10
10
B. .
5
Facebook Nguyễn Vương https://www.facebook.com/phong.baovuong Trang 1
Blog: Nguyễn Bảo Vương: https://www.nbv.edu.vn/
2 6
C. .
5
10
D. .
10
Lời giải.
Gọi I là trung điểm CD , khi đó OI  CD , hạ OK  SI tại K  OK  (Q ) .

HaOH  SA  OH  ( P )    (OH , OK )

OK 2  OH 2  HK 2 R OI  OS R R
 cos   . Ta có OI  OD 2  ID 2  , OK   ; OH 
2OH  OK 2 OI 2  SO 2 5 2
2 2 2
SI  SA  AI
HK 2  SK 2  SH 2  2 SK  SH cos  ASI  SK 2  SH 2  2 SK  SH  .
2 SI  SA
5 SO 2 R SO 2 2
SA  2 R, SI  R, SH   , SK   R.
2 SA 2 SI 5
Gọi M và N lần lượt là trung điểm OA và OB khi đó
AM  AI  AN .
Suy ra
SI 2  SA2  AM 2 SI 2  SA2  AN 2
SK 2  SH 2  2 SK  SH   HK 2  SK 2  SH 2  2 SK  SH 
2 SI  SA 2 SI  SA
2 2 2
1 9 10 OK  OH  HK 3 10
 R 2  HK 2  R 2    
10 10 10 2OH  OK 10
 10 3 10  3 10
 cos   max  ;  .
 10 10  10
Câu 3. (THPT Hương Sơn - Hà Tĩnh - 2022) Một chiếc kem Ốc quế gồm 2 phần, phần dưới là một khối
nón có chiều cao bằng ba lần đường kính đáy, phần trên là nửa khối cầu có đường kính bằng
đường kính khối nón bên dưới (như hình vẽ). Thể tích phần kem phía trên bằng 50cm3 . Thể tích
của cả chiếc kem bằng

Trang 2 Fanpage Nguyễn Bảo Vương  https://www.facebook.com/tracnghiemtoanthpt489/


Điện thoại: 0946798489 TUYỂN CHỌN VẬN DỤNG – VẬN DỤNG CAO 2022
3 3
A. 200cm . B. 150cm . C. 125cm3 . D. 500cm3 .
Lời giải
Chọn A
Gọi bán kính của khối cầu là R  R  0  . Theo bài ra ta có:
1 4 75
V1  VC  50   R 3  100  R 3  .
2 3 
Do đó, khối nón phía dưới có bán kính R ; h  3.2 R  6R .
1 1 75
Thể tích của khối nón bằng: V2   R 2 .h   R 2 .6 R  2 .R 3  2 .  150  cm 3  .
3 3 

Vậy thể tích của cả chiếc kem bằng: V  V1  V2  50  150  200 cm3 . 
Câu 4. (THPT Lê Thánh Tông - HCM-2022) Cho hình trụ tròn xoay có hai đáy là hai hình tròn I ; 7  
   
và J ; 7 . Biết rằng tồn tại dây cung EF của đường tròn I ; 7 sao cho tam giác JEF là tam
giác đều và mặt phẳng  JEF  hợp với mặt đáy của hình trụ một góc bằng 60. Thể tích V của
khối trụ đã cho là
A. V  21 . B. V  7 6 . C. V  14 . D. V  28 .

Lời giải
Chọn A

F
I
H
E
 Gọi H là trung điểm EF , có IHJ  60.
IJ h h 3
 Đặt IJ  h, tam giác vuông JIH có tan JIH   IH   . , và
IH tan 60 3
IJ h 2h 3
sin JHI   JH   .
JH sin 60 3
EF 3 2 JH 2h 3 4h
 Tam giác đều JEF có JH   EF   2.  .
2 3 3 3 3
2 2
2 2 2 h 4h
 Tam giác vuông IHE có IH  HE  IE    7  h  3.
3 9
 Vậy V(T )  r 2 . .h  7 .3  21 .
Câu 5. (Liên trường Hà Tĩnh – 2022) Cho hình nón có chiều cao bằng 2 5 . Một mặt phẳng đi qua đỉnh
hình nón và cắt hình nón theo một thiết diện là tam giác đều có diện tích bằng 9 3 . Thể tích của
khối nón được giới hạn bởi hình nón đã cho bằng

Facebook Nguyễn Vương https://www.facebook.com/phong.baovuongTrang 3


Blog: Nguyễn Bảo Vương: https://www.nbv.edu.vn/
32 5
A. .
3
B. 32 .
C. 32 5 .
18 5
D. .
3
Lời giải
Theo giả thiết tam giác SAB đều, SSAB  9 3 và SO  2 5 .

AB 2 3
S SAB  9 3   9 3  AB  6.
4
 SAB đều SA  AB  6 .
Xét SOA vuông tại O , OA  SA2  SO 2  62  (2 5) 2  4.
1 1 1 32 5
Thể tích hình nón bằng V   r 2 h    OA2  SO   42  2 5  .
3 3 3 3
Câu 6. (THPT Nguyễn Tất Thành-Đh-SP-HN-2022) Cho hình chóp S. ABCD có đáy ABCD là hình chữ
nhật, AB  2, AD  1 . Gọi M là trung điểm của DC . Biết SA   ABCD  và SA  2 . Tính bán
kính mặt cầu ngoại tiếp hình chóp S.BCM .

Trang 4 Fanpage Nguyễn Bảo Vương  https://www.facebook.com/tracnghiemtoanthpt489/


Điện thoại: 0946798489 TUYỂN CHỌN VẬN DỤNG – VẬN DỤNG CAO 2022

3 3 11 13
A. R  3 . B. R  . C. R  . D. R  .
2 2 2
Lời giải
Chọn C

Tam giác MBC vuông cân tại C . Gọi H là trung điểm của MB suy ra H là tâm đường tròn
ngoại tiếp tam giác MBC .
Từ H dựng đường thẳng d song song với SA .
Mà SA   ABCD   SA   MBC   d   MBC  .
Suy ra d là trục của đường tròn ngoại tiếp tam giác MBC .
Gọi K là trung điểm của SA .
Do AH  SA từ K dựng đường thẳng song song với AH cắt d tại I suy ra I là tâm mặt cầu
ngoại tiếp hình chóp S .BCM .
Tam giác MBC vuông cân tại C  MBC   45   ABH  45 .

Facebook Nguyễn Vương https://www.facebook.com/phong.baovuongTrang 5


Blog: Nguyễn Bảo Vương: https://www.nbv.edu.vn/
Áp dụng định lý hàm số cos cho tam giác ABH ta có
1
4   AH 2
BA2  BH 2  AH 2 5
 cos 
ABH   cos 45  2  AH 2  .
2 BA.BH 2 2 2
2 5 1
Đặt IH  x suy ra SI 2  SK 2  KI 2   2  x   và. IB 2  x 2  .
2 2
2 5 1 3
Ta có SI  IB  SI 2  IB 2   2  x    x 2   x  .
2 2 2
11
Vậy R  SI  .
2
Câu 7. (THPT Nho Quan A – Ninh Bình – 2022) Cho hình nón đỉnh S có đường cao h  a 3 . Một mặt
phẳng ( ) đi qua đỉnh S , cắt đường tròn đáy tại hai điểm A, B sao cho AB  8a và tạo với mặt
đáy một góc 30 . Tính diện tích xung quanh của hình nón.
10 7 2
A. a .
3
B. 20 7 a 2 .
C. 10 7 a 2
D. 5 7 a 2 .
Lời giải

Gọi O là tâm đường tròn đáy, I là trung điểm AB . Khi đó, góc giữa mặt phẳng ( ) và mặt đáy
  30 .
là SIO
SO
Trong tam giác SOI, ta có OI   3a .

tan SIO
Trong tam giác AIO , ta có OA2  OI 2  AI 2  9a 2  16a 2  5a
 SA  SO 2  AO 2  3a 2  25a 2  2 7 a .
Vậy S xq    OASA  10 7 a 2 .
Câu 8. (THPT Phù Cừ - Hưng Yên - 2022) Một tấm tôn hình tam giác ABC có độ dài cạnh
AB  3; AC  2; BC  19 . Điểm H là chân đường cao kẻ từ đình A của tam giác ABC . Người
ta dùng compa có tâm là A , bán kính AH vạch một cung tròn MN . Lấy phần hình quạt gỏ thành
hình nón không có mặt đáy với đỉnh là A , cung MN thành đường tròn đáy của hình nón (nhuc
hình vẽ). Tính thể tich khối nón trên.
2 114 2 3  57 2 19
A. . B. . C. . D. .
361 19 361 361
Lời giải

Trang 6 Fanpage Nguyễn Bảo Vương  https://www.facebook.com/tracnghiemtoanthpt489/


Điện thoại: 0946798489 TUYỂN CHỌN VẬN DỤNG – VẬN DỤNG CAO 2022


Theo định lý côsin trong tam giác ABC ta có BC 2  AB 2  AC 2  2  AB  AC  cos BAC
2 2 2
  AB  AC  BC   1  BAC
 cos BAC   120 hay BAC  2 .
2  AB  AC 2 3
1 3 3.
Suy ra diện tích tam giác ABC là S ABC  AB  AC  sin BAC
2 2
1 2S 3 57
Mà S ABC  AH  BC  AH  ABC  .
2 BC 19
2 AH 57
Gọi r là bán kinh đáy của hình nón. Suy ra 2 r  AH  r   .
3 3 19
2 114
Chiều cao của khối nón bằng h  AH 2  r 2  .
19
2
1 1  57  2 114 2 114
Thể tích bằng V   r 2 h        .
3 3  19  19 361
Câu 9. (Sở Bạc Liêu 2022) Trên bàn có một cốc nước hình trụ chứa đầy nước, có chiều cao bằng 3 lần
đường kính của đáy; một viên bi và một khối nón đều bằng thủy tinh. Biết viên bi là một khối cầu
có đường kính bằng đường kính của đường tròn đáy cốc nước. Người ta từ từ thả vào cốc nước
viên bi và khối nón sao cho đỉnh khối nốn nằm trên mặt cầu (như hình vẽ) thì thấy nước trong cốc
tràn ra ngoài. Tính tỉ số của lượng nước còn lại trong cốc và lượng nước ban đầu.

4 5 2 1
A. . B. . C. . D. .
9 9 3 2
Lời giải
Chọn B
Gọi R là bán kính đáy của hình trụ, h là chiều cao của hình trụ  h  6 R .
Thể tích nước ban đầu trong cốc là V1   R 2 .h  6 R 3 .
Thể tích nước tràn ra ngoài bằng thể tích của viên bi và thể tích của khối nón
4 1 8
V2   R3   R 2 .4R   R3 .
3 3 3

Facebook Nguyễn Vương https://www.facebook.com/phong.baovuongTrang 7


Blog: Nguyễn Bảo Vương: https://www.nbv.edu.vn/
10 3
Thể tích nước còn lại trong cốc là V3  V1  V2  R .
3
V3 5
Tỉ số thể tích nước còn lại và nước ban đầu là  .
V1 9
Câu 10. (Sở Hà Tĩnh 2022) Cắt hình nón  N  bởi mặt phẳng đi qua đỉnh  S  và tạo bởi với trục của
N một góc bằng 30 ta được thiết diện là tam giác SAB vuông và có diện tích 4a 2 . Chiều cao
của hình nón bằng:
A. a 3 . B. 2a 3 . C. 2a 2 . D. a 2 .
Lời giải
Chọn A
S

O I

Gọi đường sinh SB  x  x  0  . Vì tam giác SAB là tam giác vuông nên AB  x 2 .
x 2
 SI  . Theo đề bài tam giác SAB vuông và có diện tích 4a 2 nên:
2
1 1 x 2
S SAB  SI . AB  . .x 2  4a 2  x  2 2a  SI  2a
2 2 2
  30  cosIOS
Ta có IOS   SO  SO  SI .cosIOS   2a 3  a 3 .
SI 2
Câu 11. (Sở Lạng Sơn 2022) Một cái cột có hình dạng như hình bên (gồm một khối nón và một khối trụ
ghép lại). Chiều cao đo được ghi trên hình, chu vi đáy là 20cm .

Thể tích của cột bằng


52000 5000 5000 13000
A.
3
 cm 3  . B.
3
 cm 3  . C.

 cm  .
3
D.
3
 cm 3  .
Lời giải
Chọn D
Gọi r , h1 , h2 lần lượt là bán kính đường tròn đáy, chiều cao khối nón và chiều cao khối trụ.
10
Ta có: h1  10  cm  ; h2  40  cm  và chu vi đáy là 20cm suy ra 2 r  20  r   cm  .

Trang 8 Fanpage Nguyễn Bảo Vương  https://www.facebook.com/tracnghiemtoanthpt489/


Điện thoại: 0946798489 TUYỂN CHỌN VẬN DỤNG – VẬN DỤNG CAO 2022
2
1  10   10  13000
Vậy, thể tích của cột là V   r 2 h1   r 2 h2       40  
3    3  3
 cm3  .
Câu 12. (Sở Phú Thọ 2022) Cho hình trụ có bán kính đáy bằng a 3 . Cắt hình trụ bởi một mặt phẳng
song song với trục, cách trục một khoảng bằng a ta được thiết diện là một hình vuông. Thể tích
khối trụ đó bằng:
A. 2 a 3 2 . B. 4 a 3 2 . C. 6 a 3 2 . D. 3 a 3 2 .
Lời giải
Chọn C

Dựng OI  AB khi đó I là trung điểm AB .


Ta có IA  OA2  IO 2  3a 2  a 2  a 2.
Vì ABCD là hình vuông nên AB  2a 2 .
Thế tích hình trụ: V   R 2 h  6 a 3 2  đvtt  .
Câu 13. (Sở Phú Thọ 2022) Cho hình nón  có chiều cao bằng 2a . Cắt  bởi một mặt phẳng đi qua
4a 2 11
đỉnh và cách tâm của đáy một khoảng bằng a ta được thiết diện có diện tích bằng . Thể
3
tích của khối nón đã cho bằng
10πa 3 4πa 3 5 4πa 3 5
A. . B. 10πa3 . C. . D. .
3 3 9
Lời giải
Chọn A
S

A O B
I
C

Dựng mặt phẳng qua đỉnh  SBC  của hình nón, gọi I là trung điểm của BC .
4a 2 11
Theo giả thiết: SO  2a; SSBC  ; OH  d  O ,  SBC    a .
3

Facebook Nguyễn Vương https://www.facebook.com/phong.baovuongTrang 9


Blog: Nguyễn Bảo Vương: https://www.nbv.edu.vn/
1 1 1 2a 3 4a 3
Trong SOI vuông tại O có: 2
 2
 2  OI  ; SI  SO 2  OI 2  .
OH SO OI 3 3
1 2S 2a 33 BC a 33
Ta có: SSBC  SI .BC  BC    IC   .
2 SI 3 2 3
Trong OIC vuông tại I có: OC  OI 2  IC 2  a 5 .
1 10πa 3
Vậy thể tích của khối nón đã cho là: V  πSO.OC 2  .
3 3
Câu 14. (Sở Thái Nguyên 2022) Cho một dụng cụ đựng chất lỏng như hình 1 có phần trên là mặt xung
quanh và đáy của hình trụ, phần dưới là mặt xung quanh của hình nón. Biết hình trụ có cùng bán
kính đáy R và cùng chiều cao h  24cm với hình nón. Trong hình 1, lượng chất lỏng có chiều cao
bằng 12cm . Lật ngược dụng cụ theo phương vuông góc với mặt đất như hình 2. Khi đó chiều cao
của chất lỏng trong hình 2 là

A. 3cm . B. 2cm . C. 1cm . D. 4cm .

Lời giải
Chọn C
Trong hình 1 ta gọi hình nón đỉnh S có chiều cao là h  12 và đường tròn đáy tâm N có bán
kính là R
R h h 1
Áp dụng định lí Ta-lét ta có   R  R  R
R h h 2
1 1 1
Thể tích của lượng chất lỏng trong hình 1 là V   R2 h    R 2 12    R 2
3 3 4
Trong hình 2 ta gọi hình trụ có hai đường tròn đáy có tâm lần lượt là O , M ; chiều cao là x có
bán kính là R .
Thể tích của lượng chất lỏng trong hình 2 là V    R 2 x
Theo bài ra ta có V   V   R 2 x   R 2  x  1.
Câu 15. (Sở Thái Nguyên 2022) Cho tứ diện ABCD có AB  BC  CD  2 3 , AC  BD  2 ,
AD  2 2 . Diện tích mặt cầu ngoại tiếp tứ diện đã cho bằng
40 10
A. 6 . B. 24 . C. . D. .
3 3
Lời giải
Chọn C

Trang 10 Fanpage Nguyễn Bảo Vương  https://www.facebook.com/tracnghiemtoanthpt489/


Điện thoại: 0946798489 TUYỂN CHỌN VẬN DỤNG – VẬN DỤNG CAO 2022

Dựng hình lăng trụ đều ACE .DFB có cạnh đáy 2 , cạnh bên 2 2 .
Mặt cầu ngoại tiếp tứ diện ABCD là mặt cầu ngoại tiếp lăng trụ đều ACE .DFB . Gọi O , K lần
lượt là tâm hai đáy ACB , DBF .
Tâm mặt cầu ngoại tiếp lăng trụ là I (trung điểm của OK ).
2
2 2
2 3 30
Bán kính mặt cầu ngoại tiếp lăng trụ là: R  OI  OA  2   .
 3   3
 
2
 30  40
Diện tích mặt cầu ngoại tiếp tứ diện ABCD là: S  4     .
 3  3
Câu 16. (Sở Vĩnh Phúc 2022) Cho một hình nón đỉnh S có đáy là đường tròn tâm O , bán kính R  5
2
và góc ở đỉnh là 2 với sin   . Một mặt phẳng  P  vuông góc với SO tại H và cắt hình
3
nón theo một đường tròn tâm H . Gọi V là thể tích của khối nón đỉnh O và đáy là đường tròn
50 a a
tâm H . Biết V  khi SH  với a, b* và là phân số tối giản. Tính giá trị của biểu
81 b b
thức T  3a 2  2b 3 .
A. 12 . B. 23 . C. 21 . D. 32 .
Lời giải
Chọn C

Facebook Nguyễn Vương https://www.facebook.com/phong.baovuongTrang 11


Blog: Nguyễn Bảo Vương: https://www.nbv.edu.vn/

   với sin   2  cos   1  sin 2   1  4  5 .


Trong SOB vuông tại O ta có OSB
3 9 3
2 HN 2 2.SH
Suy ra tan      HN  .
5 SH 5 5
HN OB OB 3 5
Mặt khác sin     SB   ;
SN SB sin  2
45 5
Trong SOB vuông tại O ta có SO  SB 2  OB 2  5  .
4 2
Theo bài ta có thể tích khối nón đỉnh O và đáy là đường tròn tâm H là
50 1 50 50 4 SH 2 50
V  .OH . .HN 2   OH .HN 2    SO  SH  . 
81 3 81 27 5 27
 5
2 125 3 2
 SH  3
  5  2.SH  SH   54SH  135SH  125  0   .
27  SH   5  loai 
 6
2 3
Suy ra a  5; b  3 . Vậy T  3a  2b  3.25  2.27  21 .

Câu 17. (Sở Vĩnh Phúc 2022) Cho một hình nón có bán kính đáy bằng a . Mặt phẳng  P  đi qua đỉnh S
của hình nón cắt đường tròn đáy tại A và B sao cho AB  a 3 , khoảng cách từ tâm đường tròn
a 2
đáy đến mặt phẳng  P  bằng . Thể tích khối nón đã cho bằng
4
 a3  a3  a3  a3
A. . B. . C. . D. .
12 6 3 24
Lời giải
Chọn B

Gọi I là trung điểm đoạn AB dễ thấy OI là đường trung tuyến của tam giác OAB .
Dựng OK  SI tại K .
 AB  OI
Ta có   AB   SOI   OK  AB  OK   SAB  .
 AB  SO
a 3 3a 2 a
Ta có BI   OI  a 2   .
2 4 2
1 1 1 16 4 4 a
Khi đó d  O;  P    OK  2
 2
 2  2  2  2  SO  .
SO OK OI 2a a a 2

Trang 12 Fanpage Nguyễn Bảo Vương  https://www.facebook.com/tracnghiemtoanthpt489/


Điện thoại: 0946798489 TUYỂN CHỌN VẬN DỤNG – VẬN DỤNG CAO 2022
3
1 1 a a
Thể tích khối nón: V   r 2 h   .a 2 .  .
3 3 2 6
Câu 18. (Sở Vĩnh Phúc 2022) Cho hình chóp S . ABC có đáy ABC là tam giác đều cạnh 2a , cạnh bên
SA vuông góc với mặt phẳng đáy, góc giữa hai mặt phẳng  SBC  và  ABC  bằng 45 . Diện tích
mặt cầu ngoại tiếp hình chóp S . ABC bằng
25 a 2 25 a 2 25 a 2 25 a 2
A. . B. . C. . D. .
12 3 9 6
Lời giải
Chọn B

Gọi M là trung điểm của BC , G là trọng tâm tam giác ABC .


Dựng đường thẳng d qua G và song song với SA  d   ABC  , d là trục đường tròn ngoại
tiếp tam giác đều ABC .
Dựng đường trung trực cạnh SA , cắt d tại I thì I là tâm mặt cầu ngoại tiếp của hình chóp
S . ABC và bán kính R  IA .
Ta có:

   45
 SBC  ,  ABC    SMA
 SA  AM .tan 45  3a .
2 2 3a
AG  AM 
3 3 .

SA2 5 3a
Bán kính mặt cầu IA  AG 2  IG 2  AG 2   .
4 6
25 a 2
Diện tích mặt cầu: S  4 R 2  .
3
Câu 19. (Chuyên Hạ Long 2022) Cho hình trụ tròn xoay có hai đáy là hai hình tròn (O; R ) và  O; R  .
Tồn tại dây cung AB thuộc đường tròn (O ) sao cho O AB là tam giác đều và mặt phẳng
 O AB  hợp với mặt phẳng chứa đường tròn (O) một góc 60 . Khi đó diện tích xung quanh S xq
hình trụ là
4 R 2
A. S xq  .
7
3 R 2
B. S xq  .
7

Facebook Nguyễn Vương https://www.facebook.com/phong.baovuongTrang 13


Blog: Nguyễn Bảo Vương: https://www.nbv.edu.vn/
3 R 2 7
C. S xq  .
7
6 R 2 7
D. S xq  .
7
Lời giải

Gọi I là trung điểm AB . Khi đó OI  AB .


OO OO OO 2OO
Xét tam giác OOI vuông tại O có OI   và OI   . Mặt khác xét
tan 60 3 sin 60 3
OO2  OO 2 
tam giác OIA vuông tại I có AI 2  R 2  OI 2  R 2   AB 2  4  R 2  .
3  3 
Vì tam giác O AB đều nên
3 3 4 3R
OI  AB  O I 2  AB 2  OO 2  3R 2  OO 2  OO  .
2 4 3 7
6 R 2 7
Diện tích xung quanh hình trụ S xq  2 R  OO  .
7
Câu 20. (Chuyên Hạ Long 2022) Cho hình chóp S  ABCD có đáy ABCD là hình vuông cạnh
a, SA  a 7 và vuông góc với đáy. Lấy điểm M trên cạnh SC sao cho CM  a . Gọi (C ) là
hình nón có đỉnh C , các điểm B , M , D thuộc mặt xung quanh, điểm A thuộc mặt đáy của hình
nón. Tính diện tích xung quanh của (C ) .
16 7 2
A. a .
15
8 30 2
B. a .
15
32 2 2
C. a .
15
16 3 2
D. a .
9
Lời giải

Trang 14 Fanpage Nguyễn Bảo Vương  https://www.facebook.com/tracnghiemtoanthpt489/


Điện thoại: 0946798489 TUYỂN CHỌN VẬN DỤNG – VẬN DỤNG CAO 2022

Lấy điểm E thuộc đoạn thẳng SC sao cho CE  a .


Gọi hình nón  C1  ngoại tiếp hình chóp C.BDE có đỉnh C .
Gọi O  AC  BD . O  BD nên thuộc mặt đáy của hình nón  C1  và CA  2CO , điểm A thuộc
mặt đáy của hình nón (C ) . (1)
Hơn nữa CB  CD  CE  a suy ra ( BDE ) vuông góc với trục của hình nón (C ) và thiết diện
của ( BDE ) với mặt xung quanh của hình nón (C ) là đường tròn, đồng thời ( BDE ) song song với
mặt chứa đáy của hình nón (C ) . (2)
1
Từ (1) và (2) suy ra hình nón  C1  đồng dạng với hình nón (C ) với tỷ số .
2
1 2 2 3 4 2 1 2 30
SC  3a, cos SCB  , ED  EB  2a 2  a 2  a, EO  a  a  a
3 3 3 3 2 6
1 a 30 15 2
S EBD   a 2   a
2 6 6
4a 2
a 2
3 2 30
RBDE  2
 a.
a 15 15
4
6
4a 30 8 30 2
Diện tích xung quanh của hình nón (C ) : S xq     2a  a .
15 15
Câu 21. (Chuyên Lam Sơn 2022) Một cái bình thủy tinh có phần không gian bên trong là một hình nón
có đỉnh hướng xuống dưới theo chiều thẳng đứng. Rót nước vào bình cho đến khi phần không
gian trống trong bình có chiều cao 2 cm . Sau đó đậy kín miệng bình bởi một cái nắp phẳng và lật
ngược bình để đỉnh hướng lên trên theo chiều thẳng đứng, khi đó mực nước cao cách đỉnh của nón
8 cm (hình vẽ minh họa bên dướí).

Biết chiều cao của nón là h  a  b cm . Tính T  a  b .


A. 22.

Facebook Nguyễn Vương https://www.facebook.com/phong.baovuongTrang 15


Blog: Nguyễn Bảo Vương: https://www.nbv.edu.vn/
B. 58.
C. 86.
D. 72.
Lời giải
Để ý rằng có 3 hình nón đồng dạng: Phần không gian bên trong bình thủy tinh (có thể tích V ),
phần không chứa nước khi đặt bình có đỉnh hướng lên (có thể tích V1 ), phần chứa nước khi đặt
bình có đỉnh hướng xuống (có thể tích V2 ). Do tỷ số đồng dạng bằng với tỷ số của chiều cao và tỷ
số thể tích là lập phương tỷ số đồng dạng nên ta có
3 3 3
V h V h 512V (h  2) V
 3;  3
 V1  3 ;V2  .
V1 8 V2 (h  2) h h3
Mà V1  V2  V nên ta có:
512V (h  2)3V
3
 3
 V  512  h3  6h 2  12h  8  h3  h 2  2h  84  0  h  1  85
h h
Vậy T  86
Câu 22. (Chuyên Nguyễn Trãi – Hải Dương – 2022) Nhân dịp năm mới để trang trí một cây thông Noel,
ở sân trung tâm có hình nón ( N ) như hình vẽ sau. Người ta cuộn quanh cây bằng một sợi dây đèn
LED nhấp nháy, bóng đèn hình hoa tuyết từ điểm A đến điểm M sao cho sợi dây luôn tựa trên
mặt nón. Biết rằng bán kính đáy hình nón bằng 8m , độ dài đường sinh bằng 24m và M là điểm
  
sao cho 2MS  MA  0 . Hãy tính chiều dài nhỏ nhất của sợi dây đèn cần có.

A. 8 19(m) .
B. 8 13( m) .
C. 8 7 (m) .
D. 9 12(m) .
Lờí giải
    1  1
Ta có: 2 MS  MA  0  SM  SA  SM  SA  8( m) .
3 3
Trải hình nón ra như hình bên dưới

Trang 16 Fanpage Nguyễn Bảo Vương  https://www.facebook.com/tracnghiemtoanthpt489/


Điện thoại: 0946798489 TUYỂN CHỌN VẬN DỤNG – VẬN DỤNG CAO 2022

Khi đó chu vi đáy của hình nón cũng là độ dài cung AA suy ra 2 R  16 ( m)  l AA .
l 16 2
Góc   ASA  AA  
SA 24 3
Chiều dài nhỏ nhất của sợi dây đèn cần có là đoạn thẳng
2
AM  SA2  SM 2  2SA  SM  cos   242  82  2  24  8  cos  8 13(m).
3
Câu 23. (THPT Kim Liên - Hà Nội - 2022) Cắt hình trụ T  có bán kính R bởi một mặt phẳng song
song với trục và cách trục một khoảng bằng a  0  a  R  ta được thiết diện là một hình vuông có
diện tích 16a 2 . Diện tích xung quanh của hình trụ T  bằng
A. 4 a 2 5 . B.  a 2 5 . C. 8 a 2 5 . D. 16 a 2 5 .
Lời giải
Chọn C

Hình trụ có hai tâm của hai đường tròn đáy là O1 , O2 . Mặt phẳng thiết diện là hình vuông ABCD .
Hình vuông thiết diện ABCD có diện tích là 16a 2 nên cạnh AB  BC  4a .
AB
Gọi I là trung điểm của AB , suy ra IB   2a .
2
Mặt khác, khoảng cách từ trục đến mặt phẳng thiết diện là a nên O1 I  a .
2
Tam giác O1 IB vuông tại I nên R  O1 B  O1 I 2  IB 2  a 2   2a   a 5 .
Diện tích xung quanh hình trụ T  là S xq  2 Rh  2 .R.BC  2 .a 5.4a  8a 2 5 .
a 3
Câu 24. (THPT Võ Nguyên Giáp - Quảng Bình - 2022) Cho tứ diện ABCD có AB  và các cạnh
2
a m
còn lại đều bằng a . Biết rằng bán kính mặt cầu ngoại tiếp tứ diện ABCD bằng với
n
m, n* ; m  15 . Tổng T  m  n bằng
A. 15. B. 17. C. 19. D. 21.
Facebook Nguyễn Vương https://www.facebook.com/phong.baovuongTrang 17
Blog: Nguyễn Bảo Vương: https://www.nbv.edu.vn/
Lời giải
Chọn C

Gọi H , K lần lượt là trọng tâm các tam giác BCD , ACD . Do các tam giác BCD , ACD đều nên
a 3 1 a 3 a 3 AB 3 3a
AN  BN   NH  NK  .  , suy ra  ABN đều  MN   .
2 3 2 6 2 4
Do đó trục các tam giác BCD , ACD cắt nhau tại O trên đường cao NM của tam giác ABN và
O là tâm mặt cầu ngoại tiếp tứ diện.
  a 3 . tan 30  a .
Ta có: OH  NH .tan ONH
6 6
a 2 a 2 a 13
Từ đo: R  OB  OH 2  BH 2     m  13, n  6  T  m  n  19 .
36 3 6
Câu 25. (THPT Yên Lạc - Vĩnh Phúc - 2022) Cho hình trụ  T  chiều cao bằng 2a , hai đường tròn đáy
của T  có tâm lần lượt là O và O1 , bán kính bằng a . Trên đường tròn đáy tâm O lấy điểm A ,
trên đường tròn đáy tâm O1 lấy điểm B sao cho AB  5a . Thể tích khối tứ diện OO1 AB bằng
3a 3 3a 3 3a 3 3a 3
A. . B. . C. . D. .
12 3 4 6
Lời giải
Chọn D

O1
B

O
H
A C

Gọi C là điểm trên đường tròn tâm O sao cho BC || OO1 . Khi đó tứ giác OCBO1 là hình chữ nhật
và có diện tích là SOCBO1  2a 2 .
Xét tam giác vuông ABC có AC  AB 2  BC 2  a nên tam giác OAC đều.
a 3
Gọi H là trung điểm của OC , ta có AH  OC và AH  .
2
Mặt khác  ACO    OCBO1  nên AH   OCBO1  .

Trang 18 Fanpage Nguyễn Bảo Vương  https://www.facebook.com/tracnghiemtoanthpt489/


Điện thoại: 0946798489 TUYỂN CHỌN VẬN DỤNG – VẬN DỤNG CAO 2022
1 a3 3
Thể tích khối chóp A.OCBO1 là VA.OCBO1  AH .SO. ACBO1  .
3 3
1 a3 3
Thể tích khối tứ diện OO1 AB bằng thể tích khối chóp A.OBO1  VAOBO1  VA.OCBO1  .
2 6
Câu 26. (THPT Yên Lạc - Vĩnh Phúc - 2022) Một khối nón có bán kính đáy bằng 2 cm , chiều cao bằng
3 cm . Một mặt phẳng đi qua đỉnh và tạo với đáy một góc 60 chia khối nón làm 2 phần. Tính
thể tích phần nhỏ hơn (kết quả làm tròn đến hàng phần trăm).
A. 2, 47 cm 3 . B. 2,36 cm3 . C. 1, 42 cm3 . D. 1,53 cm3 .

Lời giải
Chọn C

Mặt phẳng  SMN  qua đỉnh và tạo với đáy một góc 60 .
SI  MN  
Ta có   SIO  60 là góc giữa  SMN  và đáy.
OI  MN 
SO 3 1
OI    1  OI  OA  OMAN là hình bình hành, có 2 đường chéo OA  MN
tan 60 3 2
Nên tứ giác OMAN là hình thoi, có NA  NO  OA  2     120 .
ANO  60  MON
 R 2 n  .22.120 4
Squ¹t OMN   
360 360 3
1 1 3
S OMN  OM .ON .sin120  2.2.  3.
2 2 2
4
Nên S MAN
  S qu¹t OMN  S OMN   3
3
1 1  4 
Thể tích khối nhỏ cần tính: V  .SO.S MAN   . 3.   3   1, 42 cm3 .
3 3  3 
Câu 27. (THPT Yên Phong 1 - Bắc Ninh - 2022) Cho hình nón đỉnh S tâm O có độ dài đường sinh
bằng SA  a , đường kính đáy AB . Thiết diện qua đỉnh tạo với đáy một góc 60 cắt đường tròn
2a 3
đáy theo dây cung MN  . Biết rằng khoảng cách từ A đến MN bằng a . Thể tích khối nón
3
bằng:
a 3 2 a3 6 a 3 6 a 3 6
A. . B. . C. . D. .
12 18 9 3

Lời giải
Chọn A
Biết rằng khoảng cách từ A đến MN bằng a  MN không vuông góc với AB .

Facebook Nguyễn Vương https://www.facebook.com/phong.baovuongTrang 19


Blog: Nguyễn Bảo Vương: https://www.nbv.edu.vn/
Ta có hình vẽ sau:

  60 là góc giữa  SMN  và mặt phẳng


Gọi I là trung điểm MN thì SI  MN , OI  MN  SIO
đáy.
2a 3 a 3
MN   IM  IN  .
3 3
2
2 2
2 2 2 2
a 3 2 a 2 a
Xét OIN vuông ở I có: OI  ON  IN  R     R   OI  R  .
 3  3 3
2
2 2
 a 3  2a 2
2 2 a 2
Xét SIM vuông ở I có: SI  SM  MI  a      SI  .
 3  3 3

a2
R2  2 2
OI 3  3R  a  1
Xét SIO vuông ở O có: cos 60  
SI a 2 a 2 2
3

a2 a 2
 4  3R  a   2a  R 
2 2 2 2
R .
2 2
SO 2a 3 a 2
sin 60   SO  SI .sin 60    .
SI 3 2 2

1 1 2a a 2  2a 3
V   SO.  R 2      .
3 3 2 2 12

Theo dõi Fanpage: Nguyễn Bảo Vương  https://www.facebook.com/tracnghiemtoanthpt489/

Hoặc Facebook: Nguyễn Vương  https://www.facebook.com/phong.baovuong

Tham gia ngay: Nhóm Nguyễn Bào Vương (TÀI LIỆU TOÁN)  https://www.facebook.com/groups/703546230477890/

Ấn sub kênh Youtube: Nguyễn Vương


 https://www.youtube.com/channel/UCQ4u2J5gIEI1iRUbT3nwJfA?view_as=subscriber

Tải nhiều tài liệu hơn tại: https://www.nbv.edu.vn/

Trang 20 Fanpage Nguyễn Bảo Vương  https://www.facebook.com/tracnghiemtoanthpt489/


TUYỂN CHỌN VẬN DỤNG – VẬN DỤNG CAO 2022 Điện thoại: 0946798489

VẤN ĐỀ 7. OXYZ
• |FanPage: Nguyễn Bảo Vương
• TUYỂN CHỌN CÂU HỎI VD-VDC TỪ CÁC ĐỀ THI THỬ CÁC TRƯỜNG, CÁC SỞ NĂM 2022

Câu 1. (Chuyên Vinh – 2022) Trong không gian Oxyz , cho mặt phẳng ( P ) : 2 x  y  2 z  16  0 và mặt
cầu ( S ) : ( x  2) 2  ( y  1)2  ( z  3)2  21 . Một khối hộp chữ nhật ( H ) có bốn đỉnh nằm trên mặt
phẳng ( P ) và bốn đỉnh còn lại nằm trên mặt cầu ( S ) . Khi ( H ) có thể tích lớn nhất, thì mặt phẳng
chứa bốn đỉnh của ( H ) nằm trên mặt cầu ( S ) là (Q ) : 2 x  by  cz  d  0 . Giá trị b  c  d bằng
A. 15 .
B. 13 .
C. 14 .
D. 7 .
Lời giải
Chọn B

Đầu tiên, ta có mặt cầu ( S ) tâm I (2; 1;3) , bán kính R  21


Tiếp đến ta nhận thấy: d ( I ;( P ))  9  21 nên suy ra mặt phẳng ( P ) không cắt mặt cầu ( S ) . Gọi
a, b là các kích thước mặt đáy hình hộp chữ nhật và d  d ( I ; (Q )) . Khi đó ta có:
2
 ab
  (9  d )  21  d   f ( d )
2
Ta có: V  ( d ( I ; ( P ))  d ( I ;(Q)))ab  (9  d ) ab  (9  d ) 
 2 
Xét hàm số y  f ( d )  (9  d )  21  d  trên (0; ) có f ( d )  0  d  1  max
2

f ( d )  f (1)

Suy ra thể tích khối hộp đạt max khi và chỉ khi d  d ( I ;(Q))  1, ( P)‖(Q )
|11  d |  d  8  Q  : 2 x  y  2 z  8  0
Suy ra: (Q ) : 2 x  y  2 z  d  0; d ( I ;(Q))  1   1
3  d  14  Q2  : 2 x  y  2 z  14  0
Xét điểm N (0; 0; 8) bất kì thuộc mặt phẳng ( P ) ta nhận ra để thể tích max thì chiều cao hộp
phải max tức hai điểm I và N phải nằm cùng phía với mặt phẳng (Q )
Như vậy ta nhận mặt phẳng  Q2  : 2 x  y  2 z  14  0 do
 2 xI  yI  2 zI  14  2 xN  yN  2 zN  14   0 Với (Q ) : 2 x  by  cz  d  0 , ta đồng nhất hệ số ra
b  c  d  13 .
Câu 2. (THPT Kim Liên – Hà Nội – 2022) Trong không gian Oxyz , cho mặt cầu
2 2 2
( S ) : ( x  1)  ( y  3)  ( z  2)  25 và hai đường thẳng  d1  ,  d2  lần lượt có phương trình

Facebook Nguyễn Vương https://www.facebook.com/phong.baovuong Trang 1


Blog: Nguyễn Bảo Vương: https://www.nbv.edu.vn/
 x  1  mt x  1 t
 
d1 :  y  1 và d 2 :  y  1 . Có bao nhiêu giá trị của m để  d1  ,  d 2  cắt mặt cầu ( S ) tại 4
z  1 t  z  1  mt
 
điểm phân biệt sao cho bốn điểm đó tạo thành tứ giác có diện tích lớn nhất
A. 0.
B. 3.
C. 2.
D. 1.
Lời giải
Chọn C
Ta có đánh giá khác như sau: (Gọi RC là bán kính của đường tròn thiết diện (C ) )
1 1  2 R 2  d 2  O; MP   d 2  O; NQ  
S MNPQ  MP  NQ   2 RC 2  d 2  O; MP   2 RC 2  d 2  O; NQ   2  C 
2 2  2 
 2 RC 2  O A2  const với O( 1; 1; 2) là tâm đường tròn (C ) .
Dấu bằng xảy ra khi và chỉ khi
 

d  O; MP   d  O; NQ   d  O; d1   d  O; d 2   O A;u1   O A; u  ∣
 2

m  3
| 2  m || 2m  1|  , Vậy chỉ có 2 giá trị m thỏa mãn yêu cầu đề bài.
m   1
 3
Câu 3. (Chuyên Vinh – 2022) Trong không gian Oxyz , cho mặt cầu
2 2 2
( S ) : x  y  z  4 x  12 y  6 z  24  0 . Hai điểm M , N thuộc ( S ) sao cho MN  8 và
OM 2  ON 2  112 . Khoảng cách từ O đến đường thẳng MN bằng
A. 4.
B. 3.
C. 2 3 .
D. 3 .
Lời giải
Chon B
Phương trình mặt cầu ( S ) : x 2  y 2  z 2  4 x  12 y  6 z  24  0; ta có I (2; 6; 3), R  5 và
OI  7 .
       
OM 2  ON 2  (OI  IM )2  (OI  IN )2  2OI ( IM  IN )  2OI MN
   
 2  OIMN  cos(OI , MN )  112  cos(OI , MN ).
Khi đó OM 2  ON 2  112  cos(OI , MN )  1 .
 
Suy ra OI và MN ngược hướng hay OI / / MN (vì O  MN ).
2
 MN  2
Vậy d (O, MN )  d ( I , MN )  R     3.
 2 
Câu 4. (Chuyên Lê Quý Đôn - Điện Biên - 2022) Trong không gian với hệ tọa độ Oxyz , cho mặt cầu
2 2 2
 S  :  x  1   y  3   z  4   5 và điểm M 1; 4; 2  . Xét điểm N thuộc mặt cầu  S  sao
cho đường thẳng MN tiếp xúc với mặt cầu  S  . Khi đó điểm N luôn nằm trên mặt phẳng có
phương trình là:
A. 2 x  y  z  2  0 . B. x  y  z  1  0 .
C. 2 x  y  2 z  2  0 . D. 2 x  y  2 z  2  0 .
Lời giải
Trang 2 Fanpage Nguyễn Bảo Vương  https://www.facebook.com/tracnghiemtoanthpt489/
Điện thoại: 0946798489 TUYỂN CHỌN VẬN DỤNG – VẬN DỤNG CAO 2022
Chọn C
Mặt cầu  S  có tâm I  1;3; 4  , bán kính R  5
Từ tính chất tiếp tuyến ta có MN 2  MI 2  R 2  9  5  4  MN  2 , do đó N thuộc mặt cầu tâm
M 1; 4; 2  , bán kính R  2 có phương trình x 2  y 2  z 2  2 x  8 y  4 z  17  0 1 .
N thuộc  S  viết dạng x 2  y 2  z 2  2 x  6 y  8 z  21  0  2  .
Trừ các vế (1) cho (2) và rút gọn ta được: 2 x  y  2 z  2  0 .
Vậy điểm N luôn nằm trên mặt phẳng có phương trình là: 2 x  y  2 z  2  0
Câu 5. (Chuyên Lê Quý Đôn - Điện Biên - 2022) Trong không gian với hệ trục tọa độ Oxyz , cho điểm
A  a; b; c  với a; b; c là các số thực dương thỏa mãn 5  a 2  b 2  c 2   9  ab  2bc  ca  và
a 1
Q  có giá trị lớn nhất. Gọi M , N , P lần lượt là hình chiếu vuông góc của
b  c  a  b  c 3
2 2

A lên các tia Ox, Oy , Oz . Phương trình mặt phẳng  MNP  là


A. 3 x  12 y  12 z  1  0 .B. x  4 y  4 z  12  0 .
C. 3 x  12 y  12 z  1  0 .D. x  4 y  4 z  0 .
Lời giải
Chọn A
t2 t2
Đặt t  b  c  t  0  ; b2  c 2  ; bc  .
2 4
2
5  a 2  b 2  c 2   9  ab  2bc  ca   5a 2  5  b  c   9a  b  c   28bc

 5a 2  5t 2  9at  7t 2   5a  t  a  2t   0  a  2t .

4 1
Vậy Q    f  t  với t  0 .
t 27t 3
4 1 1
Ta có f   t    2
 4 , f   t   0  t  (vì t  0 ). Ta có bảng biến thiên
t 9t 6

1 1 1
Vậy Qmax  16  t   a  ;b  c  .
6 3 12

1 1 1  1   1   1
Suy ra tọa độ điểm A  ; ;  ; tọa độ các điểm M  ;0;0  ; N  0; ;0  ; P  0;0;  .
 3 12 12  3   12   12 

x y z
Phương trình mặt phẳng  MNP  :    1  3 x  12 y  12 z  1  0 .
1 1 1
3 12 12

Facebook Nguyễn Vương https://www.facebook.com/phong.baovuongTrang 3


Blog: Nguyễn Bảo Vương: https://www.nbv.edu.vn/
Câu 6. (Cụm Trường Nghệ An - 2022) Trong không gian Oxyz , cho ba điểm A(1; 0; 0) , B (0; 2;3) ,
2
C (1;1;1) . Gọi ( P ) là mặt phẳng chứa A , B sao cho khoảng cách từ C đến ( P ) bằng . Tìm
3
tọa độ giao điểm M của ( P ) và trục Oy .
 23   23 
A. M (0; 1;0) hoặc M  0; ;0  . B. M (0;1; 0) hoặc M  0;  ; 0  .
 37   37 
 23   23 
C. M (0; 1; 0) hoặc M  0;  ; 0  . D. M (0;1; 0) hoặc M  0;  ; 0  .
 37   37 
Lời giải
Chọn D

Gọi n( P )  (a; b; c) là một véctơ pháp tuyến của mặt phẳng (P), với a 2  b 2  c 2  0 .
Ta có:
  
AB  (1; 2;3) ; ( P)  AB  n( P ) . AB  0  a  2b  3c  0  a  2b  3c .
( P ) : a ( x  1)  by  cz  0  ( P ) : ax  by  cz  a  0 .
bc 2 2
d  C; ( P)     3 b  c  2  2b  3c   b2  c 2
2 2 2
a b c 3
2
 3  b  c   4  5b 2  12bc  10c 2   17b 2  54bc  37c 2  0 ,(1).
Với c  0 , từ phương trình (1) suy ra b  0, a  0 không thỏa mãn.
Với c  0 , chia hai vế phương trình (1) cho c 2 ta được:
b
b
2
b c 1
17    54    37  0   .
c c  b  37
 c 17
b
+/ Trường hợp  1 , chọn c  1  b  1, a  1 .
c
Khi đó, ta có ( P ) : x  y  z  1  0 , suy ra ( P)  Oy  M  0;1;0  .
b 37
+/ Trường hợp  , chọn c  17  b  37, a  23 .
c 17
 23 
Khi đó, ta có ( P ) : 23 x  37 y  17 z  23  0 , suy ra ( P )  Oy  M  0;  ;0  .
 37 
Câu 7. (Cụm Trường Nghệ An - 2022) Trong không gian Oxyz , cho các điểm A  0;0;3 và B  2; 3; 5  .
Gọi  P là mặt phẳng chứa đường tròn giao tuyến của hai mặt cầu
2 2 2
 S1  :  x  1   y  1   z  3  25 và  S2  : x2  y2  z 2  2x  2 y 14  0 . Gọi M , N là hai
điểm thuộc  P sao cho MN  1 . Biết giá trị nhỏ nhất của AM  BN có dạng a b c
( a , b, c   và c là số nguyên tố). Tính a  b  c
A. 80 . B. 93 . C. 89 . D. 90 .
Lời giải
Chọn B
Mặt phẳng  P  là giao tuyến của hai mặt cầu  S1  và  S2  nên thỏa mãn hệ
2 2 2
 x  y  z  2 x  2 y  6 z  14  0
 2 2 2
 6 z  0  z  0 . Vậy  P    Oxy  .
 x  y  z  2 x  2 y  14  0
Nhận thấy A và B nằm khác phía với nhau so với mặt phẳng  P  .

Trang 4 Fanpage Nguyễn Bảo Vương  https://www.facebook.com/tracnghiemtoanthpt489/


Điện thoại: 0946798489 TUYỂN CHỌN VẬN DỤNG – VẬN DỤNG CAO 2022

Gọi A  0;0;0 và B  2; 3;0 lần lượt là hình chiếu của A và B lên  P  .


Suy ra: AA  3 , BB  5 , AB  13 .
2 2
Lại có: AM  BN  AA2  A ' M 2  BB 2  B N 2   AA  BB    AM  BN 
Mặt khác AM  MN  B N  AB   AM  B N  AB   MN  13  1 .
2
Suy ra AM  BN   3  5
2
  
13  1  78  2 13 .
Vậy a  78 , b  2 , c  13 . Vậy a  b  c  93 .
Câu 8. (Đại học Hồng Đức – 2022) Trong không gian Oxyz , cho hai đường thẳng
x 1 y 1 z  2 x 1 y  3 z 1
 d1  :   ,  d2  :   và điểm A(4;1; 2) . Gọi  là đường thẳng
2 1 2 1 2 3

qua A cắt d1 và cách d 2 một khoảng lớn nhất. Lấy u  ( a;1; c ) là một véctơ chỉ phương của  .

Độ dài của u là
A. 3 5 .
B. 86 .
C. 3 .
D. 85 .
Lời giải.

Gọi H là hình chiếu của A lên d 2 , khi đó  nằm trên mặt phẳng ( P ) qua A và nhận AH là
véctơ pháp tuyến.
  
Gọi (Q ) là mặt phẳng chứa A và  d1  . Khi đó   ( P )  (Q )  u   nP , nQ  .

Giả sử H (1  t ; 3  2t ;1  3t )  AH  (t  3; 2t  4;3t  1) .
Ta
    có

AH  ud2 , ud2  (1; 2;3)  t  3  4t  8  9t  3  0  t  1  AH  (2; 2; 2)  nP  (1;1; 1) .
   
Lấy N (1;1; 2)   d1   AN  (5;0; 4)  nQ  ud1 , AN   (4; 2; 5) .
   
Suy ra u   nP , nQ   ( 7;1; 6) | u | 86 .
Câu 9. (THPT Hồ Nghinh – Quảng Nam – 2022) Trong không gian với hệ trục tọa độ Oxyz , cho ba mặt
phẳng ( P ) : x  y  z  5  0 ; (Q ) : x  y  z  1  0 và ( R ) : x  y  z  2  0 . Úng với mỗi cặp điểm
A, B lần lượt thuộc hai mặt phẳng ( P ), (Q ) thì mặt cầu đường kinh AB luôn cắt mặt phẳng ( R )
theo một đường tròn. Tìm bán kính nhỏ nhất của đường tròn đó.

Facebook Nguyễn Vương https://www.facebook.com/phong.baovuongTrang 5


Blog: Nguyễn Bảo Vương: https://www.nbv.edu.vn/
1
A. .
3
2
B. .
3
C. 1.
1
D. .
2
Lời giải
Dễ thấy ba mặt phẳng ( P ), (Q ), ( R ) song song với nhau và mặt phẳng ( R ) nằm giữa hai mặt
phẳng ( P ), (Q ) .
Gọi ( ) : x  y  z  D  0 là mặt phẳng cách đều hai mặt phẳng ( P ), (Q ) .
| D  5 | | D  1|
Ta có   D  3  ( ) : x  y  z  3  0 .
3 3
1
Suy ra khoảng cách giưa hai mặt phẳng ( R ), ( ) là d  .
3
Khi đó mặt cầu đường kinh AB có tâm I luôn thuộc mặt phẳng ( ) cách đều hai mặt phẳng
( P ), (Q ) .
AB 2
Mặt cầu tâm I luôn cắt mặt phẳng ( R ) theo một đường tròn có bán kinh là r   d 2 . Để
4
4 AB 2
rmin thì ABmin  d [( P), (Q)]  . Vậy r   d 2  1.
3 4
Câu 10. (THPT Hương Sơn - Hà Tĩnh - 2022) Trong không gian với hệ trục tọa độ Oxyz , cho đường
x  2 y 1 z 2 2 2
thẳng d :   và mặt cầu  S  :  x  2    y  1   z  1  6 . Hai mặt phẳng
2 3 1
 P  ,  Q  chứa d và cùng tiếp xúc với  S  lần lượt tại A, B . Gọi I tà tâm mặt cầu  S  . Giá trị
cos 
AIB bằng
1 1 1 1
A.  . B. . C.  . D. .
9 9 3 3
Lời giải
Chọn A

Mặt cầu  S  có tâm I  2; 1; 1 và bán kính R  6 .


 x  2  2t
 
Phương trình tham số của đường thẳng d :  y  1  3t , ud   2; 3;1 .
z  t

Trang 6 Fanpage Nguyễn Bảo Vương  https://www.facebook.com/tracnghiemtoanthpt489/
Điện thoại: 0946798489 TUYỂN CHỌN VẬN DỤNG – VẬN DỤNG CAO 2022
Gọi H là hình chiếu của I lên d . Vì H  d  H  2  2t ; 1  3t; t 

 IH   4  2t; 3t; t  1 .
  1  5 1 3 6
Khi đó, IH .ud  0  2  4  2t   3  3t    t  1  0  t   H  1;  ;  và IH  .
2  2 2 2
Gọi M là hình chiếu của A lên IH .
IA2 R 2 2 6
Xét tam giác AIH vuông tại A có: IA2  IM .IH  IM    .
IH IH 3
30 2 30
Xét tam giác AIM vuông tại M có AM 2  IA2  IM 2  R 2  IM 2   AB  .
3 3
2 30
Tam giác AIB có IA  IB  6, AB  . Áp dụng định lý côsin trong tam giác AIB ta có:
3
IA2  IB 2  AB 2 1
cos AIB   .
2 IA.IB 9
Câu 11. (THPT Hương Sơn - Hà Tĩnh - 2022) Trong không gian với hệ toạ độ Oxyz , cho mặt phẳng
x y2 z2
   : x  y  2 z  2  0 và đường thẳng  :    Đường thẳng   là hình chiếu vuông
2 2 1
góc của đường thẳng  trên mặt phẳng    có phương trình:
x8 y 6 z  2 x 8 y 6 z  2
A.   . B.   .
3 5 4 3 5 4
x 1 y 1 z 1 x 1 y 1 z 1
C.   . D.   .
7 5 1 7 5 1
Lời giải
Chọn C

Ta có    : x  y  2 z  2  0  n  1;1;  2  .
x y2 z2 
:     u   2;  2;1 .
2 2 1
  
Gọi    là mặt phẳng qua  và vuông góc với     n   n , u    3;  5;  4  .
Đường thẳng   là hình chiếu vuông góc của đường thẳng  lên mặt phẳng    có VTCP:
   
u   n , n   14;  10; 2  . Chọn VTCP của   là u   7; 5;1 .

x y2 z 2  x  8
   
Gọi M         2 2 1   y  6  M  8;6;  2  .
 x  y  2 z  2  0  z  2


Đường thẳng   qua M  8;6;  2  và có VTCP u   7;  5;1 phương trình dạng tham số:
 x  8  7t

 y  6  5t . Với t  1  I  1;1;  1   .
 z  2  t

x 1 y 1 z 1
Phương trình đường thẳng   dạng:
  .
7 5 1
Câu 12. (THPT Lê Thánh Tông - HCM-2022) Trong không gian với hệ trục tọa độ Oxyz cho hai mặt
2
cầu  S1  :  x  5   y 2  z 2  25 ,
2
 S2  :  x  5   y 2  z 2  100 và điểm K  8;0; 0  . Đường thẳng  di động nhưng luôn tiếp xúc
với  S1  , đồng thời cắt  S2  tại hai điểm M , N . Tam giác KMN có diện tích lớn nhất bằng
A. 90 3 . B. 50 6 . C. 100 2 . D. 100 3 .
Facebook Nguyễn Vương https://www.facebook.com/phong.baovuongTrang 7
Blog: Nguyễn Bảo Vương: https://www.nbv.edu.vn/
Lời giải
Chọn A

Ta có: I1  I 2   5;0;0  , R1  5, R2  10, IK  13  R2 .

Ta có R1 , R2 không đổi nên MN  2 R22  R12  10 3 .

Do đó, tam giác KMN có diện tích lớn nhất khi d  K , MN  lớn nhất.

Ta thấy rằng d  K , MN  lớn nhất là bằng KI  R1  18 . Dấu “=” xảy ra khi K , I , T thẳng hàng
(với T là tiếp điểm).

Suy ra, tam giác KMN có diện tích lớn nhất bằng:

1 1
SKMN  .d  K , MN  .MN  .18.10 3  90 3 .
2 2

Câu 13. (Liên trường Hà Tĩnh-2022) Trong không gian với hệ trục tọa độ Oxyz cho
2 2
A( a; 0; 0), B (0; b; 0), C (0; 0; c ) với a, b, c  0 sao cho 2OA  OB  OC  5 OB  OC  36 . Tính
a  b  c khi thể tích khối chóp O. ABC đạt giá trị lớn nhất
A. 1.
B. 5
36  36 2
C.
5
D. 7
Lời giải
Chon B
Từ 2OA  OB  OC  5 OB 2  OC 2  36  2a  b  c  5 b 2  c 2  36
Ta có
(4b) 2 (3c) 2 (4b  3c) 2
36  2a  b  c  5 b 2  c 2  2a  b  c  5   2a  b  c  5  2a  b  c  4b
16 9 16  9

1
 2a  3b  4c  3 3 2a.3b.4c  3 3 24abc  368  27.24abc  abc  72  abc  12
6

Trang 8 Fanpage Nguyễn Bảo Vương  https://www.facebook.com/tracnghiemtoanthpt489/


Điện thoại: 0946798489 TUYỂN CHỌN VẬN DỤNG – VẬN DỤNG CAO 2022
 4b 3c
 16  9 a  6
 
 Vmnx  12  2a  3b  4c  b  4
 
2
36  2a  b  c  5 b  c
2
c  3

Vậy a  b  c  5
Câu 14. (THPT Nguyễn Tất Thành-Đh-SP-HN-2022) Trong không gian tọa độ Oxyz , cho đường thẳng
x 1 y 1 z 1
d:   và hai điểm A  6;0;0 , B  0;0; 6  . Khi M thay đổi trên đường thẳng d ,
2 2 1
hãy tìm giá trị nhỏ nhất của biểu thức P  MA  MB
A. min P  6 3 . B. min P  6 2 . C. min P  9 . D. min P  12 .
Lời giải
Chọn A
Cách 1:
 x  1  2t

Ta có d :  y  1  2t , vì M  d  M 1  2t;1  2t; 1  t  .
 z  1  t

2 2 2 2 2 2
P  MA  MB   2t  5  1  2t    t  1   2t  1  1  2t    t  5

 2  
2 2


2
 9t 2  18t  27  9t 2  18t  27  3  1  t    2   t  1
2

 
  
Gọi u  1  t; 2 ; v  t  1; 2 . 
    
 2    2
2 2
2
 
Ta có u  v  u  v   1  t   2   t  1 

2
3.

1 t
Dấu "  " xảy ra   1  t  0 . Suy ra P  6 3 .
1 t
Vậy min P  6 3 .
Cách 2:

Q
A2 B
A

d
I M
(C)

A1

+) Gọi  P  là mặt phẳng đi qua A và vuông góc với đường thẳng d .


Gọi  Q là mặt phẳng chứa đường thẳng d và đi qua điểm B .

Facebook Nguyễn Vương https://www.facebook.com/phong.baovuongTrang 9


Blog: Nguyễn Bảo Vương: https://www.nbv.edu.vn/
Gọi    P    Q  .
Gọi I là hình chiếu vuông góc của A lên đường thẳng d và  C  là đường tròn tâm I , bán kính
AI và đường tròn  C  thuộc mặt phẳng  P  .
Gọi A1 , A2 là giao điểm của đường thẳng  và đường tròn  C  (trong đó A1 và B nằm về hai
phía của đường thẳng d ).
Suy ra MA1  MA  MA  MB  MA1  MB  A1 B  const .
+) Ta có d  A; d   d  B; d   3 2  d  A1 ; d   d  A2 ; d   d  B; d   A1 A2  A2 B .
2 2
 A1B   A1 A2    A2 B   4IA2  d 2  B,  P    4d 2  A, d   d 2  B,  P  
 
+) Mặt phẳng  P  đi qua A  6;0;0 và có vectơ pháp tuyến nP  ud   2; 2;1
  P  có phương trình: 2 x  2 y  z  12  0  d  B,  P    6 .
2
Vậy giá trị nhỏ nhất của biểu thức P  MA  MB là A1 B  62  4. 3 2   6 3.
Câu 15. (THPT Nguyễn Tất Thành-Đh-SP-HN-2022) Trong không gian tọa độ Oxyz cho mặt phẳng
 P  : x  2 y  2 z  11  0 và điểm I  3;3;1 . Gọi  S  là mặt cầu có tâm là điểm I và cắt mặt
phẳng  P  theo một đường tròn có chu vi bằng 8 . Phương trình của mặt cầu  S  là
2 2 2 2 2 2
A.  x  3    y  3    z  1  52 . B.  x  3    y  3    z  1  64 .

2 2 2 2 2 2
C.  x  3    y  3    z  1  64 . D.  x  3    y  3    z  1  52 .

Lời giải
Chọn A
 Gọi r 2r.  8  r  4.
là bán kính đường tròn giao tuyến, có
3  2.3  2.1  11 18
 Có d  I ,  P    6.
1 4  4 3
 Gọi R là bán kính của mặt cầu  S  , có R  r 2  d 2  42  62  2 13.
2 2 2
 Suy ra  S  :  x  3    y  3    z  1  52.
Câu 16. (THPT Nguyễn Tất Thành-Đh-SP-HN-2022) Trong không gian tọa độ cho đường thẳng
x 1 y  2 z
d:   và hai điểm A 1; 1;1 , B  4; 2; 2  . Gọi  là đường thẳng đi qua A và
2 2 1
vuông góc với d sao cho khoảng cách từ điểm B đến  là nhỏ nhất. Phương trình của đường
thẳng  là
x 1 y 1 z 1 x 1 y 1 z 1
A.   . B.   .
1 1 4 1 1 4

x 1 y 1 z 1 x 1 y 1 z 1
C.   . D.   .
1 1 4 1 1 4

Lời giải
Chọn D

Trang 10 Fanpage Nguyễn Bảo Vương  https://www.facebook.com/tracnghiemtoanthpt489/


Điện thoại: 0946798489 TUYỂN CHỌN VẬN DỤNG – VẬN DỤNG CAO 2022

Đường thẳng đã cho nằm trong mặt phẳng qua A và vuông góc với d
 P  : 2( x  1)  2( y  1)  z  1  0  2 x  2 y  z  1  0
 x  4  2t

Kẻ BH   P   ( BH ) :  y  2  2t
 z  2  t

Khi đó H   P   2(4  2t )  2(2  2t )  2  t  1  0  t  1  H  2;0; 3 .
Nhận xét BH  BA , đường thẳng cần tìm là đường thẳng AH .
 x 1 y 1 z 1
Trong đó AH  1;1; 4    :   .
1 1 4
Câu 17. (Nho Quan A – Ninh Bình – 2022) Trong không gian với hệ tọa độ Oxyz , xét ba điểm
1 1 1
A( a; 0; 0), B (0; b; 0), C (0; 0; c ) thỏa mãn    1. Biết rằng mặt cầu
a b c
( S ) : ( x  2)2  ( y  1) 2  ( z  3)2  25 cắt mặt phẳng ( ABC ) theo giao tuyến là đường tròn có bán
kính là 4. Giá trị của biểu thức a  b  c là
A. 1.
B. 2.
C. 3.
D. 5.
Lời giải

x y z
Theo phương trình đoạn chắn ta có phương trình mặt phẳng ( ABC ) là:    1.
a b c
1 1 1
Với giả thiết    1 . Ta thấy mặt phẳng luôn đi qua điểm H (1; 1;1) .
a b c
Mặt cầu ( S ) : ( x  2)2  ( y  1)2  ( z  3) 2  25 có tâm I (2;1;3), Rmc  5 .
Ta gọi K là hình chiếu vuông góc của I lên mặt phẳng ( ABC ) .
2
IK  Rmc  r 2  3 và ta thấy IH  3 và IH  IK nên ta có H trùng với điểm K .

Facebook Nguyễn Vương https://www.facebook.com/phong.baovuongTrang 11


Blog: Nguyễn Bảo Vương: https://www.nbv.edu.vn/
  x y 1
( ABC ) qua H (1; 1;1) và có VTPT n  HI  (1; 2; 2)  x  2 y  2 z  1  0     1 .
1 1 1
2 2
Vậy a  b  c  2 .
x  1 t

Câu 18. (THPT Phù Cừ - Hưng Yên - 2022) Trong không gian Oxyz , cho đường thẳng d :  y  1 và
z  t

mặt phẳng ( P ) : 2 x  z  3  0 . Biết đường thẳng  đi qua điểm O (0; 0; 0) gốc toạ độ, có 1 vectơ

chỉ phương u  (1; a; b) , vuông góc với đường thẳng d và hợp với mặt phẳng ( P ) một góc lớn
nhất. Hỏi điểm nào sau đây thuộc đường thẳng  ?
A. P (0;1; 0) . B. M (2; 0; 2) . C. N ( 1;1;1) . D. Q (1; 2; 2) .
Lời giải

Ta có đường thẳng  có một vectơ chỉ phương là u  (1; a; b) .
x  1 t

Mà đường thẳng d có phương trình d :  y  1 nên suy ra một vectơ chỉ phương của d là
z  t


v  (1; 0;1) .
   
Ta lại có   d  u  v  u  v  0  1  b  0  b  1 . Suy ra u  (1; a; 1) .
Mặt khác, mặt phẳng ( P ) có phương trình ( P ) : 2 x  z  3  0 nên có một vectơ pháp tuyến là

n  (2; 0; 1) .
Giải sử  hợp với mặt phẳng ( P ) một góc   ( , ( P )) thì
 
  | u, n | |3| 3
sin  | cos(u , n ) |     .
| u || n | 5 2a 2
5  2  a2 
1 1 3 3
Mà 2  a2  2    sin    khi a  0 .
2a 2
2 
5 2a 2

10
Vì  lớn nhất khi sin  lớn nhất do đó  max khi a  0 .
x  s
 
Suy ra u  (1; 0; 1) . Vậy phương trình đường thẳng  là  y  0 . Suy ra điểm M (2; 0; 2) thuộc
 z  s

đường thẳng  .
Câu 19. (THPT Phù Cừ - Hưng Yên - 2022) Trong không gian Oxyz , cho mặt cầu ( S ) tâm I (2; 1;3)
bán kính R  4 và mặt cầu  S1  : x 2  y 2  z 2  4 x  6 z  2  0 . Biết mặt phẳng ( P ) là giao của hai
mặt cầu ( S ) và  S1  . Gọi M , N là hai điểm thay đổi thuộc mặt phẳng ( P ) sao cho MN  2 .
Giá trị nhỏ nhất của AM  BN bằng a  b 2 , với a, b   và A(0;5; 0), B (3; 2; 4) . Tính giá
b
trị gần đúng của (làm tròn đến hàng phần trăm).
a
A. 0,05. B. 0,07. C. 0,11. D. 0,13.
Lời giải

Trang 12 Fanpage Nguyễn Bảo Vương  https://www.facebook.com/tracnghiemtoanthpt489/


Điện thoại: 0946798489 TUYỂN CHỌN VẬN DỤNG – VẬN DỤNG CAO 2022

Ta có ( S ) : ( x  2) 2  ( y  1) 2  ( z  3) 2  16  x 2  y 2  z 2  4 x  2 y  6 z  2  0 .
Vi( P)  ( S )   S1   ( P) : y  0  ( P)  (Ozx) .
Ta có O (0; 0; 0), C (3; 0; 4) lần lượt là hình chiếu vuông góc của A(0;5; 0), B (3; 2; 4) xuống mặt
phẳng ( P ) .
Mà OA  5; OC  5; BC  2 .
Do đó
AM  BN  OA2  OM 2  BC 2  CN 2  (OA  BC ) 2  (OM  CN ) 2  49  (OM  CN ) 2
Lại có OM  MN  NC  OC  OM  NC  OC  MN  5  2
Dấu "=" xảy ra khi và chỉ khi O , M , N , C thẳng hàng.
Vậy AM  BN  49  (OM  CN )2  49  (5  2)2  76  10 2 .
b 10
Suy ra a  76; b  10    0,13 .
a 76
Câu 20. (Sở Hà Tĩnh 2022) Trong không gian Oxyz , cho hai mặt cầu
4 7 14
 S1  : x 2  ( y  1)2  ( z  2)2  16;  S2  : ( x  1)2  ( y  1)2  z 2  1 và điểm A  ; ;   . Gọi ( P )
3 3 3
là mặt phẳng tiếp xúc với cả hai mặt cầu  S1  ,  S2  và I là tâm của  S1  . Xét điểm M ( a; b; c ) di
động trên ( P ) sao cho IM tiếp xúc với mặt cầu  S2  , khi AM ngắn nhất thì a  b  c bằng
A. 1.
B. 1 .
7
C. .
3
7
D.  .
3
Lời giải
Mặt cầu  S1  có tâm I (0;1; 2), R1  4 và mặt cầu  S2  có tâm J (1; 1;0), R2  1 .
Do IJ  R1  R2  3   S1  ,  S2  tiếp xúc trong tại E như hình vẽ (vẽ đại diện là đường tròn).

Facebook Nguyễn Vương https://www.facebook.com/phong.baovuongTrang 13


Blog: Nguyễn Bảo Vương: https://www.nbv.edu.vn/

 1  1 4 5 2
Ta có JE  IJ  (1; 2; 2)  E  ;  ;   và ( P ) là mặt phẳng qua E vuông góc với IJ
3 3 3 3 3
 4 7 14 
có phương trình là ( P ) : x  2 y  2 z  6  0  A  ; ;    ( P ) . Theo giả thiết thì IM tiếp xúc
3 3 3
với  S2  tại N .
JN 1 1 EM 4
Tinh toán sin I    tan I    EM   2.
IJ 3 8 EI 8
Khi đó AM  AE  EM  4 2  2  3 2 . Dấu bằng xảy ra khi và chỉ khi A, M , E thẳng hàng
như hình vẽ:

 3  3  4 14 7 
Ta có AM  AE  (0; 4; 4)  (0; 3;3)  M  ;  ;   a  b  c  1 .
4 4 3 3 3
Câu 21. (Sở Thanh Hóa 2022) Trong không gian Oxyz , cho bốn điếm
A(2;3;5), B ( 1;3; 2), C ( 2;1;3), D (5; 7; 4) . Xét điếm M ( a; b; c ) di động trên mặt phắng (Oxy ) , khi
T  4 MA2  5MB 2  6 MC 2  MD 4 đạt giá trị nhỏ nhất thì a  b  c bằng
A. 11.
B. 11 .
C. 12.
D. 9.
    Lời giải
Gọi I thoả mãn 4 IA  5 IB  6 IC  0  I (5;7; 4)  D (5;7; 4) .
Khi đó
T  (4  5  6) IM 2  (4
IA2  IB 2  6 IC
5 2 4 2 4 2 2 2
 )  MI  3d ( I , (Oxy ))  d ( I , (Oxy ))  4 IA  5 IB  6 IC 
const
const.
Dấu bằng đạt tại M  h / c ( I , (Oxy ))  M (5; 7; 0) .
Câu 22. (Sở Bắc Giang 2022) Trong không gian Oxyz , biết rằng không có đường thẳng nào cắt đồng thời
cả 4 đường thẳng
x  6  t
x 3 y 3 z x 1 y 1 z x y  2 z 1 
d1 :   ; d2 :   ; d3 :   ; d 4 :  y  a  3t.
1 1 1 1 2 1 1 1 1 z  b  t

Giá trị 2b  a bằng
A. 2
B. 3
C. 2.
Trang 14 Fanpage Nguyễn Bảo Vương  https://www.facebook.com/tracnghiemtoanthpt489/
Điện thoại: 0946798489 TUYỂN CHỌN VẬN DỤNG – VẬN DỤNG CAO 2022
D. 3 .
Lời giải

Đường thẳng d1 có vec-tơ chỉ phương u1  (1;1;1) và đi qua điểm A(3; 3; 0) .

Đường thẳng d 3 có vec-tơ chỉ phương u3  (1; 1; 1) và đi qua điểm B (0; 2; 1) .

BA  (3; 1;1) .
   
Vì u1 , u3 cùng phương và u1 không cùng phương BA nên d1 / / d3 .
Gọi ( ) là mặt phẳng chứa d1 , d3 .
  
Khi đó ( ) nhận n   BA, u1   2(1; 2; 1) làm vec-tơ pháp tuyến và ( ) đi qua B (0; 2; 1)
nên nó có phương trình là:
1( x  0)  2( y  2)  ( z  1)  0  x  2 y  z  3  0.
x 1 y 1 z
Dễ thấy ( ) : x  2 y  z  3  0 cắt d 2 :   tại điểm M (0; 1;1) . d 4 có vec-tơ chỉ
  1 2 1
phương u4  (1;3;1) . Do n  u4  0 nên  , d 4 cắt nhau. Gọi tọa độ giao điểm tương ứng của
chúng là N (6  t ; a  3t ; b  t ) .

MN  (6  t ; a  1  3t ; b  1  t ).

Vì không có đường thẳng nào cắt đồng thời cả 4 đường thẳng đã cho nên suy ra MN cùng

phương với u1  (1;1;1) .

6  t a  1  3t b  1  t a  1  3t  6  t 4t  7  a
      2b  a  3
1 1 1 b  1  t  6  t 4t  10  2b
Câu 23. (Sở Hà Tĩnh 2022) Trong không gian với hệ trục tọa độ Oxyz , cho mặt cầu
( S ) : x 2  y 2  z 2  2 x  4 y  2 z  0 và điểm M (0;1; 0) . Mặt phẳng ( P ) đi qua M và cắt ( S ) theo
đường tròn (C ) có chu vi nhỏ nhất. Gọi N  x0 ; y0 ; z0  là điểm thuộc đường tròn (C ) sao cho
ON  6 . Tính y0 .
A. 3.
B. 1.
C. 2.
D. 4.
Lời giải.

Nhận thấy rằng, mặt cầu ( S ) có tâm I ( 1; 2;1) , bán kính R  6 và điểm M là điểm nằm trong
mặt cầu này. Gọi r là bán kính hình tròn (C ) và H là hình chiếu của I lên ( P ) . Dễ thấy rằng
H là tâm đường tròn (C ) . Khi đó, ta có r  R 2  IH 2  R 2  IM 2 .
Vậy để (C ) có chu vi nhỏ nhất thì r nhỏ nhất khi đó H trùng với M .

Facebook Nguyễn Vương https://www.facebook.com/phong.baovuongTrang 15


Blog: Nguyễn Bảo Vương: https://www.nbv.edu.vn/


Khi đó mặt phẳng ( P ) đi qua M (0;1; 0) và nhận vectơ IM  (1; 1; 1) làm vectơ pháp tuyến.
Phương trình mặt phẳng (P) có dạng x  ( y  1)  z  0  x  y  z  1.
Điểm N vừa thuộc mặt cầu ( S ) vừa thuộc mặt phẳng ( P ) và thỏa ON  6 nên tọa độ của N
thỏa hệ phương trình.
 x02  y02  z02  2 x0  4 y0  2 z0  0 2 x0  4 y0  2 z0  6
 2 2 2 
 x0  y0  z0  6   x02  y02  z02  6
 x  y  z  1  x  y  z  1.
 0 0 0  0 0 0

Lấy phương trình đầu trừ hai lần phương trình thứ ba ta được 2 y0  4  y0  2 .
Câu 24. (Sở Bạc Liêu 2022) Trong không gian với hệ trục toạ độ Oxyz , cho mặt cầu
2 2 2
 S  :  x  1   y  1   z  2   9 và điểm M 1;3; 1 , biết rằng các tiếp điểm của các tiếp
tuyến kẻ từ M tới các mặt cầu đã cho luôn thuộc một đường tròn  C  có tâm J  a; b; c  . Giá trị
T  2a  b  c bằng
134 62 84 116
A. T  . B. T  . C. T  . D. T  .
25 25 25 25
Lời giải
Chọn C
A

M J I

2 2 2
Ta có  S  :  x  1   y  1   z  2   9  I 1; 1; 2  ; R  9 .

2
M 1;3; 1  IM  02  42   3  5 .

Trang 16 Fanpage Nguyễn Bảo Vương  https://www.facebook.com/tracnghiemtoanthpt489/


Điện thoại: 0946798489 TUYỂN CHỌN VẬN DỤNG – VẬN DỤNG CAO 2022
Gọi A là một tiếp điểm nên AM  MI  IA  52  32  4 . 2 2

2 2 2
Mặt cầu tâm M bán kính AM  4 dạng  x  1   y  3   z  1  16 .

 x  12   y  32   z  1 2  16


Toạ độ A là nghiệm của hệ  2 2 2
 4 y  3z  1  0 .
  x  1   y  1   z  2   9

Hay A   P  : 4 y  3z  1  0 .

 x 1

J là hình chiếu của I lên mặt phẳng  P  . Đường thẳng IJ dạng  y  1  4t .
 z  2  3t


 4 y  3x  1  0  x  1
 x 1 
  11  11 23 
J  IJ   P   J là nghiệm của hệ    y   J 1; ;   J  a; b; c  .
 y  1  4t  25  25 25 
 z  2  3t  23
 z  25

84
Nên T  2a  b  c  .
25
Câu 25. (Sở Hà Tĩnh 2022) Trong không gian Oxyz, cho hai điểm A  2; 3; 5  , I  2;0; 1 và mặt phẳng

Điểm M  a; b; c  thay đổi thuộc mặt phẳng   sao cho IM  5 và độ


P
 P  : 2 x  y  2 z  5  0.
dài đoạn AM lớn nhất. Khi đó giá trị của biểu thức T  a  b  2c bằng
1
A. 1 . B. 11 . C. 6 . D.  .
3

Lời giải
Chọn B
 Có M thuộc mặt cầu  S  (tâm I , bán kính R  5 ).
4025 11

Do d I ,  P    3
 5 nên  S  cắt  P  theo giao tuyến là đường tròn  C1  và M   C1  .

4 1 4
 Gọi Q  là mặt phẳng qua A và song song với  P  , có  Q  : 2 x  y  2 z  17  0 và
4  0  2  17 11
d  I , Q    . Suy ra  S  cắt  Q  theo giao tuyến là đường tròn  C2  và A   C2  (do
4 1 4 3
IA  5 nên A   S  )

Facebook Nguyễn Vương https://www.facebook.com/phong.baovuongTrang 17


Blog: Nguyễn Bảo Vương: https://www.nbv.edu.vn/
d  I ,  P    d  I ,  Q   nên  C1  bằng  C2  và A, M lần lượt thuộc hai đường tròn đáy của hình trụ. Do
 Do

đó AM lớn nhất khi M là điểm đối xứng với A qua I , suy ra M  2;3;3 và T  2  3  2.3  11.
x 5 y 7 z 3
Câu 26. (Sở Hà Tĩnh 2022) Trong không gian Oxyz, cho ba đường thẳng d :   ,
1 2 3
x y 1 z  3 x  2 y 3 z
d1 :   và d 2 :   . Gọi  là đường thẳng song song với d đồng thời
2 1 2 1 3 2
cắt cả hai đường thẳng d1 và d 2 . Đường thẳng  đi qua điểm nào sau đây?
A.  4;10;17  . B.  4;1; 7  . C.  3; 12;10  . D. 1; 6;6  .
Lời giải
Chọn A
Gọi A    d1  A  2t1 ; 1  t1 ; 3  2t1  .
B    d 2  B  2  t2 ;3  3t2 ; 2t2  .

Ta có AB   2  t2  2t1; 4  3t2  t1 ;3  2t2  2t1  .
 
Do  là đường thẳng song song với d nên AB , ud cùng phương.
2  t2  2t1 4  3t2  t1 3  2t2  2t1
  
1 2 3
4  2t2  4t1  4  3t2  t1 3t1  5t2  8 t1  1
  
6  3t2  6t1  3  2t2  2t1 8t1  t2  9 t2  1

Đường thẳng  qua B  1;0; 2  và nhận vectơ chỉ phương AB  1; 2;3 có phương trình là
x 1 y z  2
:   .
1 2 3
4  1 10 17  2
Thay tọa độ  4;10;17  vào phương trình đường thẳng  ta được   (thỏa mãn).
1 2 3
2 2
Câu 27. (Sở Phú Thọ 2022) Trong không gian Oxyz , cho mặt cầu  S  :  x  2   y 2   z  5   24 cắt
mặt phẳng   : x  y  4  0 theo giao tuyến là đường tròn  C  . Điểm M thuộc  C  sao cho
khoảng cách từ M đến A  4; 12;1 nhỏ nhất. Tung độ của điểm M bằng
A. 6 . B. 4 . C. 0 . D. 2 .
Lời giải
Chọn B

Mặt cầu  S  có tâm I  2;0; 5 , bán kính R  2 6 .

Trang 18 Fanpage Nguyễn Bảo Vương  https://www.facebook.com/tracnghiemtoanthpt489/


Điện thoại: 0946798489 TUYỂN CHỌN VẬN DỤNG – VẬN DỤNG CAO 2022
2 2
Ta có: A     ; AI   6   12   6   6 6  A nằm ngoài mặt cầu  S  .
2

2  0  4
Khoảng cách từ tâm I đến    là: d  d  I ,       2.
2
2 2
Suy ra bán kính đường tròn  C  là: r  R 2  d 2  2 6    2   22 .
Gọi K là hình chiếu của I trên    .

Ta có IK     nên đường thẳng IK nhận vectơ n  1;1;0  làm vectơ chỉ phương.
 x  2  t

Phương trình IK :  y  t  K  2  t; t; 5  .
 z  5

Vì K     nên: 2  t  t  4  0  t  1  K  3; 1; 5  .
Gọi H là hình chiếu của A trên    .

Ta có AH     nên đường thẳng AH nhận vectơ n  1;1;0  làm vectơ chỉ phương.
x  4  t

Phương trình AH :  y  12  t  H  4  t; 12  t;1 .
z  1

Vì H     nên: 4  t  12  t  4  0  t  2  H  6; 10;1  AH  22  22  2 2 .
 KH  92  92  62  3 22  r  H nằm ngoài đường tròn  C  .
Khi đó ta có: AM  AH 2  HM 2  HM 2  8 .
Suy ra AM min khi HM min  H , M , K thẳng hàng ( theo thứ tự đó).
 2 
Khi đó: HM  HK (*).
3
 2
 x  6  3  9 
 x  0
 2 
Gọi M  a; b; c  . Từ (*) ta có:  y  10  .9   y  4 .
 3  z  3
 2 
 z  1  3 .  6 

Vậy, M  0; 4; 3 nên tung độ của M bằng 4 .
Câu 28. (THPT Bùi Thị Xuân – Huế - 2022) Trong không gian Oxyz , cho mặt cầu x 2  y 2  z 2  9 và
x  1 t

điểm M  x0 ; y0 ; z0   d :  y  1  2t . Ba điểm A, B, C phân biệt cùng thuộc mặt cầu sao
 z  2  3t

cho MA, MB, MC là tiếp tuyến của mặt cầu. Biết rằng mặt phẳng ( ABC ) đi qua điểm D (1;1; 2) .
Tổng T  x02  y02  z02 bằng
A. 21.
B. 30.
C. 20.
D. 26.
Lời giải
Mặt cầu ( S ) có tâm O (0; 0; 0) và bán kính R  3 .
Gọi A( x; y; z )  ( S ) ta có OA2  AM 2  OM 2

Facebook Nguyễn Vương https://www.facebook.com/phong.baovuongTrang 19


Blog: Nguyễn Bảo Vương: https://www.nbv.edu.vn/
2 2 2 2 2 2
 9   x  1  t0     y  1  2t0     z   2  3t0    1  t0   1  2t0    2  3t0 
 1  t0  x  1  2t0  y   2  3t0  z  9  0 (*)
Tương tự, tọa độ điểm B, C cũng thỏa mãn (*) . Hay nói cách khác, phương trình mặt phẳng
( ABC ) là: 1  t0  x  1  2t0  y   2  3t0  z  9  0
Mặt khác vì ( ABC ) đi qua D (1;1; 2) nên
1  t0  1  1  2t0  1   2  3t0   2  9  0  t0  1.
Suy ra M (0; 1;5) . Vậy T  x02  y02  z02  26 .
Câu 29. (Chuyên Hạ Long 2022) Trong không gian với hệ tọa độ Oxyz , cho điểm N (2;3; 4) . Một mặt
cầu bất kỳ đi qua O và N cắt các trục tọa độ Ox, Oy , Oz lần lượt tại A, B, C  0 . Biết rằng khi
mặt cầu thay đổi nhưng vẫn thỏa đề bài, trọng tâm G của tam giác ABC luôn nằm trên một mặt
phẳng cố định. Mặt phẳng cố định này chắn các trục tọa độ thành một tứ diện, tính thể tích của
khối tứ diện đó.
24389
A. .
3888
24389
B. .
4374
24389
C. .
8748
24389
D. .
2916
Lời giải
Giả sử A( a;0; 0)  ( S )  Ox, B (0; b; 0)  ( S )  Oy và C (0; 0; c)  ( S )  Oz .
a b c
Khi đó I là tâm của mặt cầu có tọa độ là I  ; ;  .
2 2 2
 2  a b c
Theo tính chất hình hộp, ta có OG  OI  G  ; ;  .
3  3 3 3
Do O, N  ( S )  IO  IN  I thuộc mặt phẳng trung trực của đoạn ON
a b c 29 29
 2a  3b  4c  29  2   3   4    2 xG  3 yG  4 zG 
3 3 3 3 3
29
Suy ra G  ( P ) : 2 x  3 y  4 z  .
3
 29   29 
Gọi M  ( P )  Ox  M  ; 0;0  , N  ( P )  Oy  N  0; ;0 
 6   9 
 29 
Và P  ( P )  Oz  P  0;0;  .
 12 
1 24389
Vậy VOMNP  OM .ON .OP  .
6 3888
Câu 30. (Chuyên Hoàng Văn Thụ - Hòa Bình – 2022) Trong không gian Oxyz cho hai điểm
A(4; 6; 2), B (2; 2; 0) và mặt phẳng ( P ) : x  y  z  0 . Xét đường thẳng d thay đổi thuộc ( P ) và
đi qua B , gọi H là hình chiếu vuông góc của A trên d . Biết rằng khi d thay đổi thì H thuộc
một đường tròn cố định. Diện tích của hình tròn đó bằng
A. 4 .
B.  .
C. 6 .
D. 3 .
Lời giải
Chọn C
Trang 20 Fanpage Nguyễn Bảo Vương  https://www.facebook.com/tracnghiemtoanthpt489/
Điện thoại: 0946798489 TUYỂN CHỌN VẬN DỤNG – VẬN DỤNG CAO 2022
Cách 1:
Do BHA  90 nên H thuộc mặt cầu đường kính AB, H  ( P ) , do đó, H chạy trên đường tròn là
giao của mặt cầu đường kính AB và ( P ) . Đường tròn này có tâm là hình chiếu vuông góc của I
1
lên ( P ) với I là trung điểm của AB , bán kính bằng độ dài hình chiếu vuông góc của AB trên
2
( P) .

   



Ta có BA  (2;8; 2); nP  (1;1;1), BA, n p   
 
BA  nP
Ta có | cos  |  
| BA |  nP
1  1 
r  | BA |  | sin  | | BA |  1  cos 2   6 S   r 2  6
2 2
12
Cách 2: Ta có AB 2  72, d ( A, ( P ))   4 3 , vậy hình chiếu vuông góc của AB trên ( P ) có
3
độ dài là AB 2  d 2  2 6 , bán kính r  6.S   r 2  6
Câu 31. (Chuyên Lam Sơn – 2022) Trong không gian với hệ trục tọa độ Oxyz , cho 3 đường thẳng
 x  1  2t1  x  3  t2  x  4  2t3
  
 d1  ,  d2  ,  d3  có phương trình  d1  :  y  1  t1 ,  d 2  :  y  1  2t2 ,  d3  :  y  4  2t3 tiếp xúc
 z  1  2t  
 1  z  2  2t2  z  1  t3
với 3 đường thẳng đó. Giá trị nhỏ nhất của R gần số nào nhất trong các số sau:
A. 2,1.
B. 2,2.
C. 2,3.
D. 2,4.
Lờí giải
Ta có:  d1  đi qua điểm A(1;1;1) có VTCP u1  (2;1; 2) .
 d2  đi qua điểm B(3; 1; 2) có VTCP u2  (1; 2; 2) .
 d3  đi qua điểm C (4; 4;1) có VTCPu3  (2; 2;1) .
 
Ta có u1  u2  0, u2  u3  0, u3  u1  0   d1  ,  d 2  ,  d3  đôi một vuông góc với nhau.
   
u1 , u2   AB  0, u2 , u3   BC  0, u3 , u1   CA  0   d1  ,  d 2  ,  d 3  đôi một chéo nhau.
     

Facebook Nguyễn Vương https://www.facebook.com/phong.baovuongTrang 21


Blog: Nguyễn Bảo Vương: https://www.nbv.edu.vn/

Lại có: AB  (2; 2;1); AB  u1  0 và AB  u2  0 nên  d1  ,  d 2  ,  d3  chứa 3 cạnh của hình hộp
chữ nhật như hình vẽ.

Vì mặt cầu tâm I ( a; b; c ) tiếp xúc với 3 đường thẳng  d1  ,  d2  ,  d3  nên bán kính
2 2 2
R  d  I , d1   d  I , d 2   d  I , d3   R  d  I , d1   d  I , d 2   d 2  I , d3 
   2  
 2
  AI , u     BI , u    CI , u  
2
  
 R2  


1
     2      3
 , ta thấy u 2  u 2  u 2  9 và
1 2 3
 u1   u2   u3 
     
  
AI  (a  1; b  1; c  1),  AI , u1   (2b  c  3; 2a  2c  4; a  2b  1)
 
BI  (a  3; b  1; c  2),  BI , u2   (2b  2c  6; 2a  c  4; 2a  b  7).

CI  (a  4; b  4; c  1), CI , u3   (b  2c  6; a  2c  2; 2a  2b  16).
  2   2   2   2   2   2
9 R 2   AI , u1    BI , u2   CI , u3   27 R 2   AI , u1    BI , u2   CI , u3 
2 2 2
 7  3  3  243 243
 18  a  b  c   126a  54b  54c  423  18  a    18  b    18  c   
2 2 2

 2  2  2 2 2
3 2
 Rmin  , R  2,12.
2
Câu 32. (Chuyên Lam Sơn 2022) Trong không gian với hệ trục tọa độ Oxyz cho điểm I (1; 0; 0) , điểm
x  2
7 4 4 
M  ; ;  và đường thẳng d :  y  t .N (a, b, c) là điểm thuộc đường thẳng d sao cho diện
9 9 9 z  1 t

tích tam giác IMN nhỏ nhất. Khi đó a  b  c có giá trị bằng:
A. 2.
B. 2 .
5
C. .
2
5
D. .
2
Lời giải
2
Ta có IM  .
3
1 1
Gọi H là hình chiếu của N trên đường thẳng d đi qua I , M , ta có: S MMN  IM  NH  NH
2 3
Diện tích tam giác IMN nhỏ nhất khi và chỉ khi độ dài NH nhỏ nhất.

N  d  N (2; n;1  n)  IN  (1; n;1  n).

Trang 22 Fanpage Nguyễn Bảo Vương  https://www.facebook.com/tracnghiemtoanthpt489/


Điện thoại: 0946798489 TUYỂN CHỌN VẬN DỤNG – VẬN DỤNG CAO 2022
  
Đường thẳng d có vecto chỉ phương u  (1; 2; 2)   IN , u  (2; n  3;  n  2) .
2
   5 9
2 n   
 IN , u 2 2
2  (n  3)  (n  2) 2
2 4 1
  
NH  d  N ; d        .
u 3 3 2
5  5 3
Dấu = xảy ra khi n   , suy ra: N  2;  ;   . Vậy a  b  c  2 .
2  2 2
Câu 33. (Chuyên Nguyễn Trãi – Hải Dương – 2022) Trong không gian với hệ trục Oxyz , cho mặt cầu
( S ) : x 2  y 2  z 2  1 và hai điểm A(3; 0; 0); B (1;1; 0) . Gọi M là điểm thuộc mặt cầu ( S ) . Tính
giá trị nhỏ nhất của biểu thức MA  3MB .
A. 2 34
B. 26
C. 5
D. 34
Lời giải
Gọi M ( x; y; z ) là điểm cần tìm.
Ta có : M  ( S )  x 2  y 2  z 2  1  0 .
MA  ( x  3) 2  y 2  z 2 ; MB  ( x  1) 2  ( y  1) 2  z 2 .
Suy ra: MA  3MB  ( x  3) 2  y 2  z 2  3 ( x  1) 2  ( y  1) 2  z 2
 ( x  3)2  y 2  z 2  8  x 2  y 2  z 2   8  3 ( x  1) 2  ( y  1)2  z 2
2
 1
 3  x    y 2  z 2  3 ( x  1)2  ( y  1) 2  z 2
 3
1 
 3( MC  MB)  3BC vói C  ;0; 0  .
 3 
Vậy giá trị nhỏ nhất của biểu thức MA  3MB bằng 5khi
 M  BC  ( S )  38 6 4 6 6 
    M  ; ;0  .
CM  k  CB(k  0)  25 25 
Câu 34. (THPT Đô Lương – Nghệ An – 2022) Trong không gian Oxyz cho đường tròn (C ) là giao
tuyến của mặt phẳng tọa độ ( xOy ) với mặt cầu ( S ) : ( x  6)2  ( y  6)2  ( z  3) 2  41 . Gọi d là
đường thẳng đi qua các điểm A(0; 0;12), B (0; 4;8) . Với M , N là các điểm thay đổi thứ tự trên
(C ) và d . Giá trị nhỏ nhất của độ dài đoạn MN gần với giá trị nào nhất sau đây?
A. 3,5.
B. 2,35.
C. 1,25.
D. 2,92.
Lời giải
2 2
Phưong trình (C ) : ( x  6)  ( y  6)  32 với tâm I (6; 6) và bán kính R  4 2 .
Gọi F  ( d )  Oy , IK  Oy , D  IK  (C ), G  IF  (C ), ML  Oy . Như vậy đề MN đạt giá trị
nhỏ nhẩt thì M phải thuộc cung nhỏ DG .

Facebook Nguyễn Vương https://www.facebook.com/phong.baovuongTrang 23


Blog: Nguyễn Bảo Vương: https://www.nbv.edu.vn/
  NL FL
 KH  (d ) : NL  (d )   6 x 6 x
   KH FK  NL  KH 
 LK  x  KH  d ( K : (d ))  3 2 6 2
 
LM  6  R 2  d 2 ( I ;( ML))  6  32  KL2  6  32  x 2
2
6 x 2
(6  x) 2 2
MN  ML2  LN 2  
 2 
  6  32  x2
 
2

 6  32  x 2   f ( x)

(6  x)2 2
Xét hàm số y  f ( x) 
2

 6  32  x 2  , x [4 2; 4 2] .

f ( x)  0  12 x  ( x  6) 32  x 2  0  x  x0  3,5145  [4 2; 4 2]
 min f ( x )  f (3, 5145)  2,35488
Câu 35. (THPT Lương Thế Vinh – Hà Nội – 2022) Trong không gian Oxyz , cho hai điếm
A(2; 1; 1), B (0;1; 2) và mặt phắng ( P ) : 2 x  y  2 z  2  0 . Điếm M thuộc mặt phắng ( P ) sao
cho AMB Ión nhất, khì đó cos  AMB bằng
5
A. .
13
12
B. .
13
12
C.  .
13
5
D.  .
13
Lời giải
Gọi (Q ) là mặt phẳng chửa AB vả vuông góc với ( P ) ; gọi d  ( P )  (Q ) .
Gọi H  h / c( M , d )   AMB   AHB đến đây các em có thể đưa về hàm một biến nhờ viết phương
trình của d , tham số hoá tọa độ điểm H , tinh cos  AHB tìm giá trị nhỏ nhất của biểu thức này
hoặc sử dụng
 hình học như
 sau:
Để ý AB(2; 2; 1)  nP (2;1; 2), A  ( P)  AB / /( P)  AB / / d .
Trong mặt phẳng (Q ) xét đường tròn qua hai điểm A, B và tiếp xúc với d tại M 0 ,  M 0 chính là
giao điểm của đường trung trực đoạn AB với đường thẳng d )
Nếu H  M 0  HB cắt đường tròn tại điểm N và  AHB  ANB  
AM B vậy 
AHBmax  
AM 0 B
không đổi. Dấu bằng đạt tại M  H  M 0

9 13
Ta có AB  3, IM 0  d ( A, ( P))  1, M 0 A  M 0 B  IA2  IM 02  1   .
4 2

Trang 24 Fanpage Nguyễn Bảo Vương  https://www.facebook.com/tracnghiemtoanthpt489/


Điện thoại: 0946798489 TUYỂN CHỌN VẬN DỤNG – VẬN DỤNG CAO 2022
13 13
M A 2
 M B 2
 AB 2  9
5
Khi đó cos 
AM 0 B  0 0
 4 4  .
2M 0 M 0 B 13 13
2
4
Câu 36. (THPT Kim Liên - Hà Nội - 2022) Trong không gian Oxyz , cho hai điểm A  1; 2;3 và
B  3; 2;5  . Xét hai điểm M và N thay đổi thuộc mặt phẳng  Oxy  sao cho MN  2023 . Tìm giá
trị nhỏ nhất của AM  BN .
A. 2 17 . B. 65 . C. 25 97 . D. 205 97 .
Lời giải
Chọn D
Hình chiếu vuông góc của A  1; 2;3 xuống mặt phẳng  Oxy  là C  1; 2;0  .
Hình chiếu vuông góc của B  3; 2;5  xuống mặt phẳng  Oxy  là D  3;2;0  .
 
Khi đó CD   4; 0;0   ud  1;0;0  .
x  3  t

Phương trình đường thẳng CD là  y  2 .
z  0

M  3  m; 2;0 
Để AM  BN nhỏ nhất thì hai điểm M và N thay đổi trên CD   .
 N  3  n; 2;0 
Theo bài MN  2023  n  m  2023 .
 2

Ta có AM   m  4;0; 3  AM   m  4   9 và BN   n;0; 5  BN  n 2  25 .
2 2 2
Khi đó AM  BN   m  4  9  n 2  25   m  4 9   m  2023  25 .
2 2
Đặt f  m    m  4 9   m  2023  25 .
m4 m  2023 6089
Ta có f   m    0m .
 m  4
2
9  m  2023
2
 25 8
6089
Vậy giá trị nhỏ nhất của f  m bằng 205 97 đạt được tại m  
.
8
Câu 37. (THPT Kim Liên - Hà Nội - 2022) Trong không gian Oxyz , cho mặt phẳng
 P  : 2 x  2 y  z  5  0 và mặt cầu  S  có tâm I 1;2; 2  . Biết  P  cắt  S  theo giao tuyến là
đường tròn  C  có chu vi 8π . Tìm bán kính của mặt cầu T  chứa đường tròn  C  và T  đi
qua điểm M 1;1;1 .
265 5 5
A. R  5 . B. R  . C. R  . D. R  4 .
4 4

Lời giải
Chọn B
Đường tròn  C  có chu vi 8π nên có bán kính r  4 .

Gọi J là tâm của mặt cầu T  , theo đề ta có  P  cắt  S  và  T  theo cùng một giao tuyến là
đường tròn  C  do đó IJ   P  .

Facebook Nguyễn Vương https://www.facebook.com/phong.baovuongTrang 25


Blog: Nguyễn Bảo Vương: https://www.nbv.edu.vn/

Đường thẳng IJ đi qua điểm I và nhận vectơ n P    2; 2;1 làm vectơ chỉ phương nên có
 x  1  2t

phương trình  y  2  2t .
 z  2  t

Suy ra J 1  2t;2  2t; 2  t  .

2 1  2t   2  2  2t   2  t  5 9t  9
Ta có: d  d  J ;  P      3 t 1 .
3 3
2 2
Bán kính của mặt cầu T  : R  JM  4t 2   2t  1   t  3 .
2 2 2
Ta có: d 2  r 2  R 2  9  t  1  16  4t 2   2t  1   t  3

3
 9t 2  18t  25  9t 2  2t  10  20t  15  t   .
4
265
Vậy bán kính của mặt cầu T  là R 
.
4
Câu 38. (THPT Kinh Môn - Hải Dương - 2022) Trong không gian Oxyz , cho mặt cầu  S1  có tâm
I  2;1;1 có bán kính bằng 4 và mặt cầu
 S 2  có tâm J  2;1;5  có bán kính bằng 2. Gọi  P  là mặt phẳng thay đổi tiếp xúc với hai mặt
cầu  S1  ,  S 2  . Đặt M , m lần lượt là giá trị lớn nhất, giá trị nhỏ nhất của khoảng cách từ điểm O
đến mặt phẳng  P  . Giá trị M  m bằng
A. 8 . B. 9 . C. 8 3 . D. 15 .
Lời giải
Chọn B

 S1  có bán kính R1  4 ,  S 2  có bán kính R2  2 .


Nhận xét: R1  R2  IJ  4  R1  R2 nên hai mặt cầu này cắt nhau.
1
Vì R2  R1 nên theo định lý Talet suy ra J là trung điểm đoạn MI nên M  2;1;9  .
2

P 2 2 2

qua M  2;1;9  và có vectơ pháp tuyến n   a; b; c  , a  b  c  0 nên phương trình  P  
có dạng: ax  by  cz  2a  b  9c  0 .
8c 2c
Ta có: d  I ,  P    4  4   1  3c 2  a 2  b2 .
2 2 2 2 2 2
a b c a b c
Nếu c  0 thì a  b  0 (vô lý) nên c  0 . Không mất tính tổng quát, chọn c  1 , suy ra

Trang 26 Fanpage Nguyễn Bảo Vương  https://www.facebook.com/tracnghiemtoanthpt489/


Điện thoại: 0946798489 TUYỂN CHỌN VẬN DỤNG – VẬN DỤNG CAO 2022
2 2
a b  3.
Do đó tồn tại t   0; 2  sao cho a  3 sin t , b  3 cos t .

2a  b  9c 2 3 sin t  3 cos t  9 2 3 sin t  3 cos t  9


Mặt khác: d  d  O,  P      ,
a b c 2 2 2 2 2
(vì 2 3 sin t  3 cos t  9  0, t   0; 2  ).
Biến đổi ta được: 2 3 sin t  3 cos t  2d  9 .
Điều kiện có nghiệm:
2 2 9  15 9  15
2 3   3 2
  2d  9   4d 2  36d  66  0 
2
d 
2
.

9  15 9  15
Vậy M  ,m  nên M  m  9 .
2 2
Câu 39. (THPT Lương Tài 2 - Bắc Ninh - 2022) Trong không gian với hệ tọa độ Oxyz , cho điểm
A 2;4;  2 và mặt phẳng  P  : m 2  1 x   m2  1 x  2mz  4  0 . Biết rằng,khi tham số m thay
đổi thì mặt phẳng  P  luôn tiếp xúc với hai mặt cầu cố định cùng đi qua A là  S1  và  S2  . Gọi
M và N lần lượt nằm trên  S1  và  S2  . Tìm giá trị lớn nhất của MN ?
A. 16 2 . B. 8  8 2 . C. 8 2 . D. 8  6 2 .
Lời giải
Chọn B
Gọi tâm mặt cầu tiếp xúc với  P  là điểm I  a; b; c  .
2 2 2
Do mặt cầu đi qua điểm A  2;4;  2 nên ta có:  2  a    4  b    2  c   R 2 (1)
Do mặt cầu đi tiếp xúc với  P  nên:
m 2
 1 a   m2  1 b  2mc  4 m 2
 1  a  b   2b  2mc  4
R  d  I ,  P   .
2  m  1
2 2
2  m2  1

c  0
Vì mặt cầu là cố định nên bán kính R không đổi, do đó 2mc  2b  4  0, m   .
b  2
a2 a2
Khi đó R , thay b  2, c  0 và R vào 1 ta được
2 2
a  2  I1  2; 2; 0  , R1  2 2
a 2  12a  20  0    .
 a  10  I 2  2;10;0  , R2  6 2
Từ đó ta có max MN  I1I 2  R1  R2  8  8 2 .
Câu 40. (THPT Võ Nguyên Giáp - Quảng Bình - 2022) Trong không gian Oxyz , cho ba điểm
A(1;2;3), B  0;1;0  , C (1;0; 2) và mặt phẳng ( P ) : x  y  z  2  0 . Điểm M  a; b; c  nằm trên ( P )
sao cho biểu thức MA2  2 MB 2  3MC 2 đạt giá trị nhỏ nhất. Khi đó giá trị của biểu thức
T  a  b  9c bằng
13 13
A. . B. . C. 13 . D. 13 .
9 9
Lời giải
Chọn D
     2 2 1 
Gọi I  x; y; z  là điểm thoả mãn IA  2 IB  3 IC  0  I  ; ;  .
3 3 2 
2 2 2 2 2 2 2
   
Ta có: MA  2MB  3MC  6MI  IA  2IB  3IC  2MI ( IA  2 IB  3IC )

Facebook Nguyễn Vương https://www.facebook.com/phong.baovuongTrang 27


Blog: Nguyễn Bảo Vương: https://www.nbv.edu.vn/
 6 MI 2  IA2  2 IB 2  3IC 2
Vì IA2  2 IB 2  3IC 2 không đổi nên biểu thức MA2  2 MB 2  3MC 2 nhỏ nhất  MI 2 nhỏ
nhất  M là hình chiếu vuông góc của điểm I trên ( P ) .
Gọi  là đường thẳng đi qua điểm I và vuông góc với mặt phẳng ( P ) .
 2
x  3  t

 2
Phương trình tham số của  :  y   t .
 3
 1
z  2  t

2 2 1 17  5 5 13 
Xét phương trình:  t   t   t  2  0  t  M ; ; 
3 3 2 18  18 18 9 
5 5 13
Vậy T  a  b  9c    9.  13 .
18 18 9

Theo dõi Fanpage: Nguyễn Bảo Vương  https://www.facebook.com/tracnghiemtoanthpt489/

Hoặc Facebook: Nguyễn Vương  https://www.facebook.com/phong.baovuong

Tham gia ngay: Nhóm Nguyễn Bào Vương (TÀI LIỆU TOÁN)  https://www.facebook.com/groups/703546230477890/

Ấn sub kênh Youtube: Nguyễn Vương


 https://www.youtube.com/channel/UCQ4u2J5gIEI1iRUbT3nwJfA?view_as=subscriber

Tải nhiều tài liệu hơn tại: https://www.nbv.edu.vn/

Trang 28 Fanpage Nguyễn Bảo Vương  https://www.facebook.com/tracnghiemtoanthpt489/

You might also like